Phát triển tư duy sáng tạo giải toán Hình học 8

Tài liệu gồm 315 trang, được biên soạn bởi tác giả Bùi Văn Tuyên (chủ biên) và tác giả Nguyễn Đức Trường, tuyển tập các chuyên đề Hình học 8 giúp học sinh phát triển tư duy sáng tạo giải toán Hình học 8.

1
MC LC
CHUYÊN ĐỀ 1. T GIÁC .................................................................................................................................................. 2
CHUYÊN ĐỀ 2. HÌNH THANG. HÌNH THANG CÂN. DNG HÌNH THANG .......................................................... 5
CHUYÊN ĐỀ 3. ĐƢỜNG TRUNG BÌNH CA TAM GIÁC, CA HÌNH THANG ................................................... 11
CHUYÊN ĐỀ 4. HÌNH BÌNH HÀNH ............................................................................................................................... 17
CHUYÊN ĐỀ 5. HÌNH CH NHT ................................................................................................................................ 22
CHUYÊN ĐỀ 6. HÌNH THOI VÀ HÌNH VUÔNG ........................................................................................................ 28
CHUYÊN ĐỀ 7. ĐỐI XNG TRC ĐỐI XNG TÂM ............................................................................................. 35
CHUYÊN ĐỀ 8. VẼ HÌNH PHỤ ĐỂ GIẢI TOÁN ......................................................................................................... 41
2
CHƢƠNG I: T GIÁC
CHUYÊN ĐỀ 1. T GIÁC
A. Kiến thc cn nh
1. T Giác
ABCD
là hình gm bốn đoạn thng
,AB
,BC
,CD
,DA
trong đó bất kì hai đoạn thẳng nào cũng
không cùng nm trên một đường thng.
Hình 1.1
Ta phân bit t giác li (h.1.1a) và t giác lõm (h.1.1b). Nói đến t giác mà không chú thích gì thêm, ta
hiểu đó là tứ giác li.
2. Tng các góc ca t giác bng
360
.
360A B C D
B. Mt s ví d
Ví d 1. Cho t giác
40AB
. Các tia phân giác ca góc
C
D
ct nhau ti
O
. Cho biết
110COD 
. Chng minh rng
AB BC
.
Gii (h.1.2)
Tìm cách gii
Mun chng minh
AB BC
ta chng minh
90B 
.
Đã biết
40AB
, ta tính tng
AB
Trình bày li gii Hình 1.2
Xét tam giác
COD
22
180 180
2
CD
COD C D
(vì
;
12
DD
).
Xét t giác
ABCD
360C D A B
, do đó
360
180 180 180
22
AB
AB
COD
Vy
2
AB
COD
. Theo đề bài
110COD 
nên
220AB
.
A
B
C
D
A
B
C
D
a)
b)
A
B
C
D
A
B
C
D
a)
b)
A
B
C
D
O
1
2
1
2
3
Mt khác
40AB
nên
2
20 40 :2 90B
. Do đó
AB BC
.
Ví d 2. T giác
ABCD
AB BC
và hai cnh
,AD DC
không bằng nhau. Đường chéo
DB
là đường phân giác ca góc
D
.Chng minh rằng các góc đối ca t giác này bù nhau.
Gii
.
1.3 ,h a b
Tìm cách gii
Để chng minh hai góc
A
C
bù nhau, ta to ra mt góc th ba làm trung gian, góc này bng góc
A
chng hạn. Khi đó chỉ còn phi chng minh góc này bù vi góc
C
.
Trình bày li gii
Xét trường hp
AD DC
(h.1.3a)
Trên cnh
DC
lấy điểm
E
sao cho
DE DA
.
( . . )ADB EDB c g c
AB EB
1
AE
Mt khác,
AB BC
nên
BE BC
. Vy
BEC
cân
2
CE
.
Ta có:
12
180 180E E A C
.
.Do đó
360 180B D A C
.
Xét trường hp
AD DC
(h.1.3b).
Trên tia
DA
lấy điểm
E
sao cho
DE DC
Chng
minh tương tự như trên, ta được
180AC
,.
Hình 1.3
180BD
.
Ví d 3. T giác
ABCD
có tổng hai đường chéo bng
a
. Gi
M
là một điểm bt kì. Tìm giá tr nh
nht ca tng
MA MB MC MD
.
Gii (h.1.4)
Tìm cách gii
Để tìm giá tr nh nht ca tng
MA MB MC MD
ta phi chng minh
MA MB MC MD k
(
k
là hng s).
Ghép tng trên thành hai nhóm
M
A MC MB MD
.
Trình bày li gii
Xét ba điểm
,,M A C
MA MC AC
(dấu “=” xảy ra khi
M AC
).
Xét ba điểm
M,B,D
MB MD BD
(dấu “=” xảy ra khi
M BD
).
Do đó
MA MB MC MD AC BD a
Vy
m
in MA MB MC MD a
khi
M
A
B
C
D
1
2
A
B
C
D
E
1
2
a)
b)
A
B
C
D
1
2
A
B
C
D
E
1
2
a)
b)
4
trùng giao điểm
O
của hai đường chéo
AC
BD
.
Hình 1.4
C. BÀI TP VN DNG
Tính s đo góc
1.1 Chng minh rng trong mt t giác, tng hai góc ngoài tại hai đỉnh bng tng hai góc trong tai hai
đỉnh còn li.
1.2. Cho t giác
ABCD
220AB
. Các tia phân giác ngoài tại đỉnh
C
D
ct nhau ti
K
. Tính
s đo của góc
CKD
.
1.3. Cho t giác
ABCD
AC
. Chng minh rng các đường phân giác ngoài ca góc
B
D
song
song ho trùng vi nhu.
1.4. Cho t giác
ABCD
;AD DC CB
130 ;C 
110D 
. Tính s đo góc góc
A
, góc
B
(Olympic Toán Châu Á Thái Bình Dương 2010).
So sánh các độ dài
1.5. Có hay không mt t giác mà độ dài các cnh t l vi
1,3,5,10?
1.6. T giác
ABCD
có hai đường chéo vuông góc. Biết
3;AB
6,6BC
;
6.CD
Tính đọ dài
AD
.
1.7. Chng minh rng trong mt t giác tổng hai đường chéo lớn hơn nửa chu vi nhưng nhỏ hơn chu vi tứ
giác.
1.8 Cho bốn điểm
, , ,A B C D
trong đó không có ba điểm nào thng hàng, bất kì hai điểm nào cũng có
khong cách lớn hơn 10. Chứng minh rng tn tại hai điểm đã cho có khoảng cách lớn hơn 14.
1.9. Cho t giác có độ dài các cnh là
, , ,a b c d
đều là các s t nhiên. Biết tng
S a b c d
chia hết cho
a,
cho
,b
cho
c
, cho
d
. Chng minh rng tn ti hai cnh ca t giác
bng nhau.
Bài toán gii bằng phƣơng trình tô màu
1.10. Có chín người trong đó bất kì ba người nào cũng có hai người quen nhau. Chng minh rng tn
ti mt nhóm bốn người quen nhau.
A
B
C
D
1
2
A
B
C
D
E
1
2
a)
b)
A
B
M
C
D
O
ABCD
5
CHUYÊN ĐỀ 2. HÌNH THANG. HÌNH THANG CÂN. DNG HÌNH THANG
A. Kiến thc cn nh
1. Hình thang là t giác có hai cạnh đối song song (h.2.1).
Đặc bit: hình thang vuông là hình thang có mt góc vuông (h.2.2).
Hình 2.1 Hình 2.2
2. Hình thang cân là hình thang có hai góc k một đáy bằng nhau (h.2.3).
3. Trong hình thang cân:
- Hai cnh bên bng nhau
- Hai đường chéo bng nhau (h.2.4).
Hình 2.3 Hình 2.4
4. Du hiu nhn biết hình thang cân:
- Hình thang có hai góc k một đáy bằng nhau là hình thang cân.
- Hình thang có hai góc đối bù nhau là hình thang cân.
- Hình thang có hai đường chéo bng nhau là hình thang cân.
5. Dng hình
Dng c dựng hình: thước và compa
Các bước gii mt bài toán dng hình
- Phân tích;
- Cách dng;
- Chng minh;
- Bin lun.
Đối vi mt bài toán dựng hình đơn giản ta có th không trình bày bước phân tích.
TRANG 7-8
B. Mt s ví d
A
B
C
D
1
2
A
B
C
D
E
1
2
a)
b)
A
B
M
C
D
O
A
B
C
D
A
B
C
D
A
B
C
D
1
2
A
B
C
D
E
1
2
a)
b)
A
B
M
C
D
O
A B
C
D
A
B
C
D
A
B
C
D
1
2
A
B
C
D
E
1
2
a)
b)
A
B
M
C
D
O
A
B
C
D
A
B
C
D
A
B
C
D
1
2
A
B
C
D
E
1
2
a)
b)
A
B
M
C
D
O
A
B
C
D
A
B
C
D
Để dng hình thang ta cn biết bn yếu t của nó, trong đó số đo góc cho trước không quá hai.
6
Ví d 1. Cho hình thang
( / / ),ABCD AB CD
các tai phân giác ca góc
,A
góc
D
ct nhau ti
M
thuc
cnh
.BC
Cho biết
7,AD cm
Chng minh rng một trong hai đấy của hình thang có độ dài nh hơn
4.cm
Gii(h.2.5)
*Tìm cách gii
Để chng minh mt cạnh đáy nào đó nhỏ hơn
4cm
ta có th xét tng ca hai cạnh đáy rồi chng minh
tng này nh hơn
8cm
, khi đó tồn ti một đáy nhỏ hơn
4.cm
*Trình bày li gii
Gi
N
là giao điểm ca tia
AM
và tia
.DC
Ta có :
2
//AB CD A N
(so le trong)
Mt khác,
1 2 1
A A A N DAN
cân ti D
DA DN
(1)
Xét
12
DD
nên
DM
đồng thời là đường trung
tuyến:
MA MN
( . . ) .ABM NCM c g c AB CN
Ta có:
7.DC AB DC CN DN DA cm
Vy
8.AB CD cm
Vy một trong hai đáy
,AB CD
phải có độ dài nh hơn
4cm
Ví d 2. T giác
ABCD
,.AC BD AD BC
Chng minh rng t giác này là hình thang cân.
Gii(h.2.6)
*Tìm cách gii
T giác
ABCD
có hai đường chéo bằng nhau nên để chng
minh nó là hính tháng cân, ch cn chng minh
/ / .AB CD
Mun vy ta chng minh mt cp góc so le trong bng nhau.
*Trình bày li gii
11
11
( . . )
( . . )
ADC BCD c c c C D
DAB CBA c c c B A
Mt khác:
1 1 1 1
2 2 / /COD AOB C A C A AB CD
Vy t giác
ABCD
là hình thang. Hình thang này có hai đường chéo bng nhau nên là hình thang cân.
Ví d 3. Một hình thang cân có đáy nhỏ bng cnh bên và góc k với đáy lớn bng
60
Biết chiu cao ca hình thang cân này là
3.a
Tính chu vi ca hình thang cân.
2
1
2
1
Hình 2.5
B
C
D
A
N
M
1
1
1
2
Hình 2.6
O
A
B
C
D
7
Gii(h.2.7)
*Tìm cách gii
Ta đã biết hình thang có hai cnh bên song song thì hai cnh bên bng nhau, hai cạnh đáy bằng nhau. T
đó vẽ thêm hình ph để tìm s liên h giữa đáy lớn và ba cnh còn li. Ta v
/ / ( ).AM BC M CD
Mt
khác, đề bài có cho góc
60
, gi ý cho ta vn dng tính cht của tam giác đều để tính độ dài mt cnh
theo chiu cao ca nó.
*Trình bày li gii
Ta đặt:
AD AB BC x
V
/ / ( ),AM BC M CD
ta được
,AM BC x MC AB x
V
AH CD
thì
AH
là đường cao ca hình thang cân,
cũng là đường cao của tam giác đều:
3
2
AD
AH
. Vì
3AH a
nên
3
3 2 .
2
x
a x a
Do đó chu vi của hình thang cân là:
2 .5 10 .aa
Nhn xét: Qua một đỉnh v đường thng song song vi mt cnh ben ca hình thang là mt cách v hình
ph để gii bài toán v hình thang.
Ví d 4. Dng hình thang
( / / )ABCD AB CD
biết:
2 , 5 , 40 , 70 .AB cm CD cm C D
Gii(h.2.8)
a)Phân tích
Gi s ta đã dựng được thang
( / / )ABCD AB CD
thỏa mãn đề bài. V
/ / ( )AE BC E CD
ta được
40 ; 2 ;
5 2 3
AED C EC AB cm
DE DC EC cm
ADE
dựng được ngay (g.c.g)
Đim
C
thỏa mãn điều kiên:
C
nm trên tia
DE
C
cách
D
5cm
.
Đim
B
thỏa mãn điều kiên:
B
nm trên tia
//Ax DE
( hai tia
cùng nm trên mt na mt
phng b AD) và
B
cách
A
2cm
b)Cách dng
Dng
ADE
sao cho
3 ; 70 ; 40 .DE cm D E
Dng tia
//Ax DE
( hai tia
cùng nm trên
mt na mt phng b AD). Trên tia
Ax
đặt
3AB cm
. Trên tia
DE
đặt
5DC cm
Hình 2.7
A
B
C
D
H
M
x
Hình 2.8
40
°
70
°
A
D
C
B
E
8
Ni
BC
ta được hình thang
ABCD
phi dng.
c) Chng minh
Theo cách dng t giác
ABCD
//AB CD
nên nó là hình thang.
Xét hình thang
ABCE
5 3 2CE
(cm);
2AB
cm nên
AB CE
do đó
//AE BC
40BCD AED
.
Như vậy hình thang
ABCD
2AB
cm;
5CD
cm;
70D
40C
d) Bin lun
Bài toán có mt nghim hình.
Ví d 5. Dng tam giác
ABC
, biết
70A
,
5BC
cm và
2AC AB
cm.
Gii (h.2.9)
a) Phân tích
Gi s ta đã dựng được tam giác
ABC
tho mãn đề bài.
Trên tia
AC
ta lấy điểm
D
sao cho
AD AB
.
Khi đó
2DC AC AD AC AB
cm.
ABD
cân,
70 35 125A ADB BDC
.
-
DBC
xác định được (
2CD
cm;
125D
;
5CB
cm).
- Đim
A
tho mãn hai điều kin:
A
nm trên tia
CD
A
nằm trên đường trung trc ca
BD
.
b) Cách dng
- Dng
DBC
sao cho
125D
;
2DC
cm và
5CB
cm.
- Dựng đường trung trc ca
BD
ct tia
CD
ti
A
.
- Ni
AB
ta được
ABC
phi dng.
c) Chng minh
ABC
tho mãn đề bài vì theo cách dựng, điểm
A
nằm trên đường trung trc ca
BD
nên
AD AB
.
Do đó
2AC AB AC AD DC
cm;
5BC
cm và
180 125 55ADB
125 2.55 70BAC
.
Hình 2.9
9
d) Bin lun
Bài toán có mt nghim hình.
Nhn xét: Đề bài có cho đoạn thẳng 2cm nhưng trên hình vẽ chưa có đoạn thẳng nào như vậy. Ta đã làm
xut hiện đoạn thng
2DC
cm bng cách trên
AC
ta đặt
AD AB
. Khi đó
DC
chính là hiu
AC AB
.
Cũng có thể làm xut hiện đoạn thng 2cm bng cách trên tia
AB
ta đặt
AE AC
(h.2.10).
Khi đó
2BE AE AB AC AB
cm.
AEC
cân, có
70 (180 70 ):2 55AE
.
BEC
xác định được.
Khi đó điểm
A
tho mãn hai điều kin:
A
nm trên tia
EB
A
nằm trên đường trung trc ca
EC
.
C. Bài tp vn dng
nh thang
2.1. Cho t giác
ABCD
. Các tia phân giác ca góc
A
, góc
D
ct nhau ti
M
. Các tia phân giác ca góc
B
, góc
C
ct nhau ti
N
. Cho biết
90AMD
, chng minh rng:
a) T giác
ABCD
là hình thang;
b)
NB NC
.
2.2. Cho hình thang
ABCD
vuông ti
A
D
. Gi
M
là trung điểm ca
AD
. Cho biết
MB MC
.
a) Chng minh rng
BC AB CD
;
b) V
MH BC
. Chng minh rng t giác
là hình thang.
2.3. Chng minh rng trong mt hình thang vuông, hiệu các bình phương của hai đường chéo bng hiu
các bình phương của hai đáy.
2.4. Cho hình thang
ABCD
vuông ti
A
D
. Cho biết
20AD
,
52AC
29BC
. Tính độ dài
AB
.
nh thang cân
2.5. Cho tam giác đều
ABC
, mi cạnh độ dài bng
a
. Gi
O
một điểm bt trong tam giác.
Trên các cnh
,,AB BC CA
lần lượt lấy các điểm
, , M N P
sao cho
//OM BC
;
//ON CA
//OP AB
.
Xác định v trí của điểm
O
để tam giác
MNP
là tam giác đều. Tính chu vi của tam giác đều đó.
2.6. Cho hình thang
ABCD
(
//AB CD
),
ADC BCD
. Chng minh rng
AC BD
.
Hình 2.10
10
2.7. Cho góc
xOy
s đo lớn hơn
60
nhưng nhỏ hơn
180
. Trên cnh
Ox
lấy điểm
A
, trên cnh
Oy
lấy điểm
C
. Chng minh rng
2
OA OC
AC
.
2.8. T giác
ABCD
AC BD
;
CD
BD BC
. Hi t giác
ABCD
phi hình thang cân
không?
Dng hình
2.9. Dng hình thang
ABCD
(
//AB CD
) biết
2AD
cm;
3BD
cm;
4AC
cm và góc nhn xen gia
hai đường chéo bng
70
.
2.10. Dng hình thang
ABCD
(
//AB CD
) biết
120A
;
2AB
cm,
4BD
cm và
BC a
.
2.11. Dng t giác
ABCD
biết
2,5AB
cm;
4CD
cm;
120A
;
100B
60C
.
2.12. Dng tam giác
ABC
vuông ti
B
có chu vi bng 8cm và
.
11
CHUYÊN ĐỀ 3. ĐƢNG TRUNG BÌNH CA TAM GIÁC, CA HÌNH THANG
A. KIN THC CN NH
1. Đĩnh nghĩa
Đưng trung bình của tam giác là đoạn thng nối trung điểm hai cnh ca tam giác (h3.1)
Đưng trung bình của hình thang là đoạn thng nối trung điểm hai cnh bên ca hình thang (h3.2)
(hình 3.1) (hình 3.2)
2. Tính cht
Đưng trung bình ca tam giác thì song song vi cnh th ba và bng na cnh y.
Trên hình 3.1 thì
//MN BC
2
BC
MN
.
Đưng trung bình ca hình thang thì song song vi hai cạnh đáy và bằng na tổng hai đấy
Trên hình 3.2 thì
// //AB EF CD
2
AB CD
EF
.
3. Định lý
Đưng thẳng đi qua trung điểm mt cnh ca tam giác và song song vi cnh th hai thì đi qua trung
điểm ca cnh th ba.
Đưng thẳng đi qua trung điểm ca mt cnh bên hình thang và song song với hai đáy thì đi qua
trung điểm ca cnh bên th hai.
B. MT S VÍ D
Ví d 1: Cho t giác
ABCD
. Gi
M
N
lần lượt là trung điểm ca
AB
CD
. Gi
G
là trng tâm
ca tam giác
BCD
. Chng minh
AG
chia đôi
MN
.
Gii (hình 3.3)
Tìm cách gii
12
Kết lun ca bài toán gợi ý cho ta dùng định lý đường thẳng đi qua
trung điểm mt cnh ca tam giác và song song vi cnh th hai thì đi
qua trung điểm ca cnh th ba. Gi
H
là trung điểm ca
BG
thì ta
có th dùng định lý đường trung bình để chng minh.
Trình bày li gii
Gi
O
là giao điểm ca
AG
MN
Gi
H
là trung điểm ca
BG
Theo tính cht ca trng tâm, ta có:
BH HG GN
Xét
ABG
có MH là đường trung bình
//MH AG
(Hình 3.3)
Xét
HMN
//AG MH
NG GH
nên
ON OM
Vy
AG
chia đôi
MN
Nhn xét: V thêm trung điểm ca một đoạn thng là cách v hình ph thường dùng để vn dụng định lý
đường trung bình ca tam giác.
Ví d 2: Cho t giác
ABCD
có ch vi
4a
. Gi
, , ,E F G H
lần lượt là trung đim ca các cnh
,AB BC
,,CD DA
. Chng minh rằng trong hai đoạn thng
EG
HF
một đoạn thẳng độ dài không ln
hơn
a
.
Gii (hình 3.4)
Tìm cách gii
Để chng minh một trong hai đoạn thng
EG
HF
có một đoạn thẳng có độ dài không lớn hơn
a
ta
chng minh tổng hai đoạn thng này không lớn hơn
2a
. Khi đó một trong hai đoạn thẳng có độ dài
không lớn hơn
a
.
Trình bày li gii
Gi
M
là trung điểm ca
BD
Xét
ABD
HM
là đường trung bình nên
2
AB
HM
Xét
BDC
MF
là đường trung bình nên
2
CD
MF
Xét ba điểm
M
,
H
,
F
2
AB CD
HF MH MF
Chứng minh tương tự, ta được:
2
AD BC
EG
.
Vy
4
2
22
AB CD AB CD a
HF EG a
(Hình 3.4)
Suy ra một trong hai đoạn
,HF EG
có độ dài không lớn hơn
a
.
13
Nhn xét: Phương pháp vẽ hình ph trong ví d này vn là v trung điểm của đoạn
BD
. Cũng có thể v
trung điểm ca cnh
AC
thay cho trung điểm của đoạn thng
BD
.
Ví d 3: Cho tam giác
ABC
,
6BC cm
. Trên cnh
AB
lấy điểm
D
sao cho
1
3
AD AB
. V
//DE BC E AC
. Tính độ dài
DE
Gii (hình 3.5)
Tìm cách gii
1
2
AD DB
nên ta v trung điểm
F
ca
DB
. T
F
v
đường thng song song vi
BC
thì
DE
chính là đường trung
bình ca tam giác. T đó sẽ tính được độ dài ca nó.
Trình bày li gii
Gi
F
là trung điểm ca
DB
. Khi đó:
AD DF FB
V
//FH BC H AC
Xét
AFH
//DE FH
AD DF
nên
AE EH
Xét hình thang DECB có
//FH BC
DF FB
nên
EH HC
Ta đặt
DE x
(Hình 3.5)
Ta có
DE
là đường trung bình ca
AFH
1
2
DF FH
2FH x
Ta có
FH
là đường trung bình ca hình thang
DECB
6
22
22
DE BC x
FH x x cm
Vy
2DE cm
.
Nhn xét: Phương pháp vẽ hình ph trong ví d này là ngoài vic v trung điểm ca một đoạn thng ta
còn thêm một đường thng song song vi mt cnh ca tam giác.
Ví d 4: Cho hình thang
ABCD
,
AB
là đáy nhỏ. Gi
, , ,M N P Q
lần lượt là trung điểm ca
,AD BC
,
BD
AC
.
a) Chn minh rng bốn điểm
, , ,M N P Q
thng hàng.
b) Chng minh
//PQ CD
2
CD AB
PQ
c) Hình thang
ABCD
phải có điều kiện gì để
MP PQ QN
Gii (hình 3.6)
Tìm cách gii
14
Trong hình v có nhiu đưng thẳng cùng đi qua một điểm và cùng
song song vi một đường thng nên có th vn dụng tiên đề Ơ clit
để chng minh thng hàng.
Trình bày li gii
a) Xét
ABD
MP
là đường trung bình
/ / / /MP AB MP CD
Xét
ADC
MQ
là đường trung bình
//MQ CD
Xét hình thang
ABCD
MN
là đường trung bình
//MN CD
(Hình 3.6)
Qua điểm
M
có các đường thng
,,MP MQ MN
cùng song song vi
CD
nên các đường thng trùng
nhau, suy ra bốn điểm
, , ,M N P Q
thng hàng.
b) Ta có
//MN CD
nên
//PQ CD
;
2 2 2
CD AB CD AB
PQ MQ MP
c) Ta có
;
2 2 2
AB AB CD AB
MP NQ MP PQ
2AB CD AB AB CD
(đáy lớn gấp đôi đáy nhỏ)
Nhn xét: Đưng trung bình
MN
của hình thang và đoạn thng
PQ
nối trung điểm của hai đường chéo
có tính cht ging nhau là cùng song song với hai đáy, có tính chất khác nhau là
MN
bng na tng hai
đáy còn
PQ
bng na hiệu hai đáy.
C. BÀI TP VN DNG
Đƣng trung bình ca tam giác
3.1. Cho t giác
ABCD
, đường chéo
BD
là đường trung trc ca
AC
. Gi
M
N
lần lượt là trung
điểm ca
AD
AB
. V
ME BC
,NF CD E BC F CD
. Chng minh rằng ba đường thng
,ME NF
AC
đồng quy.
3.2. Cho tam giác
ABC
. Trên cnh
AB
lấy điểm
D
, trên cnh
AC
lấy điểm
E
. Gi
,MN
lần lượt là
trung điểm ca
BE
CD
. Đường thng
MN
ct tia
AB
AC
lần lượt ti
P
Q
. Hoi hai điểm
D
E
phải có điểm kiện gì để tam giác
APQ
cân ti
A
?
3.3. Cho tam giác
ABC
. Gi
Bx
Cy
lần lượt là các đường thng cha tia phân giác ca các góc ngoài
tại đỉnh
B
C
. Gi
H
K
lần lượt là hình chiếu vuông góc ca
A
lên
Bx
Cy
.
a) Chng minh rng t giác
BCKH
là hình thang
b) Tam giác
ABC
cần điều kiện gì để hình thang
BCKH
là hình thang cân?
3.4. Cho tam giác
ABC
, trc tâm
H
. Gi
O
là giao điểm của ba đường trung trc. Chng minh rng
khong cách t
O
ti
BC
bng nửa độ dài
AH
.
15
3.5. Cho tam giác
ABC
cân ti
A
, đường cao
AH
và đường phân giác
BD
. Biết rng
1
2
AH BD
. Tính
s đo các góc của tam giác
ABC
.
3.6. Cho tam giác
ABC
cân ti
A
. Lấy điểm
D
trong tam giác. V tam giác
ADE
vuông cân ti
A
sao
cho
D
E
thuc hai na mt phẳng đối nhau b
AC
. Gi
M
,
N
,
P
lần lượt là trung điểm ca
BC
,
CD
DE
. Tính s đo các góc của tam giác
MNP
.
3.7. Cho hình thang cân
ABCD
//AB CD
,
O
là giao điểm của hai đường chéo. Gi
G
,
E
,
F
lần lượt
là trung điểm ca
OA
,
OD
BC
. Cho biết
0
60COD
. Tính s đo các góc của tam giác
GEF
.
3.8. Cho tam giác
,ABC
góc
A
nhn. V v phía ngoài ca tam giác này các tam giác vuông cân
ABM
CAN
theo th t có cạnh đáy là
AB
.AC
Gi
O
là trung điểm ca
.BC
Chng minh rng tam
giác
OMN
là tam giác vuông cân.
3.9. Tam giác
,.ABC AB AC
Trên cnh
AB
lấy điểm
,E
trên cnh
AC
lấy điểm
F
sao cho
.BE CF
Gi
M
là trung điểm ca
.EF
Chng minh rng khi
E
F
di động trên
,AB AC
thì trung
điểm
M
ca
EF
nm trên một đường thng c định.
3.10. Cho đoạn thng
AB
n
điểm
12
,,
n
O O O
không nm gia
A
B
sao cho
1 2 1 2
.
nn
O A O A O A O B O B O B a
Chng minh rng tn ti một điểm
M
sao cho
12
.
n
O M O M O M a
3.11. Cho tam giác
.
ˆ
ˆ
,
ˆ
ABC C B A
Biết rằng trung điểm của ba đường cao thng hàng. Chng minh
rng tam giác
ABC
vuông ti
.A
Đƣng trung bình ca hình thang
3.12. Cho hình thang cân
.A
BCD AB CD
V
.AH CD
Chng minh rng:
a)
HD
bằng đoạn thng nối trung điểm của hai đường chéo.
b)
HC
bằng đường trung bình ca hình thang.
3.13. Cho tam giác
,ABC
gi
M
là trung điểm ca
.AB
Trên tia đối ca tia
BC
lấy điểm
O
sao cho
1
.
2
BO BC
Đưng thng
OM
ct
OC
ti
.N
Chng minh rng
1
.
4
AN AC
3.14. Cho tam giác
,ABC
cnh
BC
c định. V ra ngoài tam giác này các tam giác
ABM
vuông cân ti
,B
tam giác
CAN
vuông cân ti
.C
Chng minh rng khi
A
di động trên mt na mt phng b
BC
thì
đường thng
MN
luôn đi qua một điểm c định.
3.15. Cho điểm
M
nm giữa hai điểm
,AB
nhưng không là trung điểm của đoạn
.AB
Trên cùng mt
na mt phng b
AB
v các tam giác
CAM
DBM
cân ti
C
D
sao cho
ˆ
ˆ
.CD
Gi
H
F
lần lượt là trung điểm ca
AD
.BC
Chng minh rng
1
.
2
HF CD
16
3.16. Chng minh rng trong các tam giác có mt góc bng nhau, xen gia hai cnh có tng bng nhau
thì tam giác cân có chu vi nh nht.
17
CHUYÊN ĐỀ 4. HÌNH BÌNH HÀNH
A. Kiến thc cn nh
1. Định nghĩa
Hình bình hành là t giác có các cạnh đối song song (h. 4.1)
2. Tính cht
Trong hình bình hành (h. 4.2):
Các cạnh đối bng nhau
Các góc đối bng nhau
Hai đường chéo ct nhau tại trung điểm ca mỗi đường.
3. Du hiu nhn biết
T giác có các cạnh đối song song là hình bình hành
T giác có các cạnh đối bng nhau là hình bình hành
T giác có hai cạnh đối song song và bng nhau là hình bình hành
T giác có các góc đối bng nhau là hình bình hành
T giác có hai đường chéo ct nhau tại trung điểm ca mỗi đường là hình bình hành.
B. Mt s ví d
Ví d 1. Cho hình bình hành
.ABCD
Trên tia đối ca tia
AD
lấy điểm
,M
trên tia đối ca tia
CB
ly
điểm
N
sao cho
.AM CN
Chng minh rằng ba đường thng
,,MN AC BD
gp nhau ti một điểm.
Gii (h. 4.3)
18
Tìm cách gii
AC
BD
là hai đường chéo ca hình bình hành
ABCD
nên chúng ct nhau tại trung điểm
O
ca
.AC
Trình bày li gii
T giác
AM CN
AM CN
nên là hình bình hành. Suy ra hai đường chéo
MN
AC
ct nhau tại trung điểm
O
ca
.AC
Mt khác,
ABCD
là hình bình hành nên hai đường chéo
BD
AC
ct nhau tại trung điểm
O
ca
.AC
Như vậy, các đường thng
,MD BD
AC
cùng đi qua trung điểm
O
ca
.AC
Nhn xét: Hai hình bình hành
ABCD
có chung đường chéo
AC
thì các đường chéo ca
chúng đồng quy tại trung điểm của đường chéo chung.
Ví d 2. Cho hình bình hành
v ra phía ngoài của hình bình hành các tam giác đều
ABM
.ADN
Chng minh rng tam giác
CMD
là tam giác đều.
Gii (h.4.4)
Tìm cách gii
19
Đề bài cho hình bình hành và các tam giác đều nên có nhiều đoạn thng hàng nhau, nhiu góc bng nhau.
Do đó có thể nghĩ đến vic chng minh tam giác bng nhau.
Trình bày li gii
Ta đặt
ABC
thì
,
180 , 360 60 60 180 60 .
o o o o o o
ADC BAD MAN
ΔMAN
ΔCDN
A
M DC AB
;
6
0 ; .
o
MAN CDN AN DN
Do đó
ΔΔMAN CDN
(c-g-c)
1M
N CN
Chứng minh tương tự, ta được
ΔΔMAN MBC
(c-g-c)
2M
N MC
T (1) và (2) suy ra
.MN CN MC
Vy
đều.
Nhn xét: Việc đặt
ABC
là mt k thuật giúp ta tính toán và so sánh góc được nhanh chóng, thun
tin.
Ví d 3. Chng minh rng nếu một tam giác có hai đường trung tuyến vuông góc vi nhau thì tng các
bình phương của hai đường trung tuyến này bằng bình phương của đường trung tuyến th ba.
Gii (h. 4.5)
Tìm cách gii
Kết lun ca bài toán gi ý cho ta vn dụng định lý Py-ta-go. Mun vy phi v đường ph to ra mt
tam giác vuông có ba cnh bằng ba đường trung tuyến.
Trình bày li gii
Gi s tam giác
ABC
là tam giác có hai đường trung tuyến
,BD CE
vuông góc vi nhau, ta phi chng
minh
2 2 2
BD CE AF
(
AF
là đường trung tuyến th ba).
Trên tia
ED
lấy điểm
K
sao cho
D
là trung điểm ca
.EK
T giác
AKCE
có hai đường chéo ct nhau
tại trung điểm ca mỗi đường nên là hình bình hành.
AK CE
.AK CE
20
Ta có
DE BC
1
2
DE BC DK BF
.DK BF
Vy t giác
DKFB
là hình bình hành
KF BD
.KF BD
Mt khác
BD CE
nên
.AK KF
Do đó
ΔKAF
vuông gi
2 2 2 2 2 2
.A AK KF AF CE BD AF
C. Bài tp vn dng
Tính cht hình bình hành
4.1. Cho tam giác
ABC
nhn. V ra phía ngoài tam giác này các tam giác
,ABD
và tam giác
ACE
vuông cân ti
.A
Gi
M
là trung điểm
.DE
Chng minh rằng hai đường thng
,MA BC
vuông góc vi
nhau.
4.2. Cho hình bình hành
.ABCD
V ra ngoài hình bình hành các tam giác
ABM
vuông cân ti
,A BCN
vuông cân ti
.C
Chng minh rng tam giác
DMN
vuông cân.
4.3. Cho tam giác nhn
ABC
có trc tâm
.H
Chng minh rng chu vi ca tam giác
ABC
lớn hơn
3
.
2
HA HB HC
4.4. Cho hình thang cân
A
BCD AB CD
và một điểm
O
trong hình này. Chng minh rng có mt t
giác mà bn cnh lần lượt bng
, , ,OA OB OC OD
và bốn đỉnh nm trên bn cnh ca hình thang cân.
4.5. Cho hình bình hành
ABCD
và đường thng
xy
không ct các cnh ca hình bình hành. Qua các
đỉnh
, , ,A B C D
v các đường thng vuông góc vi
,xy
ct
xy
lần lượt ti
, , , .A B C D
Chng minh
rng
.AA CC BB DD
4.6. Cho hình bình hành
.A
BCD AD AB
V ra ngoài hình bình hành các tam giác
ABM
cân ti
B
và tam giác
ADN
cân ti
D
sao cho
.ABM ADN
a) Chng minh rng
;CM CN
b) Trên
AC
ly một điểm
.O
Hãy so sánh
,.OM ON
4.7. Cho tam giác
ABC
cân ti
,.A AB AC
Trên tia
AB
có điểm
,D
trên tia
CA
có điểm
E
sao cho
.AD DE EC CB
Tính các góc ca tam giác
.ABC
Nhn biết hình bình hành.
4.8. Chng minh rng trong mt t giác, đoạn thng nối trung điểm hai đường chéo và các đoạn thng
ni trung điểm ca hai cp cạnh đối din gp nhau ti một điểm (định lý Giéc-gôn, nhà toán hc Pháp).
4.9. Cho t giác
.ABCD
Gi
,MN
lần lượt là trung điểm ca
AB
.CD
Gi
, , ,E F G H
lần lượt là
trung điểm ca
, , , .NA NB MC MD
Chng minh rằng ba đường thng
,,MN EF GH
đồng quy.
21
4.10. Cho đoạn thng
PQ
và một điểm
A
ngoài đường thng
.PQ
V hình bình hành
ABCD
đường chéo
BD PQ
.BD PQ
Chng minh rng mỗi đường thng
BC
CD
luôn đi qua một
điểm c định.
4.11. Trong tt c các t giác với hai đường chéo độ dài m và n cho trước và góc xen gia hai đường
chéo có độ ln
cho trước hãy xác định t giác có chu vi nh nht
Dựng hình bình hành
4.12. Cho tam giác
ABC
. Dng điểm
M AB
, điểm
N AC
sao cho
//MN BC
=BM AN
4.13. Dng hình bình hành
ABCD
biết v trí ca điểm
A
v trí các trung điểm
M
,
N
ca
BC
và
CD
.
4.14. Cho trước hai điểm
A
B
thuc hai na mt phng đối nhau b đường thng
d
. Một đoạn
thng
CD
độ dài
a
cho trước nm trên đường thng
d
. y xác định v trí của điểm
C
D
để
tng
AC CD DB
nh nht
4.15. Hai điểm dân
A
B
hai bên mt con sông hai b
d
'd
. Chiu rng con sông bng
a
. Hãy tìm địa điểm bc cu sao cho quãng đường t
A
sang
B
ngn nht (cu vuông góc vi b
sông).
22
CHUYÊN ĐỀ 5. HÌNH CH NHT
A. Kiến thc cn nh
1. Định nghĩa
Hình ch nht là t giác có bn góc vuông (h.5.1)
2. Tính cht
Trong hình ch nhật, hai đưng chéo bng nhau và ct nhau tại trung điểm mỗi đường
(h.5.2).
3. Du hiu nhn biết
- T giác có ba góc vuông là hình ch nht
- Hình thang cân có mt góc vuông là hình ch nht
- Hình bình hành có mt góc vuông là hình ch nht
- Hình bình hành có hai đường chéo bng nhau là hình ch nht.
4. Áp dng vào tam giác (h.5.3)
ABC
:
=MB MC
1
90
2
A AM BC
5. Tính chất các điểm cách đều một đường thẳng cho trước (h.5.4)
Tp hợp các điểm cách mt đường thng c
định mt khong bng h không đổi là hai đường
thng song song vi đường thng đó và cách
đường thẳng đó một khong bng h.
B. Mt s ví d
Hình 5.3
Hình 5.4
23
Hình 5.5
d 1. Cho hình ch nht
ABCD
. Trên đường chéo
BD
ly mt điểm
M
. Trên tia
AM
ly điểm
N
sao cho
M
trung đim ca
AN
. Gi
E
F
lần lượt hình chiếu ca
N
trên đường thng
BC
CD
. Chng minh rằng ba điểm
M
,
E
,
F
thng hàng.
Gii (h.5.5)
* Tìm cách gii
Xét
CAN
, đường thng
EF
đi qua trung đim ca
CN
, mun cho
EF
đi qua trung đim
M
ca
AN
ta cn chng minh
//EF AC
.
* Trình bày li gii
T giác
ENFC
có ba góc vuông nên là
hình ch nht.
Gi
O
là giao điểm ca
AC
BD
K
giao điểm ca
EF
CN
.Theo tính cht
hình ch nht,
ta có:
OA OB OC OD
;
KC KN KE FF
.
Xét
CAN
OM
đường trung
bình nên
//OM CN
.
Do đó
//BD CN
.
OCD
,
KCF
cân, suy ra:
11
DC
,
22
CF
Mt khác,
12
DC
(cp góc đồng v)
12
CF
Suy ra
//AC EF
.
Xét
CAN
đường thng
EF
đi qua trung điểm
K
ca
CN
//EF AC
nên
EF
đi qua trung đim
ca
AN
, tc là đi qua
M
. Vậy ba điểm
M
,
E
,
F
thng hàng.
d 2. Cho tam giác
ABC
cân ti
A
. T một điểm trên đáy
BC
, v đường thng vuông góc vi
BC
ct c đường thng
AC
,
AB
lần lượt ti
M
N
. Gi
H
K
lần lượt trung đim ca
BC
MN
.
Chng minh rng t giác
là hình ch nht.
Gii (h.5.6)
* Tìm cách gii
D thy t giác
hai góc vuông
90HD
nên ch cn chng minh t giác này mt
góc vuông na là thành hình ch nht.
24
A. Trình bày li gii
BKC
cân ti
A
,
AH
đường trung tuyến nên cũng là
đường cao, đường phân giác.
Do dó:
90H 
12
AA
Ta có:
//AH DN
(vì cùng vuông góc vi
BC
)
1
NA
(cp góc đồng v);
12
MA
(cp góc so le
trong).
Do dó
1
NM
(vì
12
AA
)
Vy
AMN
cân ti
A
AK
đường trung
tuyến nên
AK
cũng là đường cao,
90K 
. T
giác
90K H D
nên nó là hình ch nht.
Ví d 3. Cho tam giác
ABC
vuông cân ti
A
. Trên cnh huyn
BC
ly điểm
D
. V
DH AB
,
DK AC
. Biết
AB a
, tính giá tr ln nht ca tích
.DH CK
.
Gii (h.5.7)
* Tìm cách gii
Ta thy
DH DK AB
(không đổi). Da vào các hằng đẳng thc ta có th tìm được mi quan h gia
tích
.DH CK
vi tng
DH DK
. Mi quan h này được biếu diễn như sau:
Ta có:
2 2 2 2 2 2 2
( ) 0 2 2 4 ( ) 4x y x y xy x y xy xy x y xy
* Trình bày li gii
T giác
có ba góc vuông nên là hình ch nht.
Tam giác
HBD
90H 
;
45B 
nên tam giác vuông cân. Ta đặt:
DH x
.
DK y
thì
HB x
,
AH y
x y a
Ta có:
22
()
44
x y a
xy

(không đổi).
2
()
4
xy
xy

Hình 5.6
Hình 5.7
25
Du "=" xy ra
xy
D
là trung điểm ca
B
Vy giá tr ln nht ca tích
.DH CK
2
4
a
khi
D
là trung điểm ca
BC
.
d 4. Cho hình thang
ABCD
,
90AD
.Trên cnh
AD
một điểm
H
AH DH
90BHC 
. Chng minh rng trên cnh
AD
còn một điểm
K
sao cho
90BKC 
.
Gii (h.5.8)
* Tìm cách gii
Gi s đã chứng minh được
90BKC 
thì
BHC
BKC
hai tam giác vuông chung cnh
huyn
BC
nên hai đường trung tuyến ng vi
BC
phi bằng nhau. Do đó cần chứng minh hai đường
trung tuyến này bng nhau.
* Trình bày li gii
Gi
M
N
lần lượt trung đim ca
AD
BC
. Khi đó
MN
đường trung bình ca hình thang
ABCD
, suy ra:
//MN AB
MN AD
(vì
AB AD
)
Trên cnh
AD
lấy điểm
K
sao cho
DK AH MK MH
.
NHK
MN vừa đường cao, vừa đường trung tuyến nên tam giác
cân
KN HN
Xét
HBC
vuông ti
H
1
2
HN BC
( tính chất đường trung tuyến ng
vi cnh huyn).
Suy ra
1
2
KN BC
( vì
KN HN
)
Do đó
KBC
vuông ti
K
90BKC
Ví d 5. Cho đường thng
xy
. Một điểm
A
c định nm ngoài
xy
và mt điểm
B
di động trên
xy
Gi
O
là trung điểm ca
AB
. Hi đim
O
di động trên đường nào?
Gii (h.5.9)
V
AH xy
,
OK xy
.
Hình 5.8
Hình 5.9
26
Ta có:
AH
là mt đon thng c định. Xét
ABH
//OK AH
OA OB
nên
KH KB
.
Vy
OK
đường trung bình suy ra:
1
2
OK AH
(không di).
Đim
O
cách đường thng
xy
cho trước mt khong
không đổi là
1
2
AH
nên điểm
O
di động trên đường thng
//a xy
và cách
xy
2
AH
(đường thng
a
điểm
A
cùng nm trên mt na mt phng b
xy
).
C. BÀI TP VN DNG
Tính cht và du hiu nhn biết ca hình ch nht
5.1. Cho tam giác
ABC
vuông cân ti
A
, đường cao
AD
. Gi
M
là một điểm bt kì trên cnh
BC
. V
ME AB
,
MF AC
. Tính s đo các góc của tam giác
DEF
.
5.2. Cho hình bình hành
ABCD
. Biết
1
2
AD AC
1
2
BAC DAC
. Chng minh rng hình bình
hành
ABCD
là hình ch nht.
5.3. Cho hình ch nht
ABCD
,
8AB
,
6BC
. Điểm
M
nm trong hình ch nht. Tìm giá tr nh
nht ca tng:
2 2 2 2
.S MA MB MC MD
5.4. Cho tam giác
ABC
vuông ti
A
. Gi
O
là một điểm bt kì trong tam giác. V
OD AB
,
OE BC
OF CA
. Tìm giá tr nh nht ca tng:
222
S OD OE OF
.
5.5. Cho hình ch nht
ABCD
, đường chéo
AC d
. Trên các cnh
AB
,
BC
,
CD
DA
lần lượt ly
các điểm
M
,
N
,
P
,
Q
. Tìm giá tr nh nht ca tng:
2 2 2 2
.S MN NP PQ QM
5.6. Cho tam giác đều
ABC
cnh
a
. Trên các cnh
AB
,
AC
lần lượt lấy các điểm
D
,
E
sao cho
AD CE
. Tìm giá tr nh nht của độ dài
DE
.
Tính chất đƣờng trung tuyến cùa tam giác vuông
5.7. Cho tam giác
ABC
vuông ti
A
. Trên cnh huyn
BC
ly một điểm
M
. V
MD AB
,
ME AC
,
AH BC
. Tính s đo góc
DHE
.
5.8. Cho tam giác
ABC
vuông ti
A
, đường cao
AH
, đường trung tuyến
AD
. V
HE AB
,
HF AC
. Gi
M
N
lần lượt là trug điểm ca
HB
HC
.
a) Chng minh rng
EM//FN// AD
.
b) Tam giác
ABC
phải có thêm điều kiện gì thì ba đường thng
EM
,
FN
,
AD
là ba đường thng song
song cách đều.
27
ABC
A
AB AC
AH
AC
D
5.9. Cho tam giác vuông ti , đường cao . Trên cnh lấy đim sao cho
AD AB
. Gi
M
là trung điểm ca
BD
. Chng minh rng tia
HM
là tia phân giác ca góc
AHC
.
5.10. Cho hình ch nht
ABCD
,
15AB
,
8BC
. Trên các cnh
AB
,
BC
,
CD
,
DA
lần lượt ly
các điểm
E
,
F
,
G
,
H
. Tính giá tr nh nht ca chu vi t giác
EFGH
.
Đƣng thng song song vi một đƣờng thng cho trƣớc
5.11. Cho góc
xOy
có s đo bằng
30
. Điểm
A
c định trên tia
Ox
sao cho
2OA cm
. Lấy điểm
B
bt kì trên tia
Oy
. Trên tia đối ca tia
BA
lấy điểm
C
sao cho
2BC BA
. Hỏi khi điểm
B
di động
trên tia
Oy
thì điểm
C
di động trên đường nào?
5.12. Cho góc
xOy
có s đo bằng
45
. Điểm
A
c định trên tia
Ox
sao cho
32OA cm
. Lấy điểm
B
bt kì trên tia
Oy
. Gi
G
là trng tâm ca tam giác
OAB
. Hỏi khi điểm
B
di động trên tia
Oy
thì
điểm
G
di động trên đường nào?
5.13. Cho tam giác
ABC
cân ti
A
. Trên các cnh
AB
AC
lần lượt lấy các điểm
M
N
sao cho
AM CN
. Gi
O
là trung điểm ca
MN
. Hỏi điểm
O
di động trên đường nào?
5.14. Bên trong hình ch nhật kích thước
36
cho 10 điểm. Chng minh rng tn tại hai điểm trong s
10 điểm đó có khoảng cách nh hơn
2,3
.
5.15. Bên trong hình ch nhật kích thước
36
cho 8 điểm. Chng minh rng tn tại hai điểm trong s 8
điểm đó có khoảng cách nh hơn
2,3
.
28
CHUYÊN ĐỀ 6. HÌNH THOI VÀ HÌNH VUÔNG
A. Kiến thc cn nh
1. Định nghĩa
Hình thoi là t giác có bn cnh bng nhau (h.6.1)
Hình vuông là t giác có bn góc vuông và có bn cnh bng nhau (h.6.2)
2. Tính cht
Trong hình thoi:
Hai đường chéo ca hình thoi vuông góc vi nhau;
Hai đường chéo là các đường phân giác ca các góc ca hình thoi;
Hình vuông có đủ các tính cht ca hình ch nht và hình thoi.
3. Du hiu nhn biết
Nhn biết hình thoi:
T giác có bn cnh bng nhau là hình thoi;
Hình bình hành có hai cnh k bng nhau là hình thoi
Hình bình hành có hai đường chéo vuông góc vi nhau là hình thoi;
Hình bình hành có một đường chéo là đường phân giác ca mt góc là hình thoi.
Nhn biết hình vuông:
Hình ch nht có hai cnh k bng nhau là hình vuông;
Hình ch nhật có hai đường chéo vuông góc là hình vuông;
Hình ch nht có một đường chéo là đường phân giác ca mt góc là hình vuông;
Hình thoi có mt góc vuông là hình vuông;
Hình thoi có hai đường chéo bng nhau là hình vuông.
B. Mt s ví d
Ví d 1. Cho hình thoi
ABCD
, độ dài mi cnh là
13cm
. Gi
O
là giao điểm của hai đường chéo. V
OH AD
. Biết
6OH cm
, tính t s của hai đường chéo
BD
AC
Gii ( h.63)
C
B
D
A
D
B
A
C
29
Tìm cách gii
V thêm
BK AD
để dùng định lý đường trung bình của tam giác, định lý Py-ta-go tính bình phương
độ dài ca mỗi đường chéo.
Trình bày li gii
V
BK AD
Xét
BKD
//OH BK
( vì cùng vuông góc vi
AD
) và
OB OD
nên
KH HD
.
Vy
OH
là đường trung bình ca
BKD
.
Suy ra
1
2
OH BK
, do đó
12BK cm
.
Xét
ABK
vuông ti
K
có:
2 2 2 2 2
13 12 25 5AK AB BK AK cm
do đó
8KD cm
.
Xét
BKD
vuông ti
K
có:
2 2 2 2 2
12 8 208.BD BK KD
Xét
AOH
vuông ti
H
có:
2 2 2 2 2
6 9 117.OA OH AH
2
2
117 468.
2
AC
AC
Do đó:
2
2
208 4 2
.
468 9 3
BD BD
AC
AC
Ví d 2: Cho tam giác
ABC
cân ti
A
, hai đường cao
BE
CF
ct nhau ti
H
. Đường thng
AH
ct
EF
ti
D
, ct
BC
ti
G
. Gi
M
N
lần lượt là hình chiếu ca
G
trên
AB
AC
. Chng minh
rng t giác
DNGM
là hình thoi.
Gii ( h.6.4)
H
C
O
B
D
A
30
Tìm cách gii
Dùng định lý đường trung bình ca tam giác ta chứng minh được t giác
DNGM
là hình bình hành. Sau
đó chứng minh hai cnh k bng nhau .
Trình bày li gii
ABE ACF
( cnh huyn, góc nhn)
AE AF
BE CF
H
là trc tâm ca
ABC
nên
AH
là đường cao, đồng thời là đường trung tuyến, t đó
GB GC
và t đó
GB GC
DE DF
.
Xét
EBC
//GN BE
( cùng vuông góc vi
AC
) và
GB GC
nên
NE NC
.
Chứng minh tương tự, ta được
MF MB
.
Dùng định lý đường trung bình ca tam giác ta chứng minh được
//DM GN
DM GN
nên t giác
DNGM
là hình bình hành.
Mt khác,
DM DN
( cùng bng
1
2
ca hai cnh bng nhau) nên
DNGM
là hình thoi.
Ví d 3. Cho hình vuông
ABCD
. Lấy điểm
M
trên đường chéo
AC
. V
ME AD
,
MF CD
MH EF
. Chng minh rằng khi điểm
M
di động trên
AC
thì đường thng
MH
luôn đi qua một
điểm c định.
Gii ( h.6.5)
Tìm cách gii
H
D
N
M
E
F
G
B
C
A
2
1
1
1
N
H
E
F
D
B
A
C
M
31
MH
B
V hình chính xác ta thấy đường thng đi qua một điểm c định là điểm . Vì thế ta s chng
minh ba điểm
H
,
M
,
B
thng hàng bng cách chng minh
12
MM
.
Trình bày li gii
Gi
N
là giao điểm của đường thng
EM
vi
BC
Khi đó
BN AE
;
AE ME
( vì
AEM
vuông cân), suy ra
BN ME
.
Chứng minh tương tự, ta được:
MN MF
Ni
MB
ta được :
BMN EFM
..c g c
Suy ra:
11
BE
do đó
12
MM
T đó ba điểm
H
,
M
,
B
thng hàng.
Vậy đường thng
MH
luôn đi qua một điểm c định là điểm
B
.
Ví d 4. Cho hình vuông
ABCD
cnh
a
. Trên cnh
BC
lấy điểm
M
, trên cnh
CD
lấy điểm
N
sao
cho chu vi các tam giác
CMN
bng
2a
. Chng minh rng góc
MAN
có s đo không đổi.
Gii ( h.6.6)
Tìm cách gii
V hình chính xác ta luôn thy
45MAN
. Vì vy ta v hình ph to ra góc
90
ri chng minh
MAN
bng nửa góc vuông đó.
Trình bày li gii:
Trên tia đối ca tia
DC
lấy điểm
E
sao cho
DE BM
.
..BAM DAE c g c
suy ra
AM AE
.BAM DAE
.
Ta có:
90
o
BAM DAM
90
o
DAE DAM
hay
90
o
EAM
.
Hình 6.6
A
B
C
E
D
32
Theo đề bài,
2CM CN MN a
2CM CN MB ND a
Nên
MN MB ND
hay
MN DE ND EN
.
. . 45
2
o
EAM
MAN EAN c c c MAN EAN
.
Vy
MAN
có s đo không đổi.
Ví d 5. Cho hình vuông
ABCD
. Trên các cnh
,,AB BC CD
lần lượt lấy các điểm
,,M N P
sao cho
AM BN CP
. Qua
N
v một đường thng vuông góc vi
MP
ct
AD
ti
Q
. Chng minh rng t
giác
MNPQ
là hình vuông.
Gii (h 6.7)
(*) Tìm cách gii
T gi thiết ta nghĩ đến vic chng minh rng các tma giác bằng nhau để suy ra bn cnh ca t giác
MNPQ
bằng nhau., ta được t giác này là hình thoi. Sau đó chứng minh hai đường chéo nhau để được
hình vuông.
(*) Trình bày li gii
Gi
O
là giao điểm ca
ME
NF
.
Ta có:
AB BC CD DA
AM BN CP
Nên
BM CN DP
.
D thy t giác
là hình vuông.
EMP
FNQ
có:
90
o
EF
;
ME NF
( bng cnh hình vuông).
EMP FNQ
( hai góc có cạnh tương ứng vuông góc)
..E
MP FNQ c g c MP NQ
EP FQ
.
Ta có:
DE AM AF DP AQ
do đó
DQ CP
.
Các tam giác
,,BNM CPN DQP
AMQ
bng nhau suy ra:
MN QN QP QM
.
Do đó tứ giác
MNPQ
là hình thoi, Hình thoi này có hai đường chéo bng nhau nên là hình vuông.
C. Bài tp vn dng
(*) Hình thoi
6.1. Mt hình thoi có góc nhn bng
30
o
. Khong cách t giao điểm của hai đường chéo đến mi cnh
bng
h
. Tính độ dài mi cnh ca hình thoi.
33
6.2. Cho hình thoi
ABCD
, chu vi bng
8cm
. Tìm giá tr ln nht của tích hai đường chéo.
6.3. Cho hình thoi
ABCD
,
40
o
A
. Gi
M
là trung điểm ca
AB
. V
DH CM
. Tính s đo của góc
MHB
.
6.4. Cho hinh thoi
ABCD
. Trên na mt phng b
BD
có chứa điểm
C
, v hình bình hành
BDEF
cos
DE DC
. Chng minh rng
C
là trc tâm ca tam giác
AEF
.
6.5. Cho hình bình hành
ABCD
, hai đường chéo ct nhau ti
O
. Gi
, , ,E F G H
lần lượt là các giao
điểm các đường phân giác ca tam giác
,,AOB BOC COD
DOA
. Chng minh rng t giác
EFGH
hình thoi.
6.6. Dng hình thoi
ABCD
bieets
8AC BD cm
25
o
ABD
.
(*) Hình vuông
6.7. Cho hình vuông
ABCD
. Trên cnh
BC
lấy các điểm
E
F
sao cho
BE EF FC
. Trên cnh
AD
lấy điểm
G
sao cho
1
3
AG AD
. Tính tng
AEG AFG ACG
.
6.8. Cho hình vuông
ABCD
. Trên đường chéo
AC
ly một điểm
M
. V
,ME AD MF CD
. Chng
minh rằng ba đường thng
,AF CE
BM
đồng quy.
6.9. Cho tam giác
ABC
vuông ti
A
, đường cao
AH
. V ra phía ngoài tam giác này các hình vuông
ABDE
ACFG
. Chng minh rng:
a) Ba đường thng
,AH DE
FG
đồng quy.
b) Ba đương thẳng
,AH BF
CD
đồng quy.
6.10. Cho hình vuông
ABCD
. Trên tia đối ca tia
BA
lấy điểm
E
. Trên tia đối ca
CB
lấy điểm
F
sao
cho
AE CF
. Gi
O
là trung điểm ca
EF
. V điểm
M
sao cho
O
là trung điểm ca
DM
. Chng
minh rng t giác
là hình vuông.
6.11. Cho tam giác
ABC
,
45
o
A
. V ba đường cao
,,AD BE CF
ct nhau ti
H
. Gi
, , ,M N P Q
ln
ợt là trung điểm ca
,,AB AC HB
HC
. Chng minh rng t giác
MNPQ
là hình vuông.
6.12. Cho hình bình hành
ABCD
. V ra phía ngoài ca hình bình hành các hình vuông có mt cnh là
cnh ca hình bình hành. Gi
, , ,E F G H
lần lượt là tâm ( tức là giao điểm của hai đường chéo) ca các
hình vuông v trên các cnh
,,AB BC CD
DA
. Chng minh rng:
EG HF
EG HF
.
6.13. Dng hình vuông
ABCD
biết đỉnh
A
và trung điểm
M
ca
CD
.
6.14. Mt bàn c hình vuông có kích thước
66
. Có th dùng
9
mnh g hình ch nhật có kích thước
14
để ghép kín bàn c được không?
6.15. Mt hình ch nhật có kích thước
36
. Hãy chia hình ch nht này thành nhiu phn ( hình tam
giác, t giác) để ghép li thành mt hình vuông ( s phần được chia ra càng ít càng tt).
34
35
CHUYÊN ĐỀ 7. ĐỐI XNG TRC ĐỐI XNG TÂM
A. Kiến thc cn nh
1. Các định nghĩa
(*) Hai điểm đối xứng nhau qua đường thng
d
, nếu
d
là đường trung trc của đoạn thng ni hai
điểm đó (h.7.1)
(*) Hai điểm đối xứng nhau qua điểm
O
nếu
O
là trung điểm của đoạn thng nối hai điểm đó (h.7.2).
(*) Hai hình gọi là đối xứng nhau qua đường thng
d
( hoặc qua điểm
O
), nếu mỗi điểm thuc hình
này đối xng vi một điểm thuộc hình kia qua đường thng
d
( hoặc qua điểm
O
) và ngược li.
2. Tính cht
Nếu hai đoạn thẳng (góc, tam giác) đối xng vi nhau qua một đường thng ( hoc qua một điểm) thì
chúng bng nhau.
3. Hình có trục đối xứng, có tâm đối xng
- Hình thang cân có trục đối xứng là đường thẳng đi qua trung điểm hai đáy.
- Tương tự hình ch nht có hai trục đối xng.
- Hình thoi có hai trục, đối xứng là hai đường chéo. Hình vuông có
4
trục đối xng.
- Hình bình hành, hình ch nhật, hình thoi, hình vuông có tâm đối xứng là giao điểm hai đường chéo.
B. Mt s ví d
d 1. Cho t giác
ABCD
, hai đường thng
AB
CD
không vuông góc vi nhau. Dựng điểm
M
trên đường thng
CD
sao cho tia phân giác ca góc
AMB
vuông góc với đường thng
CD
.
Gii (h.7.3)
Hình 7.1
d
A
B
Hình 7.2
A
N
O
36
a) Phân tích
Gi s đã dựng được
M
trên đường thng
CD
sao cho tia phân giác
Mx
ca
AMB
vuông góc vi
đường thng
CD
. Trên tia đối ca tia
MB
lấy điểm
A
sao cho
MN MA
.
Vì tia
Mx
là tia phân giác ca góc
AMB
Mx CD
nên đường thng
CD
là đường phân giác ca góc
AMN
.
Xét
MAN
cân ti
M
MD
là đường phân giác nên
MD
cũng là đường trung trc, suy ra
A
N
đối xứng qua đường thng
CD
.
b) Cách dng
- Dựng điểm
N
đối xng vi
A
qua
CD
.
- Dựng giao điểm
M
ca
AB
với đường thng
CD
. Khi đó
M
là điểm cn dng.
c) Chng minh
A
N
đối xng qua
CD
nên
CD
là đường trung trc của , do đó
C
cũng là đường phân giác ca
góc
AMN
.
Nếu
Mx
là tia phân giác ca góc
AMB
thì
Mx CD
( tính cht hai tia phân giác ca hai góc k bù).
d) Bin lun: Bài toán luông có mt nghim hình.
Nhn xét: Cách dựng điểm
M
như trên còn cho ta kết qu là tng
AM MB
ngn nht.
Ví d 2. Cho hình thang
ABCD
(
//AB CD
). Trên đáy
AB
lấy điểm
K
tùy ý. V điểm
E
đối xng vi
K
qua trung điểm
M
ca
AD
. V điểm
F
đối xng vi
K
qua trung điểm
N
ca
BC
. Chng minh
rng
EF
có độ dài không đổi.
Gii (h 7.4)
x
Hình 7.3
H
A
B
C
D
N
M
37
(*) Tìm cách gii
Ta thy:
EF ED DC CF
CD
không đổi nên mun chng minh
EF
không đổi ta cn chng
minh
ED CF
không đổi.
(*) Trình bày li gii
DE
AK
đối xng nhau qua
M
nên
DE AK
//DE AK
do đó
//DE AB
.
Mt khác,
//DC AB
suy ra ba điểm
thng hàng.
Chứng minh tương tự, ta được:
BK CF
và ba điểm
thng hàng.
Ta có:
EF ED DC CF AK DC BK AB CD
( không đổi).
Nhn xét: Khi điểm
K
di động trên c đường thng
AB
thì độ dài của đoạn thng
EF
vẫn không đổi.
Ví d 3. Cho góc
xOy
khác góc bẹt và hai điểm
,MN
nằm trong góc đo. Dựng hình bình hành
sao cho
A Ox
B Oy
.
Gii (h.7.5)
a) Phân tích
Gi s đã dựng được hình bình hành
thỏa mãn đề bài. Gi
E
là giao điểm của hai đường chéo.
V điểm
F
đối xng vi
O
qua
E
. Khi đó tứ giác
AOBF
là hình bình hành.
(*) Điểm
B
thỏa mãn hai điều kin:
B Oy
//B Ft Ox
.
Đim
A
thỏa mãn hai điều kin:
A Ox
A
thuc tia
BE
.
b) Cách dng
- Dựng trung điểm
E
ca
MN
;
- Dựng điểm
F
đối xng vi
O
qua
E
;
- Dng tia
//Ft Ox
ct tia
Oy
ti
B
;
- Dựng giao điểm ca tia
BE
và tia
Ox
.
c) Chng minh
B
N
M
A
C
D
E
F
K
Hình 7.5
38
..A
OE BFE g c g EA EB
Mt khác,
EM EN
nên t giác
AMNB
là hình bình hành.
d) Bin lun: Bài toán luôn có mt nghim hình.
Ví d 4. Cho tam giác
ABC
vuông ti
A
A
B AC
. Điểm
D
thuc cnh huyn
.BC
V điểm
M
điểm
N
đối xng vi
D
lần lượt qua
AB
AC
. Chng minh rng :
a)
M
N
đối xng qua
;A
b) Xác định v trí của điểm
D
để
MN
ngn nht, dài nht.
Gii (h.7.6)
* Tìm cách gii
Mun chứng minh hai điểm
M
N
đối
xng qua
A
, ta chng minh
AM AN
180MAN 
.
* Trình bày li gii
a)
AM
đối xng vi
AD
qua
AB
nên
AM AD
12
.1AA
AN
đối xng vi
AD
qua
AC
nên
AN AD
34
.2AA
T
1
2
suy ra:
AM AN
23
2
2 2.90 180 .MAN A A BAC
Vậy ba điểm
,,M A N
thng hàng,
T đó suy ra
M
N
đối xng qua
A
2MN AD
.
b) V
AH BC
, ta có
AD AH
, do đó
22MN AD AH
.
Vy
MN
ngn nht là bng
2AH
khi
DH
(h.7.7).
Da vào quan h giữa đưng xiên và hình chiếu ta có
AD AC
suy ra
2 2 .MN AD AC
Do đó
MN
dài nht là bng
2AC
khi
DC
(h.7.8).
Hình 7.6
Hình 7.7
Hình 7.8
39
C. Bài tp vn dng
* Đối xng trc
7.1. Cho tam giác
ABD
. V điểm
C
đối xng vi
A
qua
BD
. V các đường phân giác ngoài ti các
đỉnh
, , ,A B C D
ca t giác
ABCD
chúng ct nhau to thành t giác
.EFGH
a) Xác định dng ca t giác
;EFGH
b) Chng minh rng
BD
là trục đối xng ca t giác
.EFGH
7.2. Cho tam giác nhn
.ABC
Gi
D
là điểm nm gia
B
C
. V các điểm
M
N
đối xng vi
D
lần lượt qua
AB
AC
.
a) Chng minh rng góc
MAN
luôn có s đo không đổi;
b) Xác định v trí ca
D
để
MN
có độ dài ngn nht.
7.3. Cho tam giác nhn
ABC
. Gi
lần lượt là các điểm nm trên các cnh
, , .BC CA AB
Xác định
c trí ca
để chu vi tam giác
DEF
nh nht.
7.4. Cho hai điểm
,AB
cùng thuc mt na mt phng b
xy
. Hãy tìm trên
xy
hai điểm
C
D
sao
cho
cho trước và chu vi t giác
ABCD
nh nht.
7.5. Cho tam giác
ABC
, đường phân giác
AD
và một điểm
M
trong tam giác. V các điểm
,,N P A
đối xng vi
M
lần lượt qua
,AB AC
AD
.
a) Chng minh rng
N
P
đối xng qua
;AA
b) Gi
,BC

là các điểm đối xng vi
M
lần lượt qua các đường phân giác ca góc
,B
và góc
.C
Chng minh rằng ba đường thng
,,AA BB CC
đồng quy.
7.6. Cho t giác
ABCD
vi một điểm
M
nm gia
A
B
. Chng minh rng
MC ND
nh hơn số
ln nht trong hai tng
,AC AD BC BD
.
* Đối xng tâm
40
7.1. Cho tam giác ABC và O là một điểm tuỳ ý trong tam giác. Gọi D, E, F lần lượt là trung điểm của
BC, CA, AB. Gọi A', B', C' lần lượt là các điểm đối xứng với O qua D, E, F. Chứng minh rằng ba
đường thẳng AA', BB', CC'
đồng quy.
7.2. Cho góc xOy khác góc bẹt một điểm G trong góc đó. Dựng điểm
A Ox, điểm B Oy sao cho G là trọng tâm của tam giác OAB.
7.3. Cho tam giác ABC. Vẽ điểm D đối xứng với A qua điểm B. Vẽ điểm E đối xứng với B qua C. Vẽ
điểm F đối xứng với C qua A. Chứng minh rằng tam giác ABC tam giác DEF cùng một trọng
tâm.
7.4. Dựng hình bình hành ABCD biết vị trí trung điểm M của AB, trung điểm N của BC trung điểm
P của CD.
7.5. Dựng tứ giác ABCD biết AD = AB = BC và ba điểm M, N, P lần lượt là trung điểm của AD, AB và
BC (biết M, N, P không thẳng hàng).
7.6. Cho một hình vuông gồm 44 ô vuông. Trong mỗi ô viết một trong các số 1, 2, 3, 4. Chứng minh
rằng tồn tại một hình bình hành đỉnh tâm của bốn ô vuông sao cho tổng hai số hai đỉnh đối
diện là bằng nhau.
41
CHUYÊN ĐỀ 8. VẼ HÌNH PHỤ ĐỂ GIẢI TOÁN
A. Kiến thức cần nhớ
Nhiều bài toán trong chương tứ giác cần phải vẽ hình phụ thì mới giải được. Vẽ hình phụ để tạo
thêm sự liên kết giữa giả thiết và kết luận từ đó dễ tìm ra cách giải. Một số cách vẽ hình phụ thường
dùng trong chương này là:
1. Nếu đề bài có hình thang thì từ một đỉnh có thể vẽ thêm một đường thẳng:
- song song với một cạnh bên;
- song song với một đường chéo;
- vuông góc với đáy.
Khi vẽ như vậy, một đoạn thẳng đã được dời song song với chính nó từ vị trí này đến một vị trí
khác thuận lợi hơn trong việc liên kết với các yếu tố khác, từ đó giải được bài toán.
2. Vẽ thêm hình bình hành để chứng minh hai đường thẳng song song, chứng minh quan hệ về độ
dài, chứng minh ba đường thẳng đồng quy, ba điểm thẳng hàng, tính số đo góc,
3. Vẽ thêm trung điểm của đoạn thẳng để vận dụng định lí đường trung bình của tam giác, của hình
thang, định lí đường trung tuyến ứng với cạnh huyền của tam giác vuông. Cũng có thể vẽ thêm
đường thẳng song song để tạo ra đường trung bình của tam giác, hình thang.
Dùng định lí đường trung bình có thể chứng minh các quan hệ song song, thẳng hàng, các quan hệ
về độ dài,
4. Vẽ điểm đối xứng với một điểm cho trước qua một đường thẳng hoặc qua một điểm. Nhờ cách
vẽ này ta cũng có thể dời một đoạn thẳng, một góc từ vị trí này sang vị trí khác thuận lợi cho việc
chứng minh.
B. Một số ví dụ
Ví dụ 1. Chứng minh rằng trong một hình thang tổng hai cạnh bên lớn hơn hiệu hai cạnh đáy.
Giải (h.8.1)
* Tìm cách giải
Xét hình thang ABCD (AB // CD), ta phải chứng minh AD + BC > CD - AB.
Điều phải chứng minh rất gần với bất đẳng thức tam giác. Điều này gợi ý cho ta vẽ hình phụ để có AD +
BC là tổng các độ dài hai cạnh của một tam giác.
* Trình bày lời giải
Vẽ BM // AD (M CD) ta được DM = AB và BM = AD.
Xét BMC có BM + BC > MC AD + BC > DC DM
hay AD + BC > CD AB (đpcm).
Trường hợp hai cạnh bên song song t hai đáy bằng nhau, bài toán hiển nhiên đúng.
Hình 8.1
42
Ví dụ 2. Cho hình thang ABCD (AB // CD), hai đường chéo vuông góc với nhau. Biết AB = 5cm, CD =
12cm và AC = 15cm. Tính độ dài BD.
Giải (h.8.2)
* Tìm cách giải
Ba đoạn thẳng AB, AC và CD đã biết độ dài nhưng ba đoạn thẳng này không phải ba cạnh của một tam
giác nên không tiện sử dụng. Ta sẽ dời song song đường chéo AC đến vị trí BE thì tam giác BDE vuông
tại B biết độ dài hai cạnh, dễ dàng tính được độ dài cạnh thứ ba BD.
* Trình bày lời giải
Vẽ BE // AC (E tia DC).
Khi đó BE = AC = 15cm; CE = AB = 5cm.
Ta có BE BD (vì AC BD).
Xét BDE vuông tại B có
22
BD 17 15 8
(cm).
Ví dụ 3. Hình thang ABCD có
o
A D 90 .
Biết AB = 3cm;
BC 2 2cm
CD = 5cm. Chứng minh rằng
Giải (h.8.3)
* Tìm cách giải
Nếu dời song song đoạn thẳng AD tới vị trí BH thì được BHC vuông tại H. Ta dễ dàng tính được HC =
HB, do đó tính được góc C, góc B.
* Trình bày lời giải
Vẽ BH CD (H CD) thì BH // AD, do đó
DH = AB = 3cm
suy ra HC = 5 3 = 2 (cm).
Xét BHC vuông tại H, áp dụng định lí Py-ta-go ta có
2
2 2 2
HB BC HC 2 2 2 2
(cm).
Vậy HBC vuông cân
o
C 45
do đó
o
ABC 135
suy ra
ABC 3C.
Ví dụ 4. Cho tứ giác ABCD, hai đường chéo cắt nhau tại O. Cho biết
o
AOB 60
và AC = BD = a.
Chứng minh rằng AB + CD a.
Gii (h.8.4)
* Tìm cách gii
T điều phi chng minh ta thy cn vn dng bất đẳng thức tam giác. Do đó cần
v hình ph để to ra mt tam giác có hai cnh lần lượt bng hai cnh
, AB CD
cnh th ba bằng đường chéo
AC
Hình 8.3
Hình 8.2
43
Nếu v thêm hình bình hành
ABEC
thì các yêu cầu trên được tha mãn.
* Trình bày li gii
V hình bình hành
ABEC
, ta được
//BE AC
suy ra
60
o
DBE AOB
;
; .BE AC a AB CE
Tam giác
DBE
là tam giác đều
DE a
.
Xét ba điểm
, , C D E
ta có:
CE CD DE
hay
AB CD
a
(Du
“”
xảy ra khi điểm
C
nm gia
D
E
hay
// DC AB
. Khi đó tứ giác
ABCD
là hình thang cân).
Ví d 5: Cho hình ch nht
ABCD
. V
AH BD
. Gi
K
M
lần lượt là trung điểm ca
BH
CD
. Tính s đo của góc
AKM
.
Gii (h.8.5)
* Tìm cách gii
Bài toán có cho hai trung điểm
K
M
nhưng chưa thể vn dng trc tiếp được.
Ta v thêm trung điểm
N
ca
AB
để vn dụng định lý đường trung bình ca hình ch nhật, đường trung
bình ca hình tam giác.
* Trình bày li gii
Gi
N
là trung điểm ca
AB
thì
MN
là đường trung bình ca hình
ch nht
ABCD
//MN AD
.
Mt khác,
//AN DM
nên t giác
là hình bình hành. Hình
bình hành này có
90
o
D
nên là hình ch nhật. Suy ra hai đường
chéo
AM
DN
ct nhau tại trung điểm
O
ca mỗi đường:
.OA OM ON OD
Xét
ABH
NK
là đường trung bình nên
//NK AH
NK BD
(vì
)AH BD
.
Do đó
KDN
vuông ti
K
.
Ví d 6: Cho hai điểm
A
B
thuc cùng mt na mt phng b là đường thng
d
. Tìm trên
d
một điểm
M
sao cho hai tia
, MA MB
to với đường thng
d
hai góc nhn bng nhau.
Gii (h.8.6)
* Tìm cách gii
Gi s đã tìm được điểm
Md
sao cho
12
MM
.
V điểm
A
đối xng vi
A
qua
d
thì
13
MM
, suy ra
23
MM
(
cùng
bng
1
M
). Dó đó ba điểm
A
,
M
,
B
thng hàng.
Hình 8.5
Hình 8.4
1
2
3
44
* Trình bày li gii
- V điểm
A
đối xng vi
A
qua
d
;
- V đoạn thng
AB
cắt đường thng
d
ti
M
;
- V đoạn thng
MA
ta được
12
MM
.
Tht vy, do
A
đối xng vi
A
qua
d
nên
13
MM
.
Mt khác,
23
MM
ối đỉnh) nên
12
MM
.
C. Bài tp vn dng
V thêm đƣờng thng song song
8.1. Chng minh rng nếu mt hình thang có hai cnh bên bằng nhau thì đó là hình thang cân hoặc hình
bình hành.
8.2. Cho hình thang có hai đáy không bằng nhau. Chng minh rng tng hai góc k đáy lớn nh hơn tổng
hai góc k đáy nhỏ.
8.3. Cho hình thang
/
/ ,ABCD AB CD
BD CD
. Cho biết
.AB CD BD a
Tính độ dài AC.
8.4. Cho hình thang cân
/
/ ,ABCD AB CD
đường cao bng
h
và tổng hai đáy bng
2h
. Tính góc xen gia
hai đường chéo.
8.5. Chng minh rng trong mt hình thang thì tổng các bình phương của hai đường chéo bng tng các
bình phương của hai cnh bên cng vi hai ln tích ca hai cạnh đáy.
V thêm hình bình hành
8.6. Cho tam giác
ABC
. Dựng ra ngoài tam giác này các tam giác đều
, , ABD BCE CAF
. Chng minh
rng trng tâm ca tam giác
DEF
trùng vi trng tâm ca tam giác
ABC
.
8.7. Cho tam giác đều
ABC
. Trên cnh
BC
lấy điểm
M
. Qua
M
v một đường thng vuông góc vi
AB
ct
AB
ti
H
, cắt đường thng vuông góc vi
AC
v t
C
tại điểm
K
. Gi
N
là trung điểm ca
BM
.
Chng minh rng tam giác
ANK
có s đo các góc tỉ l vi
1, 2, 3.
8.8. Dng t giác
ABCD
sao cho
2,5 ; 3 ; 4,5 ; 3,5AB cm BC cm CD cm DA cm
và góc nhn gia hai
đường thng
,AD BC
40
.
V thêm trung điểm Tạo đƣờng trung bình
8.9. Cho hình thang
/
/ ,ABCD AB CD
1
A 90 , .
2
AB CD
V
DH AC
. Gi
K
là trung điểm ca
HC
.
Tính s đo góc
BCD
.
8.10. Cho hình vuông
ABCD
, hai đường chéo ct nhau ti
O
. Gi
M
N
lần lượt là trung điểm ca
OA
CD
. Chng minh rng tam giác
MNB
vuông cân.
Hình 8.6
45
8.11. Cho tam giác
ABC
cân ti
A
, đường phân giác
BM
. T
M
v một đường thng vuông góc vi
BM
cắt đường thng
BC
ti
D
. Chng minh rng
2 .BD CM
8.12. Cho t giác
ABCD
,
90
o
CAD CBD
. Gi
E
F
lần lượt là hình chiếu ca
C
D
trên đường
thng
AB
. Chng minh rng
.AF BE
8.13. Cho đường thng
xy
. V tam giác
ABC
trên mt na mt phng b
xy
. Gi
G
là trng tâm ca tam
giác
ABC
. T
, ,A B C
G
v các đường thng song song vi nhau ct
xy
lần lượt ti
’, ’, A B C
G
.
Chng minh rng:
3 AA BB CC GG
8.14. Cho tam giác
ABC
vuông cân ti
A
. Trên các cnh
, AB AC
lần lượt lấy các điểm
M
D
sao cho
AM AD
. T
A
M
v các đường thng vuông góc vi
BD
. Chúng ct
BC
lần lượt ti
E
F
. Chng
minh rng
2
BD MF
AE
8.15. Cho t giác
ABCD
. Gi
’, ’, ’, A B C D
lần lượt là trng tâm ca các tam giác
,,BCD CDA
,DAB ABC
. Chng minh rng:
a) Các đường thng
’, ’, ’, AA BB CC DD
cùng đi qua một điểm.
b) Điểm này chia
’, ’, ’, AA BB CC DD
theo cùng mt t s.
8.16. Cho tam giác
ABC
và một điểm
O
nm trong tam giác sao cho
ABO ACO
. V
,OH AB OK AC
. Chng minh rằng đường trung trc ca
HK
đi qua một điểm c định.
V thêm hình đối xng
8.17. Cho góc
xOy
có s đó bằng
60
và một điểm
A
trong góc đó sao cho
A
cách
Ox
là 2cm và cách
Oy
1.cm
a) Tìm một điểm
B
trên
Ox
và một điểm
C
trên
Oy
sao cho chu vi tam giác
ABC
nh nht.
b) Tính độ dài nh nht ca chu vi tam giác
ABC
.
8.18. Dng tam giác biết một đỉnh, trọng tâm và hai đường thẳng đi qua hai đỉnh còn li.
46
CHUYÊN ĐỀ 9. TOÁN QU TÍCH
A. Kiến thc cn nh
1. Định nghĩa
Qu tích ca những điểm có tính cht T nào đó là tập hp tt c những điểm có tính cht T đó.
2. Các qu tích cơ bản
- Qu tích các điểm cách đều hai đầu ca một đoạn thng c định là đường thng trung trc của đoạn
thẳng đó. (1)
- Qu tích các điểm nm bên trong một góc và cách đều hai cnh ca góc là tia phân giác của góc đó.
(2)
- Qu tích các điểm cách một đường thng c định mt khong bng
h
không đổi là hai đường thng song
song với đường thẳng đó và cách đường thẳng đó một khong
h
. (3)
- Qu tích những điểm cách một điểm O c định mt khong
R
không đổi là đường tròn tâm
O
, bán kính
R
. (4)
3. Cách gii bài toán tìm qu tích các điểm có chung tính chất T nào đó
a) Phn thun: Chng minh rng nếu điểm
M
có tính cht T thì điểm
M
thuc mt hình
H
nào đó.
b) Phần đảo: Chng minh rng nếu điểm
M
thuc hình
H
thì điểm
M
có tính cht T.
c) Kết lun: Qu tích của điểm
M
là hình
H
.
4. Mt s lƣu ý khi giải bài toán tìm qu tích
a) Tìm hiểu đề bài:
Cn xét xem:
- Yếu t nào c định (vì trong các qu tích cơ bản đều có nói đến yếu t c định như điểm, đoạn thng,
góc,... )
- Yếu t nào không đổi (thường là khoảng cách không đổi, góc có s đo không đổi,... );
- Yếu t nào chuyển động (điểm nào có v trí thay đổi, liên quan đến điểm phi tìm qu tích như thế nào?
).
d) D đoán quỹ tích
V nháp vài v trí của điểm cn tìm qu tích (thường là v ba v trí).
- Nếu ba điểm này thng hàng thì ta d đoán quỹ tích là đường thẳng (đường thẳng song song, đường
trung trc, tia phân giác,...).
- Nếu ba điểm không thng hàng thì qu tích có th là đường tròn.
c) Gii hn qu tích
nhiu bài toán qu tích cn tìm ch mt phn ca hình H, phn còn li không thỏa mãn điều kin
ca bài toán, ta phi loi tr phần này. Làm như vậy gi là tìm gii hn ca qu tích.
Vic tìm gii hn ca qu tích thường làm au phn thuận, trước phần đảo.
47
B. Mt s ví d
d 1: Cho tam giác
ABC
D
một điểm đi đng trên cnh
BC
. V
DE
//
AB
,
DF
//
AE
, E
AC F AB
. Gi
M
là trung điểm ca
EF
. Tìm qu tích của điểm
M
.
Gii (h.9.1)
a) Phn thun
T giác
ADEF
// , //DE AF DF AE
nên là hình bình hình.
Suy ra
AD
EF
ct nhau tại trung điểm ca mỗi đường.
Vậy trung điểm
M
ca
EF
cũng là trung điểm ca
.AD
V
,.MK BC AH BC
Do
AH
c định nên
AH
có độ dài không đổi.
.9.1nh
Xét
AHD
MK
đường trung bình,
1
2
MK AH
(không đổi). Điểm
M
cách đường thng
BC
c
định mt khong
1
2
AH
không đổi nên điểm
M
nằm trên đường thng
//xy BC
cách
BC
mt khong
1
2
AH
(
xy
nm trên na mt phng b
BC
có cha
A
).
Gii hn: Khi điểm
D
đi động ti điểm
B
thì điểm
M
đi động tới trung đim
P
ca
AB
. Khi điểm
D
di động tới điểm
C
thì điểm
M
di động tới điểm
Q
ca
AC
. Vy đim
M
ch nằm trên đường trung
bình
PQ
ca
ABC
.
b) Phần đảo
Lấy điểm
M
bt k trên đoạn
PQ
. V tia AM ct BC ti D. V
DE
//
AB
,
DF
//
AE
, E
AC F AB
. Ta phi chng minh
M
là trung điểm ca
EF
.
c) Kết lun
Vy qu tích cua điểm
M
là đường trung bình
PQ
ca
ABC
.
Nhn xét: Đim
M
trung điểm ca
EF
. Đây tính chất ban đầu của đim
M
, chưa phải tính
chất bản theo qu tích
1
, 2 , 3 , 4
. Do đó chưa thể vn dụng để tr lời điểm
M
nm trên hình
nào.
Ta đã giải quyết vấn đề này bng cách biến đổi tính chất ban đầu của điểm
M
lần lượt như sau:
M
là trung điểm ca
EF
(tính chất ban đầu)
M
là trung điểm ca
AD
(tính cht T)
M
cách đường thng
BC
c định mt khoảng không đổi
1
2
AH
(đây mi tính chất bản ca
điểm
M
)
M
nằm trên đường thng
//xy BC
và cách
BC
mt khong
1
2
AH
.
K
D
H
E
F
Q
M
P
C
B
A
y
x
48
Như vậy ta phi chuyn tính chất ban đầu của điểm
M
qua các tính chất trung gian đến tính chất cơ bản
của điểm
M
ri theo các qu tích cơ bản tr lời điểm
M
nm trên hình nào.
d 2. Cho góc vuông
xOy
, điểm
A
c định trên tia
Ox
, điểm
B
di động trên tia
Oy
. V hình ch
nht
AOBC
. Gi
M
là giao điểm của hai đường chéo
AB
OC
. Tìm qu tích điểm
M
.
Gii (h.9.2)
a) Phn thun
M
là giao điểm của hai đường chéo hình ch nht
nên
MO MA
.
Đim
M
cách đề hai đầu mút của đoạn thng
OA
c định nên điểm
M
nằm trên đường trung trc
của đoạn thng
OA
.
.9.2nh
Gii hn: Khi điểm
B
tiến dn tới điểm
O
thì điểm
C
tiến dn tới điểm
A
khi đó điểm
M
tiến dn tới điểm
1
M
trung điểm ca
OA
. Khi điểm
B
ra xa
tận thì điểm
M
cũng ra xa tận. Vy
M
nm trên tia
1
Mt
thuộc đường trung trc ca
OA
, tia này
nm trong góc
xOy
, tr điểm
1
M
.
b) Phần đảo
Lấy điểm
M
bt k trên tia
1
Mt
. V tia
AM
ct tia
Oy
ti
B
. V hình ch nht
AOBC
. Ta phi chng
minh
M
là giao điểm của hai đường chéo.
Tht vy xét
AOB
1
//M t OB
(vì cùng vuông góc vi
OA
).
Mt khác,
11
M O M A
, nên
MA MB
. Vy
M
là trung điểm ca
AB
.
M
cũng là trung điểm ca
OC
(vì
AOBC
là hình ch nht).
Vy
M
là giao điểm của hai đường chéo.
c) Kết lun
Vy qu tích của điểm
M
tia
1
Mt
thuộc đường trung trc ca
OA
, tia này nm trong góc
xOy
, tr
điểm
1
M
.
Ví d 3: Cho góc vuông
xOy
. Điểm
A
có đỉnh trên tia
Ox
sao cho
2OA cm
. Điểm
B
di động trên tia
Oy
. V Tam giác
ABM
vuông cân ti
M
trong đó
M
O
thuc hai na mt phẳng đối nhau b
AB
.
Tìm qu tích điểm
M
.
Gii (h.9.3)
a) Phn thun
V
,MH Ox MK Oy
ta được
0
90HMK
Mt khác ,
0
90AMB
nên
HMA KMB
(hai góc có cạnh tương ứng cùng nhn)
HMA KMB
(cnh huyn góc nhn).
t
y
x
M
C
A
B
O
1
M
B
K
A
H
M
O
y
t
x
49
Suy ra
MH MK
Đim
M
nm trong góc
xOy
và cách đề hai cnh của góc đó
nên điểm
M
nm trên tia phân giác ca
xOy
.
.9.3nh
Gii hn: Khi điểm
B
trùng với đim
O
thì điểm
M
trùng với điểm
1
M
1
M
nm trên tia
Ot
1
2)OM cm
. Khi điểm
B
ra xa vô cùng thì điểm
M
ra xa vô cùng. Vy
M
nm trên tia
1
Mt
.
b) Phần đảo
Lấy điểm
M
bt k trên tia
1
Mt
. T điểm
M
v một đường thng vuông góc vi
AM
ct tia
Oy
ti
B
.
Ta phi chng minh
AMB
vuông cân ti
M
.
Tht vy , v
,MH Ox MK Oy
ta có:
MH MK
0
90HMK
HMA KMB
(hai góc có cnh
tương ứng vuông góc cùng nhn)
Do đó:
..H
MA KMB g c g
MA MB
AMB
vuông ti
M
MA MB
nên là tam giác vuông cân.
c) Kết lun
Vy qu tích của điểm
M
là tia
1
Mt
là tia phân giác ca góc
xOy
.
d 4: Cho hình bình hành
ABCD
cnh
AB
c định,
2BC cm
. Tìm qu tích điểm giao đim
O
ca
hai đường chéo.
Gii (h.9.4)
a) Phn thun
Gi
M
là trung điểm ca
AB
.
Do
AB
c định nên
M
là điểm c định.
O
giao điểm hai đường chéo ca hình bình hành
nên
OA OC
. Vy
OM
là đường trung bình
ca
ABC
1
1
2
OM BC cm
.9.4nh
Đim
O
cách điểm
M
c định mt khong
1cm
nên điểm
O
nằm trên đường tròn tâm
,M
bán kính
1cm
.
Gii hn: Vì ba điểm
,,O A B
không thẳng hàng nên điểm
O
nằm trên đường tròn tâm
M
bán kính
1.cm
b) Phần đảo
Lấy điểm
O
bt k trên đường tròn tâm
M
, bán kính
1cm
thì
1OM cm
. V điểm
C
đối xng vi
A
qua
O
, v điểm
D
đối xng vi
B
. Ta phi chng minh t giác
ABCD
là hình bình hành và
2.BC cm
Tht vy, t giác
ABCD
hai đường chéo ct nhau ti trung điểm ca mỗi đường nên hình bình
hành.
OM
là đường trung bình ca
ABC
nên
1
2.1 2
2
OM BC BO cm
c) Kết lun
M
O
D
C
B
A
50
Vy qu tích ca điểm
O
đường tròn tâm
M
, bán kính
1cm
tr giao điểm ca đường tròn này vi
đường thng
AB
.
C. Bài tp vn dng
*) Đƣờng thng song song
9.1 Cho hai đường thng
a
b
song song vi nhau cách nhau
2cm
. Tìm qu tích những điểm
M
có tng khoảng cách đến
a
b
4cm
.
9.2 Cho góc vuông
xOy
một điểm
A
c định trên tia
Ox
sao cho
OA a
. Đim
B
di động trên tia
Oy
. V vào trong góc vuông này tam giác
ABC
vuông cân ti
A
. Tìm qu tích điểm
C
.
9.3 Cho đoạn thng
AB
một điểm
C
nm gia
A
B
. Trên cùng mt na mt phng b
AB
v
các tam giác
DAC
EBC
vuông cân ti
D
E
. Gi
M
trung điểm ca
DE
. Tìm qu tích ca
điểm
M
khi điểm
C
di động gia
A
B
.
9.4 Cho đoạn thng
AB
một điểm C nm gia
A
B
. V các tam giác đều
DAC
EBC
trên
cùng mt na mt phng b
AB
. Gi
M
là trung điểm ca
DE
. Tìm qu tích của điểm
M
khi điểm
C
di động gia
A
B
.
9.5 Cho tam giác
ABC
cân ti
A
. Một điểm
D
di động trên đáy
BC
. Đường thng vuông góc vi
BC
v t
D
cắt các đường thng
AB
,
AC
lần t ti
E
F
. Gi
M
trung điểm ca
EF
. Tìm qu
tích của điểm
M
.
*) Đƣờng trung trực và đƣờng thng vuông góc
9.5 Cho góc vuông
xOy
một điểm
A
trong góc đó. Một góc vuông đỉnh
A
quay quanh
A
, mt
cnh ct
Ox
ti
B
, cnh kia ct
Oy
ti
C
. Gi
M
là trung điểm ca
BC
. Tìm qu tích của điểm
M
.
9.7 Cho hình ch nht
ABCD
. Gi
M
là một điểm trong hình ch nht hoc trên các cnh ca nó.
1) Chng minh rng:
2 2 2 2
;MA MC MB MD
2) Tìm qu tích điểm
M
nếu
MA MC MB MD
9.8 Cho tam giác đều
ABC
. Trên na mt phng b
BC
cha
A
v tia
Bx BC
trên đó lấy mt
điểm
D
. V tam giác đều
CDM
M
B
thuc hai na mt phẳng đối nhau b
CD
. Tìm qu tích
của điểm
M
khi
D
di động trên tia
Bx
.
* Tia phân giác
9.9. Cho hình vuông
ABCD
. Trên tia đối ca tia
AD
lấy điểm
E
di động. Trên tia đối ca tia
BA
lấy điểm
F
di động sao cho
DE BF
. V hình bình hành
. Hỏi điểm
M
di động trên đường nào.
9.10. Cho tam giác
ABC
vuông ti
A
. Gi
D
E
lần lượt là các điểm di dng trên hai cnh
AB
BC
sao
cho
BD BE
. T
E
v một đường thng vuông góc vi
DE
ct
AC
ti
F
. Gi
M
là trung điểm ca
DF
.
Tìm qu tích của điểm
M
.
9.11. Cho góc
xOy
có s đo bằng
60
. Mt hình thoi
ABCD
có cnh bng
a
;
60B
, đỉnh
B
di động trên
tia
Ox
, đỉnh
D
di động trên tia
Oy
, hai điểm
A
O
thuc hai na mt phẳng đối nhau b
BD
. Tìm qu tích
của điểm
A
.
* Đƣng tròn
51
9.12. Cho hình vuông
ABCD
cnh
4cm
. Tia
Ox
nm gia hai tia
DA
DC
. V tia phân giác ca góc
ADx
ct
AB
ti
E
, tia phân giác ca góc
CDx
ct
BC
ti
F
. Tia
Dx
ct
EF
ti
M
. Hi khi tia
Dx
quay quanh
D
t v trí
DA
đến v trí
DC
thì điểm
M
di động trên đường nào?
9.13. Cho góc vuông
xOy
. Một đoạn thng
2AB a
không đổi, có
A Ox
B Oy
. Tìm qu tích trung
điểm
M
ca
AB
.
9.14. Cho hình bình hành
ABCD
cnh
CD
c định,
2cmAC
. Tìm qu tích của đỉnh
B
.
1
MC LC
CHUYÊN ĐỀ 10. ĐA GIÁC – ĐAC GIÁC ĐU ............................................................................................................... 2
CHUYÊN ĐỀ 11. DIỆN TÍCH ĐA GIÁC .......................................................................................................................... 7
CHUYÊN ĐỀ 12. PHƯƠNG PHÁP DIỆN TÍCH ............................................................................................................ 15
2
CHƯƠNG II. ĐA GIÁC – DIỆN TÍCH ĐA GIÁC
CHUYÊN ĐỀ 10. ĐA GIÁC – ĐAC GIÁC ĐỀU
A. Kiến thc cn nh
1. Đa giác lồi là đa giác luôn nm trong mt na mt phng có b là đường thng cha bt kì cnh nào của đa
giác đó.
2. Đa giác đều là đa giác có tất c các cnh bng nhau và tt c các góc bng nhau.
3. B sung
- Tng các góc trong của đa giác
n
cnh
2n
2
.180n
.
- S đường chéo ca một đa giác
n
cnh
2n
3
2
nn
.
- Tng các góc ngoài của đa giác
n
cnh
2n
360
(ti mỗi đỉnh ch chn mt góc ngoài).
- Trong một đa giác đều, giao điểm
O
của hai đường phân giác ca hai góc k mt cnh là tâm của đa giác đều.
Tâm
O
cách đều các đỉnh, cách đều các cnh của đa giác đều. Có một đường tròn tâm
O
đi qua các đỉnh ca
đa giác đều gọi là đường tròn ngoi tiếp đa giác đều.
B. Mt s ví d
Ví d 1. Tìm s cnh ca một đa giác biết s đường chéo hơn số cnh là
7
.
Li gii
* Tìm cách gii. Bài này biết mi liên h gia s đường chéo và s cnh nên hiển nhiên chúng ta đặt s cnh
của đa giác là
n
biu th s đường chéo là
3
2
nn
t đó ta tìm được s cnh.
* Trình bày li gii
Đặt s cnh của đa giác là
n
3n
thì s đường chéo là
3
2
nn
theo đề bài ta có
3
7
2

nn
n
2
5 14 0 nn
2
7 0 nn
.
3n
nên
70n
7n
. Vy s cnh của đa giác là
7
.
Ví d 2. Tng tt c các góc tng và mt góc ngoài ca một đa giác có số đo là
47058,5
. Hỏi đa giác đó có
bao nhiêu cnh?
Gii
* Tìm cách gii. Nếu ta đặt
n
là s cnh,
là s đo một góc ngoài của đa giác thì
0 180
2
.180n
là mt s nguyên.
Do đó suy ra
2
.180 47058,5
n
, t đó ta có
là s dư của
47058,5
chia cho
180
. Bng cách suy
luận như vậy, chúng ta có li gii sau:
* Trình bày li gii
Gi
n
là s cnh của đa giác (
n
,
3n
).
Tng s đo các góc trong của đa giác bằng
2
.180n
.
Vì tng các góc trong và mt trong các góc ngoài của đa giác có số đo là
47058,5
nên ta có
2
.180 47058,5
n
(
là s đo một góc ngoài của đa giác với
0 180
)
2
.180 261.180 78,5
n
2 261 n
263n
.
Vy s cnh của đa giác là
263
.
3
Ví d 3. Tng s đo các góc của một đa giác
n
- cnh tr đi góc
A
ca nó bng
570
. Tính s cnh của đa
giác đó và
A
.
Gii
* Tìm cách gii. Theo công thc tính tng các góc trong, ta có
2
.180 570 nA
. Quan sát và nhìn nhn,
ta có th nhn thy ch có thêm điều kin là
n
,
3n
0 180 A
. T đó ta có lời gii sau
* Trình bày li gii
Ta có
2
.180 570 nA
2
.180 570 An
.
0 180 A
0
2 .180 570 180 n
5
70 2 .180 750 n
19 25
2
66
n
11
56
66
n
. Vì
n
nên
6n
.
Đa giác đó có
6
cnh và
6
2 .180 570 150 A
.
Ví d 4. Mt lục giác đều và một ngũ giác đều chung cnh
AD
(như hình vẽ). Tính các góc ca tam giác
ABC
.
Gii
* Tìm cách gii.
AD
là cnh ca lục giác đều và ngũ giác đều, nên d dàng nhn ra
ABD
,
ACD
,
BCD
là các tam giác cân đỉnh
D
và tính được s đo các góc ở đỉnh. Do vy
ABC
s tính được s đo các góc.
* Trình bày li gii
Theo công thc tính góc của đa giác đều, ta có
6 2 .180
120
6

ADB
30 DAB DBA
.
5 2 .180
108
5

ADC
36 DAC DCA
;
Suy ra
360 120 180 132 BDC
.
Ta có
BDC
(
DB DC
) cân ti
D
. Do đó
180 132
24
2
DBC DCB
.
Suy ra
30 36 66 BAC
;
30 24 54 ABC
;
24 36 60 BCA
.
A
D
B
C
4
Ví d 5. Cho lục giác đều
ABCDEF
. Gi
M
,
L
,
K
lần lượt là trung điểm ca
EF
,
DE
,
CD
. Gọi giao điểm
ca
AK
vi
BL
CM
lần lượt là
P
,
Q
. Gọi giao điểm ca
CM
BL
R
. Chng minh tam giác
PQR
là tam giác đều.
Gii
Các t giác
ABCK
,
BCDL
,
CDEM
có các cạnh và các góc đôi một bng nhau. Các góc ca lục giác đều bng
120
.
Đặt
BAK
CBL DCM
;
L
BA
CKA EMC DLB
120

.
Trong tam giác
CKQ
180

CQK
60 CQK
.
Trong tam giác
PBA
180

APB
60 APB
.
T đó suy ra
60 RQP RPQ
. Vy
PQR
đều.
Ví d 6. Cho bát giác
ABCDEFGH
có tt c các góc bằng nhau, và độ dài các cnh là s nguyên. Chng minh
rng các cạnh đối din ca bát giác bng nhau.
Gii
Các góc ca bát giác bng nhau, suy ra s đo của mi góc là
8 2 .180
135
8


.
Kéo dài các cnh
AH
BC
ct nhau ti
M
. Ta có:
180 135 45 MAB MBA
, suy ra tam giác
MAB
là tam giác vuông cân.
Tương tự các tam giác
CND
,
EBF
,
GQH
cũng là các tam giác vuông cân, suy ra
MNPQ
là hình ch nht.
Q
R
P
M
L
K
C
D
E
F
B
A
f
c
e
g
a
h
d
b
C
E
D
B
N
Q
P
M
A
H
G
F
5
Đặt
AB a
,
BC b
,
CD c
,
DE d
,
EF e
,
FG f
,
GH g
,
HA h
. T các tam giác vuông cân, theo
định lý Py ta go ta có:
2
a
MB
,
2
c
CN
nên
22
ac
MN b
tương tự
22
eg
PQ f
.
Do
MN PQ
nên
2 2 2 2
a c e g
bf
1
2
a c e g f b
.
Do
f
,
b
là các s nguyên nên vế phi của đẳng thc trên là s nguyên, do đó vế trái là s nguyên. Vế trai ch
có th bng
0
tc là
fb
hay
BC FG
. Tương tự ta có
AB EF
,
CD GH
,
DE HA
.
Nhn xét. Da vào tính cht s hu t, s vô t chúng ta đã giải được bài toán trên. Cũng với k thuật đó,
chúng ta có th giải được bài thi hay và khó sau: Cho hình ch nht
ABCD
. Ly
E
,
F
thuc các cnh
AB
;
G
,
H
thuc cnh
BC
;
I
,
J
thuc cnh
CD
;
K
,
M
thuc cnh
DA
sao cho hình
8
- giác
EFGHIJKM
có các
góc
bng nhau. Chng minh rng nếu độ dài các cnh ca hình 8 giác
EFGHIJKM
là các s hu t thì
EF IJ
(Tuyn sinh lp 10, THPT chuyên tỉnh Hưng Yên, năm học 2009-2010)
C. Bài tp vn dng
10.1. S đường chéo ca một đa giác lớn hơn
14
, nhưng nhỏ hơn
27
. Hỏi đa giác có bao nhiêu cạnh?
10.2. Tng s đo các góc của một đa giác
n
cnh tr đi góc A của nó bng
2570
. Tính s cnh của đa giác
đó và
A
.
10.3. Cho
ABC
có ba góc nhn và
M
là điểm bt kì nm trong tam giác. Gi
1 1 1
;;A B C
là các điểm đối xng
vi
M
lần lượt qua trung điểm ca các cnh
,,BC CA AB
.
a) Chứng minh các đoạn
1 1 1
;;AA BB CC
cùng đi qua một điểm.
b) Xác định v trí điểm
M
để lc giác
1 1 1
ABCABC
có các cnh bng nhau.
10.4. Một ngũ giác đều có
5
đường chéo và nhóm
5
đường chéo này ch có mt loại độ dài (ta gi mt loại độ
dài là một nhóm các đường chéo bng nhau). Mt lục giác đều có
9
đường chéo và nhóm
9
đường chéo này
hai loại độ dài khác nhau (hình v).
10.5. Cho ngũ giác lồi
ABCDE
có tt c các cnh bng nhau và
2ABC DBE
. Hãy tính
ABC
.
10.6. Cho ngũ giác lồi
ABCDE
có các cnh bng nhau và
A B C
a) Chng minh t giác
ABCD
là hình thang cân;
b) chứng minh ngũ giác
ABCDEF
là ngũ giác đều.
10.7. Cho ngũ giác
ABCDE
, gi
, , ,M N P Q
lần lượt là trung điểm ca các cnh
AB
,
BC
,
CD
,
EA
I
,
J
lần lượt là trung điểm ca
,MP NQ
. Chng minh rng
IJ
song song vi
ED
4
ED
IJ
.
6
ABCDEF
, , , , ,A B C D E F
AB
10.8. Cho lục giác đu .Gi lần lượt là trung đim ca các cnh ,
, , , ,BC CD DE EF FA
. Chng minh rng
A B C D E F
là lục giác đều.
10.9. Cho lc giác li
ABCDEF
có các cp cạnh đối
AB
DE
;
BC
EF
;
CD
AE
va song song va
bng nhau. Lc giác
ABCDEF
có nht thiết là lục giác đều hay không?
10.10. Chng minh rng trong bốn ngũ giác lồi bất kì luôn tìm được ba đường chéo có độ dài là ba cnh ca
mt tam giác.
10.11. Chng minh rng tổng độ dài các cnh ca một ngũ giác lồi bé hơn tổng độ dài các đường chéo ca nó.
10.12. Mun ph kín mt phng bi những đa giác đều bằng nhau sao cho hai đa giác đều nhau thì có chung
mt cnh. Hỏi các đa giác đều này có th nhiu nht bao nhiêu cnh?
10.13. Cho lc giác có tht c các góc bng nhau, các cạnh đối không bng nhau. Chng minh rng
BC EF DE AB AF CD
. Ngược li nếu có 6 đoạn thng thỏa mãn điều kin ba hiu trên bng nhau
và khác 0 thì chúng có th lập được mt lc giác có các góc bng nhau.
10.14. Chng minh rng trong mt lc giác bất kì, luôn tìm được một đỉnh sao cho ba đường chéo xut phát t
đỉnh đó có thể ly làm ba cnh ca mt tam giác.
10.15. Cho lc giác
ABCDEG
có tt c các cnh bng nhau
A C E B D G
. Chng minh rng các cp
cạnh đối ca lc giác song song vi nhau.
7
CHUYÊN ĐỀ 11. DIỆN TÍCH ĐA GIÁC
A. Kiến thc cn nh
1. Mỗi đa giác có một diện tích xác định. Diện tích đa giác là một s dương có các tính chất sau:
- Hai tam giác bng nhau thì có din tích bng nhau.
- Nếu một đa giác được chia thành những đa giác không có điểm trong chúng thì din tích ca nó bng tng
din tích ca những đa giác đó.
- Hình vuông có độ dài bng
1
thì có din tích là
1
.
2. Các công thc tính diện tích đa giác
- Din tích hình ch nht bằng tích hai kích thước ca nó
.S a b
(
,ab
là kích thước hình ch nht).
- Din tích hình vuông bằng bình phương cạnh ca nó
2
Sa
(
a
là độ dài cnh hình vuông).
- Diện tích hình vuông có đường chéo dài bng
d
2
1
2
d
.
- Din tích tam giác vuông bng na tích hai cnh góc vuông
1
.
2
S a b
(
,ab
là độ dài hai cnh góc vuông).
- Din tích hình tam giác bng na tích ca mt cnh vi chiu cao ng vi cạnh đó
1
.
2
S a h
(
,ah
là độ dài
cnh và chiều cao tương ứng)
- Din tích hình thang bng na tích ca tổng hai đáy với chiu cao:
1
.
2
S a b h
(
,ab
là độ dài hai đáy,
h
là độ dài đường cao)
- Din tích hình bình hành bng na tích ca mt cnh vi chiu cao ng vi cạnh đó
.S a h
(
,ah
là độ dài
mt cạnh và đường cao tương ứng)
- Din tích t giác có hai đường chéo vuông góc bng nửa tích hai đường chéo
12
1
.
2
S d d
(
12
;dd
là độ dài hai
đường chéo tương ứng)
- Din tích hình thoi bng nửa tích tích hai đường chéo
12
1
.
2
S d d
(
12
;dd
là độ dài hai đường chéo tương
ng)
3. B sung
- Hai tam giác có chung mt cnh (hoc mt cp cnh bng nhau) thì t s din tích bng t s hai đường cao
ng vi cạnh đó).
- Hai tam giác có chung một đường cao (hoc mt cặp đường cao bng nhau) thì t s din tích bng t s hai
cnh ng với đường cao đó.
-
ABCD
là hình thang
//A
B CD
. Hai đường chéo
AC
BD
ct nhau ti
O
thì
AOD BOC
SS
.
- Trong các hình ch nht có cùng chu vi thì hình vuông có din tích ln nht.
- Hai hình ch nht có cùng chiu cao thì t s din tích bng t s hai đáy.
- Tam giác đều cnh
a
có din tích là
2
3
4
a
.
B. Mt s ví d
Ví d 1. Cho hình ch nht
ABCD
12AB cm
,
6,8AD cm
. Gi
H
,
I
,
E
,
K
là các trung điểm tương
ng ca
BC
,
HC
,
DC
,
EC
.
a) Tính din tích tam giác
DBE
.
b) Tính din tích tam giác
EHIK
.
8
Gii
* Tìm cách gii. D dàng tính được din tích hình ch nht
ABCD
. Mặt khác, đề bài xut hin nhiu yếu t
trung điểm nên chúng ta có th vn dng tính chất: hai tam giác có chung đường cao thì t s din tích bng t s
hai cạnh đáy ứng với đường cao đó. T đó rút ra nhận xét: đường trung tuyến ca tam giác chia tam giác y
thành hai phn có din tích bng nhau.
T nhn xét quan trọng đó, chúng ta lần lượt tính đượ din tích các tam giác
BCD
,
BCE
,
DBE
,
BEH
,
ECH
,
HCK
,
CKI
,…
* Trình bày li gii
a)
ABCD
là hình ch nht nên
2
1 1 1
. . .12.6,8 40
2 2 2
D CDB BC A
S S AB AD cm
E
là trung điểm ca
CD
, suy ra:
2
1
20,4
2
BD EE BC BCD
S S S cm
b)
H
là trung điểm
2
11
.20,4 10,2
22
CHE BCE
BC S S cm
K
là trung điểm
2
1
5,1
2
HKC CHE
CE S S cm
I
là trung điểm
CH
2
1
2,55
2
CKI HKC
S S cm
Vy
2
10,2 2,55 7,65
EHIK CHE CIK
S S S cm
.
Ví d 2. Cho hình ch nht
ABCD
có din tích
2
24cm
. Lấy điểm
E
thuc
BC
F
thuc
CD
sao cho
din tích tam giác
ABE
ADF
lần lượt là
2
4cm
2
9cm
. Tính din tích tam giác
AEF
.
(Olypic Toán, Châu Á- Thái Bình Dương, năm 2001)
Gii
* Tìm cách gii. Quan sát hình v, suy lun rt t nhiên: mun tính din tích tam giác
AEF
chúng ta ch cn
tính din tích tam giác
CEF
.
Nhn thy không th và cũng không cần tính c th độ dài
CE
CF
. Chúng ta biết rng, nên ch cn tìm mi
quan h gia
CE
BC
;
CF
CD
. Phân tích như vậy, chúng ta ch cn tìm mi quan h gia
BE
BC
;
DF
CD
.
Mt khác hình ch nht
ABCD
và tam giác vuông
ABE
có chung cnh
AB
đồng thi biết din tích ca chúng
nên d dàng tìm được mi quan h gia
BE
BC
. Tương tự như vậy hình ch nht
ABCD
và tam giác
vuông
ADF
có chung cạnh AD, đồng thi biết din tích ca chúng nên d dàng tìm được mi quan h gia
DF
CD
. T đó ta có lời gii sau:
* Trình bày li gii
Ta có:
2
24
ABCD
S cm
suy ra:
2
1
12
2
ABC ACD ABCD
S S S cm
ABC
ABE
có chung đường cao
AB
nên
4
12
ABE
ABC
S
BE
BC S

K
E
I
H
C
A
B
D
C
A
B
D
E
F
9
hay
12
33
BE CE
BC BC
ADF
ADC
có chung đường cao
AD
nên
9
12
ABE
ABC
S
DF
DC S

hay
31
44
DF CF
DC CD
.
Ta có:
2
. 1 2 1 1 1 1
. . . .24 2
2. . 2 3 4 12 12 12
CEF
CEF ABCD
ABCD
S
CE CF
S S cm
S BC CD
.
Do vy
2
24 4 9 2 9
AEF ABCD ABE ADF CEF AEF
S S S S S S cm
.
Ví d 3. Cho hình thang cân
ABCD
(
AB
//
CD
). Biết
7BD cm
;
45ABD 
. Tính din tích hình thang
ABCD
.
(Olympic Toán Châu Á Thái Bình Dương 2007)
Gii
Cách 1. Ni
AC
ct
BD
ti
E
.
ABE
vuông cân
BE AC
. Din tích hình thang là:
22
1 1 49
.
2 2 2
S AC BD BD cm
Cách 2. Kéo dài tia BA lấy điểm E sao cho
AE CD
, ta được:
AED CDB
(c.g.c) suy ra:
45AED CDB
. T đó suy ra:
BDE
vuông cân ti
D
.
ABCD ABD CDB ABD AED
S S S S S
22
1 49
22
DBE
S BD cm
.
10
Cách 3. K
DH AB
,
BK CD
. Do
AB
//
CD
nên
90HDK 
DB
là phân giác
HDK
(vì
45BDK 
)
HDKB
là hình vuông mà
HAD KCB
(cnh huyn góc nhn)
suy ra
HDA BCK
SS
nên
ABCD ABKD CKB ABKD AHD DHBK
S S S S S S
2
22
49
22
BD
BK cm
.
Ví d 3. Cho
ABC
vuông ti
A
.
AH
là đường cao. Gi
M
,
N
là hình chiếu ca
H
trên
AB
,
AC
. Gi
I
là giao điểm ca
BN
CM
. Chng minh:
BIC AMIN
SS
.
Gii
Ta có:
ANH
BNH
có chung
HN
và đường cao h t
A
B
bng nhau nên
ANH BNH ANH CNH BNH CNH
S S S S S S
AHC BNC
SS
(1)
Mt khác
MA HN
nên
AHC AMC
SS
(2)
T (1) và (2) ta có:
BNC AMC BNC NIC AMC NIC
S S S S S S
Vy
BIC AMIN
SS
.
Nhn xét.
Kĩ thuật so sánh
BIC
S
vi
AMIN
S
ta so sánh
BNC
S
vi
AHC
S
t đó dẫn đến so sánh
BHN
S
AHN
S
.
Ví d 4. Gi
M
,
N
lần lượt là trung điểm các cnh
BC
CD
ca t giác li
ABCD
. Chng minh rng:
2
1
.
2
ABCD
S AM AN
.
Gii
* Tìm cách gii. Nhn thy vế phi ca phn kết luận có độ dài hai cnh ca tam giác
AMN
, mt khác d thy
2
11
. . .
2 2 4
AMN
AM AN
S AM AN

(vn dng kết qu
2
4
ab
ab
). Do vy chúng ta cn biến đổi
ABCD
S
theo
AMN
S
. Định hướng cui cùng là
4.
ABCD AMN
SS
.
* Trình bày li gii
11
2. 2.
ABCD ABC ACD AMC ANC
S S S S S
2
. 2. 2. 2.
AMC ANC AMCN AMN CMN
S S S S S
Gọi giao điểm
AM
BD
I
CMN IMN AMN
S S S
2
1
4. 4. . . 2. . 2.
24
ABCD AMN
AM AN
S S AM NH AM AN
.
Suy ra
2
1
.
2
ABCD
S AM AN
.
Ví d 5. Cho tam giác
ABC
vi
D
là điểm thuc cnh
BC
F
là điểm thuc cnh
AB
. Điểm
K
đối xng
với điểm
B
qua
DF
. Biết rng
K
,
B
nm khác phía so vi
AC
. Cnh
AC
ct
FK
ti
P
DK
ti
Q
.
Tng din tích ca các tam giác ,
PKQ
và QDC là
2
10cm
. Nếu ta cng tng din tích này vi din tích t giác
DFPQ thì bng
2
3
din tích tam giác ABC. Tính din tích tam giác ABC theo
2
cm
.
(Olympic Toán hc Tr Quc tế ti Hàn Quc KIMC 2014 (Malaysia đề ngh) )
Gii
Ta có:
D
FPQ ABC BFD APF CDQ ABC DKF APF CDQ
S S S S S S S S S
A
BC DFPQ KPQ APF CDQ
S S S S S
21
33
ABC ABC ABC
S S S
1
3
KPQ APF CDQ ABC
S S S S
2
3.10 30
ABC
S cm
.
Ví d 6: Chín đường thng có cùng tính cht là mỗi đường thng chia hình vuông thành hai t giác có t s
bng
2
3
. Chng minh rng tn ti ít nhất ba đường thng trong s đó cùng đi qua một đim.
(Thi vô địch CHLB Nga năm 1972)
Gii
* Tìm cách gii: Chng minh tn ti ít nhất ba đường thng trong s đó cùng đi qua một điểm, mà không ch ra
được c th ờng minh đó là điểm nào, chúng ta liên tưởng ti kh năng vận dng nguyên lý DDirrichle. Trong
12
trường hp này, chúng ta cn ch ra 9 đường thng mà mỗi đường thng phải đi qua ít nhất 1 trong 4 điểm c
định nào. T đó nếu mỗi điểm có nhiu nht ch có 2 đường thẳng đi qua thì nhiều nht ch có 4.2 = 8 đường
thng (nh hơn 9). Vô lý.
* Trình bày li gii:
Các đường thẳng đã cho không thể ct các cnh k nhau ca hình vuông
ABCD. Bowie vì nếu thế không th to ra hai t giác mà là tam giác và ngũ
giác.
Gi s một đường thng ct các cnh BC AD tại các điểm M N. Các
hình thang ABMN CDMN các đường cao bằng nhau do đó t s din
ch ca chúng bng t s các đường trung bình. Tức là MN chia đon thng
nối trung điểm ca các cnh AB
và CD theo t s
2
3
. Tng s các điểm chia các đường trung bình ca hình vuông theo t s
2
3
là 4.
Bi s đường thẳng đã cho là 9 và đều phải đi qua một trong s bốn điểm nói trên, nên có một điểm thuc ít
nhất 3 đường thng . Tc là có ít nhất ba đường thng trong s đó cùng đi qua một điểm.
Ví d 7: Bên trong hình vuông có cnh bằng 10 có 1000 điểm , không có ba điểm nào thng hàng. Chng minh
rng trong s các tam giác có đỉnh là các điểm đó hoặc các đỉnh hình vuông , tn ti mt tam giác có din tích
không quá
50
1001
.
Gii
* Tìm cách gii: Nhn thy rng hình vuông có din tích
10.10 100
. Suy lun mt cách t nhiên, chúng ta
nghĩ một cách t 1000 điểm và 4 đỉnh ni với nhau như thế nào để tạo thành các tam giác không có điểm chung
trong. Khi đó tổng din tích các tam giác to thành có din tích bng 100. Chúng ta s lp lun din tích nh
nht ca mt tam giác to thành tha mãn yêu cầu đề bài.
* Trình bày li gii
Gọi 1000 điểm trong hình vuông cnh bng 10 là
1 2 1000
, ,....,A A A
.
c th nht, ta ni
1
A
với các đnh của hình vuông, ta đưc bn tam giác.
Xét điểm
k
A
vi
2,3,4,....,1000.k
Nếu
k
A
nm trong một tam giác đã to ra
(chng hn
2
A
hình v), ta ni
2
A
với ba đỉnh của tam giác đó, số tam giác
tăng thêm hai (từ 1 thành 3), nếu
k
A
thuc mt cnh chung ca hai tam giác to
ra (chng hn
3
A
hình v), ta ni
3
A
với các đỉnh đối din vi cnh chung, s
tam giác cũng tăng thêm hai (từ 2 thành 4)
Như vậy, sau bước th nhất ta được tam giác. Trong 999 bước còn li, mỗi bước tăng thêm hai tam giác. Tổng
cng ta có:
4 2.999 2002
tam giác
Tng din tích của 2002 tam giác đó bằng 100. Do đó tồn ti mt tam giác có din tích không quá
100 50
2002 1001
Nhn xét. T cách gii trên, chúng ta có th giải được bài toán tng quát sau:
- Bên trong mt hình vuông cạnh a cho n đim. Chng minh rng trong s các tam giác đnh
các điểm đó hoặc các đỉnh hình vuông, tn ti mt tam giác có din tích không quá
2
22
a
n
.
13
- Bên trong một đa giác li n cnh din tích S lấy m đim. Chng minh rng trong s các tam giác
có đỉnh là các điểm đó hoặc các đỉnh đa giác, tồn ti mt tam giác có din tích không quá
22
S
mn
.
d 8: chng minh rng hai hình ch nhật cùng kích thước
ab
được xếp sao cho chúng ct nhau tại 8 điểm thì din tích
phn chung ln hn na din tích mt hình ch nht.
Gii
V CM, CN (như hình vẽ)
CM CN
.
Suy ra CA tia phân giác góc MAN góc MCN. Chng minh
tương tự, ta có: BD là phân giác ca
EBF
.
Da vào cp góc cạnh tương ng vuông góc, ta có:
MAN EBF
nên
CAE DBF
.
T đó suy ra AC BD.
Do đó
1
.
2
ABCD
S AC BD
111
. . .
222
AEBFCIDH ABCD
S S AC BD CN BD a b
Nhn xét: S dng k thut của chuyên đề tam giác đồng dng. Các bn th giải được bài toán sau: Cho hai
hình ch nhật cùng kích thước
ab
. Mt hình ch nht các cạnh màu đỏ, mt hình ch nht các cnh
màu xanh, được xếp sao cho chúng ct nhau tại 8 điểm. Chng minh rng hình bát giác tng các cnh màu
đỏ bng tng các cnh tô màu xanh.
C. Bài tp vn dng
11.1. Cho hình ch nht ABCD CD = 4cm, BC = 3cm. Gi H hình chiếu ca C trên BD. Tính din tích
tam giác ADH.
11.2. Cho hình thang ABCD độ dài hai đáy là AB = 5cm, CD = 15cm, độ dài hai đường chéo AC =16cm,
BD =12cm. Tính din tích hình thang ABCD.
11.3. Cho tam giác ABC vuông cân tại A. Các đim D, E theo th t di chuyn trên AB, AC sao cho BD = AE.
Xác định v trí điểm D, E sao cho t giác BDEC có din tích nh nht.
11.4. Cho tam giác ABC din ch S, trên cnh AB lấy điểm D sao cho AD = 2BD. Gọi E là trung điểm
của AC và I là giao điểm CD và BE. Tính đường tròn tam giác IBC.
11.5. Cho t giác li ABCD. Qua trung điểm K của đường chéo BD dựng đường thng song song với đường
chéo AC, đưng y ct AD ti E. Chng minh rng CE chia t giác thành hai phn din tích bng nhau
(biết E nm gia A và D).
11.6. Cho hình bình hành ABCD. Trên cnh Ab CD lần lượt ly các điểm M K sao cho AM = CK. Trên
đoạn AD lấy điểm P bất kì. Đoạn thng MK lần lượt ct PB PC ti E F. Chng minh rng:
PEF BME CKF
S S S
.
11.7. Cho tam giác ABC các trung tuyến AD BE vuông góc vi nhau ti O. Biết rng AC = b; BC = a.
Tính đường tròn hình vuông có cnh là a.
11.8. Đặt mt hình vuông nh vào bên trong mt hình vuông ln ri nối 4 đỉnh ca hình vuông lớn hơn tương
ng theo th t với 4 đỉnh ca hình vuông nh (như hình vẽ).
Chng minh rng:
AMNB CDQP ADQM BCPN
S S S S
.
M
N
I
C
D
H
E
F
B
A
14
11.9. Cho
ABC
vuông tại A có AH là đường cao. Trên AB, AC ly K,
L sao cho AK = AL =AH. Chng minh rng
1
2
AKL ABC
SS
.
11.10. Cho t giác ABCD. Gi M; N lần lượt trung điểm AB; CD.
Gi P; Q lần ợt trung đim BM DN. Chng minh rng
1
4
MPNQ ABCD
SS
.
11.11. Cho hình ch nht ABCD. Trên cnh AB ly hai điểm M, N sao
cho AM = MN = NB P trung điểm cnh CD. Gọi O giao điểm
ca ND MP. Biết đưng tròn tam giác DOP lớn hơn din tích tam
giác MON là 7cm
2
. Tính din tích hình ch nht ABCD.
11.12. Cho t giác ABCD AC =10cm, BD =12cm. Hai đưng chéo
AC BD ct nhau ti O, biết
0
30 .AOB
Tính din tích t giác
ABCD.
11.13. Cho t giác ABCD. Gi M, P, N, Q lần lượt trung điểm các cạnh AB, BC, CD, AD; O giao điểm
ca MN và PQ. Chng minh:
a)
AOQ BOP MPQ
S S S
b)
.
1
2
AOD BOC ABCD
S S S
11.14. Cho một hình bình hành 13 đường thng, mỗi đường thẳng đều chia hình bình hành thành hai t giác
t s din tích bng
2
5
. Chng minh rằng trong 13 đường thẳng đó, ít nhất bốn đường thẳng cùng đi qua
một điểm.
11.15. Bên trong mt hình vuông cnh bằng 1 cho 1000 điểm trong đó không ba điểm nào thng hàng.
Chng minh rng trong s các tam giác đỉnh 3 trong 1000 điểm đó, tn ti mt tam giác din tích
không quá
1
998
.
11.16. Cho 37 điểm, không ba điểm nào thng hàng, nm bên trong mt hình vuông cnh bng 1.
Chng minh rằng luôn tìm được năm điểm trong 37 điểm đó thỏa mãn: Các tam giác được to bi ba trong năm
điểm đó có diện tích không quá
1
18
.
11.17. Cho một đa giác lồi. Chng minh rng tn ti mt hình bình hành din tích không quá hai ln din
tích đa giác sao cho các đỉnh của đa giác nằm trong hoc trên biên ca hình bình hành.
11.18. Cho lc giác li ABCDEF có các cp cạnh đối song song. Chng minh
1
2
ACE ABCDEF
SS
.
11.19. Cho t giác ABCD. Gi I, E, G, H lần lượt là trung đim của AB, BC, CD, DA, đường thng CI ct BH
DE lần lượt tại M N, đường thng AG ct DE BH lần lượt ti P Q. Chng minh rng:
MNPQ IBM CEN DGP AHQ
S S S S S
.
11.20. Cho tam giác ABC, gi M, N, D lần lượt trung đim của BC, CA, AB P đim tùy ý nm ngoài
tam giác. Chng minh rng trong ba tam giác PAM, PBN, PCD luôn tn ti mt tam giác có din tích bng tng
din tích hai tam giác còn li.
P
N
B
C
A
D
M
Q
15
CHUYÊN ĐỀ 12. PHƯƠNG PHÁP DIỆN TÍCH
A. Kiến thc cn nh
1. Ta đã biết mt s công thc tính din tích của đa giác như công thức tính din tích hình tam giác, hình thang,
hình bình hành, hình ch nht, hình thoi, ...Khi y biết độ dài ca mt s yếu t, ta th tính được din tích
ca nhng hình ấy. Ngược li nếu biết quan h din tích ca hai hình chng hn biết hai tam giác din tích
bằng nhau hai đáy bằng nhau thì suy ra đưc các chiều cao tương ng bằng nhau. Như vy các công thc
tính din tích cho ta các quan h v độ dài của các đoạn thng.
2. Để so sánh hai đọ dài nào đó bằng phương pháp diện tích, ta có th làm theo các bước sau:
- Xác định quan h din tích gia các hình.
- S dng các công thc diện tích để biu din mi quan h đó bằng một đẳng thc có chứa độ dài.
- Biến đổi đẳng thc vừa tìm được ta có quan h v độ dài giữa hai đoạn thng sn so sánh.
3. Mt s bin pháp thc hin:
- S dng trc tiếp công thc tính din tích tam giác.
- S dng nh cht: Nếu hai tam giác cùng chiu cao thì t s hai đáy tương ng bng t s hai din tích.
Ngược li, nếu hai tam giác có cùng đáy thì tỉ s hai chiều cao tương ứng bng t s hai din tích.
- S dng tính cht: Nếu mt tam giác một hình bình hành cùng đáy cùng chiều cao (ng với đáy đó)
thì din tích tam giác bng na tdc hình bình hành.
B. Mt s ví d
d 1. Cho tam giác ABC, một đường thng ct cnh AB, AC lần lượt ti M N. Chng minh rng:
.
.
AMN
ABC
S
AM AN
S AB AC
Gii
Áp dng tính cht hai tam giác có cùng đường cao, ta có:
;
AMN AMC
AMC ABC
SS
AN AM
S AC S AB

T đó suy ra:
.
.
AMN AMN AMC
ABC AMC ABC
S S S
AM AN
S S S AB AC
(điều phi chng minh).
Ví d 2. Cho tam giác ABC và
ABC
AA
. Chng minh rng:
.
.
ABC
ABC
S
A B A C
S AB AC
.
Gii
Trên đường thng AB, AC lấy 2 điểm M và N sao cho
,AM A B AN A C

.
T đó suy ra:
A B C AMN
(c.g.c).
Chứng minh tương tự ví d 1, ta có:
.
.
AMN
ABC
S
AM AN
S AB AC
T đó suy ra:
.
.
ABC
ABC
S
A B A C
S AB AC
N
M
A
B
C
16
Nhn xét. d 1; 2 mt kết qu đẹp v t s diện tích. Chúng được vn dng trong nhiu bài toán v sau.
Bn nên nh tính cht này.
Ví d 3. Cho tam giác ABC, gọi D là trung điểm AB. trên cnh AC lấy điểm E sao cho
2.AE EC
.
Gọi O là giao điểm ca CD và BE. Chng minh rng:
a)
BOC AOC
SS
b) BO = 3.EO
Tìm cách gii : Vì
D
là trung điểm ca
AB
nên suy ngay ra được
DDA C B C
SS
;
DAOD BO
SS
Nên d dàng dẫn đến
BOC AOC
SS
. Nhn thy rng
BO
,
CO
hai cnh ca tam giác
BOC
,
COE
chung
đường cao k t
C
. Do đó để so sánh
BO
CO
, ta so sánh din tích tam giác
BOC
COE
. T câu a, ta so
sánh din tích tam giác
AOC
COE
, hin nhiên ta cn so sánh
AC
EA
. T đó ta có lời gii sau:
Trình bày li gii
Ta có :
DDAB
nên
DAOD BO
SS
;
DDCA CB
SS
. Suy ra :
D D DCA AOD CB BO
S S S S
Hay
BOC AOC
SS
. Áp dng t s din tích hai tam giác có chung chiu cao, ta có :
1
A3
OEC
OAC
S
EC
SE

1
3.
3
OEC
BOC
S
OE
OB OE
S OB
Nhn xét . Để chng minh
3.OB OE
chúng ta chng minh
3.
BOC OEC
SS
. Phương pháp diện ch để tìm t
s đoạn thng, ta tìm t s din tích ca 2 tam giác nhận 2 đoạn thng y làm cnh.
d 4. Cho tam giác
ABC
cân đỉnh
A
. một điểm
M
thuôc cnh
BC
, k
DM
vuông góc vi cnh
AB
,
ME
vuông góc vi
AC
. Chng minh rng tng
D+MEM
không ph thuc vào v trí điểm
M
trên cnh
BC
.
C'
B'
A'
N
M
A
B
C
O
A
B
C
E
D
17
Gii
Tìm cách gii. Nhn thấy khi điểm
M
di động trên cnh
BC
thì quan h
MD
vuông góc vi
AB
,
ME
vuông
góc vi
AC
không đổi , nên d dàng nhn biết đưc tng din tích hai tam giác
ABM
ACM
không
đổi . Do vậy chúng ta nghĩ tới phương pháp diện tích.
Trình bày li gii
K
BH AC H
c định , suy ra
BH
không đổi . Ta có:
1 1 1 1 1
. . .BH ( ) .BH
2 2 2 2 2
ABM AMC ABC
S S S AB DM AC ME AC AB DM ME AC
(vì
AB AC
)
Do đó :
DM ME
không ph thuc vào v trí ca
M
trên
BC
.
Nhn xét
Ngoài cách gii trên, chúng ta còn cách giải khác như sau: Kẻ
MI
vuông góc vi
BH
. Chúng ta chng
minh được
MI BI
,
ME IH
, t đó suy ra
DM ME BH
không ph thuc vào v trí điểm
M
trên cnh
BC
Tam giác
ABC
đều s trường hợp đặc bit ca tam giác cân, do vy vi k thut trên chúng ta giải được bài
toán sau : Cho tam giác đu
ABC
. Một điểm
M
bt thuc min trong hoc trên cnh tam giác
ABC
.
Chng minh rng tng khong cách t đim
M
đến các cnh tam giác
ABC
không ph thuộc điểm vào đim
M
.
Nếu cho điểm
M
chuyển động trên tia đối ca tia
CB
ta có :
ABM AMC ABC
S S S
Vi k thut trên chúng ta giải đưc bài toán sau : Cho tam giác
ABC
cân đỉnh
A
. Một đim
M
y ý trên tia
đối ca tia
CB
. K
MD
vuông góc vi cnh
AB
,
ME
vuông góc vi
AC
. Chng minh rng hiu
MD ME
không ph thuc vào v trí điểm
M
Bn cht ca cách gii dùng din tích kết hp vi
AB AC
đề chng minh kết qu trên bằng độ dài đường
cao ng vi cnh bên. Với tưởng y chúng ta giải được bài toán sau . Cho tam giác đều
ABC
, một điểm
M
nm min trong góc
A
, nhưng nằm ngoài tam giác
ABC
. K
MD
vuông góc vi cnh
AB
,
MK
vuông
góc vi
BC
. Chng minh rng :
MD ME MK
không ph thuc vào v trí điểm
M
.
D
E
H
B
C
A
18
d 5 . Mt hình ch nht bng giấy được gấp theo đưng chéo
AC
như hình vẽ . Din tích ca hình ch
nhật được bng
5
8
ca diện tích ban đầu . Biết din tích tam giác
AMC
2
18cm
Tính din tích hình ch nhật ban đầu
Chng t độ dài
AM
gp 3 lần độ dài
BM
Gii
Tìm cách gii. Nhn thy rng khi gp t giy hình ch nhật theo đường chéo thì phn t giy xếp chng lên
nhau chính phn tam giác
AMC
. Mt khác din tích hình nhận được bng
5
8
ca diện tích ban đầu. T đó
suy ra câu b, nhn thy
AM
,
BM
lần ợt độ dài hai cnh ca hai tam giác
AMC
,
BMC
chung đường
cao k t
C
. Do vy mun so sánh
AM
BM
chúng ta nên đi so sánh din tích tam giác
AMC
din tích
tam giác
BMC
.
Trình bày li gii .
a) Khi gp t giy hình ch nhật theo đường chéo thì phn t giy xếp chng lên nhau chính phn tam
giác
AMC
. Do vy din tích hình nhận được so vi din tích hình ch nhật ban đầu giảm đi đúng bằng
din tích tam giác
MAC
. Tc giảm đi
2
18cm
. Din tích hình nhận được bng
5
8
din tích hình ch
nhật ban đầu nên din tích tam giác
AMC
bng
53
1
38

(din tích hình ch nht)
Do đó diện tích hình ch nht là :
2
3
18: 48( )
8
cm
b) Din tích tam giác
ABC
là :
2
48:2 24( )cm
Din tích tam giác
MBC
là :
2
24 18 6( )cm
Hai tam giác
MBC
AMC
có chung đường cao
BC
nên :
18
3
3
AMC
MBC
S
AM
S MB
Suy ra :
3AM MB
d 6. Cho hình thang
D(AB//CD)ABC
. Gi
O
giao điểm ca
AC
DB
. Qua
O
k đường thng d
song song vi
DC
. Đường thng d ct
DA
BC
lần lượt ti
M
N
. Chng minh rng :
OM ON
Gii
M
A
C
D
B
19
Tìm cách gii. Khi nói v diện tích hình thang thì đặc trưng tam giác
AOD
;
BOC
din tích
bng nhau . Khai thác yếu t này , ta có :
NOM DOM BON CON
S S S S
T nhn xét trên ta mun so sánh
OM
ON
chúng ta đi so sánh tổng các đưng cao ng vi cnh
OM
ON
Trình bày li gii
K
AP
vuông góc
DC
và ct
MN
ti
I
,
BQ
vuông góc vi
DC
ct
MN
ti
J
;
DK
vuông góc vi
MN
ti
K
;
CH
vuông góc vi
MN
ti
H
ta có :
DD
11
D. ; D.
22
AC BC
S C AP S C PQ
AP BQ
nên
DDAC BC AOC BOC
S S S S
Mt khác
D
1 1 1 1
. . ( ) .
2 2 2 2
AO AOM DOM
S S S OM AI OM DK OM AI DK OM AP
1 1 1 1
.BJ . (BJ ) .BQ
2 2 2 2
BOC BON CON
S S S ON ON CH ON CH ON
Suy ra :
11
..
22
OM AP ON BQ OM ON
d 7. Cho
0
90xOy
tia
Oz
phân giác ly điểm
P
c định thuc tia
Oz
(
P
khác
O
). Qua
P
k
đường thng bt kì ct
Ox
,
Oy
ti
M
,
N
. Chứng minh khi d thay đổi thì
11
OM ON
không đổi.
Gii
J
I
Q
P
H
K
N
M
O
D
C
A
B
20
K
,PH OyPI Ox
Ta có
PI PH
không đổi , ta có :
OPM OPN OMN
S S S
Nên :
1 1 1
. . .
2 2 2
OM PI ON PH OM ON
Chia 2 vế cho
1
.
2
OM ON
Ta có :
1
PI PH
ON OM

Do
PI PH
, nên ta có :
11
OM ON
không đổi.
d 8. Trong tam giác
ABC
gi
,,
abc
h h h
độ dài các đường cao ng vi các cnh
BC
,
CA
,
AB
. Gi
,,x y z
khong cách t điểm
M
thuc miền trong tam giác đến
BC
,
CA
,
AB
. Chng minh rng :
m
in , , max , ,
a b c a b c
h h h x y z h h h
Gii
Gi s
abc
h h h c b a
2S
2S=ax+by+cz (1)
a
ax ay az x y z h
a
y
x
z
H
I
P
O
M
N
x
y
z
A
B
C
M
21
2S
2S=ax+by+cz (2)
a
cx cy cz x y z h
c
T (1) và (2) suy ra điều phi chng minh.
C. Bài tp vn dng
12.1 Cho hình vuông
DABC
E
là điểm trên cnh
DA
sao cho
2
D
144( )
ABC
S cm
D
1
3
ABE ABC
SS
Tính độ dài đoạn
AE
.
(Olimpic toán tuổi thơ toàn quốc năm 2014- 2015)
12.2 Cho hình thang
D(AB//CD)ABC
. Gi
,MN
lần lượt là trung đim ca
,DAB C
. Một đường thng song
song với hai đáy cắt
DA
E
,
MN
I
,
BC
F
. Chng minh :
IE IF
.
12. 3. Cho tam giác
ABC
. Qua điểm
O
tùy ý nm trong tam giác ta k các đường thng
; ; AO BO CO
ct
, , BC CA AB
lần lượt ti
,MN
P
. Chng minh rng:
1
OM ON OP
AM BN CP
.
12.4. Cho
ABC
trung tuyến
AM
. Một đường thng song song vi
BC
, ct cnh
, AB AC
AM
ti
, , D E F
.
Chng minh
FD FE
.
12.5. Cho hình bình hành
ABCD
. Trên
BC
lấy điểm
I
và trên
AB
lấy điểm
K
sao cho
AI CK
. Gi
O
giao điểm ca
AI
CK
. Chng minh
OD
là tia phân giác ca góc
AOC
.
12.6. Cho hình thàng
// , ABCD AB CD AB CD
. Lấy điểm
M
trên CD sao cho
BM
chia
ABCD
thành
hai phn có din tích bng nhau. Gi
N
là trung điểm
AD
. Chng minh
// MN BC
.
12.7. Cho tam giác
ABC
. Các điểm
, , MNP
theo th t thuộc các đoạn thng
, , BC CA AB
. Các điểm
, , X Y Z
theo th t thuộc các đoạn thng
, , NP PM MN
. Biết rng
; ; YZ ZX XY
theo th t song
song vi
, , BC CA AB
. Chng minh
XP MB
XN MC
.
12. 8. Cho tam giác
ABC
. Trên cnh
AB
lấy điểm
D
sao cho
3BD DA
, trên
CB
lấy điểm
E
sao cho
4BE EC
. Gi
F
là giao điểm ca
AE
CD
. Chng minh rng
FD FC
.
12. 9. Cho t giác li
ABCD
. Trên hai cnh
AB
CD
ta lần lượt lấy hai điểm
12.14. Cho lc giác ABCDEF, mỗi đường chéo AD, BE, CF chia lc giác thành hai phn bng nhau có din tích
bng nhau. Chng minh rằng AD, BE, CF đồng quy.
12.15. Cho lc giác ABCDEF. Gọi các trung điểm ca AB, BC, CD, DE, EF, FA lần lượt là L, M, N, Q, R. Biết
mỗi đoạn LP, MQ, NR đồng quy.
12.16. Cho t giác ABCD có E,F lần lượt là trung điểm ca các cạnh AD, BC. Đường thng EF cắt các đường
thng AB, CD lần lượt ti M, N. Chng minh rng MA.NC = MB.ND
22
1
CHƯƠNG III. TAM GIÁC ĐỒNG DNG
CHUYÊN ĐỀ 13. ĐỊNH LÝ TA-LÉT TRONG TAM GIÁC .....................................................................................................2
CHUYÊN ĐỀ 14. TÍNH CHẤT ĐƯỜNG PHÂN GIÁC CA TAM GIÁC .........................................................................17
CHUYÊN ĐỀ 15. CÁC TRƯỜNG HP ĐỒNG DNG CA TAM GIÁC .........................................................................29
CHUYÊN ĐỀ 16. CÁC TRƯỜNG HP ĐỒNG DNG CA TAM GIÁC VUÔNG ........................................................45
CHUYÊN ĐỀ 17. ĐỊNH LÝ MENELAUS, ĐỊNH LÝ CE VA, ĐỊNH LÝ VAN OBEN ................................................55
2
CHUYÊN ĐỀ 13. ĐỊNH LÝ TA-LÉT TRONG TAM GIÁC
A. KIến thc cn nh
- T s của hai đoạn thng: t s của hai đoạn thng là t s đô dài của chúng theo cùng một đơn vị đo.
- Đon thng t lệ: Hai đoạn thng AB và CD gi là t l với hai đoạn thng AB và CD nên có t l thc.
''
' ' ' ' ' '
AB A B AB CD
hay
CD C D A B C D

- Định lý Ta-let trong tam giác. Nếu một đường thng song song vi mt cnh ca tam giác và ct hai cnh còn
lại thì nó định ra trên cạnh đó những đoạn thẳng tương ứng t l:
Trong hình bên:
' ' ' ' ' ' '
;;
' '/ /
'B 'C'
ABC
A B AC AB AC B B C C
B C BC
AB AC B C AB AC
1. Định lý Ta-lét đảo. Nếu một đường thng ct hai cnh ca một tam giác và định ra trên hai cnh này nhng
đoạn thẳng tương ứng t l thì đường thẳng đó song song với cnh còn li ca tam giác.
Trong hình bên:
'C'/ /BC
''
'B 'C'
ABC
B
AB AC
BC
.
3
2. H qu của định lý ta-lét. Nếu mt đưng thng ct hai cnh ca mt tam giác và song song vi cnh còn
li thì nó to thành mt tam giác mi có ba cạnh tương ứng t l vi ba cnh của tam giác đả cho.
E
F
sao cho
AE CF
BE DF
. Chng minh rng nếu đường chéo
AC
đi qua trung điểm
I
của đoạn thng
EF
thì
AC
chia đôi diện tích ca t giác
ABCD
.
12.10. Cho tam giác
ABC
vuông cân ti
A
và hai đường phân giác
BD
CE
. Lấy điểm
I
bất kì trên đoạn
thng
DE
. Chng minh rng:
2
IAB I
IBC
AC
S
SS
.
12.11. Cho tam giác
ABC
vi
, , BC a CA b AB c
và ba đường cao ng vi ba cnh lần lượt có độ dài là
, ,
a b c
h h h
. Gi
r
là khong cách t giao điểm của ba đường phân giác của tam giác đến mi cnh ca tam giác.
Chng minh rng
1 1 1 1
bca
h h h r
.
12.12. Cho tm giác
ABC
có ba đường phân giác
, , AD BE CF
cùng ct nhau ti
I
. Chng minh rng
2 2 2
1
Al BI CI
AB AC BA BC CA CB
.
12.13. Hai đường chéo ca t giác
ABCD
ct nhau ti
O
, chia t giác thành bốn tam giác có đỉnh
O
, , , OAB OBC OCD OAD
. Biết s đo diện tích ca các tam giác này là các s
nguyên. Chng minh rng tích các s đo diện tích của các tam giác đó là một s chính phương.
Trong hình trên:
' ' ' '
' '/ /
ABC
AB AC B C
B C BC
AB AC BC
Chú ý: H qu trên vẫn đúng cho trường hợp đường thng a song song vi mt cnh ca tam giác và ct phn
kéo dài ca hai cnh còn li.
' ' ' 'AB AC B C
AB AC BC

4
B. Mt s ví d
Ví d 1: Cho tam giác ABC có trung tuyến AM. T một điểm E trên cnh BC ta k đường thng Ex song song
vi AM và ct tia CA, BA lần lượt ti F và G.
Chng minh:
2EF EG AM
.
Gii
* Tìm cách gii.
- Để chng minh
2EF EG AM
, suy luận thông thường là dựng đoạn thng trên tia EF, EG bằng đoạn
thng AM, ri biến đổi cng tr đoạn thng. Chng hn trong ví d này, qua A k đường thng song song
vi BC, ct EF ti I. D dàng nhn thy
EI AM
, do vy ch cn chng minh
GI IF
là xong. Tuy nhiên
để chng minh
GI IF
bng cách ghép vào hai tam giác bng nhau là khó khan, chính vì vy chúng ta
chng minh t s bng nhau có cùng mu s. Quan sát k nhn thy GI và IF có th đặt trên mu sIE!
T đó vận dụng định lý và h qu Ta-lét để chng minh
FI IG
IE IE
là xong.
- Ngoài cách trên, chúng ta có th biến đổi kết lun thành tng t s và chng minh
2
EF EG
AM AM

là xong.
Do đó vận dụng định lý Ta-lét và biến đổi linh hot t l thc là yêu cu tt yếu trong dng toán này.
* Trình bày li gii
Cách 1. Gi s E thuộc đoạn BM.
Qua A k đường thng song song vi BC ct EF ti I. Ta có AMEI là hình bình hành, suy ra
EI AM
.
Áp dụng định lý Ta-lét, xét
EFC
có AI // CE.
5
/1
IF FA EM
AM EF
IE AC MC
Xét
GED
/ / , / /AI BE AM GE
2
IG AG EM
IE AB BM
T
1
2
, kết hp vi
BM MC
Suy ra
IG IF
Ta có:
22EF EG EI IF EI IG EI AM
Cách 2.
Gi s E thuộc đoạn BM
Theo h qu định lí Ta-lét:
Xét
EFC
/ / 3
EF EC
EF AM
AM CM

Xét
ABM
/ / 4
EG BE
EG AM
AM BM

F
I
G
E
M
B
C
A
F
I
G
E
M
B
C
A
6
Cng vế theo vế
3
4
ta có:
2
EF EG EC BE EF EG BC
hay
AM AM CM BM AM BM
Suy ra
2.EF FG AM
d 2. Cho hình thang ABCD
( / / )AB CD
. Trên tia đối ca tia BA lấy điểm E sao cho
BE CD
. Gi giao
điểm ca AC vi DB và DE theo th t là I và K. Chng minh h thc
AK AC
KC CI
Gii
Tìm cách gii. Nhn thy rng chúng ta không th chng minh trc tiếp
AK AC
KC CI
, do vy nên s dng t s
trung gian. Khai thác
BE CD
//AB CD
rt t nhiên chúng ta vn dng h
qu định lý Ta-let.
Trình bày li gii
Đặt
AB a
,
BE CD b
. Theo h qu định lý Ta-let
Ta có
/ / 1
AK AE a b
AE CD
KC CD b
//AB CD
AI AB a
CI CD b
2
AI CI a b AC a b
CI b CI b
T
1
2
suy ra
AK AC
KC CI
Ví d 3. Cho tam giác
ABC
A 120 ,
AD
là đường phân giác. Chng minh rng:
1 1 1
.
AB AC AD

Gii
K
I
A
D
B
C
E
7
K
DE // AB,
ta có:
1 1 2
D A 60 ; A 60
nên tam giác
ADE
đều. Suy ra
AD AE DE.
Áp dng h qu định lý Ta lét:
DE CE AD CE
hay .
AB AC AB AC

Mt khác
AD AE
AC AC
nên
AD AD CE AE AC
1
AB AC AC AC AC
Suy ra
1 1 1
.
AB AC AD

Nhn xét. Nhng bài toán chứng minh đẳng thức có nghich đảo độ dài đoạn thng, bn nên biến đổi và chưng
minh h thức tương đương có tỉ s của hai đoạn thng.
Ví d 4. Một đường thẳng đi qua trọng tâm
G
ca tam giác
ABC
ct cnh
AB, AC
lần lượt ti
M
N.
Chng minh rng:
AB AC
a) 3;
AM AN

BM CN
b) 1.
AM AN

* Tìm cách gii. Để to ra t s
AB AC
;
AM AN
chúng ta cn vn dụng định lí Ta Lét, mà hình v chưa có yếu t
song song do vy chúng ta cn k thêm yếu t song song. K đường thng song song vi
MN
t
B
C
va
khai thác được yếu t trng tâm, va tạo ra được t s yêu cu.
* Trình bày li gii
Trường hp 1: Nếu
MN // BC,
thì li gii giản đơn (dành cho bạn đọc)
Trường hp 2: Xét
MN
không song song vi
BC.
a) Gọi giao điểm ca
AG
BC
D BD CD.
K
B
I // CK // MN I, K AD
Xét
BDI
CDK
BD CD; IBD KCD;
2
1
1
E
D
A
C
B
K
I
N
G
D
A
B
C
M
8
IDB KDC
nên
BDI CDK (g-c-g)
DI = DK
Áp dụng định lý Ta lét, ta có:
AB AI
=
AM AG
(vì
MG // BI)
;
AC AK
=
AN AG
(vì
GN // CK).
Suy ra
AB AC 2AD
3 (1)
AM AN AG
(vì
3
AD AG).
2
b) Xét
BM GI CN KG
;
AM AG AN AG

Hay
BM CN GI GK 2GD BM CN
= =1, suy ra 1.
AM AN AG AG AM AN
Nhn xét. T kết qu (1), chúng ta thy rng bi
G
là trng tâm nên
2AD
3
AG
. Vy nếu
G
không phi là trng
tâm thi ta có bài toán sau:
- Một đường bt k ct
AB, AC
và đường trung tuyến
AD
ca tam giác
ABC
lần lượt ti
M, N
G.
Chng
minh rng:
AB AC 2AD
.
AM AN AG

- Nếu thay yếu t trung tuyến bng hình bình hành, ta coa bài toán sau: Cho hình bình hành
Một đường
thng bt k ct
AB, AD
AC
lần lượt ti
M, N
G.
Chng minh rng:
AB AD AC
.
AM AN AG

Ví d 5. Mt đường thẳng đi qua trọng tâm
G
ca tam giác
ABC
ct cnh
AB, AC
lần lượt ti
P, Q
. Chng
minh rng:
PB QC 1
.
PA QA 4
(Olympic Toán, Tây Ban Nha, năm 1995)
Gii
* Tìm cách gii. V hình xong và quan sát, chúng ta nhn thy t s
PB QC
;
PA QA
đã có ở câu b, ví d 4 và có kết
qu
PB QC
1.
PA QA

Do vy khai thác yếu ty, kết hp vi bất đẳng thức đại s cho li giải đẹp
* Trình bày li gii
Da vào ví d 4, ta có:
PB QC
1
PA QA

Áp dng bất đẳng thc
2
x y 4xy;
Q
P
G
M
A
B
C
9
Ta có:
2
BP CQ BP QC BP QC 1
1 4. . hay . .
AP AQ PA QA PA QA 4



Ví d 6. Cho
ABCD
là hình bình hành có tâm
O.
Gi
M, N
là trung điểm
BO; AO.
Ly
F
Trên cnh
AB
sao cho
FM
ct cnh
BC
ti
E
và tia
FN
ct cnh
AC
ti
K.
Chng minh rng:
BA BC
a) 4;
BF BE

b) BE AK BC.
Gii
* Tìm cách gii. Vi phân tích và auy luận như câu a, ví dụ 4 thì câu a, ví d này không quá khó. Tương tự câu
a, chúng ta có kết qu:
AD AB
4
AK AF

và suy ra
AD AB AB BC
8
AK AF BF BE
Để liên kết được
BE AK
vi nhau, mà vi suy lun trên thì
BE, AK
cũng nằm mu số, do đó chúng ta liên
ng ti bất đẳng thức đại s
1 1 4
x y x y

s cho chúng ta yêu cu. Vi suy luận đó, chúng ta có lời gii
sau:
E
H
I
K
M
N
O
C
A
B
D
F
* Trình bày li gii
a) K
CI // AH // EF
(vi
I, H BD)
Xét
AOH
COI
AOH COI
ối đỉnh);
OA OB; HAO ICO
(so le trong)
AOH COI (c.g.c) IO=OH.
Áp dụng định lý Ta lét, ta có:
BA BC BH BI BH BI BO OH BO OI 2.BO
4.
BF BE BM BM BM BM BM
b) Tương tự ta có:
AD AB AD AB AB BC
48
AK AF AK AF BF BE
1 1 1 1
BC AB 8 (1)
AK BE AF BF
Áp dng bất đẳng thc
1 1 4
x y x y

(vi
x; y > 0)
Ta có:
1 1 4 4 1 1
AB 4 (2)
AF BF AF BF AB AF BF



10
T (1) và (2) suy ra:
11
BC 4
AK BE




1 1 4 1 1 4BC
BC
AK BE AK BE AK BE AK BE




4BC
4 AK BE BC
AK BE
.
Ví d 7. Cho tam giác
ABC
nhn có
AH
là đương cao. Trên
AH, AB, AC
lần lượt lấy điểm
D, E, F
sao cho
EDC FDB 90 .
Chng minh rng:
EF // BC.
(Thi hc sinh gii Toán 9, tnh Quảng Ngãi, năm học 2011 2012)
Gii
* Tìm cách gii. Để chng minh
EF // BC,
suy lun mt cách t nhiên chúng ta cn vn dụng định lý Ta lét
đảo. Do vy cn chng minh t l thc
AB AC
.
AE AF
Nhn thấy để định hướng t l thc y cũng như khái thác
được
EDC FDB 90
chúng ta cn k
BO CD; CM DB,
để có các đường thng song song ri vn
dụng định lý Ta lét. T đó chúng ta có lời gii sau:
* Trình bày li gii
K
BO CD; CM DB.
BO
CM
ct nhau ti
ID
là trung trc ca
BIC DI BC I, D, A
thng hàng.
AI AB
DE // BI .
AD AE

AI AC AB AC
IC // FD suy ra EF // BC
AD AF AE AF
nh lý Ta - lét đảo).
Ví d 8. Cho tam giác
ABC
vuông cân ti
A
BM
là đường trung tuyến. Lấy điểm
F
trên cnh
BC
sao
cho
FB 2.FC.
Chng minh
AF BM
.
Gii
I
O
M
F
E
H
A
B
C
D
11
* Tìm cách gii. Nhn thy t
FB 2.FC
suy ra:
mang tính cht trong tâm tam giác. Do vy nếu goi
G
là trong tâm tam giác,
AH
là đường trung tuyến thì d dàng nhận được
GF // AC
AH BC
nên
G
trc tâm tam giác
ABF.
Do đó ta có lời gii sau:
* Trình bày li gii
Goi
G
là trong tâm tam giác
ABC
AG
kéo dài ct
BC
ti
H AH
là đương trung tuyến ca tam giác
ABC
Mt khác
ABC
vuông cân ti
A
nên
AH BC
Ta có:
2
BG
GM
( Vì
G
là trng tâm).
2
BF
FC
(gt)
BG BF
GM FC

/ / FG AC
( theo định lý Ta lét đảo).
FG AB
nên
G
là trc tâm ca
ABF
BG AF
hay
BM AF
.
d 9: Cho tam giác
ABC
. Biết tn tại điểm
,MN
lần lượt trên cnh
,AB AC
sao cho
2
BM BN
AM CN
BNM ANC
. Chng minh tam giác
ABC
vuông.
Gii
Cách 1: Gi
P
trung đim ca
AM
,
Q
giao điểm ca
AN
vi
CP
.
Ta có:
2
BM BM BN
PM AM CN

/ / MN CP
( định lý Talets đảo)
QCN MNB ANC
QCN
cân ti
Q
Mt khác
, / / PA PM PQ MN
QA QN
nên
QA QC QN
CAN
vuông ti
C
ABC
vuông ti
C
.
G
H
M
B
A
C
F
Q
P
A
C
B
N
M
12
Cách 2: Dng
D
là điểm đối xng ca
N
qua
C
2ND CN CD CN
.
Ta có
2
2
BM BN BM BN BN
AM CN AM CN DN
/ / MN AD
( định lý Talét đảo).
12
D N N
AND
cân .
Do đó đường trung tuyến
AC
cũng là đường cao.
Vy
AC CB
ABC
vuông ti
C
.
d 10: Cho tam giác
ABC
AD
đường trung tuyến. Gi
M
điểm y ý thuc khong
BD
. Ly
E
thuc
AB
F
thuc
AC
sao cho
/ / ;CM EA CM AB∕∕
.
Gi
H
giao điểm ca
MF
AD
. Đường thng qua
B
song song vi
EH
ct
MF
ti
K
. Đường thng
AK
ct
BC
BC ti
I
. Tính t
IB
ID
?
Gii
Qua
D
k đường thng song song vi
AB
, ct tia
AI
ti
P
. Áp dụng định lý Ta-let,cho các đoạn thng song
song ta có:
// .
IB AB AB HK
DP AB
ID DP HK DP
(1).
//
AB AB BC
ME AC
HK BE BM
(2) .
1
2
1
D
A
C
B
N
M
E
I
M
H
B
A
F
D
K
P
C
13
//HK DP
//
HK AH BM
MH AB
DP AD BD
(3).
T (1) , (2) và (3) suy ra:
.2
IB BC BM BC
ID BM BD BD
. Vy
2
IB
ID
.
d 11. Cho
ABC
nhn. Hình ch nht
MNPQ
thay đổi tha mãn
M
thuc cnh
AB
,
N
thuc cnh
AC
P
,
Q
thuc cnh
BC
. Gọi giao đim ca
BN
vi
CM
X
ca
QN
vi
PM
Y
. Gi
H
giao
điểm ca
XY
vi
BC
. Chng minh rằng đường thng
AH
vuông góc vi
BC
.
Gii
Tìm cách gii. Bài toán có nhiu yếu t song song, do vy để chứng minh đường thng
AH
vuông góc vi BC,
chúng ta nên chng minh
AH
song song vi
NP
hoc
MQ
. Với định hướng y chúng ta tìm cách vn dng
định lý Ta-let đảo. Chng hn nếu chng minh
AH
song song vi
NP
, chúng ta cn chng minh
HP AN
HC AC
.
Bng cách vn dụng định Ta-lét cùng h qu biến đổi khéo léo các dãy t s bng nhau, chúng ta s có li
giải đẹp.
Trình bày li gii
Gi
Z
là giao điểm ca
XY
vi
MN
vì t giác
MNPQ
là hình ch nht,
HP ZM
//MN BC
nên:
HP ZM XM MN AN
HC HC XC CB AC
Dó đó
//AH NP
nh lý Ta-Let đảo) mà
NP BC
nên
AH BC
.
Ví d 12. Cho hình bình hành
ABCD
I
;
E
là trung điểm ca
BC
;
AD
. Qua điểm
M
tùy ý trên
AB
k
đường thng
MI
cắt đường thng
AC
ti
K
. Đường thng
KE
ct
CD
ti
N
.
Chng minh rng:
AD MN
.
Y
Z
X
M
H
B
A
Q
N
P
C
Y
Z
X
M
H
B
A
Q
N
P
C
14
Gii
Gi
P
là giao điểm của đường thng
MI
CD
Gi
Q
là giao điểm của đường thng
KN
AB
.
Nhn thy:
IBM ICP
(g.c.g) nên
BM CP
.
Ta có theo định lý Ta-lét
//AM CP
nên
AM AM KA
MB CP KC

(1).
Nhn thy
EAQ EDN
(g.c.g) nên
DN AQ
.
Theo định lý Ta-lét, ta có:
//AQ CN
nên
DN AQ KA
NC NC KC

(2).
T (1) và (2) suy ra
AM DN AM DN AM DN
MB NC AM MB DN NC AB DC

suy ra
AM DN
.
Do đó
ADNM
là hình bình hành suy ra
AD MN
.
Bài tp vn dng
13.1. Cho hình bình hành
ABCD
24AC
cm. Điểm
E
thuc cnh
AB
sao cho
1
2
AE EB
. Điểm
F
trung điểm ca
BC
. Gi
,IK
th t là giao điểm ca
AC
vi
,DE DF
. Tính các độ dài
,,AI IK KC
.
13.2. Cho tam giác
ABC
BC
cnh ln nht. Trên cnh
BC
lấy các điểm
,DE
sao cho
BD BA
;
CE CA
. Đường thng qua
D
song song vi
AB
ct
AC
ti
M
. Đường thng qua
E
song song vi
AC
ct
AB
ti
N
. Chng minh
AM AN
.
(Tuyn sinh lp 10 chuyên Toán., TP H Chí Minh, năm học 2013 2014)
13.3. Cho tam giác
ABC
vuông ti
A
, đường cao
AH
. Gi
I
là trung điểm ca
AH
. Đường vuông góc vi
BC
ti
C
cắt đường thng
BI
ti
D
. Chng minh rng
DA DC
.
P
I
M
B
A
Q
E
K
N
D
C
15
13.4. Cho hình bình hành
ABCD
. Trên đường chéo
AC
ly một điểm
I
. Tia
DI
cắt đường thng
AB
ti
M
,
cắt đường thng
BC
ti
N
. Chng minh rng:
a)
AM DM CB
AB DN CN

. b)
2
.ID IM IN
.
13.5. Cho tam giác
ABC
vuông ti
A
. V v phía ngoài hai tam giác
ABD
ACE
vuông cân ti
B
E
.
Gi
H
là giao điểm ca
AB
CD
;
K
là giao điểm ca
AC
BE
. Chng minh rng:
a)
AH AK
. b)
2
.AH BH CK
.
13.6. Cho hình vuông
ABCD
, điểm
E
thuc cnh
BC
. Gi
F
giao điểm ca
AE
CD
,
G
giao điểm
ca
DE
BF
.
a) Gi
I
K
theo th t là giao điểm ca
AB
CG
DG
. Chng minh rng
IE
song song vi
BD
b) Chng minh rng
AE
vuông góc vi
CG
.
13.7. Cho tam giác
ABC
D
là một điểm tùy ý trên
AC
. Gi
G
là trng tâm
ABD
. Gi
E
là giao điểm
ca
CG
BD
. Tính
EB CA
ED CD
.
13.8. Cho hình bình hành
ABCD
, điểm
E
thuc cnh
AB
, điểm
F
thuc cnh
BC
. Gi
I
giao điểm ca
CE
AD
, gi
K
là giao điểm ca
AF
DC
. Chng minh rng
EF
song song vi
IK
.
13.9. Cho tam giác
ABC
cân ti
A
. Trên cnh
BC
kéo dài v phía
C
lấy điểm
M
. Một đường thng
đi
qua
M
ct các cnh
lần lượt ti
N
P
. Chng minh rng
BM CM
BP CN
không đổi khi
M
thay
đổi.
(Thi hc sinh Toán 9, tỉnh An Giang, năm học 2009 2010)
13.10. Gi s
O
là giao điểm của hai đường chéo
AC
BD
ca t giác li
ABCD
. Gi
,,E F H
lần lượt là
chân các đường vuông góc k t
,BC
O
đến
AD
. Chng minh rng:
. . . .AD BECF AC BDOH
. Đẳng thc
xy ra khi nào?
13.11. Cho tam giác
ABC
vuông ti
A
. Các t giác
MNPQ
AXYZ
là các hình vuông sao cho
M AB
;
,Q P BC
;
N AC
;
,,X Y Z
tương ứng thuc
,,AB BC AC
. Chng minh
MN AX
.
13.12. Gi
M
là điểm bất kì trên đường trung tuyến trên đường trung tuyến
AD
ca tam giác
ABC
. Gi
P
giao điểm ca
BM
AC
, gi
Q
là giao điểm ca
CM
AB
. Chng minh
//PQ BC
.
13.13. Cho tam giác
ABC
AB BC
, đường phân giác
BE
và đường trung tuyến
BD
(
;ED
thuc
AC
).
Đưng thng vuông góc vi
BE
qua
C
ct
,BE BD
lần lượt ti
,FG
. Chng minh rằng đường thng
DF
chia
đôi đoạn thng
GE
.
13.14. Cho tam giác
ABC
. Lấy điểm
O
nm trong tam giác, các tia
BO
CO
ct
AC
AB
lần lượt ti
M
N
. V hình bình hành
BOCF
. Qua
N
k đường thng song song vi
BM
ct
AF
ti
E
. Chng minh
rng :
16
a)
là hình bình hành; b)
..
..
AE AM AN OM ON
AF AB AC OBOC

.
13.15. Cho hình thang
ABCD
có đáy lớn
CD
. Qua
A
k đường thng song song vi
BC
cắt đường chéo
BD
ti
M
và ct
CD
ti
I
. Qua
B
k đường thng song song vi
AD
ct cnh
CD
ti
K
. Qua
K
k đường
thng song song vi
BD
ct
BC
ti
P
. Chng minh rng:
//MP DC
.
13.16. Cho tam giác
ABC
CM
là trung tuyến. Qua điểm
Q
trên
AB
v đường thng
d
song song vi
CM
. Đường thng
d
ct
AC
,
BC
lần lượt ti
,PR
. Chng minh rng nếu
..QAQB QPQR
thì tam giác
ABC
vuông ti
C
.
13.17. Cho tam giác
ABC
trng tâm
G
. Một điểm
P
thuc cnh
BC
. Các đường thng qua
P
theo th t
song song vi
CG
BG
ct
AB
,
AC
lần lượt ti
E
F
. Gọi giao điểm ca
BG
CG
vi
EF
lần lượt
,IJ
. Chng minh rng:
a)
EI IJ JF
; b)
PG
đi qua trung điểm ca
EF
.
13.18. Cho hình thang
ABCD
(
, //AD CD AB CD
)có đường chéo
AC
bng cnh bên
AD
. Một đường thng
d
đi qua trung điểm
E
ca
CD
ct
BD
BC
ti
;MN
. Gi
;PQ
là giao điểm ca
;AM AN
vi
CD
.
Chng minh
MAD QAC
.
13.19. Cho tam giác
ABC
.
M
là điểm thuc
BC
. Chng minh rng
. . . MABC MC AB MB AC
.
13.20. Cho tam giác nhn
ABC
45A 
, các đường cao
BD
CE
ct nhau
H
. Đường vuông góc vi
AB
ti
B
ct
AC
I
. Đường vuông góc vi
AC
ti
C
ct
AB
K
. Gi
F
giao điểm ca và
CK
,
G
là
giao điểm ca
FH
EI
. Chng minh rng
G
là trng tâm ca tam giác
AIK
.
13.21. Đưng thng
d
đi qua trọng tâm
G
ca tam giác
ABC
ct cnh
AB
ti
M
, cnh
AC
ti
N
tia CB
ti
P
. Chng minh rng:
2 2 2
9
. . .
AB AC BC
AM BM AN CN BP CP
.
13.22. Cho tam giác
ABC
với đim
M
thuc min trong tam giác.Gi
,,I J K
th t giao điểm cu các tia
,,AM BM CM
vi các cnh
,,BC CA AB
. Đưng thng qua
M
song song vi
BC
ct
,IK IJ
ti
;EF
.Chng minh:
ME MF
.
17
CHUYÊN ĐỀ 14. TÍNH CHẤT ĐƯỜNG PHÂN GIÁC CỦA TAM GIÁC
Kiến thc cn nh
1. Định lý
Trong tam giác, đưng phân giác ca mt góc chia cạnh đối din thành hai đoạn thng t l vi hai cạnh đ hai
đoạn y.
ABC
DB AB
DC AC
BAD CAD

.
2. Chú ý
Định lý vẫn đúng đối đường phân giác góc ngoài ca tham giác.
()ABC AB AC
EB AB
EC AC
BAE CAE


.
* Các định lí trên có định lí đảo
BD AB
DC AC
AD là đường phân giác trong ca tam giác
EB AB
EC AC
AE là đường phân giác ngoài ca tam giác
B. Mt s ví d
Ví d 1: Cho ∆ABC, trung tuyến BM ct phân giác CD ti P. Chng minh rng
1
PC AC
PD BC

.
Gii
Dựa vào định lí Talet
11
PC AC PC AC
PD BC PD BC
D
C
A
B
E
B
A
C
B
A
C
E
18
Do CD là phân giác của ∆ABC nên
1 1 1
DA AC DA AC AB AC
DB BC DB BC DB BC
Vì vy ch cn chng minh
PC AB
PD DB
Cách 1:
V DK // BM (K thuc AM).
Theo định lí Ta-lét ta có:
PC MC MA AB
PD MK MK DB

Cách 2:
V DI // AC (I thuc BM).
Theo định lí Ta-lét ta có:
PC MC MA AB
PD DI DI DB
Cách 3:
V AN // BM (N thuc tia CD).
Do MA = MC PC = PN
PC PN
PD PD
Mt khác
( / / )
ND DA
do AN BP
PD DB
11
PN DN DA AB PC AB
PD PD DB DB PD DB
Cách 4:
V AH // CD (H thuc tia BM).
Ta có ∆AMH = ∆CMP (g.c.g)
PC = AH
PC AH
PD PD
Mt khác do PD // AH
A
C
B
D
K
M
P
P
A
B
C
M
D
I
D
C
B
A
N
M
P
C
B
A
M
D
H
19
nên theo h qu của định lí Ta-let ta có:
AH AB PC AB
PD DB PD DB
Cách 5:
Trên tia đối ca tia MB lấy điểm E sao cho MB = ME
ABCE là hình bình hành AB // CE và AB = CE
Theo h qu của định lí Ta-let ta có:
PC CE AB
PD BD DB

Ví d 2: Cho ∆ABC cân tại A và góc A = 36
0
. Chng minh rng: AB
2
= AB.BC + BC
2
Gii
*Tìm cách gii: Phân tích đề bài, chúng ta thu được
0
72BC
, nhn thy 72
2
= 2.36
0
do đó chúng ta nên kẻ
phân giác góc B (hoc góc C) là suy lun t nhiên. T đó vận dng tính chất đường phân giác trong tam giác và
biến đổi linh hot t l thức ta được li gii hay.
* Trình bày li gii.
K phân giác BD ca góc ABC (D AC), khi đó
0
12
36BB
∆ABD cân tại D và ∆BCD cân tại B AD = BC = BD
Theo tính chất đường phân giác trong tam giác ABC ta có:
BA AD BA BC
BC CD BC AC AD
Mà AB = AC, AD = BC
BA BC
BC BA BC
BA
2
BA.BC = BC
2
AB
2
= AB.BC + BC
2
Nhn xét: Tương tự chúng ta giải được bài toán sau:
Cho ∆ABC cân tại A có góc A = 108
0
. Chng minh rng: AB
2
= BC
2
- AB.BC.
Ví d 3. Cho ∆ABC có trọng tâm G và I là giao điểm của 3 đường phân giác trong. Biết rng IG // BC. Chng
minh rng AB + AC = 2.BC
*Tìm cách gii: Nhn thấy để khai thác IG // BC chúng ta nên k đưng phân giác góc A và trung tuyến ng
vi cnh BC thì s vn dụng được gi thiết.
P
A
B
C
E
M
D
2
1
A
B
C
D
20
T suy luận đó chúng ta có kết qu
2
AI
ID
. Mt khác t s
AI
ID
kết hp cới giao điểm ba đường phân giác trong
cho phép chúng ta liên tưởng ti kh năng vận dng tính chất đường phân giác trong tam giác ABD, ACD. T
đó chúng ta có lời gii sau:
*Trình bày li gii:
Gọi D, M lân lượt là giao điểm ca AI, AG vi BC.
Theo tính chất đường phân giác trong tam giác ABD, ACD ta có:
IA AB CA AB AC AB AC
ID BD CD BD CD BC

/ / 2 2
IA GA AB AC
IG BC
ID GM BC
hay AB + AC = 2.BC
Nhn xét: Vi k thut và lối tư duy trên, chúng ta có thể giải được bài toán đảo sau:
Biết AB + AC = 2.BC. Chng minh rng: IG // BC
Ví d 4. Cho ∆ABC có tỉ s gia hai cạnh chung đỉnh là 3: 2. V đường trung tuyến AM và đường phân giác
AK. Tính t s din tích ca hai tam giác AKM và AKB.
Gii
Trường hp 1: Xét
3
2
AB
AC
A
B
C
M
D
G
I
A
B
C
M
K
21
Chú ý rng
2
KB KC
KM
KC AC
KB AB
Ta có:
1 1 1 2 1
1 1 1
2 2 2 2 3 6
AKM
AKB
S
KM KC KB KC AC
S KB KB KB AB
Trường hp 2: Xét
3
2
AC
AB
Chú ý rng:
2
KC KB
KM
KC AC
KB AB
Ta có:
1 1 1 3 1
1 1 1
2 2 2 2 2 4
AKM
AKB
S
KM KC KB KC AC
S KB KB KB AB
Nhn xét: Bài này d b sót trường hp.
Ví d 5: Cho tam giác
ABC
BE
CF
là 2 đường phân giác ct nhau ti
O
. Chng minh rng nếu
1
..
2
OBOC BE CF
thì
ABC
vuông ti
A
.
Gii
* Tìm cách gii. Vi gi thiết
1
..
2
OBOC BE CF
và chng minh
ABC
vuông ti
A
, d dàng nhn thy t m
quan h độ dài mà chng minh tam giác vuông tt yếu chúng ta phải nghĩ đến định lý Py-ta-go đảo. Do đó
chúng ta cn biu din
1
..
2
OBOC BE CF
thông qua các cnh của tam giác ABC. Định hướng cui cùng là:
2 2 2
a b c
.
* Trình bày li gii:
K
M
A
C
B
22
Đặt
,,BC a AC b AB c
Theo tính chất đường phân giác, ta có:
BF BC BF BC
FA AC BF FA BC AC

BF a ac
BF
c a b a b

OF BF c OF a b c CF a b c
OC BC a b OC OC a b OC a b
Tương tự, ta có:
BE a b c
OB a b

T gi thiết
2
1.
. . 2 2
2.
abc
BE CF
OBOC BE CF
OBOC a c a b


2 2 2 2
2 2 2 2 2 2 2a b c ab ac bc a ab ac bc
2 2 2
a b c
, suy ra
ABC
vuông ti
A
.
d 6: Cho tam giác ABC vuông ti
A
G
trng tâm,
BM
đưng phân giác. Biết rng
GM AC
. Chng minh
BM
vuông góc vi trung tuyến
AD
.
Gii
O
E
F
C
B
A
I
G
D
H
M
B
A
C
23
Cách 1: (Không dùng tính chất đường phân giác). Gi
I
giao điểm ca
BM
AD
,
H
trung đim
AC
DH // AB và
1
2
DH AB
(vì
DH
là đường trung bình
ABC
)
Li có
GM AB
(cùng vuông góc vi
AC
)
GM DH
. Áp dng h qu định lý Talet:
Xét
ADH
GM DH
22
33
GM AG GM
DH AD DH
Xét
ABI
có GM //AB
1
3
GI GM GH
AI AB BH
1 3 3 3 2
3 4 4 3 2
GI AI AD
AI AG AD AI
AI

I
là trung điểm ca
AD
ABD
BI
vừa đường phân giác, vừa đườg trung tuyến, suy ra
ABD
cân ti B nên BI vừa đường
cao, vừa là đường phân giác. Do đó
BM AD
.
Cách 2:
ADH
GM DH
2
32
3
AM AG
AM AH AC AM MC
AH AD
hay
2MC AM
.
Áp dng tính chất đường phân giác trong
ABC
, ta có:
2
2
BC MC BC
AB BD
AB MA
Vy
ABD
cân ti
B
nên
BI
vừa là đường phân giác vừa là đường cao.
Do đó
BM AD
.
d 7: Cho tam giác
ABC
I
giao điểm 3 đường phân giác. Đường thng qua
I
cắt các đường thng
,,BC CA AB
lần lượt ti
sao cho
DE
nằm cùng phía đối vi
I
. Chng minh rng:
BC AC AB
ID IE IF

Gii
Áp dng tính chất đường phân giác trong và ngoài tam giác, ta có:
;;
BD BF CE CD AF AE
ID IF IE ID IF IE
E
F
I
A
B
C
D
24
Ta có:
11
BC BD CD BF CE
ID ID ID IF IE
Ta có:
22
AC AE CE AF CE
IE IE IE IF IE
T (1) và (2) cng vế vi vế, suy ra:
BC AC BF AF AB
ID IE IF IF IF
.
Ví d 8: Cho tam giác
ABC
, đường phân giác
AD
. Đặt
,AC b AB c
. Chng minh rng:
2bc
AD
bc
Gii
Cách 1: Qua
D
k đường thng song song vi
AB
ct
AC
E
.
Ta có:
1 1 2
D A A
nên
AE DE
. Ta tính
DE
theo b và c
Do
DE AB
nên theo định lí Talet thì:
1
DE DC
AB BC
Theo tính chất đường phân giác
DC AC b
DB AB c

Nên
DC b
DC DB b c

. Tc là:
2
DC b
BC b c
T (1) và (2) suy ra
DE b
c b c
. Do đó
bc
DE
bc
Tam giác
ADE
có:
2
2
bc
AD AE DE DE
bc
Cách 2: (không dùng tính chất đường phân giác). Qua
B
k đường thng song song vi
AD
, cắt đường thng
AC
K
.
1
2
1
E
D
A
B
C
25
Ta có:
1 2 1 1 1 1
;K A B A K B
ABK
cân ti
K
nên
AK AB c
Do
BK AD
nên theo định lý Talet thì:
1
AD AC b b
AD BK
BK KC b c b c

Tam giác
ABK
2BK AB AK c
(2)
T (1) và (2) suy ra:
2bc
AD
bc
Nhn xét: t kết lun bài toán, suy ra:
1 1 1 1 1
22
bc
AD bc AD b c



Tương tự như vậy đối với đường phân giác góc
B
góc
C
, thì chúng ta giải đước bài toán hay khó sau.
Cho tam giác ABC. Gi l
a
, l
b,
l
c
đ dài đường phân giác góc
A Bv C
. Đặt
,,BC a AC b AB c
. Chng
minh rng:
1 1 1 1 1 1
a b c
l l l a b c
.
Ví d 9: Cho
ABC
AD
là đường phân giác,
I
là giao điểm 3 đường phân giác và
K
là trung điểm
AB
.
Biết
0
90KIB
. Chng minh rng:
3AB AC BC
.
Gii
2
1
1
1
K
D
A
B
C
26
Trên BA lấy điểm E sao cho BE = BD
Ta có:
BDE
cân ti
B
BI
là đường phân giác nên
BI BE
Do đó
,1
KE DI
DE KI BI
KA AI
Áp dng tính chất đường phân giác trong
ABD
,
ACD
Ta có:
2
BD ID CD
BA IA CA

Do đó
3
BD CD BC
BA CA BA CA

T (1) và (2) suy ra:
2.
KE BD BE BE
KA BA BA KA
. Hay
2KE BE
T (3) và (4) suy ra:
1
3.
3
BC
AB AC BC
BA CA
C. Bài tp vn dng
14.1. Cho tam giác ABC, đường phân giác AD. Biết rng BC = 10cm và
3AB AC
. Tính độ dài BD và CD.
14.2. Gọi AI là đường phân giác ca tam giác ABC; IM, IN th t là các đường phân giác ca góc AIC và góc
AIB. Chng minh rng:
. . . .AN BI CM BN IC AM
K
I
D
C
B
A
E
27
14.3. Cho tam giác ABC chu vi bằng 18cm. Đường phân giác ca góc B ct AC tại M, đường phân giác ca
góc C ct AB ti N. Biết rng:
13
;
24
MA NA
MC NC

, tính độ dài các cnh ABC.
14.4. Cho ABC vuông tại A. Đường cao AH và đường phân giác BE ct nhau ti I. Chng minh rng:
2.CE HI
14.5. Cho hình cha nht
. Gi
M
là trung điểm
,AD N
là trung điểm
.BC
Trên tia đối ca tia
DC
lấy điểm
P
, đường thng
PM
ct
AC
ti
Q
và ct
BC
ti
S
. Đường thng
QN
ct
DC
ti
R
. Chng
minh rng:
a)
NPR
là tam giác cân. b)
MQ SQ
MP SP
.
14.6. Cho
ABC
,,AM BN CP
là các đường phân giác. Đặt
; ; .BC a AC b AB c
Chng minh rng:
2
MNP
ABC
S
abc
S a b b c c a
.
14.7. Cho
ABC
4 ; 6 ; 8AB cm BC cm CA cm
. Gi
I
là giao điểm ba đường phân giác ca tam giác
ABC
G
là trọng tâm. Tính độ dài đoạn thng
.IG
14.8. Cho hình bình hành
A
D AB
các điểm
,MN
lần lượt thuc
,AB AD
sao cho
BM DN
.
Gi
O
là giao điểm ca
BN
DM
. Đường thng
CO
cắt đường thng
AB
AD
theo th t
I
K
.
Chng minh rng:
;CD DK BI BC
14.9. Cho tam giác
ABC
vuông ti
A
. Có đường cao
AH
, đường trung tuyến
BM
và đường phân giác
CD
đồng quy ti
.O
Chng minh rng
BC BH
AC CH
.
14.10. Cho tam giác
ABC
vuông ti
A
. Hai đường phân giác
BD
CE
ct nhau
O
. Biết s đo diện tích
tam giác
BOC
bng
a
. Tính tích
.BDCE
theo
a
.
14.11. Cho tam giác
ABC
3BAC ACB
. Các điểm
,DE
thuc cnh
BC
sao cho
.BAD DAE EAC
Gi
M
là điểm thuc cnh
AB
,
MC
ct
AE
ti
I
, gi
K
là giao điểm
ME
AD
. Chng minh rng
/ / .KL BC
14.12. Cho tam giác
ABC
với đường trung tuyến
CM
. Điểm
D
thuộc đoạn
BM
sao cho
2BD MD
. Biết
rng
MCD BCD
. Chng minh rng:
ACD
là tam giác vuông.
28
29
A
B
C
A'
B'
C'
CHUYÊN ĐỀ 15. CÁC TRƯỜNG HỢP ĐỒNG DẠNG CỦA TAM GIÁC
A. Kiến thc cn nh
1. Khái niệm hai tam giác đồng dng
a. Định nghĩa
' ' 'A B C
được gọi là đồng dng vi
ABC
nếu:
' ; ' ; 'A A B B C C
' ' ' ' ' 'A B A C B C
AB AC BC

b. Tính cht
- Mỗi tam giác đồng dng vi chính nó.
- Nếu
' ' 'A B C ABC
thì
' ' 'ABC A B C
- Nếu
' ' ' " " "A B C A B C
" " "A B C ABC
thì
' ' 'A B C ABC
c. Định lí
Nếu một đường thng ct hai cnh ca tam giác và song song vi cnh còn li thì nó to thành mt
tam giác mới đồng dng với tam giác đã cho.
//
ABC
AMN ABC
MN BC
Chú ý: Định lý cũng đúng cho trường hợp đường thng ct phn kéo dài hai cnh ca tam giác và song song vi
cnh còn li.
2. Trường hợp đồng dng th nht
Nếu ba cnh ca tam giác này t l vi ba cnh ca tam giác kia thì hai tam giác đó đồng dng.
Nếu
ABC
' ' 'A B C
có:
' ' ' ' ' '
AB AC BC
A B A C B C

' ' 'ABC A B C
3. Trường hợp đồng dng th hai
Nếu hai cnh ca tam giác này t l vi hai cnh ca tam giác kia và hai góc to bi các cp cạnh đó
bằng nhau, thì hai tam giác đó đồng dng.
A
B
C
N
M
30
A
B
C
A'
B'
C'
A'
B'
C'
A
B
C
Nếu Nếu
ABC
' ' 'A B C
có:
' ' ' '
';
A B A C
AA
AB AC

' ' 'A B C ABC
4. Trường hợp đồng dng th ba
Nếu hai góc ca tam giác này lần lượt bng hai góc của tam giác kia thì hai tam giác đó đồng dng
vi nhau.
Nếu
ABC
' ' 'A B C
có:
'; 'A A B B
' ' 'A B C ABC
B. Mt s ví d
Ví d 1. Cho t giác li
BAC CAD
.ABC ACD
Hai tia
AD
BC
ct nhau ti
E
.
Chng minh rng
..AB DE BC CE
.
Gii
Tìm cách gii. Để chứng minh đẳng thức tích, thông thường chúng ta biến đổi chúng dưới dng tng t l
thc và chng minh t l thc y. Vậy để chng minh
..AB DE BC CE
chúng ta cn chng minh
.
AB CE
BC DE
Nhn thy t s
AB
BC
có th vn dụng được tính chất đường phân giác và ta có
.
AB AE
BC CE
Do vy
chúng ta cn chng minh
.
CE AE
DE CE
T đó chứng ta tìm cách chng minh
,CDE ACE
vy ch cn
chng minh
ECD BAC
là xong.
Trình bày li gii.
BAC CBA ECA
( góc ngoài tam tam giác) và
ABC ACD
nên
ECD BAC
Do đó
.C
DE ACE g g
, suy ra
1
CE AE
DE CE
B
A
E
C
D
31
Trong
ABE
AC
là đường phân giác suy ra
2
AE AB
CE BC
T
1
2
suy ra
. . .
AB CE
AB DE BC CE
BC DE
Ví d 2. Cho tam giác
ABC
vuông ti
A
có điểm
D
nm gia
A
C
. Qua
C
dng
CE
vuông góc vi
đường thng
BD
ti
E
. Chng minh:
a)
ADE BDC
.
b)
. . .ABCE AE BC AC BE
.
Gii
Tìm cách gii.
-
ADE
BDC
ADE BDC
; để tìm mt cp góc na bng nhau thật khó khăn. Do đó chúng ta tìm
cách tìm cách chng minh cp cnh k vi cp góc trên t l thông qua hai tam giác khác. Chng hn cn có
DA DE
DB DC
chúng ta nên chng minh
.ABD ECD
- Để chng minh
. . . ,ABCE AE BC AC BE
ta có vế trái là mt tng nên vế phi ta cn tách thành mt tng:
. . .AC BE AC x AC y
vi
.x y BE
Do vy ta chọn điểm
F
thuc
BD
khi đó
,x BF y FE
chng minh
. . , . . .ABCD AC BF AD BC AC FE
T đó chúng ta chỉ cn chọn điểm
F
sao cho
,ABF ACE AFE ABC ∽∽
là xong.
Trình bày li gii.
a) Xét
ABD
ECD
0
;
90ADB EDC BAD CED gt
.
DA DE
ABD ECD g g
DB DC
ADE
BDC
;
DA DE
ADE BDC
DB DC

Suy ra
ADE BDC
b) Cách 1. Gi
M
là giao điểm
AB
CE
.
Xét
, ta có
M
chung;
0
90 .
MB MC
MEB MAC MBE MCA g g
ME MA
32
Xét
,
MB MC
M
ME MA
chung
..M
AE MCB c g c
MEA MBC
Ly
F BE
sao cho
.AF AE
Xét
ABF
ACE
có:
00
90 ; 90BAF CAE DAF ABF ACE M
. . . 1
AB BF
ABF ACE g g ABCE AC BF
AC CE
Xét
AFE
ABC
0
90 ;EAF BAC AEF ACB
( cùng ph vi hai góc bng nhau)
. . . 2
AE EF
AFE ABC g g AE BC AC EF
AC BC
T
1
2
cng vế vi vế:
.
. .AB CE AE BC AC BF EF AC BE
Cách 2. Gi
J
là điểm trên cnh
Ac
sao cho
.ABJ EBC
Xét
ABJ
EBC
có:
90 ;
o
BAC BEC ABJ EBC
.A
BJ EBC g g
. . 3
AB AJ
ABCE BE AJ
BE CE
Xét
ABE
JBC
có:
;
AE BE
ABE JBC AEB JCB ABE JBC
JC BC
.
. 4AE BC BE JC
T
3
4
cng vế vi vế:
.
. .AB CE AE BC BE AJ JC BE AC
Ví d 3. Cho tam giác ABC có
2 ; 3 ; 4AB cm AC cm BC cm
. Chng minh rng
2.BAC ABC ACB
Gii
J
E
B
A
C
D
33
Tìm cách gii. V mt suy lun, mun chng minh mt góc
BAC
thành tổng các góc như đề bài. Ta hai
cách nghĩ:
Cách 1. Trong góc
BAC
dng mt góc
BAD
hoc
DAC
bng góc
ABC
chng minh phn còn li bng
. Tuy nhiên cách này vn gặp khó khăn bởi còn h s 2.
Cách 2. Trong góc
BAC
dng mt góc
BAD
bng góc
ACB
chng minh phn còn li bng
DAC ABC ACB
. Cách này tính kh thi. Tht vy, ta viết
BAC ABC ACB
nên nếu ly điểm D
trên cnh BC sao cho
BAD ACB
, thì d dàng nhn thy
ADC BAD ABC
hay
ADC ACB ABC
nên chúng ta ch cn chng minh tam giác
ACD
cân ti C là xong.
Vi suy luận như trên, chúng ta có hai cách trình bày như sau:
* Trình bày li gii
Cách 1. Trên đoạn thng
BC
lấy đim D sao cho
BAD ACB
suy ra
.A
BD CBA g g
. Suy ra
2
13
24
BD AB BD
BD cm CD BC BD cm
BA CB
CD AC
nên
ACD
cân ti C, do vy
DAC ADC
ADC ABC BAD
(tính cht góc ngoài ca tam giác)
Suy ra:
BAC BAD DAC ACB ADC ACB ABC BAD
Do đó
2.BAC ABC ACB
Cách 2. Trên đoạn thng BC lấy điểm D sao cho BD = 1 cm
3CD BC BD cm CD AC
nên tam giác ACD cân ti C
Do vy
1D
AC ADC
ABD
CBA
ABD
chung và
1
2
BD AB
BA CB

Suy ra
..A
BD CBA c g c
2B
AD BCA
A
B
C
D
34
T (1) và (2) ta có:
BAC BAD DAC ACB ADC ACB ABC BAD
Do đó
2.BAC ABC ACB
d 4. Cho tam giác ABC (AB = AC) góc đỉnh bng
20
o
; cạnh đáy BC = a; cnh bên AB = b. Chng
minh rng
3 3 2
3a b ab
.
Gii
Cách 1. Dng tia
Bx
na mt phng b BC chứa điểm A sao cho
20
o
CBx
; Tia Bx ct AC D; k
AH Bx
. Tam giác ABC cân ti A, ta có:
20 80 80 20 60
o o o o o
A B C ABH ABC CBx
Suy ra:
60 ; 90
22
oo
AB b
ABH AHB BH
Ta có:
2 2 2
AH AB BH
nh lý Pi ta go)
22
22
3
44
bb
AH b
BDC
80 ; 20 80
o o o
BCD CBD BDC
BCD
cân ti B
BD BC a
Do đó
2
b
DH BH BD a
Nhn thy:
.
BC AC
ABC BDC g g
CD BC
22
BC a
CD
AC b
, mà
2
a
AD AC CD b
b
2
2
2 2 2 2 2
3
42
bb
AD AH DH a b ab a



Vy
2
2
2 2 4 4 2 2 4 3 2 2
2
a
b b ab a b a a b b ab a b
b



3 3 2 2 3 3 2
33a a b a b a b ab
x
D
H
B
C
A
35
Cách 2.
Dng tam giác
ABE
đều sao cho E và C nm cùng phía so vi AB. Dng
ACD
cân ti A sao cho D, E nm
cùng phía vi AC và
2
0 . .
o
CAD ABC ACD ADE c g c
Gọi F G là giao đim ca BE với AD; AC. Khi đó
BG EF a
.
60
o
ABE
nên
20
o
CBG BAC CBE
CBG
cân ti B.
2
.
BC CG a CG a
BAC CBG g g CG
AC BG b a b
Ta có:
2
a
AG AC CG b
b
Ta có:
//FG CD
nên theo định lý Ta lét, ta có:
2
23
2
a
b
GF AG GF ab a
b
GF
CD AC a b b
BE BG GF FE
23
3 2 2 3
2
22
ab a
b a b ab ab a
b
3 3 2
3a b ab
d 5. Cho hình thoi ABCD,
60
o
A
. Gi M mt cnh thuc cạnh AD. Đường thng CM cắt đưng
thng AB ti N.
a) Chng minh
2
.AB DM BN
b) BM ct DN ti P. Tính góc
BPD
.
Gii
F
G
D
E
B
C
A
36
a) Ta có:
//AM BC
(do AD // BC), suy ra
NA NB
NAM NBC
AM BC
Hay
1
NA NB
AM AB
(vì BC = AB)
Ta có:
//NA DC
(Do AB // DC),
suy ra
NA CD
NAM CDM
AM DM
Hay
2
NA AB
AM DM
(vì CD = AB)
T (1) và (2) suy ra:
NA AB
AB DM
hay
2
.AB DM BN
b) T
NB AB NB BD
AB DM BD DM
Xét
BND
DBM
NB BD
BD DM
60
o
NBD BDM
Suy ra
..B
ND DBM c g c MBD BND
60
o
MBD MBN BND MBN
BPD BND MBN
nên
60
o
BPD
Nhn xét. Với thuật như trên, bn có th gii bài toán sau. Cho hình thoi ABCD có
60
o
A
v đường thng
qua C cắt tia đối ca tia BA ti M và cắt tia đối ca tia DA ti N. Gọi K là giao điểm ca DM và BN. Tính s đo
MKB
d 6. Cho
ABC
cân ti A. Ly M y ý thuc BC, k MN song song vi AB (vi
N AC
), k MP song
song vi AC (vi
P AB
). Gọi O là giao điểm ca BN và CP. Chng minh rng
OMP AMN
Gii
Tìm cách gii. Nhn thy
BPM MNC QPM ANM
P
N
D
A
C
B
M
37
OMP AMN QPM ANM
. M t khác chúng ta th y
QPM
ANM
khó th tìm
c m t c p góc n a b ng nhau. Do v y chúng ta nên tìm cách bi i thêm hai c p c nh k v i hai góc
;
OMP AMN
t l là xong.
Trình bày l i gi i
Gi s
MB MC
. G m CP và MN
Vì MNAP là hình bình hành nên
1
QPM ANM
ABC
cân t i A nên suy ra
PBM
cân t i P và
NCM
cân t i N.
PB PM AN
NC NM AP
k t h p v i
/ /
MN AP
, suy ra:
PQ PQ KM PB NA
PM PB KN PA NM
(2)
T (1) và (2) suy ra:
. .
QPM ANM c g c
QMP AMN
hay
OMP AMN
Ví d 7. Cho tam giác ABC có
2. , 4 , 8
B C AB cm AC cm
dài c nh BC?
Gi i
Tìm cách gi i. Khai thác gi thi t, t
2.
B C
chúng ta c n d ng thêm y u t ph v n d c.
ng gi i:
- Cách 1. K ng phân giác BD c c góc b ng nhau.
- Cách 2. T nh C d ng thêm m t góc b ng góc B, V i gi i sau:
Trình bày l i gi i:
Cách 1. K ng phân giác
BD
c a tam giác
ABC
.
Xét
ABC
ADB
A
chung và
2
ABC
ACB ABD
K
Q
P
N
B
C
A
M
38
Suy ra
ABC
đồng dng
ADB
(g.g)
22
4
2(cm).
8
AB AC AB
AD
AD AB AC
6( )CD cm
.
ABC
có đường phân giác
BD
nên:
. 4.6
12(cm).
2
BC CD ABCD
BC
AB AD AD
Cách 2. Trên na mt phng b
BC
không chứa điểm
A
dng tia
Cx
sao cho
BCx ACB ACB ABC
Gọi I là giao điểm ca Cx với đường thng AB.
Xét
ABC
ACE
A
chung ,
2ABC ACE ACB
Suy ra
ABC
đồng dng
ACE
(g.g)
22
8
16(cm).
4
AC AB AC
AE
AE AC AB
12( )BE cm
T
22ABC ACB BCE
Suy ra
BCE
cân ti
B
Do đó
12( )BC BE cm
Ví d 8. Cho tam giác
ABC
2AB cm
,
3AC cm
,
2,5BC cm
. Chng minh rng
2BC
.
Gii.
Tìm tòi cách gii. Bài toán y có nét đo ca d 7, do đó hoàn toàn tự nhiên chúng ta cũng nghĩ ti vic k
them yếu t phụ. Để chng minh
2BC
, chúng ta cũng có hai hướng sau.
Cách 1. Dng phân giác
BD
và chng t
ABD C
.
Cách 2. T đỉnh
C
dng thêm mt góc bng góc
B
và chng minh cp góc bng nhau.
bài toán biết khá nhiều độ dài đoạn thng nên chúng ta chng minh cp góc bng nhau bng cách chng
minh hai tam giác đồng dạng theo trường hp cnh góc cnh.
* Trình bày li gii.
C
D
A
B
E
C
B
A
39
Cách 1. K đường phân giác
BD
ca tam giác
ABC
suy ra:
AD AB AD AB
DC BC AD DC AB BC

24
(cm)
3 2 2.5 3
AD
AD
Ta có:
2 2 3
.
4
32
3
AB AC
AD AB
Suy ra:
AC AB
AB AD
Xét
ABC
ADB
A
chung,
AC AB
AB AD
suy ra
ABC
đồng dng
ADB
(c.g.c).
Do đó
ACB ABD
, vy
2ABC C
.
Cách 2. Trên tia đối ca tia
BA
lấy điểm
E
sao cho
BE BC
, suy ra
22ABC BEC BCE
Ta có:
2 3 2
;
3 2 2.5 3
AB AC
AC AE
Suy ra
AC AB
AE AC
Xét
ABC
ACE
A
chung,
AC AB
AE AC
suy ra
ABC
đồng dng
ADB
(c.g.c).
Do đó
ACE ABC
suy ra
2.ACE BCE ACB BCE
Hay
2.ABC ACB
.
d 9. Cho tam giác ABC
0
20B
. Các điểm
E
F
lần lượt nm trên các cnh
AC
AB
sao cho
0
10ABE
0
30ACF
. Tính
CFE
.
(Thi Olympic Toán quc tế Đài Loan TAIMC, năm 2012)
Gii
* Tìm tòi cách gii. Nhng bài toán tính s đo thường khó, trước hết chúng ta nên v hình chính xác, sau đó
phân tích gi thiết đ d đoán thuật k thêm yếu t ph. Trong gi thiết ta nhân thy
0
30 2ACF FC FA
. T
00
20 70BC
, khi đó
0
40BCF
chúng ta có liên tưởng gì góc
0
40
này vi
góc
0
20
0
30
đề bài không? Với suy nghĩ ấy ta lấy điểm
G
trên
AB
sao cho
0
20BCG
khi đó bài toán to
nên nhng yếu t mi:
CF
phân giác góc
ACG
, tam giác
BCG
cân ti
G
. Vi hình v chính xác chúng ta
hoàn toàn có th d đoán được
CG
song song vi
EF
. T đó định hướng để chng minh d đoán ấy bằng định
lý Ta lét đảo.
C
D
A
B
E
C
B
A
40
Trình bày li gii.
Xét
ABC
0
20B
0
70C
ACF
0
30ACF
2AFFC
Gi
D
trung điểm ca
BC
G
điểm nm
trên
AB
sao cho
GD
vuông góc vi
BC
.
Do đó
ABC
đồng dng
DBG
00
; 20 20
BD BA
GCB GBC GCF
BG BC
Mt khác
CG
BE
lần lượt là tia phân giác ca
BCF
ABC
nên
;
FC BC BA AE
FG BG BC EC

Do đó
11
22
FC BC
AF BD BA AE AF AE
FG FG BG BG BC EC FG EC
T đó ta có:
/ /EFCG
ịnh lý Ta lét đảo)
0
20CFE GCF
.
d 10. Cho tam giác
ABC
0
3 2 180AB
. Tính s đo các cạnh ca tam giác biết s đo y ba s t
nhiên liên tiếp.
Gii.
0
3 2 180 2.A B A B C C A B C A
;C B AB BC AB AC
Trên
AB
lấy điểm
D
sao cho
AD AC
D
nm gia
A
B
.
Ta có:
ACD
cân ti
A
nên
0
180
2
A
ADC
0
3 2 180AB
0
180 2( )A A B
2
2
AB
ADC A B
0
180CDB ADC C
Vy
ABC
đồng dng
CBD
(g.g)
2
. .(AB AC)
AB BC
BC AB BD AB
BC BD
(*)
Do
,,AB BC CA
là ba s nguyên liên tiếp và
a
x , ,AB m AB BC CA
nên
1AB BC
hoc
2AB BC
.
Trường hp 1. Nếu
1AB BC
thì
1AC BC
thay vào (*) ta
2
2. 2 0BC BC
, không tn ti
BC
s nguyên.
F
F
D
G
C
B
A
D
B
C
A
41
Trường hp 2. Nếu
2AB BC
thì
1AC BC
thay vào (*) ta có
2
20BC BC
(BC 2)(BC 1) 0 2BC
(vì
0BC
).
Vy
2; 3BC AC
và .
4AB
..
Nhn xét Vn dụng kĩ thuật trên, bn có th làm được bài toán đảo.
Cho tam giác
MNP
tha mãn
22
.0NP MN MP MN
. Chng minh rng
0
3. 2. 180 .MN
d 11. Cho tam giác
ABC
nhọn hai đường cao
BE
CF
. K
FI
EJ
cùng vuông góc vi
BC
(
I, J
thuc
BC
). Các điểm
K, L
lần lượt thuc
AB, AC
sao cho
/ /AKI C
,
/ / ABIJ
. Chng minh rằng ba đường
thng
, FJ, KLEI
đồng quy.
Gii.
Gi
O
là giao điểm ca
EI
FJ
, ta có:
KFI FCB
(cùng ph vi góc
IFC
) =
00
90 90 EJABC LJC L
(1)
Li có :
IKF ELJ
(cùng bù vi góc
BAC
) (2)
T (1) và (2) suy ra:
KFI
đồng dng
LJE
(g.g)
EJ
KF FI
LJ

(3)
Xét
KFI
JOE
IFO EJO
(so le trong)
FOI JOE
ối đỉnh) nên
KFI
đồng dng
JOE
(g.g)
Suy ra
EJ
FO FI
OJ
(4)
Li có
KFO LJO
(so le trong) (5)
T (3), (4) (5) suy ra
KFO
LJO
(c.g.c). Do đó
FOK JOL
, hai góc y v trí đối đỉnh. Suy ra
,,K L O
thng hang, tc là
, FJ, KLEI
đồng quy.
d 12. Cho hình thang
A
BCD CD AB
vi
//AB CD
AB BD
. Hai đường chéo
AC
BD
ct
nhau
G
. Trên đường thng vuông góc vi
AC
ti
C
lấy điểm
E
sao cho
CE AG
đoạn thng
GE
không cắt đường thng
CD
. Trên đoạn thng
CD
lấy điểm
F
sao cho
DF GB
.
a) Chng minh
FDG
đồng dng vi
ECG
.
b) Chng minh
EFGF
.
(Thi tuyn hc sinh gii lp 9, tnh Quảng An, năm học 2008 - 2009)
Gii
O
J
I
L
E
F
K
C
B
A
42
a) Ta có:
//AB CD
BG GD
AG GC

. Mà
CE AG
;
DF GB
DF GD
CE GC

Xét
FDG
ECG
có:
;
DF GD
GDF GCE
CE CG

nên
FDG
ECG
(c.g.c)
b)
FDG
ECG
12
;
GD GC
GG
GF GE
Xét
GDC
GFE
có:
;
GD GC
DGC FGE
GF GE

(vì
12
GG
)
GDC
GFE
(c.g.c)
0
90GFE GDC
. Do đó
GF FE
C. Bài tp vn dng
15.1. Cho tam giác nhn
ABC
, các đường cao
,,AD BE CF
ct nhau ti
H
.
a) Chng minh rng:
..AE AC AF AB
b) Chng minh rng:
AEF
~
ABC
c) Chng minh rng
H
là giao điểm của ba đường phân giác trong ca
DEF
15.2. Cho hình bình hành
ABCD
đường chéo
AC
lớn hơn
BD
. Gi
,HK
hình chiếu ca
C
trên đường
thng
,AB AD
. Chng minh rng:
CHK
~
BCA
15.3. Cho tam giác
ABC
vuông góc ti
A
đường phân giác
BD
cắt đường cao
AH
ti
I
. Chng minh
..AD BD BI DC
15.4. Cho tam giác
ABC
, đường phân giác
CD
. Chng minh rng
2
.CD CACB
15.5. Cho tam giác đều
ABC
. Trên tia
BA
ly đim
E
(
A
nm gia
B
E
). Gi
D
điểm đối xng vi
E
qua đường thng
BC
. Gi
F
giao điểm của đường thng
CD
AB
. Chng minh rng:
1 1 1
BC BD BF

15.6. Cho hình bình hành
ABCD
góc
A
tù. T
A
, v các đường thng vuông góc vi
BC
,
CD
ct
CD
,
BC
tương ứng ti
E
F
. Đường thng qua
A
vuông góc vi
BD
, ct
EF
ti
M
. Chng minh
ME MF
E
C
F
D
B
A
G
2
1
43
15.7. Cho tam giác đều
ABC
, gi
M
trung điểm ca
BC
. Mt góc
xMy
bng
0
60
quay quanh điểm
M
sao cho
2
cnh
,Mx My
luôn ct cnh
AB
AC
lần lượt ti
D
E
. Chng minh:
a)
2
.
4
BC
BDCE
;
b)
;DM EM
lần lượt là tia phân giác ca các góc
BDE
CED
;
c) Chu vi tam giác
ADE
không đổi.
15.8. Cho hình vuông
ABCD
. Trên cnh
AB
lấy điểm
M
. V
BH
vuông góc vi
CM
. Ni
DM
. Gi
HN
vuông góc vi
DH
()N BC
.
a) Chng minh rng tam giác
DHC
đồng dng vi tam giác
NHB
.
b) Chng minh rng
..AM NB NC MB
.
15.9. Cho tam giác
ABC
tha mãn
2.AB AC
2.AB
. Chng minh rng
ABC
là tam giác vuông.
15.10. Cho
ABC
nhn
AH
đường cao, lấy điểm
M
thuộc đoạn
BC
, k
MK
vuông góc vi
AB
ML
vuông góc vi
AC
. Đường thng qua
A
vuông góc vi
AM
ct
,MK ML
ti
E
F
.Tư
B
k
đường thng vuông góc vi
CE
ct
AH
ti
I
. Chng minh rng:
a)
AIB
đồng dng
MCE
;
b)
EM MI
FM KM
BM AI
FM AC
;
c)
,,AH BF CE
đồng qui.
15.11. Cho tam giác
ABC
các trung tuyến
,AD BE
thỏa mãn điều kin
0
30CAD CBE
. Chng minh
ABC
là tam giác đều.
15.12. Cho
ABC
. Gi
P
là giao điểm của ba đường phân giác trong ca tam giác. Một đường thẳng đi qua
P
vuông góc vi
CP
, ct
AC
BC
lần lượt ti
M
N
. Chng minh rng:
a)
2
AM AP
BN BP



;
b)
2
1
.
AM BN CP
AC BC AC BC
.
15.13. Cho tam giác
ABC
vuông ti A
A
C AB
, đường cao
AH
H
BC
. Trên tia
HC
ly điểm D sao
cho
HD HA
. Đường thng vuông góc vi
BC
ti
D
ct
AC
ti
E
.
a) Chng minh rng hai tam giác
BEC
ADC
đồng dạng. Tính độ dài
BE
theo
m AB
.
b) Gi
M
trung điểm của đon
BE
. Chng minh rng hai tam giác
BHM
BEC
đồng dng. Tính s đo
ca góc
AHM
.
c) Tia
AM
ct
BC
ti
G
. Chng minh:
GB HD
BC AH HC
.
15.14. Trong tam giác
ABC
, các điểm
tương ng nm trên các cnh
,,BC CA AB
sao cho:
, , AFE BFD BDF CDE CED AEF
.
a) Chng minh rng:
BDF BAC
.
b) Cho
5AB
,
8BC
,
7CA
. Tính độ dài đoạn
BD
.
44
15.15. Cho
ABCD
là hình bình hành. Gi s
MAB MCB
. Chng minh rng
MBC MDC
.
15.16. Gi s
D
là một điểm nm trong tam giác nhn
ABC
sao cho
90ADB ACB
AC BD AD BC
. Chng minh
2
AB CD
AD BC
.
15.17. Cho tam giác
ABC
cân ti
A
. T điểm
M
thuc cnh
BC
v
MB AB
;
MQ AC
;
;P
AB Q AC
. V
PE PQ
;
QE PQ
;E
F BC
. Chng minh rng:
BE CF
.
15.18. Cho tam giác
ABC
nhọn đường cao
BE
,
CF
. Qua
A
v các đường thng song song vi
BE
,
CF
lần lượt cắt các đường thng
CF
,
BE
ti
P
Q
. Chng minh rng:
PQ
vuông góc vi trung tuyến
AM
.
15.19. Cho tam giác
ABC
cân ti
A
20BAC
. Dựng tam giác đều
BDC
sao cho
D
,
A
cùng phía vi
BC
. Dng tam giác
DEB
cân ti
D
80EDB
C
,
E
cùng phía so vi
DB
. Chng minh tam giác
AEC
cân ti
E
.
15.20. Cho tam giác
ABC
90A
. Lấy điểm
D
thuộc đoạn thng
AC
sao cho
2CD AD
. Gi
E
điểm thuộc đoạn thng
BD
sao cho
CED ABC
. Gi
F
điểm đối xng vi
C
A
. Chng minh rng
2DEF ABC
.
45
CHUYÊN ĐỀ 16. CÁC TRƯỜNG HỢP ĐỒNG DẠNG CỦA TAM GIÁC VUÔNG
A. Kiến thc cn nh.
1. Hai tam giác vuông đồng dng nếu:
- Tam giác vuông này có mt góc nhn bng góc nhn ca tam giác vuông kia;
- Tam giác vuông này có hai cnh góc vuông t lê vi hia cnh góc vuông cu tam giác vuông kia;
- Nếu cnh huyn cnh góc vuông ca tam giác vuông này t l vi cnh huyn và mt cnh góc vuông ca
tam gaics vuông kia.
2. T s hai đường cao, t s din tích cảu hai tam giác đồng dng:
- T s hai đường cao tương ứng của hai tam giác đồng dng bng t s đồng dng.
- T s din tích cảu hai tam giác đồng dng bằng bình phương tỉ s đồng dng.
B. Mt s ví d
d 1. Cho tam giác nhn
ABC
đường cao
CK
. Dng ra phía ngoài tam giác
ABC
hai tam giác
CAE
CBF
tương ng vuông góc ti
E
;
F
tha mãn
ACE CBA
;
BCF CAB
. Chng minh rng
2
.CK AE BF
.
Li gii
Tìm cách gii. Để chng minh
2
.CK AE BF
chúng ta không th vn dụng định lý Ta-let hay xét mt cp tam
giác đồng dạng xong ngay được. Do vy, chúng ta suy luận để to ra
2
CK
, chúng ta cn ghép
CK
vào hai
cặp tam giác đồng dng. Mi cặp tam giác đồng dạng đó đều biu th
CK
dưới dng biu thc (cha
AE
hoc
BF
). D dàng nhn thy có hai cặp tam giác đồng dng thỏa mãn điều kin trên.
Trình bày li gii
F
E
K
A
C
B
ACK
CBF
90CKA BFC
;
CAK BCF
ACK CBF
(g.g)
(1)
CK BF
CA BC

.
Tương tự, ta có
EBCK CA
(g.g)
(2)
CK AE
CB AC

.
Nhân tng vế của (1) và (2) ta được:
..
CK CK BF AE
CA CB BC AC
2
.CK AE BF
.
d 2. Cho hình bình hành
ABCD
(
AC BD
). V
CE
vuông góc vi
AB
ti
E
, v
CF
vuông góc vi
AD
ti
F
. Chng minh rng
2
..AB AE AD AF AC
.
46
Li gii
Tìm cách gii. Để chng minh
2
..AB AE AD AF AC
, ta có vế trái là mt tng nên vế phi ta cn tách ra mt
tng:
. . . .AB AE AD AF AC x AC y
vi
x y AC
. Do vy ta chọn điểm
H
thuc
AC
khi đó
x AH
,
y HC
chng minh
..AB AE AC AH
,
..AD AF AC CH
. T đó chúng ta chỉ cn chọn điểm
H
sao cho
EABH AC
xong. Nhn thy tam giác
ACE
vuông ti
E
, nên tt yếu cn k thêm
BH
vuông góc vi
AC
.
Trình bày li gii
F
E
H
B
A
D
C
V
()BH AC H AC
Xét
ABH
ACE
90AHB AEC
;
BAC
chung.
Suy ra
EABH AC
(g.g)
AB AH
AC AE

. . (1)AB AE AC AH
Xét
CBH
ACF
BCH CAF
(so le trong);
0
90CHB CFB
Suy ra
. . . (2)
BC CH
CBH ACF g g BC AF AC CH
AC AF
Cng v theo vế (1) và (2) ta được:
2
. . . . . . ( )AB AE BC AF AC AH AC CH AB AE AD AF AC AH CH AC
.
d 3. Cho tam giác
ABC
vuông ti
A
. Ly một điểm
M
bt k trên cnh
AC
. T
C
v một đường thng
vuông góc vi
BM
, đường thng này ct tia
BM
ti
D
, ct tia
BA
ti
E
.
a) Chng minh:
..EA EB ED EC
.
b) Chng minh rằng khi điểm M di chuyn trên cnh AC thì tng
..BM BD CM CA
có giá tr không đổi.
c) K
, ( )DH BC H BC
. Gi
,PQ
lần lượt trung đim của các đoạn thng
,.BH DH
Chng minh
CQ PD
.
Gii
47
a) Chng minh
..EA EB ED EC
.
Xét
EBD
ECA
có :
0
90EDB EAC
,
BEC
chung nên
EBD ECA
(g-g)
T đó suy ra
..
EB ED
EAEB ED EC
EC EA
.
b) K MI vuông góc vi
()BC I BC
.
Ta có:
BIM
BDC
0
90BIM BDC
,
MBC
chung nên:
BIM BDC
(g-g)
..
BM BI
BM BD BC BI
BC BD
(1)
Tương tự:
ACB ICM
(g-g)
..
CM CI
CM CA BC CI
BC CA
(2)
T (1) và (2) cng vế vi vế, suy ra:
2
. . . . ( )BM BD CM CA BI BC CI BC BC BI CI BC
(không đổi).
c) Xét
BHD DHC
(g-g)
2.
2.
BH HD HP HD HP HD
DH HC HQ HC HQ HC
HPQ HQC
(c-g-c)
PDH QCH
00
90 90HDP DPC HCQ DPC CQ PD
.
d 4. Cho tam giác
ABC
. Lấy đim
,,E F P
lần t thuc
,,AB AC BC
sao cho
BEFP
hình bình hành.
Biết rng din tích
AEF
CFP
l t là
22
16 ;25cm cm
. Tính din tích
ABC
.
Gii
Tìm cách gii. Khi v hình xong, chúng ta có hai hướng suy lun:
B
C
A
D
M
E
H
P
Q
48
tam giác
,AEF FPC
cùng đồng dng vi tam giác
ABC
nên chúng ta tìm mi liên h gia t s hai tam giác
đồng dng.
ng th hai, đểnh din tích tam giác
ABC
, nên chúng ta tìm cách tính dinch hình bình hành. Nhn thy
tam giác
BEF
BPF
din ch bng nhau , mt khác tam giác
AEF
BEF
chung đưng cao k t
F
;
tam giác
BPF
CPF
chung đường cao k t
F
. S dng tính chất đó, kết hp với định lý Ta-lét, chúng ta
có li gii hay.
Trình bày li gii
Cách 1. Ta có:
;AEF ABC FPC ABC ∽∽
nên:
2
2
AEF
AEF
ABC
ABC
FPC
FPC
ABC
ABC
S
S
EF EF
S BC BC
S
S
S
CP CP
S BC BC
S






T đó suy ra
1
AEF EPC
ABC
SS
EF CP
BC BC
S
Hay
22
4 5 9 81
ABC AEF FPC ABC
S S S S cm
.
Cách 2. Đặt
2
BEF BEP
S S x cm
Tam giác
AEF
BEF
có chung đường cao k t
F
, suy ra:
16
FAE
FBE
S
AE AE
S BE x BE
;
Tam giác
BPF
CPF
có chung đường cao k t F, suy ra:
25
FBP
FPC
S
BP x BP
S CP CP
.
Áp dụng định lý Ta-lét, ta có:
2
16
400 20
25
AE AF BP x
xx
BE FC CP x
.
Vy
2
16 20 20 25 81
ABC
S cm
.
Nhn xét. T kết qu
2
2 2 2
( ) 2
ABC AEF FPC ABC BEFP
S S S S a b S a b a b ab
B
C
A
E
F
P
49
T đó ta có thể giải được bài toán sau:
Cho tam giác
ABC
. Ly điểm
,,E F P
lần lượt thuc
,,AB AC BC
sao cho
BEFP
hình bình hành. Đặt
22
;
AEF CFP
S a S b
(vi
;0ab
).
a) Tính din tích hình bình hành
BEFP
.
b) Xác định v trí điểm
,,E F P
trên
,,AB AC BC
để din tích hình bình hành
BEFP
đạt giá tr ln nht.
Ví d 5. Cho tam giác
ABC
. Qua điểm
F
nm trên tam giác k
// ; // ; //MN BC PQ AB IK AC
,
,
; , ; ,I M AB N P AC Q K BC
Biết rng:
2 2 2
9 ; 16 ; 25
IMF PFN FQK
S cm S cm S cm
. Tính din tích
ABC
.
Gii
m cách gii. Vi lối tư duy như ví d trên, chúng ta hoàn toàn nghĩ ti hai cách gii. Song trong ví d này
s trình y mt cách gii, bn cht ca bài toán vn dng kết qu
1
MF QK FN
BC BC BC
kết hp vi t s
din tích của hai tam giác đồng dng.
Trình bày li gii
Nhn thy
,BMFQ CNFK
là hình bình hành.
Ta có:
;;FQK ABC IMF ABC PFN ABC
Thì
IMF
ABC
S
MF
BC
S
;
FQK
ABC
S
QK
BC
S
PFN
ABC
S
FN
BC
S
1
IMF EQK PEN
ABC
S S S
MF QK FN
BC
S


3 5 4 12
ABC IMF FQK PFN
S S S S
2
144
ABC
S cm
.
F
B
C
A
M
N
P
Q
K
I
50
Nhn xét. Như vậy vi cách giải trên, chúng ta hoàn toàn đưc bài toán tng quát sau: Cho tam giác
ABC
.Qua
điểm
F
nm trong tam giác k
// ; // ; // ( , ; , ; , )MN BC PQ AB IK AC I M AB N P AC Q K BC
.
Đặt
2 2 2
; ; ( ; ; 0)
IMF PFN PQK
S a S b S c a b c
. Chng minh rng:
2
()
ABC
S a b c
.
Ví d 6. Cho tam giác
ABC
. Qua điểm F nm trên trong tam giác k
/
/ ; // ; // , ; , ; ,MN BC PQ AB IK AC I M AB I P AC Q K BC
. Đặt din tích tam giác
ABC
S
.
Tìm v trí điểm
F
để tng
APQF CNFK MPQF
T S S S
đạt giá tr ln nht.
Gii
Tìm cách gii. Tương tự ví d trên, chúng ta đặt:
2 2 2
; ; ; ; 0
IMF PEN FQK
S a S b S c a b c
Chúng ta hoàn toàn biu th tng
APFI MPQF CNFK
T S S S
theo
,,abc
. Vy hiển nhiên đ tìm giá tr ln
nht chúng ta dùng cc tr đại s vi chú ý rng
2
1
()
3
ab bc ca a b c
.
Trình bày li gii
Đặt
2 2 2
; ; ; ; 0
IMP PFN FQK
S a S b S c a b c
Ta có:
ABC IMF FQK PFN
S S S S
Hay
2
ABC
S a b c
2 M
PQF CNFK ABC IMK PFN FQK
S S S S S S S
2
2 2 2
2
()
22
2( ) .( ) .
33
T a b c a b c
T ab bc ca a b c S
Vy
2
.
3
TS
khi
abc
hay
F
là trng tâm ca tam giác
ABC
.
F
B
C
A
M
N
P
Q
K
I
51
Ví d 7. Cho tm bìa hình thanh
ABCD
0
90 , 24 ; 32 ; 64A D AD cm AB cm CD cm
. Gp tm bìa li
để cho hai điểm
C
B
trùng nhau. Tính độ dài ca nếp gp.
Gii
Tìm cách gii. Trước hết chúng ta y v xác định đường nếp gp: Gi
M
là độ dài trung điểm
BC
, qua
M
k đường thng vuông góc vi
BC
, ct
CD
ti
N
, Độ dài nếp gp cn tính chính là đ dài đoạn thng
MN
.
T đề bài
0
90 , 24 ; 32 ; 64A D AD cm AB cm CD cm
, d dàng tính được độ dài
BC
bằng định lý Py-ta-
go. T đó tính được độ i
CM
. Do vậy đ tính được
CM
trong tam giác vuông
CMN
, chúng ta ch cn tính
được độ dài hai cnh ca một tam giác vuông đồng dng vi tam giác vuông
CMN
là xong. T đó, chúng ta có
hai cách v thêm đường ph:
Cách 1: Vì
0
90AD
nên ch cn gọi giao điểm
DA
CB
E
. Sau đó tính độ dài cnh ca tam giác vuông
CDE
.
Cách 2. K
BF
vuông góc vi
CD
, khi đó
MCN FCB
. Bài toán cũng được gii.
* Trình bày li gii
Gi
M
là trung điểm ca
BC
, qua
M
k đường thng vuông góc vi
BC
, ct
CD
ti
N
. Độ dài nếp gp
cần tính chính là độ dài đoạn thng
MN
.
Cách 1. Gi
E
là giao điểm ca
AD
BC
;
F
là chân đường vuông góc k t
B
ti
CD
.
D thy
F
là trung điểm ca
CD
, t đó ta có:
2 2 2 2 2
24 32 1800 BC BF FC
Suy ra
4
20 BC cm MC cm
.
Suy ra
B
A
lần lượt là trung điểm ca
CE
DE
,
Suy ra
2 48DE AD cm
.
Ta nhn thy
MCN DCE
.
Nên
20
15
64 48
MC MN MN
MN cm
DC DE
Vậy độ dài nếp gp là
15cm
.
Cách 2. Ta có:
MCN FCB
suy ra:
20
15
32 24
MC MN MN
MN cm
CF BF
.
Vậy độ dài nếp gp là
15cm
.
Ví d 8. Cho tam giác
ABC
cân ti
A
. Trên
AB
lấy điểm
D
và trên
BC
lấy điểm
E
sao cho hình chiếu ca
DE
lên
BC
bng
1
2
BC
. Chng minh rằng đường vuông góc vi
DE
ti
E
luôn đi qua một điểm c định.
Gii
52
Gi
,MH
lần lượt là hình chiếu vuông góc ca
D
A
trên
BC
. Gi s đường thng qua
E
vuông góc vi
DE
cắt đường
thng
AH
ti
N
.
Ta có:
1
2
BH BC BM HE
.
Mt khác ta có:
HNE MED
(cùng ph vi
HEN
);
DME NHE
, nên
HNE MED
.
22
HN HE HN HE HN BM
ME DM BC DM BC DM
.
Mt khác ta có:
2.
2
BM BH HN BH BH BC
HN
DM HA BC HA HA
.
Vậy điểm
N
là điểm c định.
Nhn xét: Đim mu cht của bài là khai thác điều kiện “Hình chiếu ca
DE
bng
1
2
BC
” để t đó xác định
vic k thêm đường ph.
C. Bài tp vn dng
16.1. Cho tam giác
ABC
có hai góc
B
C
thỏa mãn điều kin
90
o
BC
. K đường cao
AH
. Chng minh
rng:
2
.AH BH CH
.
16.2. Cho tam giác nhn
ABC
có hai đường cao
BD
CE
ct nhau ti
H
. Chng minh rng:
2
..BH BD CH CE BC
.
16.3. Cho tam giác
ABC
cân ti
A
90
o
A
, đường cao
AD
, trc tâm
H
. Chng minh h thc:
2
.CD DH DA
.
16.4. Cho t giác
ABCD
90
o
ABD ACD
. Gi
,IK
th t là hình chiếu ca
,BC
trên cnh
AD
. Gi
M
là giao điểm ca
CI
BK
,
O
là giao điểm ca
AC
BD
. Chng minh rng
OM AD
.
16.5. Cho
ABC
c định có các góc
,BC
nhn và hình ch nht
MNPQ
thay đổi nhưng luôn có
,MN
trên
cnh
BC
còn
,PQ
lần lượt trên cnh
AC
AB
. Xác định v trí của các đỉnh
,PQ
sao cho hình ch nht
MNPQ
có din tích ln nht.
16.6. Cho tam giác
ABC
vuông ti
A
. Hình ch nht
MNPQ
thay đổi tha mãn
M
thuc cnh
AB
,
N
thuc
cnh
AC
,PQ
thuc cnh
BC
. Gọi giao điểm ca
BN
vi
MQ
K
, ca
CM
vi
NQ
L
. Chng
minh rng
KAB LAC
.
53
16.7. Cho tam giác
ABC
vuông ti
A
. Mt hình vuông ni tiếp tam giác
ABC
vi
D
thuc cnh
AB
,
E
thuc cnh
AC
,FG
thuc cnh
BC
. Gi
H
là giao điểm ca
BE
DG
,
I
là giao điểm ca
CD
EF
. Chng minh rng
IE HG
.
16.8. Cho hình vuông
ABCD
,
F
là trung điểm ca
AD
E
là trung điểm ca
FD
. Các đường thng
BE
CF
ct nhau ti
G
. Tính t s din tích ca tam giác
EFG
vi din tích hình vuông
ABCD
.
16.9. Cho hình ch nht
ABCD
có din tích
3
150cm
(như hình vẽ). gi
,EF
là trung điểm
AB
BC
. Gi
,MN
là giao điểm ca
,DE DF
vi
AC
. Tính tng din tích phần tô đâm.
16.10. Cho tam giác
ABC
vuông ti
A
, đường cao
AH
. Biết
2
3
AB
AC
. Tính t s
HB
HC
.
16.11. Cho tam giác nhn
ABC
,,AD BE CF
là các đường cao ct nhau ti
H
. Chng minh rng:
. . .
1
. . .
HB HC HC HA HA HB
AB AC BC BA CACB
.
16.12. Trong hình v dưới đây các tam giác
ABC
CDE
có din tích
bng nhau và
F
là giao điểm ca
CA
DE
. Biết
AB
song song vi
DE
,
9AB cm
6EF cm
. Tính độ dài theo
cm
ca
DE
.
(Olympic Toán hc tr quc tế Bulgaria (BICMC), năm 2013 –
Philippines đề ngh)
16.13. Cho hình vuông
ABCD
. Gi
,QE
lần lượt là trung điểm ca
,AB BC
. Gi
M
là giao điểm ca
DE
CQ
. Gi
I
là giao điểm ca
AM
BC
. Chng minh rng
4AM MI
.
16.14. Gi s
,AD BE
CF
là các đường phân giác ca tam giác
ABC
. Chng minh rng tam giác
ABC
đều
khi và ch khi din tích ca tam giác
DEF
bng
1
4
din tích tam giác
ABC
.
(Tuyn sinh lp 10, THPT chuyên, tỉnh Hòa Bình, năm học 2013 2014)
16.15. Cho tam giác
ABC
vuông cân,
,
CM
là trung tuyến. T
A
v đường thng vuông góc vi
MC
ct
BC
H
. Tính t s
BH
HC
.
54
(Tuyn sinh lớp 10, THPT chuyên ĐHKHTN Hà Nội, năm hc 1989 1990)
16.16. Cho tam giác
ABC
vuông ti
A
AH
là đường cao. Biết rng chu vi tam giác
,ABH ACH
lần lượt là
30 cm
,
40 cm
. Tính chu vi tam giác
ABC
.
16.17. Cho
A B C ABC
có chu vi lần lượt là
50cm
60cm
. Din tích
ABC
lớn hơn diện tích
ABC
2
33cm
. Tính din tích mi tam giác.
16.18. Qua điểm
M
thuc cnh
BC
ca tam giác
ABC
k các đường thng song song vi các cnh
AB
AC
, chúng to thành vi hai cnh y mt hình bình hành. Tìm v trí điểm
M
để hình bình hành đó có diện tích
ln nht.
(Thi hc sinh gii lp 9, TP. H Chí Minh, năm hc 2014 2015)
55
CHUYÊN ĐỀ 17. ĐỊNH LÝ MENELAUS, ĐỊNH LÝ CE – VA, ĐỊNH LÝ VAN – OBEN
A. Kiến thc cn nh
1. Định lý Menelaus.
- Menelaus sinh khoảng năm 70 và mất năm 130, những gì được biết v cuộc đời ông rt ít, thông qua mt s
tác phm khoa hc ca những người sau. Ch biết chung chung rng ông có mt thời là sinh viên trường Đại
hc Alexndrie c đại, ri làm cán b ging dạy cũng ở đó và về sau thành nhà thiên văn học La Mã. Trong
hình hc có một định lý ni tiếng mang tên ông: định lý Menelaus.
- Định lý: Cho tam giác
ABC
và ba điểm
,,
A B C
(không trùng với các đỉnh ca tam giác) lần lượt trên các
đường thng
AB
sao cho hoc c ba điểm
,,
A B C
đều nm trên phn kéo dài ca ba cnh, hoc
một trong ba điểm nm trên phn kéo dài ca mt cạnh và hai điểm còn li nm trên hai cnh ca tam giác.
Điu kin cần và đủ để
,,
A B C
thng hàng là
. . 1
A B B C C A
A C B A C B
.
Gii
Trường hp 1. Nếu trong ba điểm
,,
A B C
có đúng 2 điểm thuc cnh ca tam giác
ABC
, chng hn là
B
C
.
Nếu
,,
A B C
thng hàng. Qua
A
k đường song
song vi
BC
ct

BC
ti
M
, ta có:
;

C A AM
C B A B

B C A C
B A AM
.
Vy
. . . . 1

A B B C C A AM A C A B
A C B A C B A B AM A C
.
Ngược li, nếu
. . 1
A B B C C A
A C B A C B
.
Gi

A
là giao điểm ca

BC
vi
BC
.
Theo phn thun:
. . 1
A B B C C A
A C B A C B
. Suy ra:
A B A B
A C A C
.
Do
,

BC
lần lượt thuc cnh
nên

A
nm ngoài cnh
BC
.
Vy
A B A B
A C A C
,
AA
cùng nằm ngoài đoạn
BC
.
Suy ra
AA

. Vậy ba điểm
A ,B ,C
thng hàng.
2.Định Lí Ce-Va.
Ce-Va là kĩ sư người Ý nhưng yêu toán học. Ông sinh năm 1648, mất năm 1734. Thời thanh niên Ce-Va theo
hc đại hc Pise ri giúp vic cho qun công vùng Mantoue. Công trình nghiên cu ca ông v Hình hc và
Cơ học. Đời sau biết đến tên ông qua một định lí v hình học mang tên ông: Định Lí Ce-Va.
56
Định Lí: Cho ba điểm
A,E,F
nm trên ba cạnh tương ứng
BC,CA,AB
ca tam giác
ABC
(không trùng vi ba
đỉnh của tam giác) khi đó ba đường thng
AD,BE,CF
đồng quy khi và ch khi
DB EC FA
1
DC EA FB
.
Gii
Xét đường thng
AD,BE,CF
đồng quy
Qua
A
k đường thng song song vi
BC
, đường thng này cắt đường thng
BE,CF
ti
Q
P
.
Áp dụng định lí Talet ta có:
FA AP EC BC
;
FB BC EA AQ

AP AQ AM AP CD
CD BD MD AQ BD



T đó suy ra
DB EC FA AQ BC AP
1
DC EA FB AP AQ BC
Ngược li nếu:
DB EC FA
1
DC EA FB
Gi
M
là giao điểm ca
BE
CF
. Gi
D’
là giao điểm ca
AM
vi
BC
. Theo phn thun ta có:
D B EC FA D B DB D B DB
1
D C EA FB D C DC D B D C DB DC

.
D B DB
BD BD D D
BC BC

. Vy
AD,BE,CF
đồng quy.
3. Định lí Van Oben
Van Oben (Van Aubel) sinh ngày 20.11.1830 ti Maastricht (Hà Lan), mt ngày 03.02.1906 ti Anwerpen (B).
Ông nghiên cu và dy Toán cho các lp d b đại học Atheneum, Maastricht (Hà Lan) và đại hc Gent (B).
Trong quá trình nghiên cu ông công b nhiu tính chất, định lí đặc sc v tam giác và t giác. Sau đây là một
s định lí đặc sc mang tên ông.
Định lí: Cho
M
là điểm trong tam giác
ABC
. Gi
D, E,F
theo th t là giao điểm ca
AM,BM,CM
vi các
cnh
BC,AC,AB
. Khi đó thì
AM AE AF
MD EC FB

.
Gii
Cách 1: Qua
A
k đường thng song song vi
BC
cắt đường thng
BM,CM
ti
Q
P
.
Áp dng h qu định lí Talet ta có:

AF AP
AP // BC
FB BC
P
Q
F
D
E
A
B
C
P
Q
F
D
E
A
B
C
57

AE AQ
AQ // BC
EC BC
AF AE AQ AP PQ
FB EC BC BC
Mt khác
PQ PM AM
PQ // BC
BC MB MD
t đó suy ra
AM AF AE
MD FB EC

Cách 2: áp dụng định lí Menelaus cho
ABD
và ba điểm
F,M,C
thng hàng ta có:
AF BC MD AF CD AM
1
FB CD AM FB BC MD
(1)
Áp dụng định lí Menelaus cho
ACD
và ba điểm
E,M,B
thng hàng ta có:
AE BC MD AE BD MA
1
EC BD AM EC BC MD
(2)
T (1) và (2) suy ra
AF AE AM CD BD AM
FB EC MD BC BC MD



B. Mt s ví d
Ví d 1 (m rng Van Oben). Cho tam giác
ABC
. Trên tia đối ca tia
BA
lấy điểm
K
, trên tia đối ca tia
CA
lấy điểm
N
. Gi
E
là giao điểm ca
CK
BN
; Gi
M
là giao điểm ca
AE
BC
. Chng minh rng:
AE AK AN
EM KB NC

.
Gii
* Tìm cách gii: Vi cách suy luận như chứng minh định lí Van Oben chúng ta cũng có thể chứng minh được
bng hai cách.
*Trình bày li gii
Cách 1: Qua
A
k đường thng song song vi
BC
cắt đường thng
BN,CK
lần lượt ti
P
Q
Áp dng h qu định lí Talet ta có:
AK AQ
AQ // BC
KB BC

AN AP
AP // BC
NC BC

AK AN AQ P PQ
KB NC BC BC

Mt khác
PQ // BC
Q
P
M
E
A
B
C
K
N
58
PQ PE AE
BC BE ME
. T đó suy ra
AE AK AN
EM KB AC

Cách 2: Áp dụng định lí Menelaus cho
ABM
và ba điểm
K,E,C
thng hàng ta có:
AK BC ME
1
KB CM AE
AK CM AE
KB BC ME
(1)
Áp dụng định lí Menelaus cho
ACM
và ba điểm
E,N,B
thng hàng ta có:
BC ME
1
NC BM
AN
EA
AN BM EA
NC BC ME
(2)
T (1) và (2) ta suy ra
AK AN AE CM BM AE
KB NC ME BC BC ME



Ví d 2: nh lí Menelaus trong t giác) Cho t giác
ABCD
đường thng d ct
AB,BC,CD,DA
ti
M, N,P,Q
.
Chng minh:
MA NB PC QD
1
MB NC PD QA
.
*Tìm cách gii: Tương tự như chứng minh định lí Menelaus trong tam giác, chúng ta có nhiu cách chng
minh. Sau đây là một cách.
*Trình bày li gii
T
A,B
v
AE // BF // CD(E;F d)
Theo h qu định lí Talet ta có:
MA AE NB BF QD DP
;;
MB BF NC CP QA AE
Suy ra:
MA NB PC QD AE BF PC DP
1
MB NC PD QA BF CP PD AE
B
P
N
M
D
C
A
B
59
Ví d 3: Cho tam giác
ABC
. Trên cnh
BC
lần lượt lấy điểm
D
sao cho
BD 1
DC 2
. Lấy điểm
O
trên đoạn
AD
sao cho
AO
4
OD
. Gi
O
là giao điểm của hai đường thng
AC
BO
. Tính t s
AE
EC
.
Gii
T
BD 1
DC 2
suy ra
.
Áp dụng định lí Menelaus trong
ABC
với ba điểm
B,O,E
thng hàng, ta có:
AE BC OD AE 1 AE 4
1 3 1
EC BD OA EC 4 EC 3
Nhn xét: Ngoài cách vn dụng định lí, chúng ta có th k thêm đường thẳng song song để vn dụng định lí
Talet.
Ví d 4: Cho tam giác
ABC
nhn có
BD
CE
là đường cao.
H
là trc tâm. Qua
H
k đường thng ct các
cnh
AB,AC
ti
M,N
. Chng minh rng:
2
HM BM EM
HN DN CN



.
Gii
Áp dụng định lí Menelaus cho
B, H, D
thng hàng vi
ta có:
HM DN AB
1
HN DA BM
(1)
Áp dụng định lí Menelaus cho
C,H,E
thng hàng vi
ta có:
HM CN AE
1
HN CA EM
(2)
E
O
D
B
A
C
N
H
E
D
B
A
C
M
60
T (1) và (2) nhân vế ta có:
2
2
HM DN CN AB AE
1
HN DA CA BM EM
(3)
Mt khác
AEC ADB
(g.g)
AB AD
AB AE AC AD
AC AE
Thay vào (3) suy ra
2
2
HM DN.CN
1
HN BM EM

hay
2
HM BM EM
HN DN CN



(đpcm)
Ví d 5: Cho tam giác
ABC
vuông ti
A
có đường cao
AH
, trung tuyến
BM
, phân giác
CD
ct nhau ti
điểm
O
. Chng minh rng
BH AC
.
Gii
Tìm cách gii. Để chng minh
BH AC
bng cách ghép vào hai tam giác là không kh thi bi không khai
thác được tính đồng quy ca gi thiết. Để khai thác gi thiết này, chúng ta liên tưởng tới định lý Ce-va. Vn
dụng định lý Ce-va, chúng ta suy được
.1
BH DA
HC DB
. Đã xuất hin
BH
song chưa có
AC
. Để xut hin
AC
,
chúng ta vn dng tiếp yếu t gi thiết CD là phân giác. T đó chúng ta suy ra được:
..BH AC HC BA
. Để
BH AC
phn cui cùng là chng minh
2
.HC BC AC
.
Trình bày li gii
Theo định lý Ce-va ta có:
. . 1
BH MC DA
HC MA DB
MA MC
nên
.1
BH DA
HC DB
(1)
CD
là phân giác nên
DA AC
DB BC
(2)
T (1) và (2) ta có:
. 1 . .
BH AC
BH AC HC BC
HC BC
(3)
A
B
M
C
H
D
61
Nhn thy
2
..
HC AC
ABC HAC g g AC BC HC
AC BC
(4)
T (3) và (4) suy ra
2
.BH AC AC
hay
BH AC
.
Ví d 6: Cho tam giác
ABC
có điểm
M
nm trong tam giác. Các tia
,AM
,BM
CM
ct các cnh
BC
,
,CA
AB
tương ứng ti
D
,
E
,
F
. Gi
H
là giao điểm ca
DF
BM
. Gi
K
là giao điểm ca
CM
DE
.
Chng minh rng
AD
,
,BK
CH
đồng quy.
Gii
Tìm cách gii. Để chng minh
,,AD BK CH
đồng quy, d dàng nghĩ tới vic vn dụng định lý Ce-va đảo trong
tam giác
MBC
. Để vn dụng định lý Ce-va, chúng ta cn chng minh
. . 1
KM BH CD
KC HM BD
. Mun xut hin t s
;;
KM BH CD
KC HM BD
chúng ta cn linh hot tìm kiếm các tam giác để vn dng định lý Menelaus hoc Ce-va.
Trình bày li gii
Áp dụng định lý Menelaus trong tam giác
;AMC AMB
Ta có:
. . 1;
KM EC DA
KC EA DM
. . 1
BH DM FA
HM DA FB
Suy ra:
.
KM EA DM
KC EC DA
;
.
BH FB DA
HM FA DM
(1)
Áp dụng định lý Ce-va trong tam giác
ABC
, ta có:
. . 1 .
CD BF AE CD EC FA
BD FA EC BD AE BF
(2)
T (1) và (2) nhân vế vi vế ta được:
. . . . . . . . . 1
KM BH CD EA DM FB DA EC FA KM BH CD
KC HM BD EC DA FA DM AE BF KC HM BD
Theo định lý Ce-va đảo ta có
AD
,
,BK
CH
đồng quy.
K
E
F
A
C
M
B
H
D
62
Ví d 7:Cho tam giác
ABC
nhn có
AH
là đường cao. Lấy điểm
O
tùy ý thuc
AH
(
O
khác
;AH
). Các tia
BO
CO
ct
;AC AB
tương ứng ti
;MN
. Chng minh rng:
HA
là tia phân giác ca
MHN
.
Gii
Cách 1. Qua
A
k đường thng
xy
song song vi
BC
. Gi
;IK
lần lượt là giao điểm ca các tia
;HN HM
với đường thng
xy
.
Theo h qu định lý Ta-let, ta có:
;
AI AN AK AM
BH BN CH MC

.
Áp dụng định lý Ce-va trong
ABC
đối với ba đường thẳng đồng quy
;;AH BM CN
ta có:
. . 1 . . 1
AN BH CM AI BH CH
BN CH MA BH CH AK
1
AI
AI AK
AK
.
Xét
HKI
HA IK
;
AI AK
HIK
cân ti
H HA
là đường phân giác
MHN
.
Cách 2. Xét trường hp
ABC
A
C AB
.
Dng
ABP
cân ti
A
AH
là đường cao.
AP
ct
HM
ti
Q
. Gi
N
đối xng vi
Q
qua
AH
. Vì
,,A Q P
thng hàng suy ra
,,A N B
thẳng hàng. Khi đó
HA
là đường phân giác ca
QHN
QA N A
QP N B
.
Áp dụng định lý Menelaus cho
ACP
vi ba bim thng hàng
,,H Q M
ta có:
. . 1 . . 1
HP MC QA HB MC N A
HC MA QP HC MA N B
,
theo định lý đảo ca Ce-va thì
,,AH BM CN
đồng quy.
Theo gi thiết
,,AH BM CN
đồng quy
H
M
O
I
N
K
A
C
B
N
Q
P
N
B
C
M
A
H
63
NN

. Vy
HA
là đường phân giác
MHN
.
Xét trường hp
A
BC AC AB
Chứng minh tương tự như trên
Xét trường hp
A
BC AC AB
D chng minh.
Ví d 8: Gi s
O
là điểm bt kì nm trong tam giác
ABC
các tia
,,AO BO CO
lần lượt ct
,,BC AC AB
ti
,,M N P
. Chng minh rng:
. . .
..
AO AP BO BM CO CN
OP OM ON
không ph thuc vào v trí điểm
O
.
Gii
Tìm cách gii. Nhn thy phn kết lun ca chúng ta là mt tích các t s nên chúng ta liên tưởng tới hai định lý
có th dùng là Menelaus hoc Ce-va. Nhn thy nếu mun có
.AO AP
OP
thì
AO
OP
hay
AP
OP
không th xut hin
được nếu vn dụng định lý trên (bi c hai định lý đều không xut hin t s trên). Song nếu đảo mu s, tc là
.AO AP
OM
thì t s
AO
OM
có th xut hiện được nh vn dụng định lý Menelaus trong tam giác
AMC
hoc
AMB
.
Nhn thấy ý tưởng đó khả thi. Tiếp tc biu din các t s
;
BO CO
ON OP
một cách tương tự, chúng ta có mt li gii
hay.
Trình bày li gii
Áp dụng định lý Menelaus trong:
AMC
với ba điểm
thng hàng ta có:
. . 1 .
AO BM CN AO BC AN
OM BC NA OM BM CN
(1)
BCN
với ba điểm
,,A O M
thng hàng ta có:
. . 1 .
BO AN CM BO AC BM
ON AC MB ON AN CM
(2)
Xét
ACP
với ba điểm
thng hàng ta có:
. . 1 .
CO BP AN CO AB NC
OP BA NC OP BP AN
(3)
T (1); (2); (3) ta có:
. . .
. . . . . . .
AO AP BO BM COCN AO BO CO
AP BM CN
OP OM ON OM ON OP
. . . . . . . .
BC AN AC BM AB CN
AP BM CN
BM CN AN CM BP AN
A
N
C
M
B
P
O
64
..
. . .
..
BM APCN
BC AC AB
CM BP NA
(4)
Mt khác, áp dụng định lý Ce-va đối vi
ABC
có ba đường thng
,,AM BN CP
đồng quy ta có:
. . 1
BM CN AP
CM AN BP
(5)
T (4) và (5) suy ra:
. . .
. . . .
AO AP BO BM COON
BC AC AB
OP OM ON
Không ph thuc vào v trí điểm
O
.
Ví d 9: Trên ba cnh
,,BC CA AB
ca tam giác
ABC
lần lượt lấy ba điểm
,,H M N
sao cho
,,AH BM CN
đồng quy ta
G
. Gi
,PQ
lần lượt là giao điểm ca
HN
BM
;
HM
CN
. Tia
AP
và tia
AQ
ct
BC
lần lượt ti
E
F
. Chng minh rng:
3.
AP AQ AN AM
PE QF NB MC



Gii
Tìm cách gii. Định hướng và s la chọn định lý để vn dng là vấn đề quan trng, nó quyết định s thành
công ca bài toán. Trong bài toán này, nhn thy có nhiều đường đồng quy, mt khác phn kết lun li xut
hin tng các t s nên vic vn dụng định lý Van-Oben là điều mà chúng ta nên nghĩ tới. Để xut hin
AP
PE
nên
vn dụng định lý Van-Oben trong tam giác
ABH
đối vi
,,AE BG HN
đồng quy. Để xut hin
AQ
QF
nên vn
dụng định lý Van-Oben trong tam giác
ACH
đối vi
,,AF CG HM
đồng quy. Sau đó, vì vế phi ch xut hin
AN AM
NB MC
, chúng ta vn dụng định lý Van-Oben trong tam giác
ABC
đối vi
,,AH CN BM
đồng quy. T đó
chúng ta có li gii hay.
Trình bày li gii
Áp dụng định lý Van-Oben trong tam giác ABH vi
,,AE BG HN
đồng quy ti
P
, ta có:
AP AN AG
PE NB GH

(1)
Áp dụng định lý Van-Oben trong tam giác
ACH
vi
,,AF CG HM
đồng quy ti
Q
, ta có:
AQ AM AG
QF MC GH

(2)
T (1) và (2) cng vế vi vế, ta được:
TRANG 115-118
Q
F
H
P
G
E
B
M
N
A
C
65
AP AQ AN AM AG
2.
PE QF NB MC GH
Áp dụng định lý Van Oben cho
ABC
vi
AH,BM,CN
đồng quy ti
G
, ta có:
AG AN AM
4
GH NB MC

T
3
4
suy ra:
AP AQ AN AM
3
PE QF NB MC



(Điều phi chng minh).
Nhn xét. T kết lun ca bài toán, chúng ta nhn thy:
- Áp dụng định lý Van Oben cho
ABC
vi
AH,BM,CN
đồng quy ti
G
, ta có
AN AM AG
4
NB MC GH

do
đó chúng ta giải được bài toán sau: Trên ba cnh
BC,CA,AB
ca tam giác lần lượt lấy ba điểm
H,M ,N
sao cho
AH,BM,CN
đồng quy ti
G
. Gi
P,Q
lần lượt là giao điểm ca
HN
BM
;
HM
CN
. Tia
AP
và tia
AQ
ct
BC
lần lượt ti
E
F
. Chng minh rng:
AP AQ AN
6
PE QF NB

- Trường hp
G
là trung điểm ca
AH
thì
AN AM
1 4
NB MC

. Do đó chúng ta giải được bài toán sau: Trên
ba cnh
BC,CA,AB
ca tam giác
ABC
lần lượt lấy ba điểm
H,M ,N
sao cho
AH,BM,CN
đồng quy ti
G
.
Gi
P,Q
lần lượt là giao điểm ca
HN
BM
;
HM
CN
. Tia
AP
và tia
AQ
ct
BC
lần lượt ti
E
F
.
Chng minh rng:
AP AQ
3
PE QF

C. Bài tp vn dng
17.1. Cho tam giác
ABC
. Trên cnh
BC,CA
lần lượt lấy điểm
D
E
tha mãn
BD CE 1
DC EA 2

. Gi
O
giao điểm ca
AD
BE
. Tính t s
AO
OD
BO
OE
.
17.2. Cho tam giác
ABC
vuông ti
A
. Có đường cao
AH
, đường trung tuyến
BM
và phân giác
CD
đồng quy
ti
O
.
Chng minh rng:
BC BH
AC CH
.
17.3. Cho tam giác
ABC
có đường cao
AH
, đường trung tuyến
BM
và đường phân giác
CD
đồng quy. Đặt
a,b,c
lần lượt là độ dài ba cnh
BC,CA,AB
. Chng minh rng:
2 2 2 2
a b a b c 2.a b.
17.4. Cho tam giác
A
BC AB AC ,M
là trung điểm ca
BC
. Một đường thng qua
M
và song song vi
đường phân giác
AD
ca góc
BAC
ct
AC,AB
lần lượt
E
F
. Chng minh rng
CE BF
.
66
17.5. Cho tam giác
ABC
, lấy điểm
E
thuc cnh
AB
và điểm
F
thuc cnh
AC
. Gi
AM
là đường trung
tuyến ca tam giác
ABC
. Chng minh rằng điều kin cần và đủ để
EF
song song vi
BC
AM ,BF
CE
đồng quy.
17.6. Cho tam giác
ABC
có trung tuyến
AD
. Trên
AD
lấy điểm
K
sao cho
AK
3
KD
. Hỏi đường thng
BK
chia din tích tam giác
ABC
theo t s nào?
17.7. Cho t giác
ABCD
. Cnh
AB
ct
CD
kéo dài ti
E
, cnh
BC
ct
AD
kéo dài ti
I
. Đường chéo
AC
ct
BD
EI
lần lượt ti
M ,N
. Chng minh rng
MA NA
MC NC
.
17.8. Cho tam giác
ABC
. Ly
K
thuc cnh
AB
T
thuộc tia đối tia
BC
. Gi
F
là giao điểm ca
TK
vi
AC,O
là giao điểm ca
BF
vi
CK
. Gi
E
là giao điểm ca
AO
vi
BC
. Chng minh rng:
TB EB
TC EC
.
17.9. Cho tam giác
ABC
D
là điểm bt kì nm trong tam giác. Lấy điểm
M
tùy ý thuc
AD
. Gi giao
điểm ca
BM
AC
E
; gọi giao điểm
CM
AB
F
. Các tia
DE
CM
giao nhau ti
K
; các tia
DF
BM
giao nhau ti
H
. Chng minh rng
CH,AD,BK
đồng quy.
17.10. Cho tam giác nhn
ABC
có ba đường cao
AD,BM ,CN
ct nhau ti
H
. Chng minh rng:
HD HM HN DB MC NA
..
AD BM CN DC MA NB
.
17.11. T điểm
I
thuc min trong tam giác
ABC
, k
AI
ct
BC
ti
D
. Qua điểm
I
k
MN,PQ
RS
ln
t song song vi
BC,AB,AC
(
M ,S
thuc
AB
;
Q,R
thuc
BC
;
N,P
thuc
AC
). Chng minh rng:
IM DB
a)
IN DC
IM IP IR
b ) . . 1
IN IQ IS
17.12. Cho tam giác
ABC
vuông ti
C
có đường cao
CK
. V đường phân giác
CE
ca tam giác
ACK
.
Đưng thng qua
B
song song vi
CE
cắt đường thng
CK
ti
F
. Chng minh rằng đường thng
EF
chia
đoạn thng
AC
thành hai phn bng nhau.
17.13. Cho hình bình hành
ABCD
. Trên cnh
AB
lấy điểm
K
. Qua
K
k đường thng song song vi
AD
.
Trên đường thẳng đó lấy điểm
L
bên trong hình bình hành, trên cnh
AD
lấy điểm
M
sao cho
AM KL
.
Chng minh rằng ba đường thng
CL,DK,BM
đồng quy.
17.14. Cho tam giác
ABC
không cân có
CD
là đường phân giác. Lấy điểm
O
thuộc đường thng
CD
(
O
khác
C
D
). Gi
M ,N
lần lượt là giao điểm của đường thng
AO,BO
vi
BC
AC
. Gi
P
là giao điểm ca
đường thng
MN
AB
. Chng minh rng
CD
vuông góc vi
CP
.
17.15. Cho tam giác
ABC
có điểm
O
nằm trong tam giác. Các đường thng
AO,BO,CO
ct các cnh
BC,CA,AB
lần lượt ti
D,E,F
. Qua
O
k đường thng song vi
BC
, ct
DF,DE
lần lượt ti
M
N
.
Chng minh rng:
OM ON
.
67
17.16. Cho tam giác
ABC
có điểm
M
nm trong tam giác. Gi
D,E,F
th t là giao điểm của đường thng
AM ,BM,CM
vi các cnh
BC,AC,AB
. Chng minh rng trong các t s
AM BM CM
;;
ND ME MF
, có ít nht mt t
s không lớn hơn
2
và ít nht mt t s không nh hơn
2
.
(Thi vô địch Toán Quc tế, IMO năm
1961
)
17.17. Cho tam giác
ABC
. Gi
I
là trung điểm ca cnh
BC
. Ly
M
thuộc tia đối ca tia
CA
. Tia
MI
ct
đường thng
AB
ti
N
. Trên tia đối ca tia
BC
lấy điểm
E
, tia
EN
ct
AC
ti
P
. Tia
PI
cắt đường thng
AB
ti
Q
. Gi
F
là giao điểm ca tia
QM
IC
. Chng minh
IE IF
.
17.18. Cho tam giác
ABC
, trên ba cnh
BC,CA,AB
lần lượt lấy ba điểm
A',B',C'
sao cho
AA',BB',CC'
đồng
quy ti
K
. Gi
M ,N
lần lượt là giao điểm ca
A' C'
BB'
;
A' B'
CC'
. Tia
AM
, tia
AN
lần lượt ct
BC
ti
E;F
. Chng minh rng:
a)
EN,FM,AA'
đồng quy ti
I
b)
IA.KA' 3.IA' .KA
1
CHƯƠNG IV. HÌNH LĂNG TRỤ ĐỨNG HÌNH CHÓP ĐỀU
CHUYÊN ĐỀ 18. HÌNH HP CH NHT ....................................................................................................................... 2
CHUYÊN ĐỀ 19. HÌNH LĂNG TRỤ ĐỨNG .................................................................................................................... 8
CHUYÊN ĐỀ 20. HÌNH CHÓP ĐỀU ............................................................................................................................... 13
2
CHUYÊN ĐỀ 18. HÌNH HP CH NHT
A. Kiến thc cn nh
1. Hình hp ch nht
- Hình
18.1
cho ta hình nh ca mt hình hp ch nht.
- Hình hp ch nht có
6
mt,
8
đỉnh và
12
cnh.
- Hình lập phương có
6
mt là nhng hình vuông.
2. Din tích xung quanh và th tích ca hình hp ch nht
* Din tích xung quanh ca hình hp ch nht bằng chu vi đáy nhân với chiu cao.
xq
S
2 a b .c
* Din tích toàn phn ca hình hp ch nht bng tng ca din tích xung quanh và diện tích hai đáy.
tp
S
2 ab bc ca
* Th tích ca hình hp ch nht bng tích của ba kích thước.
V abc
* Đặc biệt đối vi hình lập phương thì:
2
xq
S 4a
2
tp
S 6a
3
Va
3. Tính cht đường chéo ca hình hp ch nht
* Bốn đường chéo ca hình hp ch nht ct nhau tại trung điểm ca mỗi đường.
* Bình phương của mỗi đường chéo bng tổng các bình phương của ba kích thước.
2 2 2 2
d a b c
3
4. Quan h v trí của hai đường thng phân bit trong không gian (h.18.2)
Cắt nhau: Nếu hai đường thng có một điểm chung.
Ví d: AB và BC.
Song song: Nếu hai đường thng cùng nm
trong mt mt phẳng và không có điểm chung.
Ví d: AB và CD.
Chéo nhau: Nếu hai đường thng không cùng
nm trong mt mt phng nào.
Ví d: AB và CC
'
.
Nhn xét. Hai đường thng phân bit cùng song song
vi một đường thng th ba thì song song.
Hình 18.2
D'
C'
B'
A'
D
C
B
A
5. Quan h song song của đường thng và mt phng (h.18.2)
Đường thng song song vi mt phẳng khi chúng không có điểm chung.
Ví d: AB //
' ' ' '
mp A BC D
.
● Nếu
''
;AB mp P AB mp P
AB
//
''
AB
thì
AB
//
m
p P
.
Nhn xét. Nếu
,A
B mp P
thì đường thng AB nm trn trong
m
p P
.
6. Quan h song song ca hai mt phng (h.18.3)
Hai mặt phng song song khi chúng không có
điểm chung.
Nếu
m
p P
chứa hai đường thng ct nhau a và b,
m
p Q
chứa hai đường thng ct nhau
'
a
'
b
trong đó a //
'
a
và b //
'
b
thì
m
p P
//
m
p Q
.
Nếu
m
p P
chứa hai đường thng ct nhau a và b
Mà a //
m
p Q
và b //
m
p Q
thì
m
p P
//
m
p Q
P
b
a
Hình 18.3
Q
b'
a'
Nhn xét. Hai mt phẳng có điểm chung thì chúng ct nhau theo một đường thẳng đi qua điểm chung y, gi
là giao tuyến ca hai mt phng.
7. Quan h vuông góc (h.18.4)
Nếu một đường thng vuông góc với hai đường thng
ct nhau ca mt phẳng thì ta nói đường thng
vuông góc vi mt phng.
Nếu đường thng
a
mp P
tại điểm O thì đường
thng a vuông góc vi mọi đường thng qua O và nm
trong
m
p P
.
Nếu
a
mp P
a
mp Q
thì
m
p P mp Q
.
a
O
Hình 18.4
m
P
n
4
B. Mt s ví d
Ví d 1. Cho hình hp ch nht
' ' ' '
.ABCD ABC D
. Gi M và N ln luột là trung điểm ca CD và
''
CD
. Chng
minh MN //
''
mp BCC B
.
Gii (h.18.5)
* Tìm cách gii
Mun chng minh MN //
''
mp BCC B
ta phi chng minh MN song song vi một đường thng ca mt phng
''
B
CC B
.
* Trình bày li gii
Xét t giác
'
MCC N
MC
//
'
NC
và MC =
'
NC
.
Vy t giác
'
MCC N
là hình bình hành, suy ra
MN
//
'
CC
.
Đưng thng MN không nm trong mt phng
''
B
CC B
còn đường thng
'
CC
nm trong mt phng
''
B
CC B
MN
//
'
CC
nên
MN
//
''
mp BCC B
.
Hình 18.5
N
M
D'
C'
B'
A'
D
C
B
A
Ví d 2. Cho hình hp ch nht
' ' ' '
.ABCD ABC D
. Trên các cnh
' ' ' '
, , ,AA BB CC DD
lần lượt lấy các điểm E, F,
G, H sao cho
''
21
,
33
AE DE DD BG CH CC
. Chng minh rng
m
p ADHG
//
''
mp EFC B
.
Gii (h.18.6)
* Tìm cách gii
Để chng minh
m
p ADHG
//
''
mp EFC B
ta tìm cách chứng minh hai đường thng ct nhau ca
m
p ADHG
tương ứng song song với hai đường thng ct nhau ca
''
mp EFC B
.
* Trình bày li gii
T giác BCHG có BG = CH, BG // CH nên là hình bình hành, suy
ra HG // BC.
Mt khác
BC
//
''
BC
nên
HG
//
''
BC
.
T giác
'
DHC F
DF
//
'
HC
'
DF HC
nên
là hình bình hành, suy ra
'
DH FC
.
Xét
m
p ADHG
có HG và DH ct nhau ti H.
Xét
''
mp EFC B
''
BC
'
FC
ct nhau ti
'
C
.
T đó suy ra
m
p ADHG
//
''
mp EFC B
.
Hình 18.6
H
G
F
E
D'
C'
B'
A'
D
C
B
A
Ví d 3. Cho hình hp ch nht
' ' ' '
.ABCD ABC D
.
a) Chng minh rng t giác
''
ADC B
là hình ch nht.
b) Tính din tích ca hình ch nht
''
ADC B
biết:
''
12, 29, 16AB AC DD
.
5
Gii (h.18.7)
a) T giác
''
ADD A
là hình ch nht, suy ra
AD
//
''
AD
''
AD AD
.
T giác
' ' ' '
ABC D
là hình ch nht, suy ra
''
BC
//
''
AD
' ' ' '
BC AD
.
Do đó
AD
//
''
BC
''
AD BC
Vy t giác
''
ADC B
là hình bình hành.
Hình 18.7
D'
C'
B'
A'
D
C
B
A
Ta có:
'
AD DD
AD DC
nên
''
AD mp DCC D
. Suy ra
'
AD DC
. Do đó hình bình hành
''
ADC B
hình ch nht.
b) Xét ∆
'
DDC
vuông ti
'
D
' '2 ' '2 2 2
16 12DC DD DC
20
.
Xét ∆
'
ADC
vuông ti D có
'2 '2 2 2
29 20 21AD AC DC
.
Vy din tích hình ch nht
''
ADC B
là:
'
. 20.21 420S DC AD
(đvdt).
Ví d 4. Cho hình hp ch nht
' ' ' '
ABCDABC D
.
a) Chng minh rng
' ' ' '
mp DCC D mp CBBC
.
b) Trong s sáu mt ca hình hp ch nht, có bao nhiêu cp mt phng vuông góc vi nhau?
Gii (h.18.8)
* Tìm cách gii
Mun chng minh
''
mp DCC D
vuông góc vi
''
mp CBBC
ta cn chng minh một đường thng
ca
''
mp DCC D
vuông góc với hai đường thng
giao nhau ca
''
mp CBBC
.
* Trình bày li gii
a)
''
DDC C
là hình ch nht nên
' ' '
DC CC
.
' ' ' '
ABC D
là hình ch nht nên
' ' ' '
DC BC
.
Vy
''
DC
vuông góc với hai đường giao nhau ca
''
mp CBBC
, do đó
''
DC
''
mp CBBC
.
Hình 18.8
D'
C'
B'
A'
D
C
B
A
Mt khác,
' ' ' '
DC mp DCC D
nên
' ' ' '
mp DCC D mp CBBC
.
b) Chứng minh tương tự như câu a), ta được các cp mt có chung mt cnh thì vuông góc vi nhau. Hình hp
ch nht có 12 canh nên có 12 cp mt vuông góc vi nhau.
Ví d 5. Cho hình hp ch nht
' ' ' '
.ABCD ABC D
. Din tích các mt
''
,ABCD BCC B
''
DCC D
lần lượt là
2 2 2
108 ,72 ,96cm cm cm
.
a) Tính th tích ca hình hp.
b) Tính độ dài đường chéo ca hình hp.
6
Gii (h.18.9)
* Tìm cách gii
Din tích các mặt đã cho là tích của hai kích thước.
Th tích ca hình hp là tích của ba kích thước. Vì
vy ta cn s dng các tích ca tng cp hai kích
thước để đưa về tích ca ba kích thước.
* Trình bày li gii
a) Gọi độ dài các cnh
'
,,AB BC CC
lần lượt là a, b, c.
Ta có : ab = 108 (1) ; bc = 72 (2) ; ca = 96 (3)
Suy ra:
. . 108.72.96abbcca
hay
2
746496abc
.
Do đó
3
746496 864abc cm
.
c
b
a
Hình 18.9
D'
C'
B'
A'
D
C
B
A
Vy th tích ca hình hp là
3
864V cm
. (4)
b) T (4) và (1) ta có:
864
8
108
abc
c cm
ab
.
T (4) và (2) ta có:
864
12
72
abc
a cm
bc
.
T (4) và (3) ta có:
864
9
96
abc
b cm
ac
.
Vậy đường chéo ca hình hp ch nhật có độ dài là:
2 2 2 2 2 2
12 9 8 17d a b c cm
.
C. Bài tp vn dng
Quan hệ song song. Quan h vuông góc
18.1. Cho hình hp ch nht
' ' ' '
.ABCD ABC D
.
a) Chng minh rng
'
m
p ACD
//
''
mp AC B
.
b) Chng minh rng
'
m
p CDB
'
m
p BCD
ct nhau. Tìm giao tuyến ca chúng.
18.2. Hình hp ch nht
' ' ' '
.ABCD ABC D
có đáy ABCD là hình vuông. Chứng minh rng
''
m
p DBB D
vuông
góc vi
''
m
p ACC A
.
18.3. Cho hình hp ch nht
' ' ' '
.ABCD ABC D
.
a) Tìm giao tuyến m ca hai mt phng
''
ACC A
''
DBB D
.
b) Chng minh giao tuyến
' ' ' '
m mp A BC D
.
c) Chng minh
''
m
p BDD B
' ' ' '
mp A BC D
.
Các mt Các đỉnh ca hình hp ch nht
18.4. Người ta ghép 480 hình lập phương nhỏ cnh 1cm thành mt hình hp ch nhật kích thước 8x12x5cm ri
sơn tất c sáu mt ca hình hp ch nht này. Hi:
a) Có bao nhiêu hình lập phương nhỏ cạnh 1cm không được sơn mặt nào?
b) Có bao nhiêu hình lập phương nhỏ cnh 1cm có ít nht mt mặt được sơn?
18.5. Mt hình lập phương cạnh n đơn vị
;2nnN
, c 6 mặt đều được sơn màu xanh. Người ta chia hình
lập phương này thành
3
n
hình lập phương cạnh 1 (đơn vị). Cho biết s hình lập phương nhỏ cạnh 1 (đơn vị)
không được sơn mặt nào là 27. Tính:
a) Giá tr ca n;
7
b) S hình lập phương nhỏ được sơn ba mặt;
c) S hình lập phương nhỏ được sơn hai mặt;
d) S hình lập phương nhỏ được sơn đúng một mt.
18.6. Mt chiếc hp hình lập phương cạnh 6cm được đặt trên mặt bàn. Tính quãng đường ngn nht mà con
kiến phi bò trên mt hp t trung điểm M ca
''
CD
đến đỉnh A.
18.7. Cho hình hp ch nht
' ' ' '
.ABCD ABC D
.
a) Hỏi có bao nhiêu đoạn thẳng mà hai đầu của nó là hai đỉnh ca hình hp ch nht?
b) Chng t rằng các đoạn thng nói trên, ch có tối đa 7 giá trị khác nhau v độ dài.
Bài 18.8 Người ta ghi vào sáu mt ca mt hình lập phương các số t nhiên t 1 đến 6. Sau đó cứ mỗi lượt,
ta cng thêm cùng mt s t nhiên vào hai mt ca hình lập phương đó. Hỏi sau mt s t, có th xy ra
sáu s bng nhau sáu mt ca hình lập phương được không?
Độ dài Din tích - Th tích
18.9 Mt hình hp ch nhật có các kích thước bằng 8, 9, 12. Tính độ dài ln nht ca một đoạn thng có th
đặt trong hình hp ch nhật đó.
18.10. Mt hình hp ch nht có tổng ba kích thước bằng 61 cm và đường chéo bng 37cm. Tính din tích
toàn phn ca hình hp ch nhật đó.
18.11 Đưng chéo ca mt hình lập phương dài hơn đường chéo mi mt ca nó là 1cm. Tính din tích toàn
phn và th tích hình lập phương đó.
8
CHUYÊN ĐỀ 19. HÌNH LĂNG TRỤ ĐỨNG
A. Kiến thc cn nh
1. Mô t hình lăng trụ đứng.
Trong hình bên cho ta hình nh mt hình lăng trụ đứng.
Các mt bên là nhng hình ch nht.
Các mt bên song song và bng nhau.
Hai đáy là hai đa giác nằm trong hai mt phng song song.
Các cạnh bên cũng như các mặt bên đều vuông góc vi hai mt
phẳng đáy.
2. Din tích xung quanh Th tích ca hình lăng trụ đứng
Din tích xung quanh của hình lăng trụ đứng bằng chu vi đáy nhân
vi chiu cao.
2
xq
S ph
(
p
là nửa chu vi đáy,
h
là chiu cao)
2
tp xq day
S S S
Th tích ca hình lăng trụ đứng bng diện tích đáy nhân với chiu cao.
.V S h
(
S
là diện tích đáy,
h
là chiu cao)
B. Mt s ví d
Ví d 1. Cho hình lăng trụ đứng
.ABC A B C
. Gi
D
,
E
,
F
theo th t là trung điểm ca
AA
,
BB
,
CC
. Chng minh rng
( ) // ( )mp AEC mp DB F

.
Gii
Tìm cách gii: Mun chng minh
( )/ / ( )mp AEC mp DB F

ta chứng minh hai đường thng giao nhau ca
()mp AEC
tương ứng song song với hai đường thng giao nhau ca
()mp DB F
.
Trình bày li gii.
Ta có:
//AD EB
AD EB
nên t giác
AEB D
là hình bình hành. Suy ra
//AE DB
. (1)
9
Xét
AC A

DF
là đường trung bình nên
//DF AC
. (2)
T (1) và (2) suy ra
( )/ / ( )mp AEC mp DB F

.
Ví d 2. Cho hình lăng trụ đứng
.ABC A B C
, đáy
ABC
là tam giác vuông ti
A
.
a) Chng minh rng
'
/ / 'mp ABB A mp ACC A

.
b) Gi
M
là điểm bt kì trên cnh
BC

. Chng minh rng
m
p AA M mp A B C
.
c) Xác định v trí của điểm
M
trên cnh
BC

để độ dài
AM
nh nht.
Gii
Tìm hướng gii: Mun chng minh
'
/ / 'mp ABB A mp ACC A

ta chng minh một đường thng ca mt
này vuông góc vi mt kia.
Trình bày li gii
a) Ta có
AB AA
AB AC
nên
A
B mp ACC A

. Mt khác
A
B ABB A

nên
'
/ / 'mp ABB A mp ACC A

.
b) Hình lăng trụ
.ABC A B C
là hình lăng trụ đứng nên
A
A mp A B C
. Mt khác
A
A mp AA M

nên
m
p AA M mp A B C
.
c) Xét
AA M
vuông ti
A
ta có:
2 2 2
AM AA A M


trong đó
AA
không đổi. Vy
AM
nh nht
AM
nh nht.
Xét
m
p A B C
AM
nh nht
A M B C

.
Vậy để độ dài
AM
nh nht thì
M
phi là hình chiếu ca
A
trên
BC

.
Ví d 3. Cho hình lăng trụ
.ABC A B C
đáy là hình tam giác vuông cân tại
A
. Biết hình tr này có chiu cao là
4m, và th tích là 18m
3
. Tính din tích toàn phn ca nó.
10
Gii
Ta có
.
V
V S h S
h
. Vy diện tích đáy của hình lăng trị này là
18
4,5
4
S 
(m
2
).
ABC
vuông cân ti
A
nên
2
1
2
S AB
.
Do đó
22
1
4,5 9 3
2
S AB AB AB
suy ra
32BC
(m).
Din tích xung quanh của hình lăng trụ
2 3 3 3 2 .4 24 12 2
xq
S ph
(m
2
)
Din tích toàn phn
24 12 2 9 33 12 2 50
tp
S
(m
2
).
Ví d 4 Một hình lăng trụ đều (tức là lăng trụ đứng có đáy là đa giác đều) có tt c 18 cnh, mi cnh dài
43
cm. Tính th tích của hình lăng trụ đó.
Gii
Tìm cách gii
Để tìm th tích hình lăng trụ đứng khi đã biết chiu cao, ta cn tính diện tích đáy. Đáy là một đa giác
đều, đã biết độ dài mi cnh nên cn biết s cnh là song.
Trình bày li gii
Gi s cnh ca một đáy là
n
. Khi đó số cnh bên là
n
. Suy ra tng s cnh của hình lăng trụ đứng là
3n n n n
. Theo đề bài, ta có
3 18 6nn
.
Vậy hình lăng trụ đứng đã cho là hình lăng trụ lục giác đều. Có th coi diện tích đáy là tổng din tích
của 6 tam giác đều, mi cnh bng
43
cm.
11
Do đó, diện tích đáy là
2
4 3 3
.6 72 3
4
S 
(cm
2
).
Th tích hình tr
. 72 3.4 3 864V S h
(cm
3
)
C. Bài tp vn dng.
Chng minh song song, vuông góc, tính chiu cao.
19.1 Cho hình lăng trụ đứng
.ABC A B C
. Gi
E
G
lần lượt là trng tâm tam giác
ABB
ACC
. Trong
mt bên
ABB A

v
//EM BB
M
AB
. Trong mt bên
ACC A

v
//GN CC
N
AC
. Chng minh
rng
//m
p MNGE mp BCC B

.
19.2 Cho hình lăng trụ đứng
.ABC A B C
có cạnh đáy
10AB AC
cm,
12BC
cm. Gi
M
là trung điểm
BC

.
a) Chng minh rng
B
C mp AA M
.
b) Cho biết
17AM
cm. Tính din tích toàn phn của hình lăng trụ.
19.3 Một hình lăng trụ đều có tng s mt, s đỉnh và s cnh là 26. Biết th tích của hình lăng trụ là 540cm
3
,
din tích xung quanh là 360cm
2
. Tính chiu cao của hình lăng trụ đó.
19.4 Hình hộp đứng
.'ABCD A B C D
có đáy là hình thoi
ABCD
cnh
a
, góc nhn
30
. Cho biết din tích toàn
phn ca hình lăng trụ đứng bng hai ln din tích xung quanh ca nó. Tính chiu cao của hình lăng trụ đứng.
Tính din tích, th tích.
19.5 Hình lăng trụ đứng
.ABC A B C
5AB
cm,
12AC
cm và chiu cao
10AA
cm. Biết din tích xung
quanh ca hình tr là 300cm
2
. Tính din tích ca nó.
19.6 Một hình lăng trụ đứng có đáy là hình thoi với các đường chéo bng 16cm và 30cm. Din tích toàn phn
của hình lăng trụ là 2680cm
2
, tính th tích ca nó.
19.7 Hình lăng trụ ngũ giác đều
.ABCDE A B C DE
có cạnh đáy bằng
a
. Biết hiu gia các din tích xung
quanh của hai hình lăng trụ đứng
.ABCE A B C E
.CDE C D E
2
4a
. Tính din tích xuang quanh ca hình
lăng trụ đã cho.
19.8 Cho hình lăng trụ đứng
.'ABCD A B C D
có đáy
ABCD
là hình thang vuông ti
A
D
. Biết
B AD a
,
45BCD
3AC a
. Tính
a) Th tích hình lăng trụ đứng;
b) Din tích toàn phần hình lăng trụ đứng.
19.9 Có mt tm bt hình ch nhật kích thước
ab
,
ab
. Dùng tm bt này để dng mt chiếc lu tri có
dạng hình lăng trụ đứng, hai đáy (tức là hai ca) là hai tam giác vuông cân. C tm bt thành hai mái lu sát mt
đất.
a) Chng minh rằng dù căng tấm bt theo chiu dài hay rng thì din tích mặt đất bên trong lều là như
nhau.
b) Trong hai trường hợp trên, trường hp nào có thch không khí bên trong lớn hơn.
12
19.10 ng
. '
ABCD A B C D
t th
tích là 1280cm
3
chi u cao là
20cm. Tính giá tr nh nh t c a di n tích xung quanh.
19.11 M t chi ng có d ng, chi u c nh 18cm.
a) Tính di n tích gi làm m t xung quang c
b) Tính th tích c
c) N u gi nguyên chi u cao c i gi dài c iêu l th
hai l n.
13
M
H
C
D
B
A
S
CHUYÊN ĐỀ 20. HÌNH CHÓP ĐỀU
A. Kiến thc cn nh
1. Mô t hình chóp hình chóp đều.
Hình chóp có đáy là một đa giác
Các mt bên là những tam giác chung đỉnh. Đường
thẳng đi qua đỉnh ca vuông góc vi mt phẳng đáy
được gọi là đường cao ca hình chóp.
Hình chóp đều là hình chóp có mặt đáy là một đa giác
đều, các mt bên là nhng tam giác cân bng nhau.
Trong hình chóp đều, chân đường cao trùng vi tâm
của đa giác đáy, ví dụ SH. Đường cao ca mi mt
bên v t đỉnh S gọi là trung đoạn ca hình chóp, ví
d SM
2. Hình chóp cụt đều.
Cắt hình chóp đều bng mt mt phng song song với đáy. Phần hình chóp nm gia mt phng
đó và mặt phẳng đáy gọi là hình chóp cụt đều
(Hình 20.2).
Mi mt bên ca hình chóp cụt đêu là hình thang
cân.
3. Din tích xung quanh của hình chóp đu
Din tích xung quanh của hình chóp đều bng
tích ca nửa chu vi đáy và trung đoạn ca hình
chóp.
.
xq
S p d
(p: là nửa chu vi đáy, d là trung đoạn)
Din tích xung quang ca hình chóp cụt đều bng:
Din tích mt mt bên nhân vi s mt bên.
Din tích xung quanh của hình chóp đều tr đi diện tc xung quanh của hình chóp đều nh hoc
'.
xq
S
p p d
(Trong đó:
,'pp
là na chu vi đáy lớn, đáy nhỏ.
d: là trung đoạn ca mt bên.)
4. Th tích hình chóp đều.
1
.
3
V S h
(S là diện tích đa giác dấy; h là chiu cao)
Th tích ca chóp cụt đều ln tr đi thể tích của hình chóp đều nh; hoc
1
2 1 2
1
.
3
V S S S S h
M'
M
O'
C'
B'
O
C
A
A
D
A'
D'
14
(Trong đó:
12
,SS
là diện tích hai đáy, h là chiều cao)
B. Mt s bài toán ví d
Ví d 1. Cho hình chóp tam giác đều S.ABC, đường cao SH. Trên các cnh SA, SB, SC lần lượt ly các
điểm A’, B’, C; sao cho SA’=SB’=SC’. Chứn g minh rng:
a)
( ' ' ') // ( )mp A B C mp ABC
. b)
( ) ( )mp SCH mp SAB
.
Gii
Tìm hướng gii
Mun chng minh
( ' ' ') // ( )mp A B C mp ABC
ta chng minh hai cnh ca
' ' 'A B C
tương ứng song song vi
hai cnh ca
ABC
.
Trình bày li gii
a) Xét
SAC
; ' 'SA SC SA SC
nên
'
' ' //
'
SA SC
A C SA
SA SC

(1).
Chứng minh tương tự, ta được:
' ' //A B AB
(2).
T (1) và (2) suy ra
( ' ' ') // ( )mp A B C mp ABC
.
b) Xét
ABC
có H là giao điểm của ba đường trung tuyến. Gọi M là trung điểm ca AB, ta có:
;CM AB SM AB
. Vy
()AB mp SCM
.
Mt khác
()AB mp SAB
nên
( ) ( )mp SAB mp SCM
hay
( ) ( )mp SAB mp SCH
.
Ví d 2. Cho hình chóp
.S ABC
có đáy là tam giác đều và
SA
là đường cao ca hình chóp. Gi M là trung
điểm ca
BC
.
a) Chng minh rng
( ) ( )mp SAM mp SBC
.
b) Cho biết
30
o
SAM
, chng minh rng din tích tam giác
BSC
bng tng din tích ca các tam giác
ABS
ACS
.
B'
C'
H
M
C
A
B
S
A'
15
Gii
Tìm cách gii
B
C mp SBC
nên mun chng minh
()m
p SBC mp SAM
, ta ch cn chng minh
BC
vuông góc
vi
AM
SM
.
Trình bày li gii
a)
;S
A mp ABC SA AB SA AC
.
.
. .SAB SAC c g c SB SC
Xét
SBC
cân ti
;S SM BC
Xét
ABC
đều
AM BC
. Suy ra
()BC mp SAM
.
Mt khác
()BC mp SBC
nên
( ) ( )mp SBC mp SAM
b) Xét
SAM
vuông ti
, 30
o
A SAM
nên
1
2
SA SM
hay
2SM SA
.
Din tích
BCS
11
. .2 .
22
BC SM BC SA BC SA
. (1).
Tng din tích các
ABS
ACS
là:
1 1 1
. . .
2 2 2
AB SA AC SA SA BC BC SA BC
. (2)
T (1) và (2) suy ra điều phi chng minh.
Ví d 3. Cho hình chóp ct t giác đều
. '. ' ' 'ABCD A B C D
. Mt mt phng song song với đáy của hình chóp
ct ct các cnh
', ', ', 'AA BB CC DD
lần lượt ti
, , ,M N P Q
. Chng minh rng t giác
MNPQ
là hình vuông.
Gii
Gọi S là đỉnh ca hình chóp sinh ra hình chóp ct. Vì
/ / mp MNPQ mp ABCD
nên hình chóp ct
.ABCD MNPQ
là hình chóp cụt đều. Các mt bên của nó đều là hình thang cân.
Suy ra
// ; //NP BC MQ AD
.
Mt khác
//BC AD
nên
//NP MQ
.
Chứng minh tương tự ta được
//MN PQ
.
Do đó tứ giác
MNPQ
là hình bình hành.
M
A
B
C
S
16
Xét
SBC
//NP BC
nên
.1
BC SB
NP SN
Xét
SAB
//MN AB
nên
.2
AB SB
MN SN
.
T (1) và (2)
BC AB
NP MN

, mà
BC AB
nên
NP MN
.
Hình bình hành
MNPQ
có hai cnh k bng nhau nên là hình thoi.
Hai đường thng
MP
AC
cùng nm trong mt phng
S
AC
và hai đường thẳng này không có đim chung
(vì nm trong hai mt phng song song) nên
//MP AC
. Chứng minh tương tự, ta được
//NQ BD
.
Ta có:
AC SC SB BD
MP SP SN NQ
. Vì
AC BD
nên
MP NQ
.
Hình thoi
MNPQ
có hai đường chéo bng nhau nên là hình vuông.
Ví d 4. Cho hình chóp tam giác đều
.S ABC
có độ dài cạnh đáy là 12 cm, độ dài cnh bên là 8 cm. Hãy tính:
a) Th tích ca hình chóp; b) Din tích toàn phn ca hình chóp.
Gii
Q
D'
P
C'
N
B'
C
A
B
D
S
A'
M
17
Tìm hướng gii
Để tính th tích và din tích toàn phn của hình chóp đều khi đã biết độ dài ca cạnh đáy và cạnh bên, ta cn
tính chiều cao và trung đoạn ca hình chóp.
Trình bày li gii
a) Gọi M là trung điểm ca
AC
O
là giao điểm của ba đường trung tuyến ca
ABC
Ta có
BM
là đường cao của tam giác đều nên:
3
63
2
AB
BM 
2
43
3
BO BM
SBO
vuông ti
O
nên ta có:
2
2 2 2 2
8 4 3 16
4 cm
SO SB OB
SO

Din tích
ABC
2
2
3 144 3
36 3 cm
44
AB

Th tích ca hình chóp là:
3
11
. 36 3.4 48 3 cm
33
V S h
b) Tam giác
SMA
vuông ti M nên
2 2 2 2 2
8 6 28 2 7 cm .SM SA MA SM
Din tích xung quanh ca hình chóp là:
2
12.3
. .2 7 36 7 cm
2
xq
S p d
.
Din tích toàn phn ca hình chóp là :
2
36 7 36 3 36 7 3 157,6 cm
tp
S
.
b) Tam giác
SMA
vuông ti
M
nên
2 2 2 2 2
86SM SA MA
28 2 7 (cm)SM
.
O
M
C
A
B
S
18
Din tích xung quanh ca hình chóp là:
2
12.3
. .2 7 36 7 (cm ).
2
xq
S p d
Din tích toàn phn ca hình chp là:
2
36 7 + 36 3 = 36 7 3 157,6 (cm ).
tp
S
Ví d 5. Cho hình chóp cụt tam giác đều
. ' ' 'ABC A B C
có cnh bên bng
17cm
, cạnh đáy lớn bng
28cm
, cnh
đáy nhỏ bng
12cm
. Tính din tích xung quanh ca hình chóp ct.
Gii ( h.20.7)
* Tìm hướng gii
Để tính din tích xung quanh ca hình chóp cụt đều khi đã biết độ dài
cạnh đáy lớn, độ dài cạnh đáy nhỏ còn phi tính chiu cao ca mt bên
* Trình bày li gii
Trong mt bên
''A B BA
v
'A H AB
ta được:
' ' 28 12
8 ( ).
22
AB A B
AH cm

Xét
'A AH
vuông ti
H
, ta có:
2 2 2 2 2
' ' 17 8 225 ' 15 ( )A H AA AH A H cm
.
Din tích xung quanh ca hình chóp ct là :
2
(12 28).15
.3 900 (cm )
2
xq
S

.
Hình 20.7
C. Bài tp vn dng
Chng minh song song, vuông góc. Tính chiu cao
20.1. Cho hình chóp t giác đều
.S ABCD
. Trên các cnh
, , ,SA SB SC SD
lần lượt lấy các điểm
', ', ', 'A B C D
sao cho
' ' ' '.SA SB SC SD
Chng minh rng:
a) Bốn đim
', ', ', 'A B C D
cùng thuc mt mt phng. nhn xét gì v mt phng
( ' ' ' ')A B C D
mp
( ).ABCD
b)
( ) ( )mp SAC mp SBD
.
20.2. Cho hình chóp t giác đều
.S ABCD
. Cho biết
.SA SC
Chng minh rng các mt bên nhng tam giác
đều.
20.3. Cho hình chóp
.S ABC
, c bn mt những tam giác đu cnh bng
a
. Gi
, , , M N P Q
lần t
trung điểm ca
, , , SC SB AB AB
. Chng minh rng t giác
MNPQ
là hình vuông.
20.4. Cho hình chóp tam giác đều
.S ABC
, các mt bên là nhng tam giác vuông cân ti
S
.
a) Chng minh rng mi mt bên vuông góc vi hai mt còn li.
b) Gọi độ dài mi cạnh đáy là
a
. Tính chiu cao ca hình chóp.
20.5. Mt hình chóp ct t giác đều din ch xung quanh bng tng diện ch hai đáy. Biết cạnh đáy lớn
bng
6cm
, cạnh đáy nhỏ bng
4cm
. Tính chiu cao ca hình chóp cụt đều.
28
17
12
C
B
A
C'
A'
D'
H
19
20.6. Cho hình chóp ct t giác đều
1 1 1 1
.ABCD A BC D
cnh
11
, ( )AB a AB b a b
. Mt mt phng song
song với hai đáy của hình chóp ct ct các cnh
1 1 1 1
, , , AA BB CC DD
lần lượt ti
2 2 2 2
, , , A B C D
chia hình
chóp ct ln thành hai hình chóp ct nh din tích xung quanh bng nhau. Gi
c
cnh ca hình vuông
2 2 2 2
A B C D
. Chng minh
22
2
2
ab
c
.
Tính din tích, th tích
20.7. Cho hình chóp t giác đều
.S ABCD
cạnh đáy bng
2a
cnh bên bng
10a
. Tính th tích hình
chóp.
20.8. Cho hình chóp lục giác đều
.S ABCDEF
2AD a
din tích tam giác
SAD
2
a
. Tính din tích
xung quanh ca hình chóp.
20.9. Cho hình chóp tam giác đu
.S ABC
các cạnh bên đều bng
a
. Chng minh rng khi các cnh bên
vuông góc vi nhau từng đôi một thì din tích xung quanh s ln nht.
20.10. Cho hình chóp t giác đều
.S ABCD
cnh bên dài
5cm
din tích xung quanh bng
2
48cm
. Tính
th tích hình chóp.
20.11. Cho hình chóp tam giác đều
.S ABC
các cnh bên bng
17cm
chiu cao bng
15cm
. Gi
', ', 'A B C
lần lượt là trung điểm ca
, , .SA SB SC
Tính th tích hình chóp ct
' ' '. .A B C ABC
20.12. Cho hình lập phương
. ' ' ' 'ABCD A B C D
cnh
a
. T hình lập phương này ct ra hình chóp
.'C BDC
.
Chng minh rng:
a) Hình chóp
.'C BDC
là hình chóp đều.
b) T s gia din tích xung quanh và diện tích đáy của hình chóp là
3
.
c) T s gia th tích hình chóp và th tích hình lp phương là
1
6
.
- 1 -
NG DN GII ĐÁP SỐ
Chƣơng 1. TỨ GIÁC
CHUYÊN ĐỀ 1. T GIÁC ........................................................................................................................ - 2 -
CHUYÊN ĐỀ 3. ĐƢỜNG TRUNG BÌNH CA TAM GIÁC, HÌNH THANG .................................. - 15 -
CHUYÊN ĐỀ 4. HÌNH BÌNH HÀNH ..................................................................................................... - 23 -
CHUYÊN ĐỀ 5. HÌNH CH NHT ...................................................................................................... - 29 -
CHUYÊN ĐỀ 6. HÌNH THOI VÀ HÌNH VUÔNG ............................................................................... - 37 -
CHUYÊN ĐỀ 7. ĐỐI XNG TRỤC. ĐỐI XNG TÂM ...................................................................... - 45 -
CHUYÊN ĐỀ 8. V HÌNH PH ĐỂ GII TOÁN TRONG CHƢƠNG T GIÁC .......................... - 54 -
CHUYÊN ĐỀ 9. TOÁN QU TÍCH ....................................................................................................... - 63 -
- 2 -
CHUYÊN ĐỀ 1. TỨ GIÁC
1.1.
Trƣờng hp hai góc ngoài tại hai đỉnh k nhau (h. 1.5)
Gi
11
, CD
là s đo hai góc trong,
22
, CD
là s đo hai góc
ngoài tại hai đỉnh k nhau là
C
D
.
Ta có:
2
2 1 1 1 1
180 180 180
o o o
C D C D C D
. (1)
T giác
ABCD
có:
11
360
o
A
B C D
. (2)
T (1) và (2) suy ra:
22
C D A B
.
Trƣờng hp hai góc ngoài tại hai đỉnh đối nhau (h 1.6)
Chứng minh tương tự, ta được:
22
A B C D
.
Hình 1.5
Hình 1.6
1.2. (h.1.7)
Ta có:
Gi
220
o
CDx DCy A B
( bài 1.1).
110
2
o
CDx CDy

.
Do đó
22
110
o
CD
.
Xét
CKD
có:
22
1
80 180 110 70
o o o o
CKD D C
.
Hình 1.7
1.3. (h.1.8)
T giác
ABCD
có:
360 360 2 .
oo
B D A C C
1 2 1 2
;B B D D
nên
1 1 1 1
180 180
oo
B D C B D C
. (1)
Xét
BCM
11
180
o
B M C
. (2)
2
1
2
1
A
B
D
C
2
1
2
1
A
B
D
C
2
1
2
1
y
x
K
A
B
D
C
1
1
2
1
2
M
N
A
D
B
C
- 3 -
T (1) và (2) suy ra
11
//D M DN BM
.
Hình 1.8
1.4. (h.1.9)
V đường phân giác ca
C
D
chúng ct nhau ti
E
.
Xét
ECD
có:
110 130
180 60
2
oo
oo
CED
.
( . . ) 60
o
ADE CDE c g c AED CED
.
( . . ) 60
o
BCE DCE c g c BEC CED
.
180
o
AEB
Do đó
,,A E B
thng hàng.
Vy
65 .
o
BAD EAD ECD
Do đó
360 (65 110 130 ) 55
o o o o o
ABC
.
Hình 1.9
1.5. (h.1.10)
Gi s t giác
ABCD
CD
là cnh ln nht.
Ta s chng minh
CD
nh hơn tổng ca ba cnh còn li.
(1)
Tht vy, xét
ADC
CD AD AC
.
Do đó
CD AD AB BC
.
Ta thy nếu các cnh t l vi 1, 3, 5, 10 thì không tha
mãn điều kin (1) nên không có t giác nào mà các cnh t
l vi 1, 3, 5, 10.
Hình 1.10
1.6. (h.1.11)
Gi
O
là giao điểm của hai đường chéo.
Xét
,AOB COD
vuông ti
O
, ta có:
2 2 2 2 2 2
AB CD OA OB OC OD
.
Tương tự ta được:
2 2 2 2 2 2
BC AD OA OB OC OD
.
Do đó:
2 2 2 2
AB CD BC AD
.
Suy ra
2 2 2 2 2
3 6 6.6 1,44 1,2AD AD AD
.
Hình 1.11
1.7. (h.1.12)
2
1
2
1
E
D
A
C
B
A
B
D
C
?
6
6,6
3
O
B
A
C
D
- 4 -
Gi
O
là giao điểm của hai đường chéo
,AC BD
ca t
giác
.ABCD
Gọi độ dài các cnh
, , ,AB BC CD DA
lần lượt là
, , ,a b c d
.
Vn dng bất đẳng thức tam giác ta được:
,OA OB a OC OD c
.
Do đó
O
A OC OB OD a c
hay
AC BD a c
(1)
Tương tự
AC BD b d
(2)
T (1), (2) suy ra
2
a b c d
AC BD

.
Xét các
ABC
ADC
ta có:
; 2 .AC a b AC c d AC a b c d
(3)
Tương tự
2.BD a b c d
(4)
T (3) và (4) suy ra
.AC BD a b c d
T các kết qu trên ta được điều phi chng minh.
Hình 1.12
1.8.
Trước tiên ta chng minh bài toán ph:
Cho
, 90 .
o
ABC A
Chng minh rng
222
BC AB AC
.
Gii (h.1.13)
V
BH AC
.
nên
H
nằm trên tia đối ca tia
AC
.
Xét
HBC
HBA
vuông ti
H
, ta có:
2
2 2 2 2 2
2 2 2 2 2 2
2. . 2. .
BC HB HC AB HA HA HC
AB HA HA AC HA AC AB AC HA AC
Vì HA.AC >= 0 nên
222
BC AB AC
( dấu “= “xảy ra khi
HA
tc là khi
ABC
vuông)
Vn dng kết qu trên để giải bài toán đã cho
Trƣờng hp t giác
ABCD
là t giác li (h.1.14)
Ta có
360
o
A B C D
Suy ra trong bn góc này phi có mt góc lớn hơn hoặc bng
90
o
, gi s
.
Hình 1.13
Hình 1.14
d
c
b
a
O
A
B
D
C
H
B
A
C
D
A
C
B
- 5 -
Xét
ABD
ta có:
2 2 2 2 2
10 10 200BD AB AD
200 14BD BD
.
Trƣờng hp t giác
ABCD
là t giác lõm (h.1.15)
Ni
CA
, ta có:
360
o
ACD ACB BCD
.
Suy ra trong ba góc này phi có mt góc lớn hơn hoặc bng
120
o
.
Gi s
120
o
ACB ACB
là góc tù.
Xét
ACB
có:
2 2 2 2 2
10 10 200AB AC BC
200 14AB AC
.
Vy luôn tn tại hai điểm đã cho có khong cách lớn hơn 14.
Hình 1.15
1.9. (h.1.16)
Ta chng minh bằng phương pháp phản chng.
Gi s không có hai cnh nào ca t giác bng nhau. Ta có th
gi s
.a b c d
Ta có
.a b c BD c d
.
Hình 1.16
Do đó:
2.a b c d d
Ta đặt
a b c d S
thì
2Sd
(*)
Ta có:
. ( )S a S ma m N
(1)
. ( )S b S nb n N
(2)
. (p )S c S p c N
(3)
. (q )S d S q d N
(4)
T (4) và (*)
2qd d
do đó
2q
a b c d
nên t (1), (2), (3),(4) suy ra
2m n p q
Do đó
3; 4; 5; 6q p n m
T (1), (2), (3), (4) suy ra
1 1 1 1
; ; ;
a b c d
m S n S p S q S
D
A
C
B
d
c
b
a
D
A
C
B
- 6 -
Ta có:
1 1 1 1 1 1 1 1
1
6 5 4 3
a b c d
m n p q S
T đó
19
1
20
, vô lý.
Vậy, điều gi s là sai, suy ra tn ti hai cnh ca t giác bng nhau.
1.10. Coi mỗi người như một điểm , ta có chín điểm
, , ,...A B C
Nối hai điểm với nhau ta được một đoạn
thng. Ta tô màu xanh nếu hai người không quen nhau, ta tô màu đỏ nếu hai người quen nhau. Ta s chng
minh tn ti mt t giác có các cạnh và đường chéo cùng tô màu đỏ.
Xét hai trường hp:
* Trường hp có một điểm là đầu mút ca bốn đoạn thng màu xanh là
, , ,AB AC AD AE
v nét đứt (h.1.17)
Hình 1.18
Hình 1.17
A
B
C
D
E
A
B
C
D
E
Xét tam giác
ABC
có hai đoạn thng
,AB AC
màu xanh nên đoạn thng
BC
màu đỏ vì bt kì tam giác nào
cũng có một đoạn thẳng màu đỏ. Tương tự các đoạn thng
, , , ,CD DE EB BD CE
cũng có màu đỏ ( nét v
liền) ( hình 1.18). Do đó, tứ giác
BCDE
có các cạnh và đường chéo được tô đỏ, nghĩa là tồn ti mt nhóm
bốn người đôi một quen nhau.
* Trường hp mọi điểm đều là đầu mút ca nhiu nhất là ba đoạn thng màu xanh. Không th mọi điểm đều
là đầu mút của ba đoạn thẳng màu xanh vì khi đó số đoạn thng màu xanh là
9.3
2
N
.
Như vậy, tn ti một điểm là đầu mút ca nhiu nhất là hai đoạn thng màu xanh, chng hạn đó là điểm
,A
do đó
A
là đầu mút ca ít nhất là sáu đoạn thẳng màu đỏ, gi s đó là
, , , , ,AB AC AD AE AF AG
( h. 1.19)
Trong sáu điểm
, , , , , B C D E F G
tn tại ba điểm là đỉnh ca mt tam giác có ba cạnh cùng màu ( đây là
bài toán cơ bản v phương pháp tô màu) chẳng hạn đó là
BCD
( h. 1.20)
- 7 -
j
Hình 1.20
Hình 1.19
A
B
C
D
E
F
G
B
A
C
D
E
F
G
Trong
BCD
có mt cạnh màu đỏ (theo đề bài) nên ba cnh ca
BCD
cùng màu đỏ. Khi đó, tứ giác
ABCD
là t giác có các cạnh và đường chéo được tô đỏ, nghĩa là tồn ti mt nhóm bốn người đôi một quen
nhau.
- 8 -
CHUYÊN ĐỀ 2. HÌNH THANG. HÌNH THANG CÂN. DNG HÌNH THANG
2.1 (h.2.11
1
1
2
1
2
1
A
B
N
D
M
C
a) Xét
MAD
:
90
O
M
1
0
1
0
90 90
2
180 / /
O
AD
AD
A D AB CD
Vy t giác ABCD là hình thang.
b) Ta có:
0
180ABC BCD
( hai góc k vi mt cnh bên)
0
90
2
ABC BCD

hay
11
0
90BC
Xét
NBC
có:
0
11
0 0 0
180 ( ) 180 90 90N B C
Vy
NB NC
2.2.
a) Gi
E
là giao điểm ca tia
BM
vi tia
CD
ABM DEM
(g.c.g)
AB DE
MB ME
CBE
CM
va là trung tuyến vừa là đường cao nên là tam giác cân
CB CE
CB CD DE CB CD AB
(vì
AB DE
).
- 9 -
b)
CBE
cân ti
,C CM BM
(1)
12
C C MH MD
(tính chất điểm nm trên tia phân giác).
HCM DCM
(cnh huyn góc nhn)
CH CD CHD
cân
CM DH
(2)
T (1) và (2) suy ra
//BM DH
do đó tứ giác
là hình thang.
2.3. (h.2.13)
Xét hình thang
ABCD
vuông ti
A
D
.
Gi s
AB CD
, áp dụng định lí Py-ta-go ta có:
2 2 2 2 2 2
;.AC AD DC BD AD AB
Suy ra
2 2 2 2 2 2
( ) ( ).AC BD AD DC AD AB
Do đó
2 2 2 2
.AC BD CD AB
2.4. (h.2.14)
V
BH CD
ta được
; 20.AB DH BH AD
Xét
BHC
vuông ti
H
có:
2 2 2 2 2
29 20 441 21.HC BC BH HC
Xét
ADC
vuông ti
D
có:
2 2 2 2 2
52 20 2034 48.CD AC AD CD
Do đó
48 21 27 27.DH CD HC AB
Nhn xét: Bài này đã vẽ thêm đường cao
BH
của hình thang. Đó là một cách v hình ph thường dung khi
gii bài toán v hình thang.
2.5.
Hình 2.13
- 10 -
T giác
MONB
//OM BC
nên là hình thang. Hình thang này có
()MBN ONB ABC
nên là
hình thang.
Chứng minh tương tự ta đưc các t
;ONCP OMAP
cũng là hình thang cân.
Suy ra:
; ; .MN OB NP OC MP OA
Do đó
MNP
là tam giác đều
MN MP NP
OB OC OA O
là giao điểm của ba đường trung trc ca
ABC
.
Trong tam giác đều, giao điểm của ba đường trung trực cũng là giao điểm của ba đường cao, ba đường
trung tuyến.
Chiu cao
h
của tam giác đều cnh
a
được tính theo công thc:
3
2
a
h
.
2 2 3 3
.
3 3 2 3
aa
OA h
Do đó chu vi của
MNP
là:
3
.3 3
3
a
a
.
2.6. (h.2.16)
Trên na mt phng b
CD
có cha
A
v
Cx
sao cho
.DCx ADC
Tia
Cx
ct tia
AB
ti
E
.
Khi đó hình thang
AECD
là hình thang cân.
AC DE
.DAB CEB
Xét
ABD
có góc
DBE
là góc ngoài nên
DBE DAB DBE CEB
(vì
DAB CEB
).
Do đó
.DBE DEB DE BD AC BD
2.7.
Xét trường hp
OA OC
(h.2.17)
AOC
là tam giác cân.
0
60O
nên
0
60 .A C AC OA OC
Hình 2.16
- 11 -
Do đó
2.
2
OA OC
AC OA OC AC
Xét trường hp
OA OC
(h.2.18)
Trên tia
Ox
lấy điểm
D
, trên tia
Oy
lấy điểm
B
sao cho
;.OB OA OC OD
Các
;OAB OCD
cân ti
O
nên:
0
180
//
2
O
OAB ODC AB CD
T giác
ABCD
là hình thang.
Mt khác
OCD ODC
nên
ABCD
là hình thang cân
AC BD
.
Gi
K
là giao điểm ca
AC
BD
. Ta có:
;.AC AK KC BD BK KD
( ) ( ).AC BD AK BK KC KD
(1)
;.AK BK AB KC KD CD
(2)
T (1) và (2) suy ra:
BF CE
BM CM
. Do đó
BF CE
(do
BM CM
)
Cách 2: (dùng menelaus)
Xét tam giác
ABC
với ba điểm
,,F E M
thng hàng, ta có:
. . 1
EA MC FB
EC MB FA
(4)
Do
2
BAC
AEF AFE
nên
AFE
cân
A
. Suy ra
AE AF
(5)
T (4) và (5) suy ra:
BF CE
. Điều phi chng minh.
2.8.(h.2.19)
Qua A vẽ một đường thẳng song song với CD cắt tia CB tại B'. Hình thang
AB'CD có hai góc ở đáy bằng nhau nên là hình thang cân.
Vậy nếu B' trùng với B thì tứ giác ABCD là hình thang cân.
Nếu B' không trùng với B, ta có AC = B'D.
Mặt khác, AC = BD nên B'D = BD.
Do đó DBB' cân
o
DB'B DBB' 90 ,
vô lí.
Vậy B' trùng với B và tứ giác ABCD là hình thang cân.
2.9. (h.2.20)
a) Phân tích
Vẽ BE // AC (E tia DC) ta được
Hình 2.17
Hình 2.18
Hình 2.19
- 12 -
o
DBE 110 ,
- BDE dựng được ngay (c.g.c);
- Điểm A thoả mãn hai điều kiện: A nằm trên tia Bx // DE và cách B là 2cm.
- Điểm C thoả mãn hai điều kiện: C nằm trên tia ED và cách E là 2cm.
b) Cách dựng
- Dựng BDE sao cho
o
DBE 110 ,
BD = 3cm, BE = 4cm.
- Dựng tia Bx // DE và trên đó đặt BA = 2cm (hai tia Bx và ED cùng nằm trên một nửa mặt phẳng bờ BE).
- Trên tia ED đặt EC = 2cm.
- Nối AD, BC ta được hình thang ABCD phải dựng.
c) Chứng minh
Tứ giác ABCD theo cách dựng có AB // CD nên là hình thang.
Xét hình thang ABEC có AB = EC = 2cm nên AC // BE và AC = BE = 4cm.
o
DOC DBE 110
o
BOC 70 .
Hình thang ABCD theo cách dựng có
AB = 2cm, BD = 3cm, AC = 4cm và
o
BOC 70 .
d) Biện luận
Bài toán có một nghiệm hình.
2.10. (h.2.21)
Cách dựng
- Dựng ABD sao cho
o
A 120 ,
AD = 2, DB = 4.
- Dựng tia Dx // AB (hai tia Dx và AB cùng nằm trên một nửa mặt phẳng bờ AD).
- Dựng cung tròn tâm B, bán kính a cắt Dx tại C.
- Nối BC ta được hình thang ABCD phải dựng.
Biện luận
Vẽ AH CD thì
o
DAH 30 .
Do đó
1
DH AD 1
2

cm
22
AH 2 1 3.
Nếu
a3
thì đường tròn (B; a) không cắt tia Dx nên bài toán không có nghiệm hình.
Nếu
a3
thì đường tròn (B; a) có chung với tia Dx một điểm, bài toán có một nghiệm hình.
Nếu
3 a 4
thì đường tròn (B; a) cắt tia Dx tại hai điểm C và C', bài toán có hai nghiệm hình.
Nếu a 4 thì đường tròn (B; a) cắt tia Dx tại một điểm C D nên bài toán có một nghiệm hình.
2.11. (h.2.22)
a) Phân tích
Giả sử ta đã dựng được tứ giác ABCD thoả mãn đề bài.
Ta thấy AB = 2,5cm dựng được ngay.
BE = AC = 4cm, CE =
H
ình
AB
2.20
=
2cm.
Hình 2.21
- 13 -
Trên tia BC lấy điểm C'. Vẽ đoạn thẳng C'D' // CD và C'D' = CD. Khi đó
o
C' C 60
và DD' // CC'.
b) Cách dựng
- Dựng AB = 2,5cm.
- Trên cùng một nửa mặt phẳng bờ AB dựng các tia Ax và By sao cho
o
BAx 120 ,
o
ABy 100 .
- Trên tia By lấy điểm C'.
- Dựng đoạn thẳng C'D' sao cho
o
BC'D' 60
và C'D' = 4cm.
- Dựng DD' = BC' (D Ax).
- Dựng DC // D'C' (C By).
Tứ giác ABCD là tứ giác phải dựng.
Các bước còn lại, bạn đọc tự giải.
2.12. (h.2.23)
a) Phân tích
Giả sử đã dựng được ABC thoả mãn đề bài.
Trên tia đối của tia BC lấy điểm D; trên tia đối của tia CB lấy điểm E sao
cho BD = BA, CE = CA.
Khi đó DE = DB + BC + CE = BA + BC + CA = 8cm.
ABD vuông cân tại B nên
o
D 45 .
Góc ACB là góc ngoài của tam giác cân CAE nên
ACB 2E
o
m
E.
2

ADE dựng được (g.c.g).
Điểm B thoả mãn hai điều kiện: B nằm trên đoạn thẳng DE và AB DE.
Điểm C thoả mãn hai điều kiện: C nằm trên đoạn thẳng DE và nằm trên đường trung trực của AE (vì C
cách đều hai đầu đoạn thẳng AE).
b) Cách dựng
- Dựng ADE sao cho DE = 8cm;
o
D 45
o
m
E.
2
- Dựng AB DE (B DE).
- Dựng đường trung trực của AE cắt DE tại C.
- Nối AC ta được ABC phải dựng.
c) Chứng minh
ABD vuông tại B có
o
D 45
nên là tam giác vuông cân BA = BD.
Điểm C nằm trên đường trung trực của AE nên CA = CE.
ABC có AB + BC + CA = BD + BC + CE = DE = 8cm;
Hình 2.22
Hình 2.23
- 14 -
o
B 90
o
o
m
ACB 2E 2. m .
2
d) Biện luận
- Nếu m 90 thì bài toán không có nghiệm hình.
- Nếu 0 < m < 90 thì bài toán có một nghiệm hình.
- 15 -
CHUYÊN ĐỀ 3. ĐƢỜNG TRUNG BÌNH CỦA TAM GIÁC, HÌNH THANG
3.1. (h.3.7)
Gọi O là giao điểm của AC và BD.
Ta có AC BD và OA = OC.
Xét ABD có MN là đường trung bình
MN // BD và OA MN (vì OA BD).
Xét ABC có ON là đường trung bình
ON // BC và ON ME (vì ME BC).
Xét ACD có OM là đường trung bình OM // CD và OM NF (vì NF CD).
Xét OMN có OA, ME, NF là ba đường cao nên chúng đồng quy.
3.2. (h.3.8)
Gọi O là trung điểm của BC.
Xét EBC có OM là đường trung bình
OM // CE và
CE
OM .
2
Xét DBC có ON là đường trung bình
ON // BD và
BD
ON .
2
Ta có
1
M AQP;
1
N APQ
(so le trong).
APQ cân tại A
11
Q P N M
OM = ON CE = BD.
3.3. (h.3.9)
a) Gọi D và E thứ tự là giao điểm của AH và AK với đường
thẳng BC.
ABD có BH vừa là đường phân giác, vừa là đường cao nên
là tam giác cân HA = HD.
Tương tự ta có KA = KE.
Xét ADE có HK là đường trung bình nên HK // DE HK // BC.
Do đó tứ giác BCKH là hình thang.
b) Ta có
1 1 1 1
H B ;K C
(so le trong).
Hình thang BCKH là hình thang cân
1 1 1 1
H K B C
ABD ACE ABC ACB
ABC cân tại A.
3.4. (h.3.10)
Gọi M và N lần lượt là trung điểm của BC và CA.
Gọi F và G lần lượt là trung điểm của AH và BH.
Hình 3.7
Hình 3.8
Hình 3.9
- 16 -
Ta có MN là đường trung bình của ABC; FG là đường trung bình của ABH.
Suy ra MN // AB và
1
MN AB.
2
FG // AB và
1
FG AB.
2
Do đó MN // FG và MN = FG. Dễ thấy OM // AD, ON // BE.
OMN và HFG có:
MN = FG;
OMN HFG;
ONM HGF
(hai góc có cạnh tương ứng song song).
Vậy OMN = HFG (g.c.g)
AH
OM HF .
2
3.5. (h.3.11)
Gi
M
là trung điểm ca
BD
thì:
1
2
MD BD AH
ABC
cân ti
A
,
AH
là đường cao nên
HB HC
.
Ta có:
HM
là đường trung bình ca
//BCD HM AC
. Hình 3.11
Hình thang
có hai đường chéo bng nhau nên là hình thang cân.
ADH DAM
(c c c)
1 1 1
90A D C B C
. (1)
Ta đặt
B C x
thì (1)
90 36
2
x
x x x
.
Vy
ABC
36 ; 108B C A
.
3.6. (h.3.12)
ABD
ACE
có:
AB AC
;
(cùng ph vi góc
DAC
);
AD AE
.
Do đó
ABD ACE
(c g c)
BD CE
11
BC
. Hình 3.12
Gi
H
K
lần lượt là giao điểm của đường thng
BD
vi
CE
CA
.
Ta có:
1
90B BKA
1
90C CKH
90H
.
Xét
CBD
MN
là đường trung bình
//MN BD
1
2
MN BD
.
Xét
CED
NP
là đường trung bình
//NP CE
1
2
NP CE
.
1
1
2
1
M
D
H
A
B
C
2
1
1
H
K
P
N
M
E
A
B
C
D
Hình 3.10
- 17 -
BD CE
nên
MN NP
.
Ta có:
90MNP H
(hai góc có cạnh tương ứng song song).
Do đó
MNP
vuông cân ti
90NN
;
45MP
.
3.7. (h.3.13)
ADC
BCD
có:
AD BC
,
AC BD
,
CD
chung.
Do đó
ADC BCD
(c c c)
ACD BDC COD
cân.
Mt khác
60COD 
nên
COD
đều.
Ta có:
OE ED
nên
CE
là đường trung tuyến Hình 3.13
của tam giác đều, do đó
CE
là đường cao.
Vy
CE BD
.
Xét
EBC
vuông ti
E
EF
là đường trung tuyến ng vi cnh huyn nên
1
2
EF BC
.
Chứng minh tương tự, ta có:
1
2
GF BC
.
Xét
AOD
EG
là đường trung bình nên
1
2
EG AD
1
2
EG BC
(vì
)AD BC
.
Vy
1
2
EF FG EG BC GEF



đều
60G E F
.
3.8. (h.3.14)
Gi
D
E
th t là trung điểm ca
AB
AC
.
Ta có:
OD
OE
là đường trung bình ca
ABC
nên
//OE AD
OE AD
;
//OD AE
OD AE
.
BDO BAC
;
CEO BAC
ng v) .
MAB
vuông cân ti
M
nên
MD AB
MAD
vuông cân
AD MD
. Hình 3.14
Tương tự,
NE AC
NEA
vuông cân
AE NE
.
NOE
có:
M
D OE AD
;
90ODM OEN BAC
;
O
D NE AE
.
Vy
OMD NOE
(c g c)
OM ON
OMD NOE
.
Do đó
180 90 90MON MOD DOE NOE MOD BDO OMD
.
F
E
G
B
O
D
C
A
E
D
O
N
M
B
C
A
- 18 -
Vy
vuông cân.
3.9. (h.3.15)
V đường phân giác
AD
thì
AD
là một đường thng c định.
Gi
O
là trung điểm ca
BC
thì
O
là một điểm c định.
Gi
,PQ
lần lượt là giao điểm của đường thng
OM
với các đường thng
AC
AB
.
Xét
EBC
ON
là đường trung bình
//ON BE
1
2
ON BE
.
Xét
ECF
MN
là đường trung bình Hình 3.15
//MN CF
1
2
MN CF
.
BE CF
nên
ON NM OMN
cân
11
MO
.
Ta có:
1
1 2
P M P
;
11
Q O P Q
.
Xét
APQ
BAC
là góc ngoài nên
1
BAC P Q
.
Mt khác
nên
21
//A P OP AD
.
Vy
M
nm trên một đường thẳng đi qua
O
và song song vi
AD
. Đó là một đường thng c định.
3.10. Gi
M
là trung điểm ca
AB
O
là một điểm tùy ý không nm gia
A
B
.
Trường hp
O
nằm trên tia đối ca tia
AB
hay tia đối ca tia
BA
(h.3.16), ta chứng minh được
2
OA OB
OM
. (1)
Hình 3.16
Trường hp
O
không thng hàng vi
A
B
(h.3.17).
Gi
N
là trung điểm ca
OB
, khi đó
MN
là đường trung bình ca
OAB
,
2
OA
MN
.
Xét
ta có:
OM ON MN
2
OA OB
OM

. (2) Hình 3.17
T (1) và (2) suy ra:
2
OA OB
OM
. (*)
Áp dng h thức (*) đối vưới
n
điểm
1 2 3
, , ,...,
n
O O O O
ta có:
1
1
2
1
2
1
N
P
Q
O
D
M
F
B
C
A
E
M
O
B
A
N
M
B
O
A
- 19 -
11
1
2
O A O B
OM
;
22
2
2
O A O B
OM
; …;
2
nn
n
O A O B
OM
.
Cng tng vế các bất đẳng thức trên ta được:
1 1 2 2
12
... ...
2 2 2
nn
n
O A O B
O A O B O A O B
O M O M O M

1 2 1 2
... ...
2 2 2 2
nn
O A O A O A O B O B O B
aa
a
.
Như vậy điểm cần tìm chính là trung điểm
M
ca
AB
.
3.11. (h.3.18)
Gi
'AA
,
'BB
,
'CC
là ba đường cao ca
ABC
.
Gi
,,M N P
là trung điểm của các đường cao đó.
Gi
,,D E F
theo th t là trung điểm ca
,BC CA
AB
.
Ta có:
,,EF FD DE
là các đường trung bình ca
ABC
// , // , //EF BC FD CA DE AB
. Hình 3.18
M
là trung điểm ca
'AA
nên
M FE
.
N
là trung điểm ca
'BB
nên
N FD
. Vì
P
là trung điểm ca
'CC
nên
P DE
.
Theo đề bài, ba điểm
,,M N P
thẳng hàng nên các điểm này ch có th nm trên mt trong các cnh
,DE DF
hoc
EF
ca
DEF
.
Nếu ba điểm
,,M N P
cùng nm trên
DE
thì
N
trùng vi
D
,
M
trùng vi
E
, khi đó
ABC
vuông
ti
C
, trái vi gi thiết góc
C
là góc nh nht ca
ABC
.
Nếu ba điểm
,,M N P
cùng nm trên
DF
thì cũng lập luận như trên,
ABC
vuông ti
B
, trái vi gi
thiết
BA
.
Vy ba điểm
,,M N P
nm trên
EF
.
Lp luận tương tự như trên ta được
ABC
vuông ti
A
.
3.12. (h.3.19)
a) V
BK CD
ta được
//AH BK
//AB HK
AB HK
.
ADH BCK HD KC
.
Ta có:
2HD KC CD HK HD CD AB
2
CD AB
HD

. Hình 3.19
Theo ví d 4 thì đoạn thng
PQ
nối trung điểm của hai đường chéo bng na hiệu hai đáy. Vậy
HD PQ
.
M
P
N
D
E
F
A'
B'
C'
B
C
A
P
Q
B
K
D
C
H
A
- 20 -
b) Ta có:
22
CD AB CD AB
HC CD HD CD

.
Đưng trung bình ca hình thang bng na tổng hai đáy. Do đó
HC
bằng độ dài đường trung bình ca
hình thang.
3.13. (h.3.20)
Gi
D
là trung điểm ca
BC
.
V
//BE ON
,
//DF ON
(
,E F AC
).
Ta có:
1
2
OB BD DC BC
.
Hình 3.20
Xét
ABE
//MN BE
MA MB
nên
NA NE
. (1)
Xét hình thang
ONFD
//BE ON
OB BD
nên
NE EF
. (2)
Xét
CBE
//DF BE
BD DC
nên
EF FC
. (3)
T (1), (2), (3) suy ra:
AN NE EF FC
, do đó
1
4
AN AC
.
3.14. (h.3.21)
Gi
O
là trung điểm ca
MN
.
V
OF BC
;
AH BC
;
MD BC
NE BC
.
Ta có:
OF//AH// MD// NE
BMD ABH
(cnh huyn - góc nhn)
MD BH vµ BD=AH
(1)
Tương tự,
CNE ACH
NE CHvµCE AH
(2)
T (1) và (2) suy ra
B
D = CE AH .
D thy
OF
là đường trung bình ca hình thang
MD NE BH CH BC
OF
2 2 2

(không đổi).
Ta có:
FD FE; BD CE FB FC.
Vy
O
nằm trên đường trung trc ca
BC
và cách
BC
mt khoảng không đổi là
BC
2
. Do đó
O
là mt
điểm c định.
Suy ra
MN
đi qua một điểm c định là điểm
O.
3.15. (h. 3.22)
* Tìm hướng gii
F
E
N
M
O
D
B
C
A
Hình 3.21
- 21 -
Điu phi chng minh là
1
2
HF CD
gợi ý cho ta nghĩ đến định lý đường trung bình ca tam giác. Ta v
đường trung bình
EG
ca
MCD
thì
1
.
2
EG CD
Ch còn phi chng minh
HF EG
* Trình bày li gii:
Gi
E
là trung điểm ca
CM
,
G
là trung điểm ca
DM
. Khi đó
EG
là đường trung bình ca
1
MCD EG CD.
2
(1)
CAM DBMc©n t¹i C D mµ C D
nên các góc đáy
ca chúng bng nhau:
CAM CMA DMB DBM
CA// DM vµ CM // DB
(Vì có các cặp góc đồng v bng
nhau).
Xét
CMB
EF
là đường trung bình
EF//MB.
Xét
HG
là đường trung bình
HG//AM.
Suy ra:
EF// HG
(vì cùng song AB). Vy t giác
EFGH
là hình thang.
Xét hình thang
ACDM
EH
là đoạn thng nối trung điểm hai đường chéo nên
EH // AC
.
Tương tự, xét hình thang
CDBM
có:
FG // DB.
Do đó
EHG CAM,FGH DBM.
Mt khác:
CAM DBM
(chng minh trên) nên
EHG FGH
Vây hình thang
EFGH
là hình thang cân
HF EG
(2)
T (1) và (2) suy ra
1
.
2
HF CD
3.16. (h.3.23)
V
c©n t¹i A.ABC
V cnh
AB
lấy điểm
M
, trên tia đối ca tia
CA
ly
N
sao cho
BM CN
Như vậy
AB AC AM AN.
(1)
Ta phi chng minh chu vi
ABC
nh hơn chi vi
Mun vy ta phi chng minh
BC MN
Ta v
MD// NE // BC (D AC,E tia ®èi cña tia BA)
Hình thang cân
MDCB h×nh thang c©n MB DC,
BM CN nªn DC = CN
Xét hình thang cân
, có
Hình 3.22
Hình 3.23
- 22 -
BC // NE vµ DC = CN nªn MB = BE
Vëy
BC
là đường trung bình ca hình thang
V
MH EN
thì
HN BC
(xem bài 3.12)
Xét
vuông ti
H cã HN < MN BC MN
(2)
T (1) và (2) suy ra chu vi
ABC
nh hơn chu vi
AMN.
- 23 -
CHUYÊN ĐỀ 4. HÌNH BÌNH HÀNH
4.1 (h.4.6)
V hình bình hành
DAEF
. Khi đó
AF ®i qua M
Gi
H
là giao điểm ca
MA víi BC.
Ta có:
EF AD AB
00
ABF DAE 180 mµ BAC DAE 180 nªn AEF BAC.
AEF CAB (g.c.g) EAF ACB
Ta có:
0 0 0
CAH EAF 90 ACB CAH 90 H 90 .
Do đó
MA BC
4.2. (h.4.7)
Ta đặt
00
ADC th× DAM 90 ;NCD 90 .
DAM vµ NCD
có:
0
AM CD( AB);DAM NCD( 90 )
AD CN( BC)
Do đó:
DAM NCD (c.g.c)
DM DN (1)
DMA NDC
Kéo dài
MA t CD i H.
Ta cã: MA AB MH CD
Xét
0
MDH cã DMA ADM 90
0
NDC ADM 90
0
Hay MDN 90 (2)
T (1) và (2) suy ra
vuông cân ti
D
4.3. (h.4.8)
V
HM//AC (M AB). HN//AB (N AC)
CH AB nªn CH HN. V× BH AC n BH HM.
Xét
HBM vu«ngi H cã BM > HB (1)
Xét
HCN vu«ng i H cã CN > HC (2)
Xét hình bình hành
ANHM
AM AN AM MH HA (3)
Hình 4.6
Hình 4.7
- 24 -
T (1), (2), (3) suy ra:
BM CN AM AN HB HC HA
Do đó:
(MB AM) (CN AN) HA HB HC hay AB + AC > HA + HB + HC.
Chứng minh tương tự, ta được:
BC BA HA HB HC
CA CB HA HB HC.
Cng tng vế ba bất đẳng thức trên ta được:
2(AB BC CA) 3(HA HB HC)
Do đó:
3
AB BC CA (HA HB HC).
2
4.4. (h.4.9)
Qua
O
dng một đường thng song song vi
BC
ct
AB
CD
lần lượt ti
E vµ G
. Qua
O
dng mt
đường thng song song vi
CD
ct
AD
ti
H
.
Qua
E
dng một đường thng song song vi
OC
ct
BC i F.
Khi đó tứ giác
EFGH
tho mãn đề bài.
Tht vy, các t giác
AEOH,HOGD
là nhng hình thang
cân.
OA EH;OD HG (1)
T giác
EFCO
là hình bình hành
OC EF (2)
OE CF.
Suy ra:
OG BF.
Vy t giác
OBFG
là hình bình hành
OB GF. (3)
T (1), (2), (3) suy ra t giác
EFGH
tho mãn đề bài.
4.5. (h.4.10)
Gi
O
là giao điểm ca
AC vµ BD
. V
'
OO xy.
Ta có:
' ' ' ' '
AA //BB //CC //DD //OO .
Xét hình thang
''
AA C C
' ' ' ' ' '
OA OC vµ OO //AA nªn OA OC
Do đó:
'
OO
là đường trung bình ca hình thang
''
' ' ' ' ' '
AA CC
AA C C OO hay AA C C 2OO .
2
Xét hình thang
, cũng chứng minh tương tự, ta có:
' ' '
BB DD 2OO .
Hình 4.8
Hình 4.9
Hình 4.10
- 25 -
T đó suy ra:
' ' ' '
AA CC BB DD .
4.6 (h.4.11)
a)
ABCD
là hình bình hành nên
ABC ADC.
Ta đặt
00
ABC m ,ABM n ,
khí đó
00
MBC CDN m n .
MBC vµ CDN
có:
MB CD( AB); MBC CDN (cmt)
BC DN( AD). VËy MBC CDN (c.g.c) CM CN.
b) Các
ABM vµ AND
là nhng tam giác cân có góc đỉnh bng nhau mà
AB AD nªn AM > AN
(bn
đọc t chng minh)
Xét
ACM vµ CAN CM = CN; CA chung vµ AM > AN nªn ACM ACN
Xét
OCM vµ OCN cã CM = CN; CO chung vµ ACM ACN nªn OM > ON
4.7 (h.4.12)
V hình bình hành
BDEF
thì
EF BD (1); ED = FB.
Ta có:
AD CE;AB AC BD EA. (2)
T (1) và (2) suy ra
EF EA
Ta có:
CEF DAE
(so le trong):
DEA DAE
(hai góc đáy của tam giác cân).
Suy ra:
CEF DEA. CEF DEA (c.g.c) CF AD.
T đó suy ra:
BF CF BC FCB ®Òu
.
Ta đặt
00
BAC m ,ADE n
V tia
Fx
là tia đối ca tia
FC
CFE DAE
nên
0.
EFx BAC m
Ta có:
0 0 0 0
120 120 . (*)BFx hay m n
Trong
CEF
ta có
0 0 0
; 60 .ECF D n CFE CEF n
Do đó:
0 0 0 0 0 0 0 0 0 0
60 60 180 3 60 20 .n n n n n
T (*)
00
100m
. Suy ra
0
40 .ABC ACB
4.8 (h 4.13)
Hình 4.11
Hình 4.12
- 26 -
Gi
, , , , ,M N P Q E F
lần lượt là trung điểm ca
, , , , ,AB BC CD DA AC BD
. Ta phi chng minh
,MP NQ
EF
cùng đi qua một điểm.
Xét
ABC
MN
là đường trung bình
//MN AC
2
AC
MN
Chứng minh tương tự, ta có :
//PQ AC
2
AC
PQ
.
Suy ra
//MN PQ
MN PQ
.Do đó tứ giác
MNPQ
là hình bình hành.
Chứng mnih tương tự ta được t giác
MEPF
là hình bình hành.
Hai hình bình hành
MNPQ
MEPF
có chung đường chéo
MP
nên các
đường chéo
,MP NQ
EF
đồng quy tại trung điểm ca mỗi đường.
4.9. (h. 4.14)
Bn chng minh t giác
MGHN
MFNE
là hình bình hành.Hai hình
bình hành này có chung đường chéo
MN
nên các đường chéo
MN, EF
GH
đồng quy.
4.10. (h4.15)
Qua
A
v đường thng
xy // PQ
.
Trên tia Ax lấy điểm
M
, trên tia
Ay
lấy điểm N sao cho
.AM AN PQ
Như vậy các điểm
M
N
c định.
T giác
AMBD
có hai cạnh đối din song song và bng nhau nên là hình bình hành
//BM AD
.
Mặt khác , BC // AD nên ba điểm B, M ,C thẳng hàng ( tiên đề Ơ- clit)
Do đó đường thẳng BC đi qua điểm c định M.
Chứng minh tương tự , ta được đường thẳng CD đi qua điểm c định N.
4. 11 ( h4.16)
Xét t giác ABCD có AC = m; BD = n và
BOC
V hình bình hành ADBE và hình bình hành CAEF.
Khi đó EF = AC = m; CF = AE = BD = n ;
EAC BOC

.
Như vậy hình bình hành CAEF hoàn toàn được xác
định, do đó hai đường chéo AF và CE không đổi.
D thy t giác BFCD là hình bình hành => BF = CD.
Chu vi t giác ABCD là :
A
B CD BC AD AB BF BC BE AF CE
- 27 -
Dấu “=” xảy ra
, , / /
, , / /
A B F thang hang AB CD
C B E thang hang AD BC




< => ABCD là hình bình hành .
Vy chu vi ca t giác ABCD nh nht khi và ch khi ABCD là hình bình hành.
4. 12 ( h.4.17)
a ) Phân tích
Gi s đã dựng được MN // Bc sao cho BM = AN.
V ND // AB
D
BC
T giác MNDB là hình bình hành
= > DN = BM mà BM = AN nên DN = AN =>
NAD
cân
21
AD
Mt khác
( so le trong ) nên
Do đó AD là đường phân giác ca ca góc A
Đim D dựng được suy ra các điểm N và M cũng dựng được.
b ) Cách dng
-, Dưng đường phân giác AD ca tam giác ABC
- Dng DN // AB
N
AC
- Dng MN // BC
M
AB
Các bước còn li , bạn đọc t gii.
4. 13 ( h.4.18)
a ) Phân tích
Gi s đã dựng được hình bình hành thỏa mãn đề bài.
Gọi O là giao điểm của hai đường chéo và K là giao điểm ca MN và AC
Xét
CBD
MN
là đường trung bình
//BDMN
. Xét
COB
MB MC
//MK OB
nên
CK KO
.
Vy
MK
là đường trung bình nên
1
2
MK OB
Chứng minh tương tự ta được
1
2
KN OD
Mt khác,
OB OD
nên KM=KN.
Vậy điểm K là trung điểm của MN xác định được .
D thy
1 1 1
2 2 4
OK KC OC OA KC AC
suy ra
1
3
KC KA
.
Đim C nằm trên tia đối ca tia KA và cách K mt khong
1
3
AK
Điểm C xác định được thì các điểm
B
D
cũng xác định được .
b)Cách dng:
- Dựng đoạn thng
MN
.
- Dựng trung điểm
K
ca
MN
.
- 28 -
- Dng tia
AK
.
- Trên tia đối ca tia
KA
dựng điểm C sao cho
1
3
KC KA
.
- Dựng điểm B sao cho
M
là trung điểm ca
CB
.
- Dựng điểm
D
sao cho
N
là trung điểm ca
CD
.
- Dựng các đoạn thng
,AB AD
ta được hình bình hành phi dng.
Bạn đọc gii tiếp các bước còn li.
4.14. (h.4.19)
Gi s đã xác định được v trí ca
C
Dd
để tng
AC CD DB
nh
nht.V hình bình hành
'
CDBB
(chú ý
CD
'
BB
ngược chiu nhau).
Khi đó
'
BB CD a
(không đổi);
'
DB CB
. Điểm
'
B
c định.
Ta có tng
AC CD DB
nho nht
AC DB
nh nht (vì
không đổi
'
AC CB
nh nht
'
,,A C B
thng hàng.
T đó ta xác định được điểm
Cd
như sau:
-Qua B v một đường thng song song với d, trên đó lấy điểm
'
B
sao cho
'
BB a
(
'
BB
ngược chiu vi
CD).
-Lấy giao điểm
C
ca
'
BA
và d.
-Ly
Dd
sao cho
(
CD
'
BB
ngược chiu).
Khi đó tổng
AC CD DB
nh nht.
Phn chng minh dành cho bạn đọc.
4.15. (h.4.20)
Gi s đã xác định được v trí
CD
ca cu (
'
;C d D d
)sao cho tng
AC CD DB
nh nht
- 29 -
CHUYÊN ĐỀ 5. HÌNH CHỮ NHẬT
5. 1. ( h.5.10)
T giác
AEMF
có ba góc vuông nên là hình ch
nht
=>AE = MF.
Tam giác FMC vuông ti F.
0
45C
nên là tam giác
vuông cân => CF = MF. Do đó AE = CF.
Tam giác ABC vuông cân, AD là, đường cao nên
đồng thời là đường trung tuyến , đường phân giác
nên
0
1
; 45
2
AD DC BC EAD FCD
..E
DA FDC c g c DE DF va EDA FDC
Ta có
00
90 90EDA FDC ADF EDA
Hay
0
90EDF
Do đó
DEF
vuông cân
00
45 ; 90E F EDF
5. 2. ( h.5.11)
Gọi O là giao điểm ca AC và BD,
ta có OA = OC.
1
2
AD AC
nên AD = AO
V
;AH OD OK AB
V hình bình hành ABCA’
Ta có
' , 'AC A D AA CD a
'AA d
Khi đó A’ là điểm c định .
Ta có tng AC + CD + DB nh nht nh nht ( Vì CD
= a không đổi)
'DA DB
nh nht A’ , D, B thng hàng.
T đó ta xác định v trí ca CD ca cẩu như sau:
+) V
AH d
+) Trên tia AH lấy A’ sao cho AA’ = a
+) Lấy giao điểm D của A’B và d’
+) V
D
C d C d
Khi đó AC + CD + DB nhỏ nht .
Phn chng minh rành cho bạn đọc.
- 30 -
Xét
AOD
cân ti
A
,
AH
là đường cao
AH
cũng là đường trung tuyến, cũng là đường phân giác.
Do đó
HO HD
.
1
2
BAC DAC
nên
3 2 1
A A A
.
AOK AOH
(cnh huyn, góc nhn)
1
2
OK OH OD
1
1
30
2
OK OB B
.
Xét
ABH
vuông ti
H
1
B
30
nên
60HAB 
suy ra
90DAB 
.
Hình bình hành
ABCD
có mt góc vuông nên là hình ch nht.
5.3. (h.5.12)
h.5.12
ABCD
là hình ch nht nên
AC BD
22
8 6 10
.
Ta đặt
,
MC y
.
Xét ba điểm
M
,
A
,
C
ta có:
MA MC AC
.
Do đó
10xy
2
100xy
hay
22
2 100x y xy
.
1
Mt khác,
2
0xy
hay
22
20x y xy
,
2
T
1
2
suy ra
22
2 100xy
22
50xy
.
Dấu “
” xảy ra khi và ch khi
M
nm gia
A
C
MA MC
M
là trung điểm ca
AC
.
Chứng minh tương tự, ta được
22
50MB MD
, dấu “
” xảy ra
M
là trung điểm ca
BD
.
Vy
2 2 2 2
100MA MC MB MD
.
Do đó giá trị nh nht ca tng
S
100
khi
M
là giao điểm của hai đường chéo
AC
BD
.
5.4. (h.5.13)
x
y
D
A
B
C
M
- 31 -
nh 5.14
Q
P
D
C
N
M
B
A
h.5.13
V
AH BC
,
OK AH
. T giác
ADOF
KOEH
là hình ch nht nên
OF AD
OE KH
. Xét
AOD
vuông ti
D
, ta có
2 2 2 2
OD AD OA AK
.
Do đó
2 2 2 2 2 2 2 2
OD OF OE OD AD OE AK KH
2
2
22
AK KH
AH

(không đổi).
Dấu “
” xảy ra khi và ch khi
O
nm gia
A
H
AK KH
khi và ch khi
O
là trung điểm ca
AH
.
Vy giá tr nh nht ca tng
S
2
2
AH
khi
O
là trung điểm ca
AH
.
5.5.( H. 5 .14 )
T giác
ABCD
là hình ch nht nên
0
A B C D 90
Áp dụng định lý Py-ta-go, ta có:
2 2 2 2 2 2
MN BM BN ; NP CN CP ;
2 2 2 2 2 2
PQ DP DQ ; QM AQ AM .
Do đó:
2 2 2 2
S MN NP PQ QM
2 2 2 2 2 2 2 2
BM BN CN CP DP DQ AQ AM
Vn dng bất đẳng thc
2
22
ab
ab
2

(du
""
xy ra khi
ab
),
ta được :
2 2 2 2
AM BM BN CN CP DP DQ AQ
S
2 2 2 2
22
2 2 2 2
22
2 AB BC
AB BC CD AD
AC d .
2 2 2 2 2
Vy giá tr nh nht ca tng
S
2
d
khi
M,N,P,Q
lần lượt
là trung
điểm ca các cnh hình ch nht.
E
F
D
H
A
B
C
O
nh 5.15
B
H
K
C
F
E
A
D
1
1
1
- 32 -
nh 5.16
B
A
D
H
C
E
M
O
5.6. ( h.5.15)
V
DH BC,EK BC
DF EK
T giác
có 3 góc vuông nên là hình ch nht.
Suy ra
DF HK
HBD
vuông ti K có
0
B 60
nên
0
1
1
D 30 BH BD.
2
KCE
vuông ti
K
co
0
C 60
nên
0
1
11
E 30 CK CE AD
22
Ta có :
1 1 1 a
DE DF HK BC BD AD BC AB .
2 2 2 2



Vy giá tr nh nht ca
DE
a
2
khi
D
E
lần lượt là trung điểm ca
AB
AC
5.7. ( h.5.16)
T giác
ADME
có ba góc vuông nên là hình ch nht nên
AM DE.
Gi
O
là giao điểm ca
AM
DE
, ta có :
OA OM OD OE.
Xét
AHM
vuông ti H, ta có :
1
HO AM
2
1
2
HO DE
Xét
HDE
HO
là đường trung tuyến ứng với cạnh
DE
1
2
HO DE
nên
HDE
vuông tại
H
90DHE
.
5.8. (h.5.17)
a) Tứ giác
AFHE
có ba góc vuông nên là hình chữ
nhật
OA OF OH OE
.
Xét
ABC
vuông tại
A
AD
đường trung tuyến nên
AD DB DC
DAC
cân
1
AC
Mặt khác
2
CA
(cùng phụ với
B
);
(hai góc ờ đáy của tam giác cân)
suy ra
11
AE
Gọi
K
là giao điểm của
AD
EF
.
Xét
AEF
vuông tại
A
1 1 1 1
90 90 90E F A F K
.
Do đó:
AD EF
. (1)
Ta có:
OEM OHM
(c.c.c)
90 .OEM OHM EM EF
(2)
Chứng minh tương tự, Ta được:
FN EF
. (3)
Hình 5.17
- 33 -
Hình 5.19
G
M
N
P
F
E
H
D
C
B
A
Từ (1), (2), (3) suy ra:
// //EM FN AD
(vì cùng vuông góc với
EF
).
b) Ba đường thẳng
EM
,
FN
AD
là ba dường thẳng song song cách đều
KF KE K O AD AH ABC
vuông cân.
5.9. (h.5.18)
Vẽ
DE BC
,
DF AH
.
HAB
FDA
có:
90HF
;
AB AD
;
HAB FDA
(cùng phụ với
FAD
)
Do đó
HAB FDA
(cạnh huyền - góc nhọn)
AH FD
. (1)
Tứ giác
FDEH
có ba góc vuông nên là hình chữ nhật
HE FD
. (2)
Từ (1) và (2) suy ra:
.AH HE
Ta có
1
2
AM EM BD
.
AHM EHM
(c.c.c)
AHM EHM
.
Do đó tia
HM
là tia phân giác của
AHC
.
5.10. ( h.5.19)
Gi M, N, P lần lượt là trung điểm ca HE, HF, FG.
Theo tính chất đường trung bình ca tam giác, tính chất đường
trung tuyến ng vi cnh huyn ca tam giác vuông, t có :
EF = 2MN; FG = 2CP; GH = 2NP; HE = 2AM.
Do đó chu vi của t giác EFGH là :
E
F FG GH HE 2 AM MN NP PC
.
Xét các điểm A, M, N, P, C ta có :
AM MN NP PC AC
(Không đổi).
2 2 2 2 2
AC AB BC 15 8 289 AC 17
.
Vy chu vi t giác
EFGH 2.17 34
(Du
""
xy ra
M,N,P
nm trên AC theo th t đó
EF/ /AC/ /FG
HE/ /BD/ /FG
).
Do đó giá trị nh nht ca chu vi t giác EFGH là 34.
5.11. (h.5.20).
Hình 5.18
- 34 -
Gọi M là trung điểm ca BC.
V
AH Oy
,
CE Oy
MD Oy
.
Xét
AOH
vuông ti H, có:
0
O 30
nên
1
AH OA 1cm
2

.
MDB AHB MD AH 1cm
.
Xét
BCE
, d thấy MD là đường trung bình nên :
CE 2MD 2cm
.
Đim C cách Oy mt khoảng là 2cm nên C di động trên đường thng
a / /Oy
và cách Oy là 2cm.
5.12. (h.5.21)
Gọi M là trung điểm ca OB.
Khi đó
G AM
và AG = 2GM.
Gọi N là trung điểm của AG, ta được
AN NG GM
.
V AD, NE, GF cùng vuông góc vi Oy.
Ba đường thẳng AD, NE và GF là ba đường thẳng song song cách đều nên
DE EF = FM
.
Hình 5.20
a
C
y
x
E
D
M
B
H
A
O
Hình 5.21
x
y
a
G
N
A
B
M
F
E
D
O
- 35 -
Ta đặt FG = x thì EN = 2x và
FG AD
EN
2
. Do đó
x AD
2x AD 3x
2
.
Xét
DOA
vuông cân ti D
22
OA 2DA
.
Do đó
2
2
2DA 3 2 DA 3 cm FG 1cm
.
Đim G cách Oy mt khoảng không đổi là 1cm nên điểm G di động trên đường thng
a / /Oy
và cách Oy là
1cm.
5.13. (h.5.22)
V
N
D // AB D BC
Ta có:
1
DB
(cặp góc đồng v)
BC
nên
1
DC
NDC
cân. Do đó
ND NC
.
Mt khác,
AM NC
nên
ND AM
.
Suy ra t giác
ANDM
hình bình hành, trung đim
O
ca
MN
cũng là trung điểm
O
ca
AD
.
Ta điểm
A
BC
c định, theo d 5, thì điểm O di
động trên đường thng
//a BC
cách
BC
mt khong
2
AH
(AH là đường cao ca
ABC
).
5.14. (h.5.23)
Chia hình ch nhật kích thưc 3x6 thành 9 hình ch nht nh
kích thước 1x2. 10 điểm nm trong 9 phn nên tn ti hai
điểm chng hn A và B thuc cùng mt phn.
D thy
AB
độ dài đường chéo ca mi hình ch nht nh, tc
là:
22
1 2 5 2,3AB
.
5.15. (h.5.24)
a
K
Hình 5.22
1
O
N
D
B
C
A
M
Hình 5.23
A
B
- 36 -
Chia hình ch nhật kích thước 3x6 thành 7 phần như hình
5.24. 8 điểm nm trong 7 phn nên tn tại hai điểm chng
hn A B cùng thuc mt phn. D thy
22
1 2 5 2,3AB
.
Hình 5.24
B
A
- 37 -
CHUYÊN ĐỀ 6. HÌNH THOI VÀ HÌNH VUÔNG
6.1. (h.6.9)
Gi s ABCD là hình thoi,
. Hai đường chéo ct nhau ti
O
. V
;OH AD BK AD
thì
OH // BK
OH
là đường
trung bình ca tam giác
BKD
1
1
2
OH BK
Xét
ABK
vuông ti
K
,
1
30 2
2
o
A BK AB
T (1) và (2) suy ra:
1
4
OH AB
, do đó
4 4.AB OH h
.
6.2 (h.6.9)
Gọi O là giao điểm ca hai đường chéo.
Ta đặt
;OA x OB y
thì
2 ;BD 2y.AC x
Ta có
8:4 2AB cm
22
4xy
.
T bất đẳng thc
22
2x y xy
suy ra
22
4
2
22
xy
xy
.
Do đó
. 2 .2 4 8AC BD x y xy
.
Vy giá tr ln nht ca tích
.AC BD
khi
xy
AC BD ABCD
là hình vuông.
6.3 (h.6.10)
Gi
N
là trung điểm ca
CD
.
Ta
AM // CN
AM CN
nên t giác
hình bình
hành.
// .AN CM
Mt khác,
DH CM
nên
DH AN
ti K.
Xét
HCD
//KN CH
NC ND
nên
KH KD
.
ADH
AK
vừa đường cao vừa đường trung tuyến nên
ADH
cân.
AH AD
.
Mt khác,
AB AD
nên
AH AB ABH
cân.
Suy ra
ADH AHD
ABH AHB
.
Xét t giác
ABHD
360
o
ADH DHA BHA ABH A
.
2 360 40
oo
DHA BHA
2 320 160
oo
BHD BHD
Mt khác,
90
o
DHM
nên
160 90 70
o o o
MHB
6.4 (h.6.11)
Ta có
AC DB
// DB EF
nên
AC EF
. (1).
V điểm
M
sao cho
D
là trung điểm ca
EM
.
Xét tam giác
CEM
CD
là đường trung tuyến mà
1
2
CD EM
nên tam giác
CEM
vuông ti
C
CM CE
.
T giác
MDFB
có hai cạnh đối song song và bng nhau nên là hình bình hành.
Suy ra
DB
MF
ct nhau tại trung điểm ca mỗi đường.
Hình 6.8
K
H
O
B
D
C
A
Hình 6.9
y
x
O
A
C
D
B
Hình 6.10
K
M
H
B
D
C
A
N
- 38 -
Mt khác
O
là trung điểm ca
BD
nên
O
là trung điểm
MF
.
T giác
OA OC
,
OM OF
nên là hình bình hành.
Suy ra,
// AFCM
CE AF
. (2).
Xét tam giác
AEF
AC
CE
là hai đường cao ct nhau ti
C
nên
C
là trc tâm.
Nhn xét: Nếu v hình bình hành
DBFE
v phía có điểm
A
thì kết lun ca bài toán vn
đúng.
6.5 (h.6.12)
Ta có
OE OH
OG OH
(hai tia phân giác ca hai góc k bù)
Suy ra,
,,E O G
thng hàng.
Chứng minh tương tự ta
,,H O F
thng hàng.
Ta có:
//AB CD BAC ACD
.
EAO ACG
(mt na ca hai góc bng nhau).
A
OE COG g c g OE OG
Chứng minh tương tự ta được
OF OH
.
T giác
EFGH
hai đường chéo ct nhau tại trung điểm ca ca mỗi đường nên hình bình hành. Hình bình
hành có hai đường chéo vuông góc nên là hình thoi.
6.6 (h.6.13)
Gi s đã dựng được hình thoi
ABCD
thỏa mãn đề bài.
Gi
O
là giao điểm của hai đường chéo.
Ta có
AC BD
;OA OC OB OD
.
Do đó
8
:2 4OA OB cm
.
Trên tia
OD
lấy điểm
E
sao cho
OE OA
.
Khi đó
4BE cm
AOE
vuông cân.
Suy ra
45
o
AEB
.
T đó
AEB
dựng được ngay (g-c-g).
Đim
O
thỏa mãn hai điều kin:
O
nm trên
BE
O
nằm trên đường trung trc ca
AE
.
Hình 6.11
M
F
O
B
C
A
D
E
Hình 6.12
H
G
O
C
A
B
D
E
F
Hình 6.13
E
B
C
O
D
A
- 39 -
Đim
C
thỏa mãn hai điều kin:
C
nm trên tia
AO
sao cho
OC OA
.
Đim
D
thỏa mãn hai điều kin:
D
nm trên tia BO sao cho
OB OD
.
Các bước còn li bạn đọc t gii.
6.7 (h.6.14)
Các t giác
, , ABEG AEFG AECG
là hình bình hành nên:
// ; // ; // AB EG AE GF AF CG
Suy ra
1 1 2 2 3 3
; ; ;E A F A C A
Do đó
2 3 1 2 3
1 45
o
E F C A A A BAC
6.8. (h.6.15)
* Tìm cách gii
Mun chng minh
, AF CE và BM
đồng quy, ta chng minh chúng là
các đường thng chứa các đường cao ca
BEF
.
* Trình bày li gii
T giác
MEDF
có ba góc vuông nên là hình ch nht.
;ME DF MF DE
.
ADC
vuông cân
45
o
CAD ACD
Do đó
AEM
CFM
vuông cân.
AE ME
AE DF
.
CF MF DE CF
.
11
.
. 90
o
ABE DAF g c g B A H
(
H
là giao điểm ca
AE và CF
).
Chứng minh tương tự, ta được
CE BF
.
Gi
N
là giao điểm ca
;EM BC
;
K
là giao điểm ca
;.BM EF
Ta có
MF MN
(vì
M
nm trên tia phân giác ca góc
C
).
M
E BN AE
.
..M
FE NMB g c g MFE NMB
.
Ta có:
90
o
NMB FMK
(vì
90
o
NMF
)
90 90
oo
MFE FMK K BM EF
Vậy ba đường thng
, AF CE và BM
là ba đường cao ca
BEF
nên chúng đồng quy.
6.9 (h.6.16)
a) Gi
K
là giao điểm của hai đường thng
.DE và FG
T giác
AGKE
có ba góc vuông nên là hình ch nht.
Gọi O là giao điểm ca
.AH và EG
11
. .AEG ABC c g c G C
Ta li có:
11
CA
(cùng ph vi
ABC
); Và
.
Hình 6.14
3
2
1
3
2
1
E
F
B
C
A
D
G
Hình 6.15
2
1
1
K
H
F
N
B
C
D
A
E
M
- 40 -
11
GA
. Do đó
OAG
cân
OG OA
.
Chứng minh tương tự, ta được
OE OA
OG OE
.
Xét hình ch nht
AGKE
O
là trung điểm của đường chéo
EG
nên đường chéo
AK
phải đi qua
O
hay
đường thng
AH
đi qua
K
.
Vậy ba đường thng
AH
,
DE
,
FG
đồng quy.
b)
BCF
KAC
BC KA
(cùng bng
EG
);
BCF KAC
(vì
12
90 90 CA
;
CF AC
).
Do đó
BCF KAC
2
2
FC
.
Gi
M
là giao điểm ca
BF
KC
.
Ta có
23
90 CC
2
3
90 FC
90 M
. Vy
.BF FC
Chứng minh tương tự, ta được
CD KB
.
Xét
KBC
có các đường thng
, , AH BF CD
chứa ba đường cao nên chúng đồng quy.
6.10
- 41 -
( ) ADE CDF c g c
DE DF
ADE CDF
.
Ta có
90ADE CDE
90CDF CDE
hay
90EDF 
T giác
có hai đường chéo ct nhau tại trung điểm mỗi đường nên là hình binh hành. Hình bình
hành này có hai cnh k bng nhau nên là hình thoi.
Hình thoi này có
90EDF 
nên là hình vuông.
6.11. (h.6.18)
FAC
vuông ti
F
,
45A 
nên là tam giác vuông cân
AF FC
.
AFH
CFB
90 AFH CFB
;
AF FC
;
FAH FCB
(hai góc có cạnh tương ứng vuông góc).
Do đó
AFH CFB
(g.c.g)
AH BC
.
Vn dụng định lí đường trung bình ca tam giác ta chứng minh được
MNPQ
là hình bình hành.
Ta có
11
;
22
MQ AH MN BC
AH BC
nên
MQ MN
.
- 42 -
Hình bình hành
MNPQ
có hai cnh k bng nhau nên là hình thoi.
Bạn đọc t chng minh
90M 
suy ra
MNPQ
là hình vuông.
6.12 (h.6.19).
Ta đặt
B
90

.
Khi đó
90EBF GCF
.
EFB GFC
(c.g.c)
EF GF
EFB GFC
.
Ta có
90CFE EFB
90CFE GFC
hay
90EFG 
.
Chứng minh tương tự, ta được
FG GH HE
.
T giác
EFGH
có bn cnh bng nhau nên là hình thoi.
Hình thoi này có
90EFG 
nên là hình vuông, suy ra
EG HF
EG HF
.
6.13 (h.6.20).
- 43 -
a) Phân tích
Gi s dựng được hình vuông
ABCD
tho mãn đề bài.
Gi
N
là trung điểm ca
AM
. V
NH AD
.
Qua
M
v một đường thng vuông góc vi
AM
cắt đường thng
AD
ti
E
.
Xét
//NH MD
AN NM
nên
1
2
AH HD AD
.
Mt khác,
1
2
MD MC CD
nên
MD AH
.
Ta có
DME HAN
(cùng ph vi
DMA
).
1
(g.c.g)
2
DME HAN ME AN AM
.
Vậy E xác định được, t đó xác định được
, , D C B
.
b) Cách dng
Dựng đường thng
d AM
ti
M
;
Trên
d
lấy điểm
E
sao cho
1
2
ME AM
.
Dng
MD AE
Dựng điểm
C
sao cho
M
là trung điểm ca
CD
;
Dng
// ADCx
// Ay CD
chúng ct nhau ti
B
.
T giác
ABCD
là hình vuông phi dng.
c) Chng minh
Tht vy, t giác ABCD có các cặp đối song song nên là hình bình hành.
Hình bình hành này có
90D
nên là hình ch nht.
Gọi N là trung điểm ca
AM
. V
NH AD
thì
1
2
AH AD
.
HAN DME
(cnh huyn, góc nhn)
AH DM AD DC
.
Hình ch nht có hai cnh k bng nhau nên là hình vuông.
d) Bin lun
Có hai cách lấy điểm
E
trên đường thng
d
(v hai phía của điểm
M
) nên bài toán có hai nghim hình là
các hình vuông
ABCD
ABC D
.
6.14. (h.6.21).
- 44 -
Hình 6.21
Tô màu bàn c như hình 6.21. Lúc này trên bàn cờ
20
ô đen và 16 o trắng.
Mi mnh g
14
khi đặt lên bàn c che lấp 2 ô đen và 2 ô trắng.
Do đó 9 mảnh g
14
ch che lấp được 18 ô đen.
Như vậy vi mọi cách đặt 9 mnh g lên bàn c bao gi cũng còn thừa hai ô đen không được che lp.
Vy kông th dùng 9 mnh g
1x4
để lp kín bàn c.
6.15. (h.622)
- 45 -
CHUYÊN ĐỀ 7. ĐỐI XỨNG TRỤC. ĐỐI XỨNG TÂM
7.1. (h.7.9)
a)
C
đối xng vi
A
qua
BD
nên
ABD
đối xng vi
CBD
qua
BD
.
Do đó
ABD CBD
, suy ra
12
12
; ; B B D D BA BC
DA DC
.
Ta có
BD
BE
là các tia phân giác trong và ngoài tại đỉnh
B
nên
BD BE
.
Chứng minh tương tự, ta được
BD DH
.
Suy ra
//EF HG
T giác
EFGH
là hình thang.
Ta có
34
DD
(cùng ph vi hai góc bng nhau).
11
AC
(mt na ca hai góc bng nhau). Suy ra
HG
.
Hình thang
EFGH
có hai góc k một đáy bằng nhau nên là hình thang cân.
b)
ADH CDG
(g.c.g)
DH DG
.
Chứng minh tương tự, ta được
BE BF
.
Đưng thng
BD
đi qua trung điểm hai đáy của hình thang cân nên là trục đối xng ca hình thang cân
EFGH
.
7.2.
(h.7.1
0)
- 46 -
a) Các đoạn thng
AM
AN
đối xng vi
AD
lần lượt qua
AB
AC
nên
1 2 3 4
; AN=AD; ; AM AD A A A A
.
Ta có
23
22M
AN MAD NAD A A BAC
(không đổi).
b) Xét
AMN
AM AN
(cùng bng
AD
) nên là tam giác cân. Tam giác cân này có góc
MAN
không
đổi nên cạnh đáy
MN
ngn nht
cnh bên
AM
ngn nht
AD
ngn nht (vì
AM AD
)
AD BC D
là hình chiếu ca
A
trên
BC
.
7.3 (h.7.11)
V điểm
M
đối xng vi
D
qua
AB
và v điểm
N
đối xng vi
D
qua
AC
. Khi đó
;MF DF EN ED
.
Chu vi
DEF DF FE ED MF FE EN
.
Chu vi
DEF
nh nhất khi độ dài đường gp khúc
MFEN
ngn nht. Mun vy bốn điểm
, , ,M F E N
phi thng hàng theo th t đó.
Do đó ta phải tìm điểm
D
trên
BC
sao cho
MN
nh nht.
Theo kết qu bài 7.2, để
MN
nh nht thì
D
là hình chiếu ca
A
trên
BC
. Khi đó
E
F
lần lượt là giao
điểm ca
MN
vi
AC
AB
(h.7.12).
Ta chng minh với cách xác định
, , D E F
như vậy thì chu vi
DEF
bng
MN
MN
nh nht. (1)
- 47 -
Hình (h.7.12)
Khi D, E, F nhng v trí khác nhau thì chu vi
DEF
bằng độ dài đường gp khúc MFEN do đó lớn hơn
MN. (2).
T (1) và (2) suy ra điều phi chng minh.
Chú ý: Ta có nhận xét điểm E là chân đường cao v t đỉnh B, điểm F là chân đường cao v t đỉnh C ca
ABC
Tht vy. xét
DEF
có các đường BFCE l ợt là các đường phân giác ngoài tại đỉnh F và E. Hai
đưởng thng này ct nhau ti A nên tia DA là tia phân giác ca
EDF
.
Ta có: DC
DA nên DC là tia phân giác ngoài tại đỉnh D ca
DFE
Mt khác, EC là đường phân giác ngoài tại đỉnh E.
Đim C là giao điểm của hai đưởng phân giác ngoài nên FC là đường phân giác trong. Kết hp vi FB
đường phân giác, suy ra FC
FB hay CF
AB
Chứng minh tương tự ta được BE
AC
Như vậy ba điểm D, E, F có th được xác định bi chân của ba đường cao ca tam giác
7.4 ( h 7.13)
Hình (h.7.13)
Gi s đã dựng được hai điểm C và D thuc xy sao cho CD=a và chu vi t giác ABCD nh nht.
V hình bình hành BMDC ( điểm M phía gn A).
Khi đó BM=CD=a và DM=BC.
D
E
F
A
B
C
M
N
y
C
B
M
D
a
A
- 48 -
V điểm N đối xng với điểm M qua xy, điểm N là một điểm c định và DN=DM.
Ta có AB+ BC+CD+DA nh nht
BC + DA nh nhất ( Vì AB và CD không đổi)
DM + DA nh nht
DN + DA nh nht
D nm gia A và N.
T đó ta xác định điểm D như sau:
- Qua B v đường thng song song với xy và trên đó lấy điểm M sao cho BM=a (điểm M phía gn
A).
- V điểm N đối xng vi M qua xy.
- Lấy giao điểm D ca AN vi xy.
- Lấy điểm C thuc xy sao cho DC=MB=a (DC và MB cùng chiu ).
- Khi đó tổng AB+BC+CD+DA nh nht
Phn chng minh dành cho bạn đọc.
7.5. (h .7.14)
Hình (h.7.14)
a) AN đối xng vi AM qua AB
AN=AM và
(1)NAB MAB
AP đối xng vi AM qua A
AP=AM và
MAC PAC
(2)
AA’ đối xng vi AM qua AD nên
'MAD A AD
Mt khác,
BAD CAD
nên
'MAB CAA
(3)
T (1) và (3) suy ra
'NAB MAB CAA
Ta có :
' 'ACA AP A PAC MAB MAC BAC
Chứng minh tương tự, ta được :
'A AN BAC
.
Suy ra :
''A AP A AN
ANP
cân tại A có AA’ là đường phân giác nên AA’ cũng là đường trung trc ca NP, suy ra N và P đối
xứng qua AA’.
A'
D
N
P
Q
A
B
C
M
- 49 -
b) Gọi Q là điểm đối xng ca M qua BC.
Chứng minh tương tự như trên ta được BB’ là đường trung trc của NQ và CC’ là đường trung trc ca PQ.
Vậy AA’, BB’ và CC’ là ba đường trung trc ca
ANP
nên chùng đồng quy.
7.6
Trước hết ta chng minh bài toán ph:
Cho tam giác ABC, điểm M trong tam giác ( hoc trên cạnh nhưng không trùng với đỉnh ca tam giác).
Chng minh rng MB+MC < AB +AC (h7.15).
Hình (h.7.15)
Tht vy, xét
ANP
, ta có BD<AB+AD hay MB+MD<AB+ AD (1)
Xét
MCD
có MC< DC+MD (2)
Cng tng vế của (1) và (2) ta được:
MB+MD+MC < AB+AD+DC+MD
MB+MC < AB+AC
Bất đẳng thc trên vẫn đúng nếu điểm M nm trên mt cạnh nhưng không trùng với đỉnh ca tam giác.
Bây gi ta vn dng kết qu trên để gii quyết bài toán đã cho.
V điểm E đối xng với D qua đường thng AB ( h.7h.16).
Hình (h.7.16)
Khi đó AE=AD, ME=MD và BE=BD
Vì điểm M nm gia A và B nên hoặc điểm M nm trong
BEC
hoặc điểm M nm trong
AEC
hoặc điểm
M nm trên cnh EC.
A
B
C
M
D
E
A
D
C
B
M
- 50 -
Ta có:
ME MC AE AC
ME MC BE BC
hay
MD MC AD AC
MD MC BD BC
Do đó MD + MC
m
ax ,AD AC BD BC
.
7.7. (h.7.17)
Hình (h.7.17)
Ta có AC’ và BO đối xng nhau qua F nên AC=BO và AC//BO (1)
BO và CA’ đối xứng nhau qua D nên BO=Ca’ và BO//CA’ (2)
T (1) và (2) suy ra : AC’= CA” và AC’//CA’.
Do đó tứ giác ACA’C’ là hình bình hành.
Chứng minh tương tự ta được t giác ABA’B’ là hình bình hành.
Hai hình bình hành ACA’C’ và ABA’B’ có chung đường chéo AA’ nên các đường chéo AA’, BB’ và CC’
đồng quy.
7.8 ( h.7.18)
Hình (h.7.18)
a) Phân tích
Gi s đã dựng được điểm A thuc Ox và B thuc Oy sao cho G là trng tâm ca
AOB
.
Tia OG ct AB tại trung điểm M ca AB và
3
2
OM OG
V điểm N đối xng với O qua điểm M. T giác ANBO là hình bình hành
E
A'
D
C'
B'
F
A
B
C
O
x
y
N
M
O
A
B
G
- 51 -
NA//Oy; NB//Ox, t đó xác định được A và B.
b) Cách dng
- Trên tia OG lấy điểm M sao cho
3
2
OM OG
Dựng điểm N đối xng với điểm O qua M.
T N dng mt tia song song vi Oy ct Ox ti A.
Khi đó G là trọng tâm ca tam giác AOB.
c) Chng minh
T giác ANBO là hình bình hành, suy ra AB và ON ct nhau tại trung điểm ca mỗi đường.
Mặt khác, M là trung điểm của ON nên M là trung điểm ca AB.
Vậy OM là đường trung tuyến ca tam giác AOB.
Ta có
3
2
OM OG
nên G là trng tâm ca tam giác AOB.
d) Bin lun: Bài toán luôn có mt nghim hình.
7.9. (h.7.19)
Hình (h.7.19)
V đường trung tuyến AM của tam giác ABC và đường trung tuyến DN ca tam giác DEF. Gi G là giao
điểm của hai đường trúng tuyến này. Gi H và K lần lượt là trung điểm ca GA và GD.
Xét
FCE
có AN là đường trung bình
1
/ / , .
2
AN CE AN CE
Do đó AN//BM và AN=BM, suy ra ANMB là hình bình hành
1
/ / , .
2
MN AB MN AD
K
N
E
F
D
H
G
M
A
B
C
- 52 -
Mặt khác , HK là đường trung bình ca
GAD
nên HK//AD và
1
.
2
HK AD
T đó MN//HK và MN=HK
Suy ra MNHK là hình bình hành, hai đường chéo HM và NK ct nhau tại G nên G là trung điểm mỗi đường.
Do đó GM=GH=HA
G là trng tâm ca
ABC
GN=GK=KD
G là trng tâm ca
DFE
Vy
ABC
DFE
có cùng trng tâm.
7.10 (h.7.20)
Hình (h.7.20)
a) Phân tích
Gi s đã dựng được hình bình hành ABCD thỏa mãn đề bài.
Gọi O là giao điểm của hai đường chéo. Ta có M và P đối xng qua O.
Gọi Q là giao điểm của NO và AD thì Q và N đối xng qua O.
V các đường trung trc ca
MN
NP
chúng ct nhau ti
O
.
Gi
Q
là điểm đối xng ca
O
qua
N
. T giác
AOBQ
là hình bình hành.
Đim
A
tho mãn hai điều kin:
A nm tn đường trung trc ca
MN
và
QA
song song với đưng trung trc ca
NP
.
Đim B tho mãn hai điều kin: Hình 7.21
B nằm trên đường trung trc ca
NP
QB
song song với đưng trung trc ca
MN
.
Khi đó hai điểm
,CD
còn lại được xác định d dàng.
b) Cách dng
- Dựng các đường trung trc
1
d
ca
MN
2
d
ca
NP
, chúng ct nhau ti
O
;
- Dựng điểm
Q
đối xng vi
O
qua
N
;
- Qua
Q
dng một đường thng song song vi
2
d
ct
1
d
ti
A
;
- Qua
Q
dng một đường thng song song vi
1
d
ct
2
d
ti
B
;
N
Q
P
M
O
B
A
D
C
- 53 -
- Dựng điểm
C
đối xng vi
B
qua
P
;
- Dựng điểm
D
đối xng vi
A
qua
M
.
Khi đó tứ giác
ABCD
là t giác phi dng.
Các bước còn li, bạn đọc t gii.
7.1. (h.7.22)
Hình vuông có 4 4 = 16 ô vuông, chia thành 8 cặp đối xng nhau qua tâm hình
vuông. Xét các cp hai s hai ô đối xứng qua tâm đó. Hình 7.22
Tng hai s ca mi cp nh nht là 1 + 1 = 2,
ln nht là 4 + 4 = 8.
Có 7 tng (là 2, 3, 4, 5, 6, 7, 8) mà có 8 cp s nên phi có hai cp có tng bng nhau. V trí ca 4 s trong
hai cặp này là đỉnh ca mt hình bình hành phải tìm (trường hợp đặc bit: 4 s này nm trong 4 ô có tâm
thng hàng, ta nói hình bình hành "suy biến" thành đoạn thng).
- 54 -
CHUYÊN ĐỀ 8. VẼ HÌNH PHỤ ĐỂ GIẢI TOÁN TRONG CHƢƠNG TỨ GIÁC
8.1. (h.8.7)
Xét hình thang
ABCD
(
//AB CD
).
Trường hp hai cnh bên song song:
Khi đó tứ giác
ABCD
là hình bình hành. Hình 8.7
Điu kin
AD BC
đề bài được tho mãn.
Trường hp hai cnh bên không song song:
V
//AE BC
(
E CD
) ta được
ABCE
là hình bình hành
AE BC
.
Mt khác,
AD BC
nên
1
AE AD D E
(1)
Ta li có
1
//AE BC C E
(2)
T (1) và (2) suy ra
DC
, do đó hình thang
ABCD
là hình thang cân.
8.2. (h.8.8)
Xét hình thang
ABCD
//AB CD
AB CD
.
Ta phi chng minh
A B C D
.
V
//AM BC
(
M CD
) khi đó
1
CA
. Hình 8.8
Ta có
1
A A C
;
1
MD
(tính cht góc ngoài ca
)
BD
.
Do đó
A B C D
.
8.3. (h.8.9)
V
// , BE AC E CD
. Ta được
CE AB
BE AC
.
Ta có
AB CD CE CD DE
.
AB CD a
nên
.
Hình 8.9
Tam giác
BDE
vuông cân
22BE a AC a
.
8.4. (h.8.10)
Qua
B
v
//BE AC
(
E
đường thng
CD
) ta được
BE AC
CE AB
.
Do đó
2DE DC CE DC AB h
.
Ta có
BD AC
(hai đường chéo ca hình thang cân) Hình 8.10
BE AC
nên
BD BE
.
BDE
cân ti
B
,
BH
là đường cao nên cũng là đường trung tuyến, suy ra
; DH HE h BH h
.
- 55 -
Do đó các tam giác
HBD
,
HBE
vuông cân
1
45DE
.
Suy ra
BDE
vuông ti
B
90COD EBD
8.5. (h.8.11)
Trường hp hình thang có hai góc k một đáy cùng tù, hai góc kề
đáy kia cùng nhọn
V
AH CD
,
BK CD
thì
HK AB
.
Ta có
2 2 2 2
––AC HC AD DH
(
2
AH
);
2 2 2 2
BD KD BC KC
(
2
BK
). Hình 8.11
Cng tng vế hai đẳng thức trên ta được
2 2 2 2 2 2 2 2
AC HC BD KD AD BC DH CK
2 2 2 2 2 2 2 2
––AC BD AD BC CH CK DK DH
22
––AD BC CH CK CH CK DK DH DK DH
22
AD BC HK CH CK HK DK DH
22
AD BC HK CH CK DK DH
22
AD BC HK CD CD
22
2.AD BC ABCD
.
Trường hp mỗi đáy có một góc tù (hoc mt góc vuông), mt góc nhọn: Cũng chứng minh tương tự.
8.6. (h.8.12)
V hình bình hành
DAFH
.
Gi
N
là giao điểm của hai đường chéo
DF
, AH M
là giao
điểm ca
EH
BC
.
Ta có
, NA NH ND NF
.
Ta đặt
ADH AFH

thì
60BDH HFC
180DAF
;
360BAC BAD CAF DAF
3
60 60 60 180
60
.
BDH
HFC
có:
()BD HF AD
;
BDH HFC
(chng minh trên);
()DH FC AF
. Hình 8.12
Do đó
BDH HFC
(c.g.c)
HB HC
. (1)
Chứng minh tương tự ta được
BAC HFC
(c.g.c)
BC HC
. (2)
- 56 -
T (1) và (2) suy ra
HB HC BC
.
T giác
BHCE
có các cp cạnh đối bng nhau (cùng bng
BC
) nên là hình bình hành
MB MC
MH ME
.
Xét
AEH
AM
AN
là hai đường trung tuyến nên giao điểm
G
ca chúng là trng tâm
2
3
EG EN
2
3
AG AM
.
Xét
ABC
AM
là đường trung tuyến mà
2
3
AG AM
nên
G
là trng tâm ca
ABC
.
Xét EDF có EN là đường trung tuyến mà
2
3
EG EN
nên
G
là trng tâm ca
EDF
.
Vy
ABC
EDF
có cùng trng tâm
G
.
8.7. (h.8.13)
vuông ti
H
60ABC 
nên
30HMB 
.
CAK
vuông ti
C
60ACB 
nên
30KCM 
.
Suy ra
KMC KCM
(cùng bng
HMB
).
Do đó
KMC
cân
KC KM
.
V hình bình hành
BKMD
//BD KM
BD KM
.
Do đó
BD AB
(vì
KM AB
) và
BD KC
(vì cùng bng
KM
).
ABD ACK
(c.g.c)
12
AA
AD AK
. Hình 8.13
Tam giác
ADK
cân,
AN
là đường trung tuyến nên là đường cao, đường phân giác
AN DK
,
90AHK 
.
Ta có
21
60 60A BAK BAC A BAK
hay
60 60 :2 30DAK NAK
.
Do đó
90 30 60AKN
.
Xét
ANK
: : 30 :60 :90 1:2:3NAK NKA ANK
.
8.8. (h.8.14)
a) Phân tích
Gi s đã dựng được t giác
ABCD
tho mãn đề bài.
V hình bình hành
DABE
ta được
3,5cmBE AD
;
2,5cmDE AB
. Gi
O
là giao điểm của hai đường thng
AD
BC
.
Do
//BE AD
nên
40CBE O
.
Tam giác
BCE
dựng được (c.g.c). Tam giác
CDE
dựng được
(c.c.c)
Đim
A
thỏa mãn hai điều kin:
A'
D'
Hình 8.14
3,5
C
3,5
3
2,5
E
D
B
A
O
- 57 -
-
A
nằm trên đường thng qua
D
và song song vi
BE
.
-
A
nằm trên đường thng qua
B
và song song vi
DE
.
b). Cách dng
-Dng
CBE
sao cho
40B 
,
3cmBC
,
3,5cmBE
.
-Dng
CDE
sao cho
CE
đã biết
4,5cmCD
,
2,5cmED
.
-Qua
D
dng một đường thng song song vi
BE
. Qua
B
dng một đường thng song song vi
DE
chúng ct nhau ti
A
ABCD
.
T giác
ABCD
là t giác phi dng.
c).Chng minh
Theo cách dng,
ABED
là hình bình hành nên
2,5cmAB DE
;
3,5cmAD BE
;
40COD CBE
.
T giác
ABCD
2,5cmAB
;
3cmBC
;
4,5cmCD
;
3,5cmDA
;
40COD 
, tha mãn
đề bài.
d).Bin lun
Bài toán có hai nghim hình là t giác
ABCD
và t giác
A BCD

8.9. (h.8.15)
Gi
M
là trung điểm ca
CD
.
Xét
HCD
KM
là đường trung bình nên
//KM HD
do
đó
KM AC
(vì
HD AC
).
T giác
ADMB
//AB MD
AB DM
1
2
CD



nên
ABMD
là hình bình hành .
Hình bình hành này có
90A 
nên là hình ch nht. Suy ra
AM BD
OA OM OB OD
.Xét
KAM
vuông ti
K
KO
là trung tuyến nên
11
22
KO AM BD
.
Xét
KBD
KO
là trung tuyến nên
1
2
KO BD
nên
KBD
vuông ti
K
do đó
90BKD 
.
8.10. (h.8.16)
Gi
E
là trung điểm ca
OB
thì
ME
là đường trung
bình
ca
AOB
//ME AB
1
2
ME AB
.
Do đó
//ME NC
ME NC
.
Hình 8.15
O
H
K
M
D
C
B
A
Hình 8.16
F
E
O
N
M
D
C
B
A
- 58 -
T giác
là hình bình hành
//CE MN
CE MN
.
Ta có
ME BC
ti
F
(vì
AB BC
),
BO AC
(tính chất đường chéo hình vuông).
Xét
MBC
E
là trc tâm nên
CE MB
do đó
MN MB
1
MAB
EBC
AB BC
;
45MAB EBC
;
MA EB
(mt na của hai đoạn thng bng
nhau).
Vy
MAB EBC
(c.g.c)
MB EC
MB MN
2
T
1
2
suy ra
MNB
vuông cân.
8.11. (h.8.17)
Gi
E
là giao điểm của đường thng
DM
vi
AB
.
Tam giác
BDE
BM
vừa llaf đường phân giác va là
đường cao nên là tam giác cân, do đó
BD BE
MD ME
.
Gi
N
là trung điểm ca
BE
thì
MN
là đường trung bình
ca
// BDE BD MN
12
M B
do đó
1
1 2
M B B
NBM
cân
BN MN
.
T giác
là hình thang cân
BN CM
MN CM
Xét
MBE
vuông ti
M
MN
là đường trung tuyến nên
1
2
MN BE
2BE MN
2BD CM
.
8.12. (h.8.18)
Ta có
//CE DF
(cùng vuông góc vi
AB
).
T giác
FECD
là hình thang.
Gi
M
,
N
lần lượt là trung điểm ca
EF
CD
,
MN
là đường trung bình ca hình thang
CEFD
. Do đó
//MN CE
MN EF
.
Ta có
1
2
AN BN CD
(tính chất đường trung
tuyến ca tam giác vuông).
NAB
cân.
Mt khác
NM
là đường cao nên cũng là đường trung tuyến
MA MB
AF BE
.
Hình 8.17
D
E
N
M
2
1
C
B
A
Hình 8.18
F
E
M
N
D
C
B
A
- 59 -
8.13. (h.8.19)
V đường trung tuyến
AM
. Gi
N
là trung điểm
AG
Qua
M
N
v các đường thng
AA
ct
xy
ti
M
N
.
Xét hình thang
BB C C

21B
B CC MN


.
Xét hình thang
AA G G

22A
A GG NN

.
Xét hình thang
NN M M

2
2 4NN MM GG NN MM GG
.
T
1
2
suy ra
2A
A BB CC GG MM NN
4AA BB CC GG GG
.
Do đó
3AA BB CC GG
8.14. (h.8.20)
Trên tia đối ca tia
AB
lấy điểm
N
sao cho
AN AM
..A
CN ABD c g c
CN BD
ACN ABD
CAE ABD
(cùng ph
BAE
) nên
ACN CAE
//AE CN
//MF CN
(vì cùng song song vi
AE
).
Xét hình thang
// CNAE
AM AN
nên
EF EC
.
Suy ra
22
MF CN MF BD
AE


.
8.15. (h.8.21)
a). Gi
, , , , ,M N P Q E F
lần lượt là trung điểm ca
, , , ,AB BC CD DA AC
BD
. Theo định lý Giéc-gon (bài 4.8) thì ba đường thng
MP
,
NQ
,
EF
đồng qui tại điểm
O
là trung điểm ca mỗi đọan thẳng đó.
Gọi giao điểm ca
AO
vi
DN
G
v
//QH AG
.
Xét
NQH
ta được
NG GH
.
Xét
ADG
ta được
GH HD
.
Vy
1
1
3
NG GH HD HG D N
A
là trng tâm
BCD
nên
A DN
1
3
NA DN
2
.
Hình 8.19
C'
M'
G'
N'
A'
B'
y
x
M
G
N
M
C
B
A
Hình 8.20
F
F
M
N
C
A
B
Hình 8.21
A'
H
G
O
F
E
Q
P
N
M
D
C
B
A
- 60 -
T
1
2
suy ra
GA
do đó
AA
đi qua
O
.
Chứng minh tương tự các đường thng
BB
,
CC
,
DD
đều đi qua
O
.
Suy ra
AA
,
BB
,
CC
,
DD
đồng qui ti
O
.
b). Ta có
1
2
OA QH
1
2
QH AA
nên
1
4
OA AA

. Suy ra
1
3
OA OA
hay
1
3
OA
OA
.
Chứng minh tương tự ta được
1
3
OB OC OD
OB OC OD
.
8.16. (h.8.22)
Gi
,,E F M
lần lượt là trung điểm ca
,,OB OC BC
. Theo tính chất đường trung tuyến ng vi
cnh huyn ca tam giác vuông ta có
1
2
EH EB EO OB
;
1
2
FK FC FO OC
. Theo tính
chất đường trung bình ca tam giác ta có t giác
OFME
là hình bình hành
1O
EM OFM
.
Mt khác
2HEO ABO
;
2KFO ACO
ABO ACO
nên
2H
EO KFO
.
T
1
2
suy ra
HEM MFK
.
HEM
MFK
1
2
HE MF OB




;
HEM MFK
(chng minh trên);
E
M FK OC
.
Do đó
..H
EM MFK c g c
3M
H MK
Gi
N
là trung điểm ca
OA
, ta có
1
4
2
NH NK OA




T
3
4
suy ra
MN
là đường trung trc ca
HK
.
Vậy đường trung trc ca
HK
đi qua điểm c định
M
là trung điểm ca
BC
.
8.17. (h.8.23)
Hình 8.22
N
F
F
K
O
H
M
C
B
A
- 61 -
a). V điểm
M
đối xng vi
A
qua
Ox
. V điểm
N
đối xng
A
qua
Oy
.
Hai điểm M và N là hai điểm c định.
Đon thng MN ct Ox ti B, ct Oy ti C.
Khi đó chu vi
ABC
là nh nht.
Tht vậy, vì M đối xng vi A qua Ox nên
AM MB
. Vì N đối xng vi A qua Oy nên .
Chu vi
ABC
là:
AB BC CA MB BC CN MN
.
Do đó chu vi
ABC
nh nht bng MN.
b) V
MH AN
, ta có :
0
MAH O 60
(hai góc có cạnh tương ứng vuông góc cùng nhn).
0
AMH 30
.
Xét
vuông ti H,
0
AMH 30
nên
11
AH AM .4 2cm
22
.
Xét
vuông ti H, ta có:
2
2 2 2 2
MN MH HN MH HA AN
2 2 2
2 2 2
2 2 2 2
MH HA AN 2HA.AN
MH HA AN 2HA.AN
AM AN 2HA.AN 4 2 2.2.2 28
M
N 28 5,3 cm
.
Vậy độ dài nh nht ca chu vi
ABC
là 5,3cm.
8.18. (h.8.24)
Hình 8.23
O
60
°
y
x
H
C
B
N
M
A
- 62 -
a) Phân tích
Gi s đã dựng được
ABC
có đỉnh A ti v trí A cho trước có trng tâm G ti v trí G cho trước, có đỉnh
Bb
, đỉnh
Cc
với b, c cho trước.
Gi M là giao ca tia AG vi BC.
Ta có :
3
AM AG
2
nên M xác định được.
Gi D là một điểm trên b. V điểm E đối xng vi D qua M.
Khi đó đường thng
b'
đi qua C và E là đường thẳng đối xng vi b qua M.
Điểm C thỏa mãn hai điều kin :
Cc
C b'
.
Điểm
Bb
B
tia CM.
b) Cách dng :
- Dựng điểm M thuc tia AG sao cho :
3
AM AG
2
.
- Dựng đường thẳng
'b
đối xứng với
b
qua
M
,
'b
cắt
c
tại
C
- Dựng giao điểm
B
của đường thẳng
b
với tia
CM
;
- Vẽ các đoạn thẳng
AB
,
AC
ta được
ABC
phải dựng.
Các phần còn lại bạn đọc tự giải.
Hình 8.24
c
b
'
b
E
C
G
M
D
B
A
- 63 -
CHUYÊN ĐỀ 9. TOÁN QUỸ TÍCH
9.1. (h.9.5)
Xét trường hợp điểm
M
nằm trên nửa mặt phẳng bờ
a
không chứa
b
.
a) Phần thuận
Vẽ
MH a
, đường thẳng
MH
cắt
b
tại
K
.
Ta có:
4MH MK cm
;
2MK MH cm
Suy ra
(4 2):2 1MH cm
Điểm
M
nằm trên nửa mặt phẳng bờ
a
không chứa
b
cách
a
1cm
nên điểm
M
nằm trên đường
thẳng
d
song song với
a
và cách
a
1cm
b) Phần đảo
Lấy điểm
M
bất kì trên đường thẳng
d
. Vẽ
MH a
cắt đường thẳng
b
tại
K
.
Ta có:
MH lcm
;
2HK cm
3MK cm
.
Do đó
4MH MK cm
.
c) Kết luận
Vậy quỹ tích của điểm M là đường thẳng
//da
cách
a
lcm (
d
năm trên nửa mặt phảng bờ
a
không
chứa
b
).
Xét trường hợp điểm M nằm trên nửa mặt phẳng bờ
b
không chứa
a
.
Cũng chứng minh tương tự như trên, ta được quỹ tích của điểm
M
là đường thẳng
' //db
và cách
b
là lcm
(
'd
nằm trên nửa mặt phẳng bờ
b
không chứa
a
).
Kết hợp cả hai trường hợp ta được: Quỹ tích của điểm
M
là hai đường thăng
d
'd
nằm ngoài phần mặt
phẳng giới hạn bởi
a
b
sao cho
//da
và cách
a
là lcm;
' //db
và cách
b
là lcm.
9.2. (h.9.6)
a) Phần thuận
Vẽ
CH Ox
ta được
11
CA
(cùng phụ với
2
A
).
HAC OBA
(cạnh huyền, góc nhọn)
CH OA a
Điểm
C
cách đường thẳng
Ox
một khoảng bằng
a
nên
C
nằm trên đường thẳng
//d Ox
cách
Ox
một
khoảng
a
cho trước.
Hình 9.5
- 64 -
Giới hạn: Nếu B trùng với O thì C trùng với C
1
(C
1
d và C
1
A OA).
Nếu B ra xa cùng thì điểm C cũng ra xa cùng. Vậy điểm C nằm
trên tia C
1
t của đường thẳng d.
b) Phần đảo
Lấy điểm C bất kì trên tia C
1
t. Vẽ đoạn thẳng AC. Từ A vẽ AB AC
(B Oy). Ta phải chứng minh tam giác ABC vuông cân tại A.
Thật vậy, vẽ CH Ox.
HAC và OBA có: HC = OA = a; (cùng phụ với
Do đó HAC = OBA (g.c.g) AC = AB.
Vậy ABC vuông tại A.
c) Kết luận
Vậy quỹ tích của điểm C là tia C
1
t // Ox và cách Ox một khoảng bằng a.
9.1. (h.9.7)
a) Phần thuận
Gọi O là giao điểm của hai tia AD và BE. Như vậy O là một điểm cố định.
Xét AOB có nên
Tứ giác OECD có ba góc vuông nên là hình chữ nhật.
Hai đường chéo DE và OC cắt nhau tại trung điểm của mỗi đường nên trung điểm M của DE cũng là trung
điểm của OC.
VOH AB, MK AB tMK đường trungnh của OHC suy ra
Điểm M cách đường thẳng AB cho trước một khoảng là nên điểm M nằm trên đường thẳng xy // AB
và cách AB là
Giới hạn: Khi điểm C di động dần tới A thì điểm M dần tới trung điểm P của OA. Khi điểm C di động dần
tới B thì điểm M dần tới trung điểm Q của OB. Vậy điểm M chỉ di động trên đường trung bình PQ của
OAB (trừ hai điểm P và Q).
b) Phần đảo
Lấy điểm M bất kì trên đoạn thẳng PQ (M không trùng với P, Q). Vẽ tia OM cắt AB tại C. Vẽ CD OA,
CE OB. Ta phải chứng minh các DAC, EBC vuông cân và M là trung điểm của DE.
Thật vậy, xét OAB có OP = PA, PQ // AB nên MO = MC.
Xét DAC vuông tại D có nên là tam giác vuông cân tại D.
Tương tự, EBC vuông cân tại E.
T giác OECD có ba góc vuông nên là hình ch nht. Do đó hai đường chéo ct nhau tại trung điểm mi
đường.
o
H O 90 ;
11
CA
2
A ).
o
A B 45
o
AOB 90 .
1
MK OH.
2
OH
2
OH
.
2
o
A 45
Hình 9.6
Hình 9.7
- 65 -
Do M là trung điểm OC nên M cũng là trung điểm DE.
c) Kết lun
Vy qu tích của đim M là đường trung bình PQ ca tam giác OAB tr hai điểm P Q.
9.4 ( h 9.8)
Gi O là giao điểm ca hai tia ADBE. Như vậy O là điểm c định .
Giải tương tự bài 9.3,
ta được qu tích của điểm M là đường trung bình PQ ca tam giác OAB
tr hai điểm PQ.
9.5 ( h. 9.9)
a) Phn thun.
V
AH BC
th AH//DE và
( tính cht ca tam giác cân) .
Ta có
11
EA
( cp góc so le trong) ;
12
FA
( cặp góc đồng v ).
nên
11
EF
. Suy ra
AEF
cân.
Ta có : ME = MF suy ra
AM EF
.
T giác AHDM có ba góc vuông nên là hình ch nht nên MD = AH (
không đổi).
Đim M cắt đường thng BC cho trước mt khong bng AH nên điểm
M nằm trên đường thng xy // BC và cách BC mt khong bng AH.
Gii hn: Khi điểm D trùng vi B thì E trùng vi BF trùng vi
1
F
(
1
F
nm trên tia CA
1
AF AC
). Khi đó điểm M trùng vi
1
M
(
1
M
là giao điểm ca xy vi
1
BF
). Tương tự , khi điểm D trùng vi C thì
điểm M trùng vi
2
M
. Vậy điểm M ch nằm trên đường thng
12
MM
của đường thng xy.
b) Phần đảo
Lấy điểm M bất kì trên đoạn
12
MM
. Qua M v một đường thng vuông góc vi BC ct BC , AB, AC lần lượt
ti D, E, F ta phi chng minh M là trung điểm ca EF.
Ta có
1 2 1 2
E A ; F A
12
AA
nên
. Do đó
AEF
cân. Vì AM là đường cao cũng đồng
thời là đường trung tuyến
ME MF
c) Kết lun:
Vy qu tích của điểm M là đoạn thng
12
MM
của đương thẳng xy // BC và cách BC mt khong AH.
9.6 (h 9.10)
a) Phn thun
Q
P
O
A
B
D
E
C
M
D
H
M2
M1
E1
F1
C
B
A
F
- 66 -
V đoạn thẳng MO, MA ta được
1
MO MA BC
2

.
Điểm M cách đều hai đoạn thng OA c định nên điểm M
nằm trên đường trung trc ca AO.
Gii hn: Khi điểm C di động tới điểm O thì điểm B di
động
11
B (AB AO)
. Khi đó điểm M di động ti
2
M
trung điểm ca OH.
Khi B di động tới O thì C di động ti
11
C (AC AO)
. Khi
đó điểm M di động trên
2
M
là trung điểm ca OC.
Vậy điểm M di động trên đoạn thng
12
MM
.
(hình 9.10)
b) Lấy điểm M bất kì trên đoạn thng
12
MM
trên tia Ox, lấy điểm B
B0
, sao cho MB = MA. Tia MB
ct Oy tại điểm C. Ta phi chng minh
ABC
vuông tại A và M là trung điểm ca BC.
Tht vy ta có MB = MA mà MO = MA ( vì M nằm trên đường trung trc ca OA)
nên MB = MO (1)
MOB
cân
11
BO
.
Xét
OBC
vuông ti O có
OO
11
B BCO 90 O BCO 90 MOC MCO
(vì cùng ph
1
O
).
Nên
MOCcân MO MC
. (2)
T (1) và (2) suy ra MB = MC. Vậy M là trung điểm ca BC.
Xét
ABC
có MA = MB = MC nên
1
MA BC
2
ABC
vuông cân ti A.
c) Kết lun:
Qũy tích của điểm M là đoạn thng
12
MM
thuc trung trc ca OA.
2
) Tìm quỹ tích cùa điểm M.
a) Phẩn thuận
Ta có: MA + MC = MB + MD. (2)
Suy ra (MA + MC)
2
= (MB + MD)
2
2 2 2 2
MA MC 2MA.MC MB MD 2MB.MD.
2MA.MC 2 MB. MD. 3
Hình.9.11
Từ (1) và (3)
2 2 2 2
MA MC 2MA.MC MB MD 2MB.MD.
2
2
(MA MC MD) MB .
Suy ra MA - MC = MB - MD. (4) hoặc MA - MC = MD - MB. (5)
Từ (2) và (4) ta có:
MA MC MB MD
MA MC MB MD
Do dó 2MA = 2MB MA = MB.
Vậy điểm M nằm trên đường trung trực của AB.
Từ (2) và (5) ta có:
MA MC MB MD
MA MC MD MB
Do đó 2MA = 2MD MA = MD.
Vậy điếm M nằm trên đường trung trực của AD.
Giới hạn: Vì M nằm trong hình chữ nhật
hoặc trên các cạnh của nó nên M nằm
trên hai đoạn thẳng EF và GH nối trung
điểm hai cặp cạnh đối diện cùa hình chữ
nhật.
x
y
1
M
O
C
A
B
C
- 67 -
b) Phẩn đảo (h.9.12)
Lấy điểm M bất kì trên đoạn thẳng GH. Hình.9.12
Khi dó MA = MD; MB = MC.
Vậy MA + MC = MD + MB. Nếu M EF ta cũng có kết quả trên.
c) Kết luận: Quỹ tích của điểm M là hai doạn thắng EF và GH nối các trung điểm của hai cặp cạnh đôi diện
của hình chữ nhật.
9.8. (H.9.13)
a) Phần thuận
MAC và ADBC có: MC = DC;
12
C C
(vì cùng cộng với ACD cho
0
60
); CA = CB.
Vậy MAC = DBC (c.g.c)
MAC DBC 90 . Suy ra MA AC
tại A.
Do đó điểm M nằm trên một dường thẳng đi
qua A và vuông góc với AC.
Giới hạn: Khi điểm D trùng với B thì điểm M
trùng với A. Khi điểm D ra xa vô cùng thì
điểm M cũng ra xa vô cùng. Vậy điểm M chỉ
nằm trên tia Ay. Hình.9.13
b) Phần đảo
Lấy điểm
M
bt k trên tia
Ay
. V đoạn thng
MC
. Trên tia
Bx
lấy điểm
D
sao cho
CD CM
. Ta phi
chng minh
MCD
đều.
Tht vy,
DBC
ˆ
ˆ
90AB
,
CM CD
,
CA CB
.
Do đó
MAC DBC
(cnh huyn, cnh góc vuông)
Suy ra
12
60C C MCD BCA
.
MCD
cân có
60MCD
nên là tam giác đều.
c) Kết lun
Qu tích của điểm
M
là tia
Ay AC
(tia
Ay
na28mt rên na mt phng b
AC
không cha
B
).
9.9 (H.9.14)
DCE BCF
(c.g.c)
CE CF
12
CC
.
- 68 -
Ta có
12
90 90C BCE C BCE
.
Hình bình hành
CE CF
90ECF
nên
là hình vuông
ME MF
.
V
MH AB
,
MK AD
ta được
90HMK
.
Mt khác
90EMF
nên
HMF KME
(hai góc có cạnh tương ứng vuông góc cùng nhn).
Suy ra
HMF KME
(cnh huyn, góc nhn)
MH MK
.
Đim
M
nm trong góc vuông
EAB
và cách đều hai cnh ca góc này nên
M
nm trên tia phân giác
Ax
ca góc
EAB
.
Lưu ý: Bài toán không hi qu tích của điểm
M
, mà ch hỏi điểm
M
nằm trên đường nào do đó trong lời
gii ta ch trình bày ni dung ca phn thun.
9.10 (H.9.15)
a) Phn thun
Xét
EDF
vuông ti
E
EM
là đường trung tuyến nên
1
2
EM DF DM
.
BDM BEM
(c.c.c)
12
BB
.
Vậy điểm
M
nm trên tia phân giác
Bx
ca góc
B
.
Gii hn:
* Khi điểm
D
trùng vi
A
thì điểm
M
trùng với điểm
1
M
(
1
M
là giao điểm ca tia
Bx
vi
AC
).
* Khi điểm
D
trùng vi
B
thì điểm
M
trùng với điểm
2
M
(
2
M
là trung điểm ca
1
BM
).
b) Phần đảo
Lấy điểm
M
bt k trên đoạn thng
12
M M
.
Lấy điểm
D
trên cnh
AD
sao cho
1MD MA
Lấy điểm
E
trên cnh
BC
sao cho
BE BD
.
- 69 -
Tia
DM
ct cnh
AC
ti
F
.
Ta phi chng minh
M
là trung điểm ca
DF
EF DE
.
Tht vy
BMD BME
(c.g.c)
2MD ME
MAD
cân
11
DA
.
Ta có
11
90DF
,
12
90AA
11
DA
nên
12
3F A MF MA
T
1 , 2 , 3
suy ra
MD ME MF
.
Vy
M
là trung điểm ca
DF
DEF
vuông ti
E EF DE
.
c) Kết lun
Vy qu tích của điểm
M
là đoạn thng
12
M M
ca tia phân giác ca góc
B
.
9.11 (H.9.16)
a) Phn thun
V
AH Ox
,
AK Oy
. Khi đó
180 60 120HAK
.
Mt khác
180 60 120BAD
nên
12
HAK BAD A A
.
HAB KAD
(cnh huyn, góc nhn)
AH AK
.
Đim
A
nm trong góc
xOy
và cách đều hai cnh ca góc
xOy
nên
A
nm trên tia phân giác
Ot
ca góc
xOy
.
Gii hạn: Khi điểm
B
trùng vi
O
hoc khi
D
trùng vi
O
thì điểm
A
trùng vi
1
A
(
và cách
O
mt khong
1
OA a
). Khi
AB Ox
thì
AD Oy
, điểm
A
trùng với điểm
2
A
(
2
A Ot
và cách
O
mt
khng
2
2OA a
).
b) Phần đảo
- 70 -
Lấy điểm
A
bất kì trên đoạn thng
12
A A
. V
AH Ox
,
AK Oy
thì
AH AK
(tính cht tia phân giác).
Trên đoạn thng
HO
lấy điểm
B
, trên tia
Ky
lấy điểm
D
sao cho
AD AB a
. V hình bình hành
ABCD
, ta phi chng minh
ABCD
là hình thoi cnh
a
,
ˆ
60B
.
Tht vy, hình bình hành
ABCD
AB AD a
nên nó là hình thoi cnh
a
.
HAB KAD
(cnh huyn, cnh góc vuông)
12
AA
180 60 120BAD HAK
. Do đó
1
ˆ
2080 601B
.
c, Kết lun
Vy qu tích của điểm
A
là đoạn
12
AA
thuc tia phân giác
Ot
ca góc
xOy
.
9.12. (H.9.17)
Ta có:
1 2 3 4
D D ,D D
00
2 3 1 4
D D D D 90 :2 45
Trên tia đối ca tia
AB
ly
N
sao cho
51
AN CF
AND CDF c.g.c
DN DF và D D
Do đó
0
4 5 4 1
D D D D 45
. Suy ra
0
NDF FDE 45
NDE FDE
(c.g.c)
NED FED
Do đó
DAE DAE
(g.c.g)
DM DA 4cm.
Đim
M
cách điểm
D
cho trước mt khoảng không đổi là
4cm
nên điểm
M
nằm trên đường tròn tâm
D
,
bán kính
4cm
.
9.13. (H.9.18)
a) Phn thun
V đoạn thng
OM
ta có
1
OM= AB=a
2
( tính cht trung tuyến ca tam giác vuông).
Đim
M
cách điểm
O
cho trước mt khong
a
cho trước nên
M
nằm trên đường tròn tâm
O
, bán kính .
x
5
4
3
2
1
M
N
F
E
C
D
A
B
2
Gii hn:
Khi điểm
B
di động ti
O
thì
A
tới điểm
1
A Ox
1
OA 2a
. Khi đó điểm
M
di động ti
1
M
là trung
điểm ca ca
1
OA .
Khi điểm
A
di động ti
O
thì
B
tới điểm
1
B Oy
1
OB 2a.
Khi đó điểm
M
di động ti
2
M
là trung
điểm ca ca
1
OB .
Vậy điểm M nm trên cung M
1
M
2
của đường tròn tâm, bán kính a
b) Phần đảo
Lấy điểm
M
bt kì trên cung
12
M M .
Trên tia
Ox
lấy điểm
A
sao cho
MA MO.
(1)
Tia
AM
ct tia
Oy
ti
B
. Ta phi chng minh
M
là trung điểm ca
AB
AB 2a.
Tht vy, vì
MA MO
nên
MOA
cân
1
1
A =O
.
Xét
AOB
vuông ti
O
00
1 2 1 2
A +B 90 O +B 90
=>
2
2
O =B
( cùng ph vi
1
O
)
Do đó
MOB
cân
MB MO.
(2)
T (1) và (2) suy ra
MA MB MO a,
do đó
AB 2a.
c) Kết lun:
Qu tích của điểm
M
là cung
12
MM
của đường tròn tâm
O
, bán kính
a
.
9.14. (H.9.19)
a) Phn thun
Gi
O
là điểm đối xng vi
D
qua
C
thì
O
là một điểm c định.
x
y
2
1
2
1
M
O
A
B
M
2
M
1
A
1
B
1
- 2 -
T giác
ABOC
AB/ /OC;AB OC
( vì cùng bng
CD)
nên
ABOC
là hình bình hành
OB AC 2cm.
Đim
B
cách điểm
O
c định mt khong
2cm
nên điểm
B
nm trên
đường tròn tâm
O
bán kính
2cm
Gii hn:
B, C, D
không thng hàng nên
B
nằm trên đường tròn tâm
O
, bán kính
2cm
tr giao điểm
của đường tròn này với đường thng
CD.
b) Phần đảo
Lấy điểm
B
bất kì trên đường tròn tâm
O
bán kính
2cm
( tr các giao điểm của đường tròn này vi
đường thng
CD
). Suy ra
OB 2cm
. V hình bình hành
Ta chng minh hình bình hành có
AC 2cm
.
Tht vy,
AB / / CD
AB CD AB / /CO
AB CO.
Do đó tứ giác
ABOC
là hình bình
hành, suy ra
AC OB 2cm.
c) Kết lun
Vy qu tích ca điểm
B
là đường trong tâm
O
bán kính
2cm
.
2
2
D
A
C
O
B
- 1 -
Chƣơng II: ĐA GIÁC – DIỆN TÍCH ĐA GIÁC
Chuyên đề 10. ĐA GIÁC – ĐA GIÁC ĐỀU
10.1. Gi s cnh của đa giác là n, điều kin
n N, n 3
Ta có
2
( 3)
14 27 28 3 54
2
nn
nn
2 2 2
11 3 15 11 3 15
2 2 2 2 2 2
7 9 8
nn
nn
10.2. Tổng các góc trong trừ đi một góc của đa giác bằng
0
2570
nên
00
n 2 .180 A 2570
00
A ( 2).180 2570n
00
0 A 180
0 0 0 0
0 ( 2).180 2570 180n
55
16 17
18 18
n
. Vì
n N n 17
.Vậy đa giác đó có 17 cạnh
10.3.
a) Ta có
1 1 1
AMBC ,BMCA ,CMAB
là các hình bình hành .
Suy ra các đường chéo
1 1 1
AA , BB , CC
đồng quy ( xem bài 7.7)
b) Theo tính chất các hình bình hành ta có:
1 1 1 1 1 1
AC A C MB; AB A B MC; BC B C AM
Để hình lục giác
1 1 1
AB CA BC
các cạnh
bằng nhau thì
MB MC AM
hay điểm
M
là giao điểm ba đường trung trực của tam giác
ABC
.
10.4. Xét các đường chéo xuất phát từ cùng một đỉnh. Ta chọn một đỉnh nào đó rồi đánh số
1
các đỉnh tiếp
theo theo chiều kim đồng hồ đánh lần lượt số
2, 3,
Đường chéo ngắn nhất nối đỉnh
1
với đỉnh
3
.
Đường chéo dài nhất là đường chéo nối đỉnh
1
với đỉnh
11
. Từ đó ta có
9
loại độ dài khác nhau.
10.5. Ta có:
12
11
DBE ABC B B ABC.(1)
22
EA AB EAB
cân
0
2 1 1
EAB
E B B 90
2
0
2
BCD
B = 90 -
2
CB CD
Thay vào (1) ta được:
00
BCD EAB 1
90 90 ABC
2 2 2
0
EAB ABC BCD 360
.
Tổng các góc của ngũ giác bằng
0
540 .
C
1
A
1
B
1
A
B
C
M
- 2 -
0 0 0
CDE DEA 540 360 180
0 0 0
11
CDE DEA
D E 90 90 90 AD CE.
22
Mặt khác
EAD
cân tại
E
,
CDE
cân tại
D
AD CE
nên
AD
CE
tại trung điểm mỗi đường
AEDC
là hình bình hành.
AC DE AB BC CA ABC
đều
0
ABC 60
Vậy
0
ABC 60
10. 6. a)
ABC
BCD
AB BC; ABC BCD;BC=CD
ABC BCD c.g.c AC BD.
ABD và DCA có AB DC; AC DB ; AD
chung
ABD DCA c.c.c BAD CDA
BAH CDK BH CK BC/ /CD ABCD
là hình thang cân.
b) Chứng minh tương tự câu a, ta có
ABCE
là hình thang cân.
Ta có:
ABC cân BAC BCA
A C CAE ACD
ACE CDA c.g.c ACDE là
hình thang cân).
( Chứng minh tương tự câu a).
Ta có:
AB / / CK ( ABCD
là hình thang cân),
BC / / AK ( ABCE
là hình thang cân),
AB BC.
Suy ra
ABCK
là hình thoi
1
12
A C C
ACDE là hình thang cân
11
2 1 1 3
C E E C C C
ABC CDE BAC CDE
1
2
1
2
1
E
A
C
B
D
- 2 -
Chứng minh tương tự, ta được
BAE AED
Do đó:
A B C D E
AB BC CD DE EA
(gt).
ABCDE
là ngũ giác đều.
10.7. Nối
CE
, gọi
K
là trung điểm của
CE
. Ta có
QK
là đường trung bình của
tam giác
ACE
suy ra
1
QK / / AC và QK AC
2
.
M, N
lần lượt là trung điểm của
AB
BC.
Ta có
MN
là đường trung bình của tam giác
ABC
, suy ra
MN / / AC
1
MN AC
2
. Từ đó ta có:
MN / / QK
MN QK MNKQ
là hình bình hành
M, J, K
thẳng hàng và
MJ JK.
Xét
MKP
I, J
lần lượt là trung điểm của
MP
MK.
Ta có
I J
là đường trung bình của tam giác
MKP I J/ /PK
1
I J PK(1)
2
Xét
,CDE PK
là đường trung bình
1
/ / ;
2
PK DE PK DE
(2).
1
1
3
2
1
H
K
E
A
D
C
B
J
I
P
K
Q
N
M
A
C
B
E
D
- 2 -
T (1) và (2)
/ / IJ DE
1
4
IJ DE
.
10.8
Nhn thy
’; ; ’; ’; ’; AA F BA B AB C DC D ED E FE F
bng nhau (c.g.c)
A B B C C D E F A F
(1)
’’BA B
có BA’=BB’
’’BA B
cân ti B
0
0
180
' ' ' ' 30
2
B
BA B BB A
Tương tự đối vi
’’AA F
ta có
0 0 0
'F' ' ' 30 ' 'F' 180 'F' ' ' 120AA AF A B A AA BA B
Chứng minh tương tự ta được
0
' ' ' ' 'D' ' ' ' ' ' ' ' ' ' 120A B C B C C D E D E F E F A
(2)
T (1) và (2) ta có điều phi chng minh.
10.9
Lc giác ABCDEF không nht thiết phi là lục giác đều . Tht vy
* Trên mt phng lấy điểm O tu ý , v 3 tia OA; OC;OE sao cho độ dài 3 đoạn OA, OB, OC đôi một
khác nhau và độ ln của 3 góc AOC; COE; EOA cũng đôi một khác nhau
* V các hình bình hành OABC, OCDE, OAFE khi đó ta có được lc giác li ABCDEF
Rõ ràng
/ / , ; / / , , / / , AB CD AB DE BC EF BC EF CD FA CD FA
nhưng ABCDEF không phải
lục giác đều.
10.10 .
Gi s AD là đường chéo ln nht của ngũ giác ABCDE. Gọi O là giao điểm ca AC và BD
Xét
AOD
AD OA OD
; OA AC OD BD
nên
AD AC BD
Mt khác
;AC AD BD AD
nên AD, AC, BD là độ dài 3 cnh ca mt tam giác
10.11 .
F'
F
E'
E
D'
D
C'
C
B'
A'
B
A
F
E
D
C
B
A
O
- 3 -
Áp dng tính cht v quan h gia các cnh ca tam giác, ta có
A
B BC CD DE EA AN NB BP PC CQ QD DK KE EM MA
(1)
Mt khác
; AN PC AC BP DQ BD
; CQ KE CE DK MA DA
EM NB EB
(2) . T (1) và (2) ta có điều phi chng minh
Nhn xét : Nhng bài toán v bất đẳng thc bạn nên đưa về bất đẳng thc tam giác.
10.12. Gọi đa giác đều trên có n cạnh, để xếp các đa giác đều bng nhau không có khe h thì
0
00
( 2).180
360 360 ( 2).180 2 ( 2) 2 4 4 ( 2) 4 (n 2) n 3;4;6
n
n n n n n n
n
Vậy đa giác có nhiều nht là 6 cnh.
10.13.
Theo gi thiết
0
0
(6 2)180
120
6
A B C D E F
Gi s
; ; BC EF DE AB AF CD
.
Qua A k đường thng song song vi BC
Qua C k đường thng song song vi DE
Qua E k đường thng song song vi FA
Chúng ct nhau to thành tam giác PQR
Ta có ABCP là hình bình hành nên
00
120 60APC B QPR
Tương tự
0
60PRQ
, do đó
PQR
đều ,
PR PQ QR
, tc là
BC EF DE AB AF CD
Ngược li , gi s có 6 đoạn thng
1 1 1 1 1 1
;BC ;CD ;DE ;EF;FAAB
Tho mãn điều kin
1 1 1 1 1 1
-EF -AB -CDBC DE AF a
. Dựng tam giác đều PQR vi cnh bằng a. Đặt
trên các tia QP, RQ và PR các đoạn thẳng tương ứng bằng đoạn thng lớn hơn trong các cặp
1
AB
1
DE
;
1
CD
1
FA
;
1
EF
1
BC
. Dng thêm các hình bình hành t đó ta xác định được lc giác cn tìm
10.14 .
Xét đường chéo dài nht ca lc giác
P
Q
K
M
N
E
D
C
B
A
Q
P
R
F
E
D
C
B
A
- 4 -
Trường hp 1 : Trường hợp đường chéo dài nht ca lc giác chia lc giác thành một ngũ giác và một tam
giác
Gi s đường chéo dài nht ca mt lc giác là AE. Chia lục giác thành ngũ giác và tam giác. Nếu 3 đường
chéo t đỉnh A không là độ dài 3 cnh ca mt tam giác thì AC+ AD
AE (1)
Ta s chứng minh 3 đường chéo k t E s tho mãn tính chất đó.
Gọi I là giao điểm của EB và AK, K là giao điểm ca EC và AD. Ta có
AI AK AC AD
kết hp vi (1)
AI AK AE 
(2)
Ta li có
AI IE AK KE AE
(3)
Mt khác
EB EC EI EK
nên t (3)
EB EC AE 
. Vy EA, EB, EC làm thành 3 cnh ca mt tam giác
Trường hp 2 :
Trường hợp đường chéo dài nht ca lc giác chia lc giác thành hai t giác
Gi s AD là đường chéo dài nht ca lc giác chia lc giác thành hai t giác. Nếu 3 đường chéo
xut phát t A không to thành 3 cnh ca mt tam giác thì AC+ AE
AD (4) Gi I, K lần lượt là giao
điểm hai đường chéo ca t giác ADEF và ABCD.
T (4) suy ra
AI AK AC AE AD
(5)
Ta li có
2 AI DI AK DK AD
. Kết hp vi (5) suy ra
DI DK AD
. Do đó
DB DF DA
Vy DA, DB, DF làm thành 3 cnh ca mt tam giác
10. 15 .
K
I
G
E
D
C
B
A
K
I
G
E
D
C
B
A
K
H
D
C
B
A
T ng các góc c a l c giác ABCDEG là ( 6 2 ) . 180
0
= 720
0
Theo gi thi t ta có
0
360A E C B D G
D ng
EDK ABC
DK BC
( . . )EDK ABC c g c EK AC
(1)
T
0
360
( . . ) (2)
EDK CDE AGE CDK AGE
CDK AGE c g c CK AE
T (1) và (2) suy ra ACKE là hình bình hành
0
180ACD DCK CAE
DCK GAE
nên
0
180 / /ACD GAE CAE CD AG
.
ch ng
c
/ / ; / /AB DE BC EG
E
D
C
B
A
- 6 -
Chuyên đề 11. DIỆN TÍCH ĐA GIÁC
11.1 Áp dụng định lý Py-ta-go trong tam giác vuông BCD, ta có
2 2 2 2 2 2
3 4 25 5BD BC CD
nên BC = 5cm
2 . 3.4
2,4( )
5
BCD
S BC CD
CH cm
BD BD
Xét tam giác vuông CDH , ta có
2 2 2 2 2 2
4 2,4 10,24 3,2DH CD CH
Nên DH = 3,2 cm . K
AK vuông góc BD, Ta có
ABD CBD
SS
Nên
2,4 AK CH cm
. Vy
1
2
ADH
S
DH. AK =
1
2
3,2.2,4=3,86 (cm
2
)
11.2.
Qua A k đường thng song song vi BD, cắt đường thng CD ti E
ABDE là hình bình hành
AE = BD = 12cm, DE = AB = 5 cm
CE = 20cm
Ta có
2 2 2 2 2 2
12 16 400 20AE AC CE ACE
vuông ti A
2
11
. 16.12 96( )
22
ACE
S AC AE cm
Mt khác
ADE ABC
SS
( vì AB= DE và đường cao k t A và C của hai tam giác đó bằng nhau)
2
96
ABCD ABC ACD ADE ACD ACE ABCD
S S S S S S S cm
11.3
Ta có
1
..
2
ADE
S AD AE BD
2
11
.
22
ADE
S AD AB AD AB AD AD
K
G
E
D
C
B
A
- 7 -
2
22
11
.
2 4 8
ADE
AB
S AD AB AD AB



2
2
2
11
2 2 8 8
ADE
AB AB
S AD AB



Vy
22
2
3
2 8 8
BDEC ABC ADE
AB AB
S S S AB
không đổi
Do đó min
2
3
8
BDEC
S AB
khi
,BD
lần lượt là trung điểm
,AB AC
11.4
Gi
,KH
lần lượt là hình chiếu ca
,DC
trên
BE
1
2
ABE BCE ABC
S S S
1
3
CDB ABC
SS
11
33
BDE ABE CBE
S S S
1
3
BDE
CBE
S DH
S CK
31
3
44
IBC IBD CDB
S S S S
11.5
- 8 -
Do
//KE CA
nên
CAE CAK
SS
Vy
CBAE ABC CEA ABC CKA ABCK
S S S S S S
ABCK
S
=
111
222
CKB KBA BCD BAD ABCD
S S S S S
T đó suy ra điều phi chng minh.
11.6
Ta có
BCD
có chung canh
BC
và đường cao ng vi canh
BC
bng nhau nên
BCD BCP
SS
1
1
2
BCP ABCD
SS
Ta có
.AM CK AB CD BM DK
K
BQ CD
, ta có
.
.1
.
2 2 2 2
BMKC ABCD
BM CK DK KC BQ
CD BQ
S BQ S

Suy ra
PBC BMKC
SS
hay
PEF BEFC BME KFC BEFC
S S S S S
Vy
PEF BME KFC
S S S
11.7
- 9 -
Ta có
,AD BE
là các đường trung tuyến nên
O
là trong tâm suy ra :
2 ; 2OB OE OA OD
Áp dụng định lý Py ta go ta có
2
2 2 2
4
b
OA OE AE
2
2 2 2
4
a
OB OD BD
22
2 2 2 2
4
ab
OA OB OE OD
2 2 2 2
22
44
OA OB a b
OA OB

2 2 2
2
44
AB a b
AB

22
2
5
ab
AB
Vy din tích hình vuông canh
AB
là:
22
5
ab
(đvdt)
11.8
- 10 -
Qua
,,M N P
k các đường thng song song vi cạnh hình vuông ABCD như hình vẽ. khi đó ra được
IKHE là hinh vuông và các tam giác INM, KPN, HQP, EMQ bng nhau
Ta có
1 9 3 8 6 10AMNB CDQP IMN HPQ
S S S S S S S S S S
2 11 7 4 12 9AMQD BNCP EMQ KPN
S S S S S S S S S S
1 2 3 4 5 6 7 8
;;;S S S S S S S S
9
10
.;S S AD IE IN
1
1 12
.S S AB IK ME
9 10 11 12
S S S S
Do đó
AMNH CDQP ADQM BCPN
S S S S
11.9
Ta có
2
1 1 1
. ; .
2 2 2
AKL ABC
S AK AL AH S AH BC
Gi
M
là trung điểm ca
BC
, ta có
`
2.BM MC AM BC AM
Mt khác
2AH BC
T đó suy ra :
2
1 1 1 1
. . .
2 2 2 2
AKL ABC
S AH AH BC S
- 11 -
Đẳng thc xy ra khi
HM
hay
vuông cân
11.10
Ta có :
33
44
ACP ABC
AP AB S S
3 3 3 3
4 4 4 4
ACQ ACD APCQ ACQ ABC ACD
CQ CD S S S S S S
3
4
APCQ ABCD
SS
Ta có
11
33
PQM APQ
MP AP S S
11
33
PNQ PQC
NQ CQ S S
1
3
MNPQ APCQ
SS
(2)
T (1) và (2) suy ra
1 3 1
.
3 4 4
MPNQ ABCD ABCD
S S S
11.11
Ta có
2
7
DPN MNP DOP MON
S S S S cm
- 12 -
2
3
MNP
DPN
S MN
S DP

2
1
32
MNP
DPN MNP
S
SS

2
2.7 14
MNP
S cm
Ta có
2
. 3 . 6. 6.14 84
ABCD MNP
S AB AD MN AD S cm
11.12
K
;BH AO DI OC
BHO
1
90 ; 30
2
BHO BOH BH BO
DIO
1
90 ; 30
2
DIO DOI DI DO
Ta có
11
..
22
ABCD ABC ACD
S S S AC BH AC DI
1 1 1 1
.
2 2 2 2
ABCD ABC ACD
S S S AC BH DI AC BO DO



2
1 1 1 1
. .10.12 30
2 2 4 2
AC BD AC BD cm
11.13
- 13 -
V
,,AA BB MM
vuông góc vi
PQ
Ta có : T giác
MNPQ
là hình bình hành
OQ OP
11
. ; .
22
AOQ BOP
S OQ AA S OP BB


1 1 1 1
.2. .
2 2 2 2
BDQ BOP
S S OQ AA BB PQ MM PQ MM
1
.
2
MPQ NOQ BOP MPQ
S PQ MM S S S
b) Chứng minh tương tự, ta có
DOQ COQ NPQ
S S S
.
AOQ BOP DOQ COP MPQ NPQ
S S S S S S
AOD BOC MPNQ
S S S
(1)
K
,MI AC BK AC
. Suy ra, ta có:
MPFE
S MP MI
1 1 1
.
2 2 4
AC BK AC BK
1
.
2
ABC
S AC BK
1
2
MPFE ABC
SS
Tương tự, ta có:
NQEF
1
2
ACD
SS
.
Do đó:
11
22
MPNQ ABC ACD
S S S
1
.
2
ABCD
S
(2)
- 14 -
T (1) và (2) suy ra:
1
2
AOD BOC ABCD
SSS
.
11.14. Các đường thẳng đã cho không thể ct các cnh k nhau ca
hình bình hành
ABCD
. Bi vì nếu thế không th to ra hai t giác
mà là tam giác và ngũ giác.
Gi s một đường thng ct các cnh
BC
AD
tại các điểm
M
N
.
Các hình thang
ABMN
CDNM
có các đường cao bng nhau do
đó tỉ s din tích ca chúng bng t s các đường trung bình. Tc là
MN
chia đoạn thng nối trung điểm ca các cnh
Ab
CD
theo
t s
2
5
. Tổng các điểm chia đường trung bình ca hình bình hành theo t s
2
5
4
.
Bi s đường thẳng đã cho là
13
và đều phải đi qua một trong s bốn điểm nói trên mà
13 3.4 1
, nên có
một điểm thuc ít nht bốn đường thng. Tc là có ít nht bốn đường thng trong s đó cùng đi qua một
điểm.
11.15. Theo b đề v đa giác bao, tồn ti một đa giác có
n
đỉnh
1000n
n
điểm trong s
1000
điểm
đã cho và
1000 n
điểm còn lại đã cho nằm trong đa giác.
Ta ni một điểm đã cho chẳng hn
1
A
vi
n
đỉnh của đa giác
n
cạnh, ta được
n
tam giác. Ni
n
điểm
cùng nm trong một tam giác đã tạo ra với ba đỉnh của tam giác đó, số tam giác tăng thêm hai (từ mt thành
ba). Tng cng ta có:
2
. 1000 1 1998n n n
(tam giác).
1000n
nên
1998 998n
.
Tn ti mt tam giác có din tích không quá
1
998
diện tích đa giác. Do đó tam giác có diện tích không quá
1
998
.
11.16. Chia hình vuông thành chín hình vuông nh có cnh bng
1
3
, din tích mi hình vuông nh
1
9
. Vì
37 4.9 1
nên tn ti mt hình vuông nh chứa năm điểm, ba điểm vào trong năm điểm ấy cùng là đỉnh
mt tam giác có din tích ca hình vuông nhỏ. Do đó các tam giác được to bi ba trong năm điểm đó có
din tích không quá
1
18
.
11.17. Gi
a
là đường thng cha cnh
AB
của đa giác.
Gi
C
là đỉnh của đa giác cách xa
AB
nht. Qua
C
k
đường thng
//b AB
.
Gi
,DE
là các đỉnh của đa giác các xa
,AC
nht v hai
phía ca
AC
. Qua
D
k đường thng
c//AC
, qua
E
k
đường thng
//d AC
.
- 15 -
Gi
MNPQ
là hình bình hành to bởi các đường thng
, , ,a b c d
các đỉnh của đa giác nằm trong hoc trên
biên ca hình bình hành.
Hin nhiên
da giác ACD ACE
S S S
.
1
2
ACD ACE MNPQ
S S S
nên
giác
1
.
2
MNPQ da
SS
tc là
giác
2.
MNPQ da
SS
.
11.18. V hình bình hành
;;ABCQ CDER AFEP
.
Ta có:
2.
ABCDREF APE
SS
2. 2.
CER ACQ PQR
S S S
2.
AEC ABCDEF FPQR ABCDE
SS SS
.
Vy
1
2
AEC ABCDEF
SS
.
11.19. Ta có:
1 1 1
,
2 2 2
ACG ACD ACI ACB AGCI ABCD
S S S S S S
(1)
Tương tự
1
2
DEBH ABCD
SS
(2)
T (1) và (2) suy ra:
AGCA DEBH ABCD
S S S
MNPQ IBM CEN DGP AHQ
S S S S S
.
11.20. Gi
G
là trng tâm ca tam giác
,ABC
ta có:
2 2 2
,,
3 3 3
PAG PAM PBG PBN PGC PCD
S S S S S S
ta s
chng minh trong ba tam giác
,,PAG PBG PCG
tn
ti
mt tam giác có din tích bng tng din tích hai tam
giác kia.
Xét trường hp
A
B
cùng nm v phía
C
đối vi
đường thng
PG
.
H
,,AH PG BK PG CF PG
,
DE PG
// // //AH DE CF BK
, theo gi thuyết
DA DB
2,AH BK DE G
là trng tâm
2CG GD
- 16 -
2CF DE CF AH BK
GPAP GCG PB
S S S
NPAP MCD PB
S S S
Tương tự với các trường hp còn li, trong ba tam giác
, , PAM PBN PCD
luôn tn ti mt tam giác có din
tích bng tng din tích hai tam giác còn li.
- 17 -
Chuyên đề 12. PHƢƠNG PHÁP DIỆN TÍCH
12.1. Ta có:
22
144 cm
ABCD
S AD
nên
12AD cm
.
Mt khác:
1
3
ABE ABCD
SS
nên
11
23
AB AE AB AD
22
12 8 (cm)
33
AE AD
.
12.2. Ta có:
;AM MB DN CN
, mà
AMND
BCNM
hình thang có chung đường cao, nên
AMND BCNM
SS
.
Ta có:
AMI BMI
SS
AM BM
và đường cao h t
;EF
bng
nhau.
Ta có
DEN FCN
SS
ND NC
và đường cao h t
;EF
bng
nhau.
EMN FMN
SS
.
K
;EK MN FH MN
nên
EK FH
.
Suy ra
EKI FHI
nên
EI FI
.
12.3. Đặt din tích
, , ,OBC OAC OAB ABC
lần lượt là
1 2 1
;;;S S S S
.
Áp dng t s diện tích hai tam giác có chung đường cao và tính
cht dãy t s bng nhau, ta có:
OBM OCM
ABM ACM
SS
OM
AM S S

1
OBM OCM
MBM ACM
SS
S
S S S

.
OMN OCN
ABN BCN
SS
ON
BN S S

2
OAN OCN
ABN BCN
SS
S
S S S

.
OAP OBP
ACP CBP
SS
OP
CP S S

3OAP OBP
ACP CBP
S S S
S S S

.
123
S S S S
. T đó suy ra
2
3
1
1
S
SS
OM ON OP
AM BN CP S S S
.
12.4. V
,,AI EF BH EF CK EF
.
- 18 -
Ta có:
MB MC
nên
ABM ACM
SS
;
FBM FCM
SS
.
Suy ra
ABF ACF
SS
ADF BDF AEF CEF
S S S S
11
( ) ( )
22
DF BH AI EF AI CK
BD CK
suy ra
DF EF
.
12.5. V
;DM AI DN CK
( , )M AI N CK
.
Ta có:
1
2
DCK ABCD
SS
1
2
ADI ABCD
SS
,
Nên
11
..
22
DCK ADI
S S AI DM CK DN
.
Mt khác
AI CK
, suy ra
DM DN
D
cách đều hai cnh ca góc
AOC
. Vy
OD
là tia phân giác ca
góc
AOC
.
12.6. Trên tia đối ca tia
DC
lấy điểm
E
sao cho
DE AB
T giác
ABDE
là hình bình hành, mà
N
là trung điểm
AD
N
là trung điểm
BE
.
Ta có:
ABN DEN
, nên
ABN DEN
SS
ABCD BEC
SS
.
Đim
M
thuc
CD
sao cho
BM
chia
ABCD
thành hai phn có din tích bng nhau
11
2 2
BMC ABCD BEC
S S S
1
2
CM CE
CM ME
BCE
,MC ME NB NE
MN
là đường trung bình ca
BCE
//MN BC
A
B
D
C
N
M
E
- 19 -
12.7. Ni
; ; ; ; MX BX BY CX CZ
. Ta có
MXP MXY PXY
MXN MXZ NXZ
XP S S S
XN S S S

T đó, với chú ý rng
// BP YZ
thì
PXY BXY
SS
//CN ZX
thì
NXZ CXZ
SS
. Ta có:
1
MXY BXY
MXZ CXZ
XP S S
XN S S
Mt khác, vì
// YZ BC
nên ta có:
2
XMB YMB XMB YMB MXY BXY
XMC ZMC XMC ZMC MXZ CXZ
MB S S S S S S
MC S S S S S S


T (1) và (2) suy ra:
XP MC
XN MN
12.8. K
; DH AE CK AE
. Ta có:
1
;
4
AEC
ABE
S EC
S BE

1
4
ADE
ABE
S AD
S AB

(vì
4. ; 3.BE EC BD DA
).
AEC ADE
ABE ABE
SS
SS

AEC ADE
SS
CK DH
Suyra HFD KFC
FD FC
0
90 ; ;H K FHD FCK CK AH
.
A
C
B
P
Z
N
Y
M
X
H
A
F
B
E
C
D
K
- 20 -
12.9. K
’, ’, , DD BB EH FK
cùng vuông góc vi
AC
.
Do
// EH BB
'
EH AE
BB AB

Do
// EH BB
'
EH AE
BB AB

AE CF
gt
AB CD
' DD'
EH FK
BB

Ta có
HIE KIF
(vì
, KIFHIFIEEI
)
HE FK
vy
’’BB DD
. Suy ra
ACD ABC
SS
Vy
AC
chia đôi diện tích t giác
ABCD
.
12.10. Ta thy
ABD ACE
(g.c.g) nên
AD AE
Vy tam giác
ADE
vuông cân ti
A
0
ADE AED 45
Do đó
// DE BC
nên
1
I
BC DBC
SS
Dng
( )DH BC H BC
. Ta thy
DBH DBA
(c.huyn g.nhn)
Nên
DH DA
1
.
2
2
1
.
2
DBC
DBA
BC DH
S BC
Suyra
S AB
AB DA
22
DBC
ABC DBC
S
hay
SS
T (1) và (2) suy ra
2
IBC
ABC IBC
S
SS
2
IBC
IAB ICA
S
SS

12.11. Vn dng hai tam giác có chung mt
cnh thì t s din tích bng t s hai
đường cao ng vi cạnh đó là:
; ; .
OBC OAC OAB
a ABC b ABC c ABC
r S r S r S
h S h S h S
1
OBC OAC OAB
a b c ABC
r r r S S S
h h h S

Vy
1 1 1 1
abc
h h h r
.
12.12. Gi
, , M N P
là điểm đối xng ca
I
qua
, , AB AC BC
.
B’
A
C
B
E
D
I
F
K
H
D’
H
I
E
C
B
A
D
B
A
r
O
C
r
r
- 21 -
Gi
IM
giao
AB
ti
; H IN
giao
AC
ti
; K IP
giao
BC
ti
J
.
Ta có
2.MAI BAC BAI
nên
2
.
.
1
.
AMI
ABC
AHM AHIK
ABC ABC
S AM AI
hay
S AB AC
AI S S
AB AC S S

Tương tự , ta có
2
2
.
BPI BHIJ
ABC ABC
BI S S
AB AC S S

2
3
.
CNI CKIJ
ABC ABC
CI S S
AB AC S S

T (1), (2), (3) cng vế vi vế ta có điều phi chng minh.
12.13. Gi
1 2 3 4
, , ,S S S S
theo th t là din tích ca các tam giác
, , , OAB OBC OCD OAD
.
Suy ra
1
2
1
S OA
S OC
Tương tự
4
3
2
S OA
S OC
T
1
, 2
, ta có
14
23
SS
SS
Suy ra
1 3 2 4
..S S S S
nên
2
1 2 3 4 1 3
.S S S S S S
là mt s chính phương.
12.14. Gi s
, , AD BE CF
không đồng quy, gọi giao điểm các cặp đường chéo
AD
BE
CF
. N CF
AD
P
. Do
M
không thuc
CF
nên
M
nm trong mt trong hai t giác
FABC
hoc
CDEF
, gi s
M
nm trong t giác
FABC
. Theo gi thiết
, AD BE
cùng chia đa giác
ABCDEF
thành hai
phn có din tích bng nhau nên
1
2
ABCD BCDE ABCDEF
S S S
MAB MDE
SS
1
MAB
MDE
S
S

.
1
.
MAMB
MD ME

.
. . 1MAMB MD ME PD EN
Chứng minh tương tự, ta có:
DEF
1
2
CDEF A ABCDEF
S S S
PCD PFA
SS
.
. . 2PC PD PF PA NF MA
J
H
A
N
C
K
B
I
P
M
B
1
A
C
B
D
O
E
A
C
B
D
M
P
N
F
- 22 -
EFAB
1
2
FABC ABCDEF
S S S
NEF NBC
SS
.
. . 3NE NF NB NC MB BC
T
1
, 2
3
suy ra
. . . . . . . . . .MAMB PC PD NE NF MAMB PC PD NE NF
(Vô lí)
Vy
, , AD BE CF
đồng quy.
12.15. Gọi giao điểm ca
LP
MQ
O
, ta có
OLA OLB
SS
Đặt
1 OLA OLB
S S S
Tương tự
2 OMB OMC
S S S
3
;
ONC OND
S S S
4 OPD OPE
S S S
5
;
OQF OQE
S S S
6 ORF ORA
S S S
1
.
2
ABCDEF LBCDP MCDEQ
S S S S S
OLRM OPEQ
SS
1 2 4 5
S S S S
OABC ODEF
SS
Cng hai vế vi
ta được
1
2
RABCNO NDEFRO
S S S
NR
chia lc giác thành hai phn có din tích bng nhau nên
O
phi thuc
NR
. Vy
, , LP MQ NR
đồng quy.
12.16. Ta có:
EAM FAM EAM FAM FAE
EBM FBM EBM FBM FBE
1
MA S S S S S
MB S S S S S
EDN FDN EDN FDN FDE
ECN FCN ECN FCN FCE
2
ND S S S S S
NC S S S S S
, EF
lần lượt là trung điểm ca
AD
BC
nên ta có:
EAF
;
FDE
SS
3
E
BF ECF
SS
T
1
, 2
3
suy ra:
MA ND
MB NC
. . .MANC MB ND
N
M
C
A
E
D
B
F
B
E
A
C
D
M
P
N
F
Q
R
L
O
1


 ............................................................................ 2
 14. TÍNH CHNG PHÂN GIÁC CA TAM GIÁC .................................................. 11
 NG HNG DNG CA TAM GIÁC .................................................. 17
 NG HNG DNG CA TAM GIÁC VUÔNG .................................. 27
 NH LÝ CE-NH LÝ VAN-OBEN.............................. 34
2
C
13.1.

1
3
AI AE AE
IC CD AB

1
6
4
AI AC cm

1
2
CK CF CF
KA AD BC
; 
1
8
3
CK AC cm
. Suy ra
2
4 6 8 10IK cm
.
13.2.

..
//AB
AM BD AC BD AC BD
DM AM
AC BC BC BC
..
EN//AC
AN CE ABCE AB AC
AN
AB CB CB BC
. 
AM AN
.
13.3.

M

AC
,
N

MI

AB

AHC

MI

nên
//HCMI

//MN BC
.
 Do
// 1
IB HB
AH CD
ID HC

Do
//
IN AI IM
MN BC
HB AH HC
. 
2
IN HB
IM HC
K
I
F
C
A
B
D
E
N
M
E
D
A
B
C
M
D
I
H
B
A
C
N
3
 
//
IB IN
BN DM
ID IM



BN AC
nên
DM AC

DM

AC
suy ra
DA DC
.
13.4.
a)
BMN

//BM CD

1
MN BN MN ND BN NC MD BC
ND NC ND NC ND NC


MAD

//BN AD

2
AM DM
AB DN
 
AM DM CB
AB DN CN

.

ADI

//AD NC

3
ID IA
IN IC

DIC

DC//AM

4
IM IA
ID IC
 
2
.
ID IM
ID IM IN
IN ID
13.5.
a)
//BD AC
suy ra
AH AC AH AC AH AC
BH BD AH BN BD AC AB BD AC
N
M
D
B
C
A
I
K
H
E
D
A
B
C
4

.
1
AB AC
BD AB AH
AB AC
AB//CE
suy ra
AK AB AK AB AK AB
KC CE AK KC BD EC AC BD EC

.
1
AB AC
CE AC AK
AB AC
 
AH AK
.
b)
// 3 , // 4
AH AC CK CE
BD AC CE AB
BH BD AK AB

,AC CE BD AB
.
 
2
.
AH CK
AH BH CK
BH AK
.
13.6 a)
IK KE
IB ED
.
Do
//BK DF

1
IK IG IB IK CD
CD GC CF IB CF

//AK DF

2
KE BE AB
ED EC CF


AB CD
 
IK KE
IB ED

//IE BD
.

BD AC

//IE BD IE AC
.

ACI

,CB AI IE AC
nên
E

AE CG
13.7
F

BG

AC

`A FD


M

CG
sao cho
//DM BG
.

CA CD CF FA CE FD
hay
22CA CD CF CA CF CD
.

G

ABD
nên
2GB GF
.

//MD BG
suy ra
2 2 2
11
EB CA GB CF CD GF CF
ED CD MD CD MD CD

//GF MD
nên
1
GF CF EB CA
MD CD ED CD
13.8
O

,AF CE

G
F
K
B
A
D
C
F
I
M
E
G
F
A
B
C
D
5
//
DI//
OE OA
AE CK
OC OK
OC OF
CF
OI OA



..
OE OE OC OA OF OF
OI OC OI OK OA OK
//
OE OF
EF IK
OI OK

13.9
//N
H AB H BC
BM CM MH CM MH CM CH
BP CN NH CN CN CN CN

//NH AB
suy ra
CH CN CH BC
BC AC CN AC
.

BM CM BC
BP CN AC


M

13.10.
A
T BD T BD

AT AO
nên
.
. 2
ABD
AD BE BD AT S
Suy ra
. . . . 1
AO
AD BE BD AO AD BE AC BD
AC

//OH CF
nên
2
AO OH
AC CF
 
. . . . . .
OF
AD BE AC BD AD BE CF AC BD OH
CF

T

O
hay
AC BD
.
13.11
,xy

,MNPQ AXYZ

,,abc

,,BC CA AB

AH BC

AH h
suy ra
.
. 2
ABC
a h bc S

2 2 2
a b c
.

22
2 2 2 2
22a h a h ah b c bc b c
a h b c
1 1 1 1
1
a h b c
ah bc a h b c

Theo talet
1
x x MN MQ AM MB
a h BC AH AB AB
O
I
K
C
A
B
D
E
F
H
N
A
B
C
M
P
T
H
F
E
O
B
C
A
D
H
Z
X
P
Q
M
B
A
C
N
Y
6
D
M
E
F
A
B
C
Q
P
O
E
G
M
D
F
B
A
C
K
O
E
F
G
M
A
H
C
N
B
1
y y XY ZY BY CY
b c AC AB BC BC
1 1 1 1
2xy
a h b c
 
xy
hay
MN AX
13.12. Qua
A
k ng thng song song vi
BC
, lt ct
BP
CQ
kéo dài ti
E
F
.
Áp dng h qu nh lý Ta-lét ta có:
AF AM AE
CD MD BD
, mà
CD BD
nên
AF AE
.
Áp dng h qu nh lý Ta-lét ta có:
;
AF AQ AE AP
BC QB BC PC
Suy ra:
AP AQ
PQ
PC QB
//
BC
o Ta-lét).
13.13. Gm ca
CG
AB
K
m ca
DF
BC
M
. Ta có
BCK
cân (vì có
BF
vng cao va là phân giác)
F
m ca
CK
.
ACK
,FK FC AD CD
suy ra
DF
ng trung bình
FD
//
.AK
BCK
,FK FC FM
//
BK
suy ra
M
m ca
BC
.
Xét tam giác
DBC
có trung tuyn
DM
,
theo bài toán 13.12. thì
GE
//
BC
, suy ra
.
OE OG
BM MC
BM MC

OE OF
hay
DF

n thng
.GE
13.14. a) Gi
G
m ca
NE
AC
,
H
m
CF
AB
.
nh lý Ta-lét ta có:
NE
//
CH
GE CF
EN FH
NE
//
BM
//
CH
.
GM NB NO
MC BH OC
CN
//
BF
CF BN
FH BH
7
M
K
I
A
B
D
C
P
R
P
M
A
C
B
Q
Suy ra
GE GM
ME
EN MC
//
NC MONE
là hình bình hành.
b) Ta có
BM
//
HC
NE
//
HF
nh lý Ta-lét ta có:
.
..
.
AM AN AM AN AB AN AN AE
AB AC AC AB AH AB AH AF
(1)
Ta có
/ / ; / / .OM NG OB CH
nh lý Ta-lét, ta có
.
. . / /
.
OM ON OM ON NG NC NG
HC
OBOC OC OB NC HC HC
//
NG AN
NG HC
HC AH
.
//
.
AN AE OM ON AE
NE HF
AH AF OBOC AF
(2)
T u phi chng minh.
13.15. T giác
ABKD
/ / ; / /AB DK BK AD
nên
ABKD
là hình bình hành,
suy ra
DK AB
(1)
T giác
ABCI
/ / ; / /AB CI AI BC
nên
ABCI
là hình bình hành,
Suy ra
CI AB
(2)
T (1) và (2) ta có
DK CI DI KC
Áp dnh lý Ta-lét vào
ABM
vi
//AB DI
, ta có
BM AB
MD DI
.
Áp dnh lý Ta-lét vào
CBD
vi
/ / ,KP BD
ta có
BP DK
PC KC
hay
.
BP AB
PC KC
,
AB AB BM BP
DI KC
DI KC MC PC

//MP CD
nh lý Ta-o).
13.16. Trong tam giác
BQR
//CM QR
nên
CM MB
QR QB
(h qu nh lý Ta-lét)
..
QR QA
CM MB MB
QB QP
(do
..
QR QA
QAQB QPQR
QB QP
).
Mt khác, trong tam giác
ACM
//PQ CM
nên
QA AM
QP CM
.
QA
CM MB
QP
nên
.
AM
CM MB
CM
22
.CM MAMB AM
(vì
MA MB
)
CM AM BM
8
Vy tam giác
ABC
vuông ti
C
.
13.17.
a) Gi
BM
CN
ng trung tuyn ca tam giác
ABC
. Gm ca
BG
EP
H
, ca
CG
FP
.T
T
/ / , / / ,HI PF EP CN
nh lý Ta-lét, ta có
1
3
EI EH NG
EF EP NC
Suy ra
1
3
EI EF
 ta có
1
3
FJ EF

1
3
EI EJ FJ EF
b) T
//PE CN
nh lý Ta-lét ta có:
2
.
3
PH CG
PE CN
T
/ / ,PF BM
nh lý Ta-lét ta có
2
,
3
PT BG PH PT
PF BM PE PF

//TH EF
nh lý Ta-o).
Gi
,OK
m ca
PG
HT
EF
. Ta có
.
HO PO OT
EK PK KF
T 
KE KF
u
phi chng minh.
13.18. Gi
I
m cng thng
d
AB
.
Áp dnh lý Ta-lét ta có:
//
DP DE EC NC QC
AB CD
AB BI AI NB AB
O
K
I
J
F
T
E
H
G
N
M
A
B
C
P
I
P
M
Q
E
D
C
A
B
N
9
A
B
C
M
N

DP QC
, theo gi thit
AC AD
ADC
cân ti
A
ADP ACQ ADP ACQ
(c.g.c) suy ra
MAD QAC
13.19. K
//MN AB
(hình v). Ta có:
..
MN MC MC
MN AB
AB BC BC
.
NA MB MB
NA AC
AC BC BC
AM AN NA
(bng thc tam giác).
Hay
..
MC MB
AM AB AC
BC BC
Vy
. . .AM BC MC AB MB AC
.
13.20.
Tam giác vuông
ACK
0
45A
nên là tam giác vuông cân,
CE
ng cao nên
,AE EK IE
ng
trung tuyn ca
AIK
.
Ta s chng minh
2IG GE
(bng cách chng minh
2FI EH
).
Ta có
2FI CF
(vì
CIF
vuông cân),
CF BH
(vì
BFCH
là hình bình hành).
2BH EH
(vì
BEH
vuông cân) nên
2FI EH
. Do
//EH FI
nh lý Ta-lét ta có
2
IG FI
GE EH
suy ra
2.IG GE
Vy
G
là trng tâm
.AIK
13. 21. Qua
A
C
k ng thng song song vng thng
d
, cng thng
BG
lt ti
A
C
.
Áp dng ví d 4, ta có:
3; 3
AB AC AC BC
AM AN CN CP
(1)
MN
ct tia
CB
ti
P
 cách chng minh ví d 4, ta có:
;3
BA BA BA BC BA BC
BM BG BP BG BM BP
(2)
G
F
H
K
I
D
E
A
C
B
10
T (1) và (2) suy ra:
9
AB AC AC BC AB BC
AM AN CN CP BM BP
( ) ( ) ( )
9
. . .
AB AM MB AC AN NC BC CP BP
AM BM AN CN BPCP
2 2 2
9
. . .
AB AC BC
AM BM AN CN BP CP
u phi chng minh).
Nhn xét: Da vào bi toán trên ta ging thng
d
ng tâm
G
ca tam giác
u
ABC
, cnh
a
, ct cnh
AB
ti
M
, cnh
AC
ti
N
, tia
CB
ti
P
. Chng minh rng
2
1 1 1 9
.
. . .AM BM AN CN BPCP a
13.22 Gi EF ct AB, Ac tnh lý Ta Lét, ta có:
MP IB
(1)
MQ IC
ME IC
(2)
MP BC
MQ BC
(3)
MF BI
T (1), (2) và (3) nhân v vi nhau
c:
MP ME MQ IB IC BC
. . . .
MQ MP MF IC BC IB
ME
1
MF

Hay ME=MF
C'
A'
P
N
G
A
B
C
M
B
P
K
M
C
I
Q
F
J
A
E
11
 14. TÍNH CHNG PHÂN GIÁC CA TAM GIÁC
14.1. Ta có: 2AAAB=3AC suy ra
AB 3
AC 2
ng phân giác ca góc BAC nên
BD 3 BD 3
BD CD 3 2 10 5


Suy ra:
3.10
BD 6(cm);
6

CD=4 (cm)
14.2. Áp dng tính chng phân giác vào các tam giác ABC, ABI, AIC:
BI AB AN AI CM IC
;;
IC AC NB BI MA AI
BI AN CM AB AI IC AB IC
. . . . . 1
IC NB MA AC BI AI AC BI
14.3 Xét
ABC
ng phân giác ca
ABC
nên:
AM AB AB 1 1
AB BC
MC BC BC 2 2
 
Gng phân giác ca
ACB
, suy
ra:
NA AC AC 3 3
AC .BC
NB BC BC 4 4

Ta có: AB+BC+AC=18 Suy ra
BC 3
BC BC 18
24
9
.BC 18 BC 18(cm)
4
 
T c AB= 4 (cm); AC = 6 (cm)
14.4: Ta có:
C
D
B
A
N
M
C
D
B
A
B
C
M
N
A
12
0
11
AIE BAH ABI (A B) 45 C AEI
22
Suy ra
AIE
cân ti A => AI=AE (1)
Áp dng tính chng phân giác ca
ABH
BAC
, ta có:
IH BH AB BH
(2)
IA BA AI IH
EC BC AB BC
(3)
EA BA AE EC


T (2) và (3) suy ra:
BH BC
(4)
IH EC
ABC
vuông cân ti A nên BC=2.BH
T t hp vi (4), suy ra EC = 2.IH
14.5.
a) Ta có: CN//DM, CN=DM và
0
NCD 90
nên CDMN là hình ch nht => MN//CD.
Gm ca AC và MN.
AOM và
CON có AM=CN;
0
AMO CNO 90 ;MAO NCO
AMO CNO(g c g) MO ON
 
Áp dng h qu nh lý Ta- lét, ta có
MO//CP=>
MO QO NO QO
;NO/ /CR
CP QC CR QC

Suy ra
NO MO
CR CP
mà MO=NO suy ra CR=CP
NRP
NC PR,CR CP
Nên
NRP cân
b) MN//RP nên
QNM NRP;MNP NPR
NRP NPR QNM MNP 
=>MN là tia phân giác
QNP
Ta có: NS nh N ca PNQ.
Áp dng tính chng phân giác trong và ngoài ca NPQ, ta có:
;
H
I
E
C
A
B
13
14.6. Theo tính chng phân giác ca ABC, ta có:
, ta có:
Mt khác: (1)
: : (2)
(3)
T (1), (2) và (3) ta có:
14.7. Gi D, M lm ca AI, AG vi BC.
Áp dng tính cht ng phân giác trong tam giác ABD, ta có:
O
R
C
N
D
P
M
B
Q
A
S
C
M
N
P
B
A
14
Mt khác G là trng tâm
󰇡 󰇢 nh lý Ta-o)
14.8. Gm cu ng thng BN và CD
BM//DE nên
Mà BM = DN nên (1)
Ta có: DN//BC nên (2)
T (1) và (2) suy ra: ng phân giác 


 
 cân ti D


 
cân ti B
14.9. K   nên MI//AH
Mt khác MA = MC nên HI = CI
Áp dng tính chnh lý Ta-lét, ta có:
(li gii khác, các bn xem  nh lý Cê-va)
t BC = x; CA = y; AB = z
Theo tính chng phân giác ca , ta có:
D
A
C
G
I
B
M
C
B
M
K
D
E
A
O
N
I
B
D
O
C
I
H
M
A
15
AO
là phân giác
BAD
nên
(2)
OB AB OB AB
OD DA OB OD AB DA

T (1) và (2) suy ra:
OB x z
BD x y z


OC x z
CE x y z

. T 
2
2
2 2 2
..
. . 2 2 2
x y x z
OBOC OBOC x xy xz yz
BDCE BDCE x y z xy yz xz
x y z


2 2 2
y z x
nên
2
2
.1
.2
2
OBOC x xy xz yz
BDCE
x xy yz xz


Hay
. 2. . (3)BDCE OBOC
 ý rn nu k
BH OC
, mt khác d thy
0
135BOC
, nên
BHO
vuông cân ti
H

12
..
24
HOC
S BH OC OBOC
, suy ra
. 2 2 (4)OBOC a
T (3) và (4) suy ra:
. 4 2BDCE a
.
14.11. Trên
AE
lm
N
sao cho
MN BC
T gi thit
EAC ECA EAC
cân ti
(1)E AE EC
 thit
2.AEB EAC ECA ECA EAB BAE
cân ti
B MAN
cân ti
M
(vì
MN BE
)
(2)AM NM
Vy ta có
LM NM
LC EC
(vì
MN EC
)
AM
AE
(theo (1) và (2))
KM
KE
(theo tính chng phân giác) suy
ra
KL BC
nh lý Ta o).
14.12.
BCM
CD
ng phân giác nên
2 2.
BC BD
BC CM
CM MD
i ca tia
MC
lm
P
sao cho
MC MP
suy ra
2.CP CM
CP BC CBP
cân ti
C
, mà
CD
là phân giác nên
(1)CD BP
.
16
Mt khác:
.
. ,CMA PMB c g c

CAM PBM
suy ra
(2)AC BP
T (1) và (2), ta có:
CD AC
hay
ACD
vuông
ti
C
.
17
 15. NG HNG DNG CA TAM GIÁC
15.1.
a) Xét
ABE
ACF
0
90AEB AFC
;
BAC
chung
( . )ABE ACF g g
..
AB AE
AE AC AF AB
AC AF
b) T
..
AE AF
AE AC AF AB
AB AC
Xét
AEF
ABC
AE AF
AB AC
;
BAC
chung
( . . )AEF ABC c g c
.
c) Ch, ta có:
AEF ABC AEF ABC
Chc:
( . )CAB CDE g g ABC CED
T 
AEF CED EB
là tia phân giác
DEF
.
Ch, ta
DA
là tia phân giác
EDF
. T u phi chng minh.
15.2.
CBH
CDK
có:
0
90 ,
( . )
CHB CKD HBC KDC BCD
CH CK
CBH CDK g g
CB CD
CD AB
nên
CH CK
CB CD
.
CHK
BCA
CH CK
CB AB
ABC HCK
(cùng bù vi
BAD
) suy ra
( . . )CHK BCA c g c
.
15.3.
IAB
DCB
;ABI CBD IAB DCB
(hai góc cùng ph vi
ABC
)
AB BI
IAB DCB
BC BD
.
ABC
BD
ng phân giác nên:
AB AD
BC DC
.

..
BI AD
AD BD BI DC
BD DC
.
18
15.4.
Ta có:
CDB A
(tính ch
trên cnh
BC
ly
E
sao cho
CDE A
.
ACD
DCE
có:
12
;C C A CDE
( . )
AC CD
ACD DCE g g
CD CE
2
..CD AC CE AC BC
.
15.5.
Ta có:
AEC BDC
0
60DBC EBC
0
60DBC ACB
nên
AC BD
.
Suy ra:
( . )ACF BDC AEC AEC ACF g g
AC AE AB AE AB AE
AF AC AF AB AB AF AB AE

1
AB AE AB AB
BF BE BF BE
1 1 1
1
AB AB
BF BE BF BE AB
1 1 1
BD BF BC
u phi chng minh.
15.6.
T gi thit suy ra
C
là trc tâm
AEF
nên
AC EF
.
Kt hp vi
BD AM
ED AF
theo tính cht góc có c ng vuông góc ta có:
;ICD MFA CDI MAF
ICD MFA
(1)
IC MF
ID MA


( . ) (2)
IC ME
ICB MEA g g
IB MA
T (1) và (2) kt hp vi gi thit
IB ID
suy ra
ME MF
.
15.7. a)
19
Trong tam giác
BDM
ta có:
0
11
120DM
0
2
60M
nên ta có:
0
11
120MM
Suy ra
13
DM
0
60BC

(1)BMD CEM
Suy ra
BD CM
BM CE
, t 
..BDCE BM CM
2
BC
BM CM
, nên ta có:
2
.
4
BC
BDCE
b) T (1) suy ra:
BD MD
CM EM
BM CM
nên ta có:
BD MD
BM EM

BMD MED
.
T 
12
DD

DM
là tia phân giác ca góc
BDE
.
Ch ta
EM
là tia phân giác ca góc
CED
.
c) Gi
là hình chiu ca
M
trên
,,AB DE AC
.
Theo tính chng phân giác, ta có:
,DH DI EI EK AH AK
.
T  
ADE
bng:
2AD DE EA AD DH EK EA AH
Vy chu vi tam giác
ADE
i.
15.8. a)
Xét
DHC
NHB
có:
0
90DHC NHB CHN
;
0
90HCD HBC BCH
Suy ra:
( . )DHC NHB g g
.
b)
Xét
BCH
có:
00
90 ; 90MHB BHC MBH HCB CBH
Suy ra
( . )MBH BCH g g
(1)
MB HB
BC HC

( . )DHC NHB g g
(2)
NB HB
DC HC

BC CD
nên t (1) và (2), suy ra:
MB NB AM CN
, suy ra
..AM NB NC MB
.
20
15.9. i ca tia AC lm D sao cho AD = AB.
T 
BAC=2BDA
nên
BDC=2ABC.
T 
2
ABC ΔBDC .
AC BC
BC DC AC
BC DC
2 2 2 2 2
3 4.BC AC BC AC AC
Nên
2 2 2
AB BC AC
. Vy
ABC
là tam giác vuông ti C.
15.10. a) Ta có
0
BIA=MCE 90 1 .IBH
Li có
00
IAB+ 180 ;CME+ 180BAH EMB
0
BAH=EMB 90 2ABC IAB CME
T (1) và (2) suy ra
A
IB ΔMCE . .gg
b)
AK ΔMEAM
0
9
0 ,MKA MAE AME
chung
2
AK ΔMEA . . 3
MA MK
M g g MA ME MK
ME MA
:
2
AL ΔMFA . . 3
MA ML
M g g MA MF ML
MF MA
T (3) và (4) suy ra
. .
ME ML
ME MK MF ML
MF MK
Ta có:
.
.
AMB
AMC
S
MB AB MK
MC S AC ML

Mt khác:
ME ML MK MF
MF MK ML ME
E
L
A
K
B
H
M
C
F
I
21
Suy ra
..
5
..
MB AB MF MB AB MC
MC AC ME MF AC ME
Mt khác
AIB ΔMCE
, suy ra
MC AI
ME AB
(6)
T (4) và (5) suy ra
.AI
.AB
MB AB AI
MF AC AC

c)
BF ΔAICM
MB MF
AI AC
..I
AC BMF MBF AIC c g c AIC MBF
00
90 90AIC ICB AI BC MBF ICB
hay BF vuông góc vi CI.
u phi chng minh.
15.11. Ta có:
A
DC ΔBEC .gg
suy ra:
22
1
2
1
1
2
CB
CA CD CB
CA CB CA CB
CB CE CA
CA
2.CA CD
Mt khác
00
30 60 2DAC C
T  u.
15.12. a) Ta có :
00
11
180 180
22
AB
APB A B
00
0
360 180
90
2 2 2
A B C C
Xét
CMB
1
M MBC MCP
0
11
90 .
2
C
M APB M
ΔAMPAPB
11
; APB ΔAMP
A
APB M A A
2
. . 1
AM AP
AM AB AP
AP AB
, ta có
A
PB ΔPNB .gg
2
. . 2
BN BP
BN AB BP
BP AB
T (1) và (2) suy ra
2
,
AM AP
BN BP



u phi chng minh.
b) xét
AMP ΔAPB
(chng minh trên);
APB ΔPNB
(chng minh trên);
B
D
C
E
A
2
1
2
1
P
B
N
C
M
A
22
AMP ΔPNB . . MP
AM MP
AM BN PN
PN BN
Hay
2
..AM BN MP
2
..AM BN MP
CMN
ng cao nên
CMN
cân ti C
;.CM CN MP PN
2 2 2
2
2
. . . .
. . .
..
..
. . .
AM BC BN AC CP AM BC BN AC CM MP
AM BC BN AC CM AM BN
AM BC BN BN AC CM
CM AM CM BN AC
CM AC BN AC AC CM BN AC B
C
t

2
. . .AM BC BN AC CP AC BC
Suy ra
2
1,
.
AM BM CP
AC BC AC BC
u phi chng minh.
15.13. a)
ΔCABCDE
0
= CAB 90 , CDE DCE chung
Suy ra
ΔCAB .
CD CA
CDE g g
CE CB
Xét
ΔBECADC
,,
CD CA
ACB
C
chung
E CB
(chng minh trên)

Δ
BEC . .ADC c g c
Suy ra
0
BEC= 135ADC
( vì tam giác AHD vuông cân ti H theo gi thit)
Nên
0
45AEB
i A.
Suy ra
2 2.BE AB m
b) Ta có
11
. .
22
BM BE AD
do BEC ADC
BC BC AC
M
B
H
O
D
C
E
A
23
22AD AH m
( tam giác AHD vuông cân ti H)
nên
1 1 2
. . =
22
2
BM AD AH BH BH
do ABH CBA
BC AC AC BE
AB
Xét
Δ EC BBHM
BM BH
BC BE
cCBE hung

Δ
BEC . .cBHM c g
suy ra
00
= BEC 13 4 55BHM AHM
c) Ta có AG còn là phân giác góc BAC
GB AB
GC AC
ΔDEC //
AB ED AH HD
ABC ED AH
AC DC HC HC
GB HD GB HD GB HD
GC HC GB GC HD HC BC AH HC
15.14.
t
, , .AFE BFD BDF CDE CED AEF
Ta có
0
180 *BAC

Gng phân giác ca tam giác DEF, suy ra OD, OE, OF lt vuông góc vi BC,
AC, AB.
0
OF O ODF 90 1E ED
Ta có
0
270 2OFD OED ODF
0
1 2 180 **
*
** BA B C DF
Ch câu a), ta có
;BC
AEF BDF AEF ABC


Suy ra
O
F
B
D
C
E
A
24
5 5 5
8 8 8
7 7 7
8 8 8
5 7 5 24
7 7 5 5
7
3 3
BD BA BF BF
BD BD
BF BC
CD CA CE CE
CD CD
CF CB
AE AB CE BF
CE BF
AF AC
CD BD


Ta li có: CD + BD = 8 (4)
T
2 5. 4 ,3 Bv Dà 
15.15. K t
M
ng thng song song vi các cnh
AB
,
BC
ct các cnh ti
E
,
F
,
G
,
H
(hình v).
Ta có:
AGM CFM ABC
.
Mt khác
MAB MCB

GGM CFM
AG MG
CF MF

mt khác
AG DH
;
CF MH
;
MG FB
nên
DH BF
MH MF
(1)
Ta li có
DHM BFM BCD
(2)
T (1) và (2) suy ra
DHM BFM
MDC MBC
.
15.16. V phía ngoài
ABC
v
BCE
vuông cân ti C
90ADB ACE ACB
AD BD
AC BC
(vì
..AC BD AD BC
)
AD BD
AC CE


ABD AEC
(c.g.c) (1)
12
AA
BAE DAC
T (1)
AB AE
AD AC


ABE ADC
(c.g.c)
..
AB BE
AB CD AD BE
AD DC
.
Mt khác
ABE
vuông cân nên
2.BE BC
.

. 2. .AB CD AD BC
hay
.
2
.
AB CD
AD BC
.
15.17. Ly
N
trên
PQ
sao cho
MN BC
.
Ta có:
PBE PMN
(cùng ph vi
PMB
)
BPE MPN
(cùng ph vi
EPM
)
nên
PBE PMN
(g.g)
BE BP
MN MP

.
BP
BE MN
MP

(1)
H
G
F
E
M
C
A
B
D
2
1
B
C
A
D
E
N
F
E
P
Q
B
C
A
M
25
, ta có:
.
CQ
CF MN
MQ
(2)
Mt khác
BPN CQM
(g.g)
BP CQ
MP MQ

(3)
T (1), (2) và (3) suy ra
BE CF
.
15.18. Gi
H
  m ca
BE
CF
. Gi
I
  m ca
AH
PQ
.Ta có:
90ABQ ACP BAC
;
BAQ PAC
suy ra
ABQ ACP
(g.g)
AQ AB
AP AC
AQ AP
AB AC

. Mt khác
APHQ
là hình bình hành nên
AP HQ
AQ HQ
AB AC

.
Ta li có:
180BAC AQH PAQ
suy ra
ABC QAH
(c.g.c)
ABC QAH
;
AB BC BM
QA AH AI

(vì
2.BC BM
,
2.AH AI
).

ABM QAI
(c.g.c)
BAM AQI
180QAM AQI
AM PQ
.
15.19. Gi
P
m ca
AB
DE
;
Q
m
ca
BD
CE
.
DEC
D
C DE DB
60 80 140EDC
nên
1
180 20
2
DEC DCE EDC
.
Ta có
ABD DBC ABC
nên
20ABD
.
BDP
EDQ
20 DEQ DBP
;
BD ED
;
EDB
chung
BDP EDQ
(c.g.c)
EQ BP
;
PD DQ
.
BPD
ABC
80 PDB ABC
;
20 DBA BAC
BPD ABC
(g.g)
AB BC BD ED
BP PD PD PD
hay
AB ED
BP PD
// AE BD
nh lý Ta -o)
EAP PBD
(so le trong)
20 40 EAP EAC
.
Mt khác
40 ACE ACD DCE
EAC ACE
ACE
cân ti
E
.
15.20. Gi
K
i xng vi
B
qua
A
. Gi
M
m ca
BD
CK
.
BCK
CA
ng trung tuyn (
AB AK
),
2. CD AD
nên
D
là trng tâm tam giác
MC MK
.
BCK
AK AB
,
MC MK
nên
AM
ng trung bình
//AM BC
AMB EBC
ABC DEC
ABM ABC MBC DEC EBC ECB
.
K
I
M
H
Q
P
F
E
B
C
A
Q
P
E
D
A
B
C
26
AMB
EBC
AMB EBC
,
ABM ECB
AMB EBC
(g.g)

BC BE
MB AM
.
Ta có
AB AK
,
AC AF
BK CF
nên
BCKF
là hình thoi
BC CK
AM MC
.
BF BC BE BE
MB MB AM MC

BF BE
MB MC
EBF CMB
EBF CMB
(c.g.c)
BEF MCB
kt hp vi
BCKF
là hình thoi nên
180 180 2. DEF BEF MCB FBC ABC
hay
2.DEF ABC
.
M
K
F
E
D
B
C
A
27
 16. CÁC NG HNG DNG CA TAM GIÁC VUÔNG
16.1. Ta có
90ABC BAH AHB BAH
90ABC ABC
ACH BAH
.
T 
ABH CAH
(g.g)
AH BH
CH AH

2
.AH BH CH
16.2. K
HI BC
ti
I
.
BIH
DBC
90BIH BDC
DBC
  
BIH BDC
(g.g)
..
BH BI
BH BD BI BC
BC BD
(1)
 ta có
CIH CEB
(g.g)
CH CI
CB CE

..CH CE BC CI
(2)
T (1) và (2) cng tng v ta có:
2
. . . .BH BD CH CE BI BC BC CI BC BI CI BC
.
16.3. Ta có:
90BAD BCH ABC
90C
DH ADB
.
Suy ra:
CDH ADB
(g.g) nên
CD DH
AD DB
.
Ta li có
CD DB
nên
2
.CD DA DH
.
16.4. Qua
O
k ng thng song song vi
AD
, cng thng
BI
,
CK
lt ti
E
,
F
OE BI
,
OF CK
.
Xét
BEO
AIB
có:
BEO AIB
;
90ABI BOE OBI
BEO AIB
(g.g)
BO EO
AB IB

(1)
Ch, ta có:
CFO DKC
(g.g)
CO OF
CD CK

(2)
Xét
AOB
DOC
AOB DOC
;
ABO DCO
AOB DOC
(g.g)
BO OC
AB CD

(3)
H
A
C
B
I
H
E
D
B
C
A
H
D
B
C
A
F
E
O
M
K
I
A
D
B
C
28
T (1), (2) và (3) suy ra
EO OF
IB CK
OE IB
OF CK

(4)
Ta có:
//BI CK
nên
IB BM
CK MK
(5)
Ta có:
BEO NFO
(g.g)
OE BO
OF ON

(6)
T (5) và (6) suy ra
BM BO
MK ON

OM // NK
nh lý Ta-o) hay
OM AD.
16.5. Gng cao ca ABC, AH ct PQ ti I.
t
BC a; AH h; PQ x; MQ y
Ta có:
AI h y
APQ ACB
nên
PQ AI x h y a(h y)
x
BC AH a h h

MNPQ
a
S xy h y y
h
Vì a, b là các hng s n nht khi
(h y)y
ln
nht. Áp dng h thc:
2
ab
ab ,
2



ta có:
2
22
MNPQ
h y y h a h ah
h y y S . .
2 4 h 4 4




Vy giá tr ln nht ca S là
ah
4
khi
h
h y y y
2
tc là P, Q lt là trung
m ca AC, AB.
16.6. Ly U, V theo th t thuc AK, AL sao cho
0
ABU ACV 90 ,
ta có:
BU BK
NA // BU (1)
NA NK

NA CA
MN // BC (2)
MA BA

MA ML
MA // CV (3)
CV CL

T (1), (2) và (3) suy ra
BU NA MA BK CA ML
. . . .
NA MA CV NK BA CL
BU BQ CA MN BQ.CA BQ CA NP
. . . .
CV NM BA CP BA.CP MQ BA CP
(vì
MQ NP
)
BU BA CA BA
..
CV CA BA CA

(vì
BMQ BCA, CNP CBA) ∽∽
29
Hay
BU AB
CV AC
0
A
BU ACV 90

ABU ACV (c.g.c)
Vy
KAB LAC.
16.7.
Ta có:
0
ADE EDG BDG 180 ,
0
EDG 90
nên
0
ADE BDG 90
Mt khác, ta li có:
0
ADE AED 90
nên
BDG AED
BGD DAE(g.g) (1)
Ch, ta có:
EFC DAE(g.g) (2)
T (1) và (2) suy ra:
BG EF
BGD EFC (3)
DG FC
S dnh lý Ta-lét trong
BHG,
ta có:
HG BG
DE // BG
HD DE

.
DE DG
(tính cht hình vuông) nên
HG BG
(4)
HD DG
, ta có:
IE DE EF
(5)
EF FC FC

T (3), (4) và (5) ta có:
HG IE
HD IF
, suy ra:
HG IE
HG HD IE IF

hay
HG IE
DG EF
. Mà
DG EF
nên ta có
HG IE.
16.8.
ED EF
nên
GED EFG
SS
AF 2.EF
nên
GAF EFG
S 2.S .
Ta li có
GBC GEF
nên
2
GBC
GBC EFG
EFG
S
BC
S 16S
S EF




E
FG GED GAF GBC EFG EFG
S S S S 1 1 2 16 .S 20.S
EFG GED GAF GBC ABCD
1
S S S S .S
2
Vy
EFG
EFG ABCD
ABCD
S
11
S .S .
40 S 40
16.9. Ta có:
EM AE 1
AME CMD DM 2.EM.
DM DC 2
t
AEM
S x.
Ta có:
AEM
ADM
ADM
S
EM 1
S 2x.
S DM 2
Ta có:
2
AEM ADM ADE ABD ABCD
11
S S S S S x 2x 37,5cm
24
30
22
AMD
x 12,5cm S 25cm
, ta có:
22
CNE CND
S 12,5cm ; S 25cm .
2
DMN ACD AMD CND
S S S S 75 25 25 25cm .
din tích phm là:
2
12,5 12,5 25 50cm .
16.10. Các tam giác AHB và CHA có chung chiu cao k t A
nên
AHB
CHA
S
HB
(1).
HC S
Ta li có
AHB CHA (g.g)
nên
22
AHB
CHA
S
AB 2 4
(2).
S AC 3 9
T (1) và (2) suy ra:
HB 4
.
HC 9
16.11. D thy
CH CE
CHE CAF (g.g) .
CA CF

HBC
ABC
1
HB.CE
S
HB.HC
2
.
1
AB.AC S
AB.CF
2

 ta có:
HAC
HAB
ABC ABC
S
S
HC.HA HA.HB
;.
BC.BA S CA.CB S

T 
HBC HCA HBC
ABC
S S S
HB.HC HC.HA HA.HB
1.
AB.AC BC.BA CA.CB S

16.12. Cách 1. V m H thun GC. Gi K là giao
m ca AH và DF. Ta có:
AB 9 3
EF 6 2

CE 2.BE.
Vì hai tam giác ABC và CDE có din tích bng nhau nên hai hình
bình hành ABCH và DECG có din tích bng nhau.

CH 2.HG.
Suy ra:
DE GC 9 4,5 13,5cm
DF DE EF 13,5 6 7,5cm
.
Cách 2. K ng cao CI ca
ABC,
CI và EF ti J. Ta có:
CJ EF 6 2
.
CI AB 9 3
Hai tam giác ABC và CDE có din tích bng nhau nên
AB.CI DE.CJ
AB CJ AB 2 3 3
DE .AB .9 13,5cm.
DE CI DE 3 2 2
Suy ra:
DF DE EF 13,5 6 7,5cm
16.13. Ta có:
CBQ DCE (c.g.c) BCQ CDE
0
CDE CED 90
nên
0
BCQ CED 90

0
EMC 90 .
Vng dng
DC DM MC
CE MC ME
DC 2.CE
DM 2MC; MC 2ME DM 4.ME
31
EI // AD
nên
AM DM
4 AM 4.MI.
MI ME
16.14.
* Chứng minh điều kin cn. ng phân giác trong ca tam
giác ABC ta cn chng minh:
2
DEF
ABC
S
11
S 2 4




Do tam giác
ABC
u và
,,AD BE CF
ng phân giác
ca tam giác nên ta có
1
2
DE EF DF
DEF
AB BC AC
ABC 
22
11
24
DEF
ABC
S
DE
S AB
* Chu ki. Cho tam giác
ABC
,
,,AD BE CF
ng phân giác ca tam giác, tha
mãn
1
4
DEF
ABC
S
S
, ta cn chng minh
ABC
u.
t
,
, , , 0BC a AC b AB c a b c
AD
ng phân giác ca
BAC
nên ta có:
DB c DB c ac
DB
DC b DB DC c b c b
ac ab
DC a DB a
c b c b

Ch, ta có:
; ; ; .
ab bc bc ca
EC EA FA FB
a c a c a b a b
Ta có
1
ABC AEF BDF CDE CDE
DEF AEF BDF
ABC ABC ABC ABC ABC
S S S S S
S S S
S S S S S
. . .
1
. . .
1
( ).( ) ( ).( ) ( ).( )
2
( )( )( )
AF AE BF BD CE CD
AB AC BA BC CACB
bc ab ac
a b a c a c b c a c b c
abc
a b b c c a
Theo gi thit ta có :
21
( )( )( ) 4
abc
a b b c c a
222
( )( )( ) 8 ( ) ( ) ( ) 0a b b c c a abc a b c b c a c b a
a b c ABC
u.
A
B
C
D
E
F
32
16.15.
Gm
AH
CM
I
. Gi
K
m ca
BH
, thì ta có
//AHMK
.
D thy ba tam giác vuông
,,AMC IAC IMA
ng dng mà
2.AC AM
nên
2. 4.IC IA AM
Suy ra:
1 2. 1
42
HK IM BH HK
HC IC HC HC
.
16.16. Ta có:
ABH CAH
nên t s chi vi bng t s ng dng, suy ra:
30 3
40 4 3 4
AH AH AH HC
HC HC
t
0
34
AH HC
kk
Áp dnh lý Py-ta-go, ta có:
2 2 2
5AH HC AC AC k
Mà chu vi
CAH
40 cm
nên
10
3 4 5 40
3
k k k k cm
.
Suy ra
40 50
10 , ,
33
AH cm HC cm AC cm
Ta có
ABC HAC
nên t s chu vi bng t s ng dng, suy ra:
50/ 3 5 5
.40 50
40/ 3 4 4
ABC
ABC
HAC
C
AC
C cm
C HC
16.17. Nhn xét rng t s chu vi cng dng bng t s ng dng. T s din tích ca hai
ng dng b s ng dng nên:
2
' ' ' ' ' '
50 25
' ' '
60 36
A B C A B C
ABC ABC
SS
A B C ABC
SS



I
K
H
M
B
A
C
A
H
B
C
33
2
' ' ' ' ' '
' ' '
' ' '
25 25
75
36 25 33 11
A B C A B C
A B C
ABC A B C
SS
S cm
SS

2
33 75 108
ABC
S cm
.
16.18. m
M
trên cnh
BC
v ng thng song song
vi
AB
ct
AC
ti
E
, v ng thng song song vi
AC
ct
AB
ti
D
.
Ta có
DBM ABC EMC ∽∽
22
;
EMC
DBM
ABC ABC
S
S
BM CM
S BC S BC
Ta có:
2 2 2
11
1 1 1
22
EMC
MDAE DBM
ABC ABC ABC
S
SS
BM CM BM CM
S S S BC BC BC BC




(áp dng bng thi s:
2
22
1
.
22
MDAE ABC
xy
x y S S
)
Vy khi
M
m ca
BC
thì hình bình hành
AEMD
có din tích ln nht là:
1
.
2
ABC
S
.
E
D
A
B
C
M
34
 17. NH LÝ CE-NH LÝ VAN-OBEN
17.1. t
1
2
BD CE
DC EA

suy ra
12
; 2;
33
BD CD AE
BC DB AC
.
Áp dnh lý Menelaus trong
ABC
vm
,,B O E
thng hàng ta có:
11
. . 1 . . 1 6
32
AO BD CE AO AO
OD BC EA OD OD
Áp dnh lý Menelaus trong
BEC
vm
,,A O D
thng hàng ta có:
2 2 3
. . 1 . .
3 1 4
BO AE CD BO BO
OE AC DB OE OE
.
17.2. Tam giác
ABC
,,AH CD BM
ng quy ti
O
.
nh lý Ce-va, ta có:
. . 1
BH CM AD
HC MA DB
1
CM
MA
BD BC
AD AC
(tính chng phân giác)
Suy ra
.1. 1
BH AC BC BH
HC BC AC CH
(cách gii khác, bc xem
 ng phân giác)
17.3. Áp dnh lý Ce-ng quy
,,AH BM CD
, ta có:
. . 1
AD BH CM
BD CH AM
AM CM
Nên
.1
AD BH AD CH
BD CH BD BH
Mt khác,
CD
ng phân giác nên:
AD AC b
BD BC a

suy ra
CH b
BH a
hay
..aCH b BH
(1)
Áp dnh lý Py-ta-go cho các tam giác vuông ta có:
2 2 2 2
2. .a BC HB HC HB HC
2 2 2 2
b AC HA HC
2 2 2 2
c AB HA HB
T 
2 2 2
2 . 2 . . 2 . . .a b a b c a b aCH a a HC b HC a b BH b HC
(theo (1))
D
O
H
M
B
A
C
D
O
H
A
B
C
M
O
D
A
B
C
E
35
2
2 . 2a ab a b
17.4. Cách 1: (không dùng Menelaus)
Ta gii vt t
T
//AD FM
//ME AD
Áp dnh lý Ta-lét, ta có:
BA BF
BD BM
(1) và
CE CA
CM CD
(2)
Mt khác, theo tính chng phân giác ta có:
BA CA
BD CD
(3)
T (1), (2), (3) suy ra:
BF CE
BM CM

B
F CE doBM CM
Cách 2: (dùng Menelaus)
Xét tam giác
ABC
vm
,,F E M
thng hàng, ta có:
. . 1
EA MC FB
EC MB FA
(4)
Do
2
BAC
AEF AFE
nên
AEF
cân
A
, Suy ra
AE AF
(5)
T (4), (5) suy ra
BF CE
u phi chng minh)
17.5. Xét
. . .
AE BM CF AE CF
EB MC FA EB FA
(1)
Nu
,,AM BF CE
nh
lý Ce-va:
. . 1
AE BM CF
EB MC FA
.
T (1) suy ra:
. 1 //
AE CF AE AF
EF BC
EB FA EB CF
nh lý
Ta-o).
Nu
//
AE AF
EF BC
BE CF

. T (1) suy ra:
. . . 1
AE BM CF AE CF
EB MC FA EB FA

,,AM BF CE
nh lý Ce-o)
E
A
B
C
M
F
E
F
A
B
C
D
M
36
17.6. Gi
E
m cng thng
BK
AC
. Áp dnh lý Menelaus trong
ACD
i ba
m
thng hàng, ta có:
. . 1
AK BD CE
KD BC EA
12
3. . 1
23
CE CE
EA EA
.
Mt khác
ABE
BCE
ng cao k t
B
, suy ra:
3
2
ABE ABE
BCE BCE
SS
AE
S CE S
17.7. Áp dnh lý Menelaus trong
AEC
vi
m
,,M D B
thng hàng, ta có:
. . 1
MA DC BE
MC DE BA
Áp dnh lý Menelaus trong
ΔABC
vi ba
m
N, I, E
thng hàng, t có:
NA IC EB
. . 1
NC IB EA
suy ra
MA DC BE NA IC EB
. . . .
MC DE BA NC IB EA

MA NA IC DE AB
. . .
MC NC IB DC AE
(1)
Áp dnh lý Menelaus trong
ΔBEC
vi
I, D, A
thng hàng, nên
IC AB DE
. . 1
IB AE DC
(2)
T (1) và (2) suy ra:
MA NA
MC NC
17.8. Áp dnh lý Ce-va trong
ΔABC
vng thng quy
AE, BF, CK,
ta có:
EB FC KA
. . 1
EC FA KB
(1)
Áp dnh lí Menelaus trong
ΔABC
vm thng
hàng
T, K, F,
ta có:
TC KB FA
. . 1
TB KA FC
(2)
T (1) và (2) nhân v vi v c:
TB EB
TC EC
17.9. Gi
BC
giao vi
AD
ti
G
Áp dnh lý Menelaus trong
ΔABM, ΔAMC
ta
c:
K
A
B
C
D
E
M
N
B
I
D
E
C
A
37
DM FA HB
. . 1 (1)
DA FB HM
DM EA KC
. . 1 (2)
DA EC KM
c:
EC FA KC HM
. . (3)
EA FB KM HB
AG,BE,CF
ng quy
GB EC FA EC FA GC
. . 1 . (4)
GC EA FB EA FB GB
T (3) và (4):
GC KC HM GB KC HM
. . . 1
GB KM HB GC KM HB
u phi chng minh.
17.10. Áp dng t s din tích hai tam giác có
chung c
HBC HCA
HAB
ABC ABC ABC
SS
HD HM HN S
1
AD BM CN S S S
Áp dnh lý Ce-va, ta có:
DB MC NA
. . 1
DC MA NB
.
T u phi chng minh.
17.11. a) Áp dng h qu nh lý Ta-lét, ta có:
MI AI
MI//BD
BD AD
IN AI
IN//CD
CD AD
MI IN MI DB
BD CD NI DC


b) Gm cng thng
BI
AC
;
F
là giao
m cng thng
CI
AB
Ch câu a, ta có:
IP AF IR CE
;.
IQ BF IS AE

Áp dnh lý Ce-va trong
ΔABC
i vi
AD, BE, CF
ng quy, ta có:
BD CE AF IM IP IR
. . 1 . . 1
CD AE BF IN IQ IS
u phi chng
minh.
17.12. Ta có:
BEC A ACE KCB KCE BCE

ΔBCE
cân ti B nên
BE BC
.
Mt khác
BF//CE
nh lý Ta-lét, ta có:
38
CK EK CK FK EK BK CF BE
FK BK FK BK FK BK

BC BE
nên
CF BC
(1)
FK BK
CE
ng phân giác ca góc
ACK
nên
AE AC
(2)
KE CK
BC AC
ABC CBK g.g (3)
BK CK
T (2) và (3)
CF AE
(4)
FK KE

Gi s ng thng
EF
ct cnh
AC
ti
D
. Áp dnh lý Menelaus vào
ACK
b cát tuyn
DEF
ct
các cnh, ta có :
AD CF KE
. . 1 (5)
CD KF AE
T (4) và (5)
AC
1 AC CD.
CD
17.13. Gi
N
m cng thng
BM
CL
. Ta có t giác
MLKA
là hình bình hành. Gi
s ng thng
ML
ct cnh
BC
ti
P
 :
LP LK;MD CP
. Ta s chng minh
D,N,K
thng hàng.
Áp dnh lý Menelaus vào
BMP
b cát tuyn
CLN
ct các cnh, ta có :
BN ML PC BN AK MD
. . 1 . . 1
NM LP CB NM KB AD
m
K,N,D
thng hàng.
o vào
ABM
)
Vng thng
CL,DK,BM
ng quy.
17.14. Áp dnh lý Ce - va vào
ABC
, ta có :
CN AD BM
. . 1 1
NA DB MC
Áp dnh lý Menelaus vào
ABC
vm
N,M,P
thng hàng, ta có :
39
CN AP BM
. . 1 2
NA PB MC

T (1) và (2)
CN AD BM CN AP BM
. . . .
NA DB MC NA PB MC

AD AP
DB PB

T gi thit
CD
ng phân giác ca
ABC
AD CA AP CA
DB CB PB CB
CP
ng phân giác ngoài ca
ABC
CD CP
17.15. Qua
A
k ng thng
xy
song song vi
BC
ct
DM,DN
lt ti
H
I
Theo h qu cnh lý Ta - let , ta có :
AH AF AI AE
;
BD BF CD EC

Áp dnh lý Ce - va vào
ABC
vng thng
AD,BE,CF
ng quy ti
O
,
ta có :
AF BD CE AH BD CD
. . 1 . . 1
BF CD EA BD CD AI
AH
1 AH AI
AI
MN
//
AI
, Theo h qu cnh lý Ta - let , ta có :
OM DO ON
AH DA AI

AH AI
nên
OM ON
17.16. K ng trung tuyn
AI,BK,CP
ca
ABC
có trng tâm
G
chia tam giác thành 6 tam giác
BGI,BGP,CGK,AGK,AGP,CGI
.
m
M
nm trong m c trên cnh.
Gi s
M
nm trong hoc trên cnh ca
AGK
.
nh lý Van- Oben, ta có :
AM AF AE AF AE
2
MD FB EC PB KC
Mt khác :
BM BF BD BF BD
2
ME FA DC FA IC
Dy ra khi
M
trùng vi
G

17.17. Áp dnh lý Menelaus vào
ABC
vm
M,N,I
thng hàng, ta có :
40
IB MC NA MC NA
. . 1 . 1 (1)
IC MA NB MA NB
Áp dnh lý Menelaus vào
ABC
vm
Q,P,I
thng hàng, ta có :
IC PA QB PA QB
. . 1 . 1 (2)
IB PC QA PC QA
Áp dnh lý Menelaus vào
ABC
vm
N,E,P
thng hàng, ta có :
EB PC NA
. . 1 (3)
EC PA NB
Áp dnh lý Menelaus trong
ABC
vi
, , Q M F
thng hàng, ta có:
. . 1
FC QB MA
FB QA MC
(4).
T (1) và (2) suy ra:
..
MC NA QB PA
MA NB QA PC

..
PC NA QB MA
PA NB QA MC
.
T (3) và (4) suy ra:
. . . .
EB PC NA FC QB MA
EC PA NB FB QA MC
.
T 
EB FC EB FC EB FC
BE FC IE IF
EC FB EB EC FB FC BC BC

.
17.1 a) Áp dnh lý Menelaus cho tam giác
ABE
vm thng hàng
, , A M C

ta có:
. . 1 .
AM EA BC AM C A A B
ME A B C A ME BC EA
.
Áp dnh lý Menelaus cho tam giác
AFC
vm thng hàng
, , A N B

ta có:
. . 1 .
FN AB CA FN A F B C
NA B C A F NA CA AB
Xét
. . . . . . . . 1
AM EA FN C A A B EA A F B C C A A B B C
ME A F NA BC EA A F CA AB BC CA AB
(do
, AA BB

CC
ng quy ti
K
nh lý Ce-va)
 -va ta
, AA EN
FM
ng quy ti
I
.
b) Áp d nh Van - Oben cho tam giác
ABA
;
ACA
;
AEF
, ta có:
AM AK AC
ME KA C B


(1);
I
F
E
K
C'
B'
A'
C
B
A
M
N
41
AN AK AB
NF KA B C


(2);
AM AN AI
ME NF IA

(3).
c:
2.
AK AC AB AI
KA C B B C IA

.
Áp dnh lý Van-Oben cho tam giác
ABC
, ta có:
AC AB AK
C B B C KA


(4).
Thay vào (4) ta c:
3. 3. . .
AK AI
IA AK KA AI
KA IA


.
Phát triển tư duy sáng tạo giải toán Hình học 8-Bùi Văn Tuyên (Chủ biên)
1
Chương IV. HÌNH LĂNG TRỤ ĐỨNG HÌNH CHÓP ĐỀU
CHUYÊN ĐỀ 18. HÌNH HP CH NHT ...................................................................................................................... 2
CHUYÊN ĐỀ 19. HÌNH LĂNG TRỤ ĐỨNG .................................................................................................................... 6
CHUYÊN ĐỀ 20. HÌNH CHÓP ĐỀU ............................................................................................................................... 12
Phát triển tư duy sáng tạo giải toán Hình học 8-Bùi Văn Tuyên (Chủ biên)
2
CHUYÊN 18. HÌNH HP CH NHT
18.1 (h.18.10)
a) Xét hình bình hành
ACC A

//AC A C

//AC
mp
()BA C

.
Xét hình bình hành
ABC D

//AD BC

//AD
mp
()BA C

. Hình 18.10
AC
AD
ct nhau ti
A
nên mp
( )//ACD
mp
()BA C

.
b) (h.18.11)
Mt phng
cũng là mặt phng
()CDA B

.
Mt phng
cũng là mặt phng
()BCD A

.
Hai mt phẳng này có hai điểm chung là
C
A
nên chúng ct nhau theo giao
tuyến
CA
.
Hình 18.11
18.2 (h.18.12)
T giác
ADD A

là hình ch nht nên
DD D A
.
T giác
DCC D

là hình ch nht nên
DD D C
.
Suy ra
DD
mp
()A B C D
. Do đó
DD D B
.
T giác
DBB D

//DD BB

DD BB

nên là hình bình hành. Hình bình
hành này có
DD D B
nên là hình ch nht.
Hình 18.12
Gọi O là giao điểm ca AC và BD, Gi O
là giao điểm ca A
C
và B
D
Ta có OO
là đường trung bình ca hình ch nht DBD
B
nên OO
DB
Ta li có AC BD (tính chất đường chéo hình vuông) BD mp(ACC
A
)
Mt phng (DBB
D
) cha BD nên mp(DBB
D
) mp(ACC
A
)
18.3. (h.18.12)
a) Gọi O là giao điểm ca AC và BD, Gi O
là giao điểm ca A
C
và B
D
Ta có O AC mà AC
mp(ACC
A
) nên O mp(ACC
A
)
O BD mà BD
mp(BDD
B
) nên O mp(BDD
B
)
Vậy O là điểm chung ca hai mt phng (ACC
A
) và (BDD
B
)
Chứng minh tương tự, O
là điểm chung ca hai mt phng (ACC
A
) và (BDD
B
)
Phát triển tư duy sáng tạo giải toán Hình học 8-Bùi Văn Tuyên (Chủ biên)
3
Hai mt phng (ACC
A
) và (BDD
B
) có hai điểm chung là O và O
nên chúng ct nhau theo gia tuyến là
đường thng OO
b) Trong mt chéo (DBB
D
) có OO
là đường trung bình nên OO
B
D
ti O
Chứng minh tương tự ta được OO
A
C
ti O
Đưng thng OO
vuông góc vi hai đường thng giao nhau ca mp(A
B
C
D
) nên OO
(A
B
C
D
)
c) Ta có OO
(A
B
C
D
) mà OO
mp(BDD
B
) nên mp(BDD
B
) mp(A
B
C
D
)
18.4. a) Các hình lập phương nhỏ không được sơn mặt nào là các hình lập phương ở bên trong. Chúng to thành
mt hình hp ch nhật có độ dài các cnh là:
8 2 = 6 (cm); 12 2 = 10 (cm); 5 2 = 3 (cm)
Th tích ca hình hp ch nhật đó là: 6.10.3 = 180 (cm
3
)
Vy có tt c 180 hình lập phương nhỏ không được sơn mặt nào
b) Có tt c 480 hình lập phương nhỏ trong đó có 180 hình không được sơn mặt nào. Vy s hình lập phương
nh có ít nht mt mặt được sơn là: 480 – 180 = 300 (hình)
18.5. (h.18.13)
a) Các hình lập phương đơn vị không được sơn mặt nào
bên trong hình lập phương đã cho, chúng tạo thành
mt hình lập phương có cạnh dài
27 3
(đơn vị)
Do đó cạnh ca hình lập phương đã cho dài là:
n = 3 + 2 = 5 (đơn vị dài)
b) mỗi đỉnh có mt hình lập phương đơn vị sơn ba mặt.
Có tt c 8 đỉnh nên có 8 hình lập phương đơn vị được sơn ba mặt.
c) mi cnh có ba hình lập phương đơn vị được sơn hai mặt. Có tt c 12 cnh nên có 3.12 = 36 hình lp
phương đơn vị được sơn hai mặt
d) mi mt có 9 hình lập phương đơn vị được sơn một mt. Có tt c 6 mt nên có 9.6 = 54 hình lập phương
đơn vị được sơn một mt.
Hình 18.13
Phát triển tư duy sáng tạo giải toán Hình học 8-Bùi Văn Tuyên (Chủ biên)
4
18.6. (h.18.14)
Khai trin hình lập phương rồi tri phng 3 mt (ABCD), (CDD
C
) và (ADD
A
) ta được hình dưới:
Hình 18.14
* Xét trường hp kiến bò qua cnh DD
để tới đỉnh A; Đoạn đường ngn nht mà kiến phi bò t M đến A là:
22
1
(6 3) 6 117 10,8(cm)MA
* Xét trường hp kiến bò qua cnh DC để tới đỉnh A; Đoạn đường ngn nht mà kiến phi bò t M đến A là:
22
2
(6 6) 3 153 12,4(cm)MA
* Xét trường hp kiến bò qua cnh CC
để tới đỉnh A; D thấy đoạn đường mà kiến phi bò t M đến A dài hơn
nhiu so với hai trường hp trên.
Kết lun: Đoạn đường ngn nht mà kiến phi bò t M đến A là: 10,8 (cm)
18.7. (h.18.15)
a) Hình hp ch nhật có 8 đỉnh. S đoạn thẳng mà hai đầu ca nó
là hai đỉnh ca hình hp ch nhật là: (8.7) : 2 = 28 (đoạn thng)
b) 28 đoạn thng này chia làm 7 nhóm, mỗi nhóm 4 đoạn thng
dài bng nhau (chng hn AB = CD = D
C
= A
B
).
T đó suy ra trong 28 đoạn thng này ch có tối đa 7 giá trị khác nhau v độ dài.
18.8. Lúc đầu tng 6 s 6 mt là:
1 2 3 4 5 6 21
. Đó là một s l. Sau mỗi lượt tổng này tăng thêm
mt s chn nên tng các s 6 mt luôn là mt s l, không chia hết cho 6. Do đó không thể xy ra c 6 s
bng nhau.
18.9. Áp dng công thức tính độ dài đường chéo ca hình hp ch nht:
2 2 2 2 2 2 2
d a b c 8 9 12 289
d 289 17
.
Vậy độ dài ln nht ca một đoạn thng có th đặt trong hình hp ch nht là 17.
A
B
C
D
D'
C'
B'
A'
M
D
A
1
C
A'
C'
B
A
2
D'
M
A
B
C
D
D'
C'
B'
A'
hình 18.15
Phát triển tư duy sáng tạo giải toán Hình học 8-Bùi Văn Tuyên (Chủ biên)
5
18.10. Gọi ba kích thước ca hình hp ch nht là a, b, c. Ta có:
2 2 2 2
a b c 61 1
a b c 37 2
T (1) suy ra
2
2 2 2 2
a b c 61 a b c 2 ab bc ca 3721
Do đó:
2
2 ab bc ca 3721 1369 2352 cm
Vy din tích toàn phn ca hình hp ch nht là: 2352cm
2
.
18.11. Gọi a là độ dài ca mi cnh ca hình lập phương và d là độ dài đường chéo ca hình lập phương đó. Ta
có:
22
d 3a d a 3 cm
.
Độ dài đường chéo mi mt ca hình lập phương đó là
a2
.
Ta có:
a
3 a 2 1 a 3 2 1 a 3 2 cm
.
Din tích toàn phn ca hình lập phương là:
2
22
S 6a 6 3 2 59,39 cm
.
Th tích ca hình lập phương là:
3
33
V a 3 2 31,14 cm
.
Phát triển tư duy sáng tạo giải toán Hình học 8-Bùi Văn Tuyên (Chủ biên)
6
H
G
N
M
E
F
C'
B'
A'
C
B
A
Hình 19.7
M
B'
C'
A'
C
B
A
CHUYÊN ĐỀ 19. HÌNH LĂNG TRỤ ĐỨNG
19.1. (h.19.6)
G F là giao điểm ca
AB'
BA'
.
Gọi H là giao điểm ca
AC'
CA'
.
Vì E là trngj tâm ca
ABB'
nên :
21
BE BF BA'
33

.
Vì G là trng tâm ca
nên :
21
CG CH CA'
33

Ta có :
EM/ /BB' EM/ /AA';GN/ /CC' GN/ /AA'
.
Xét
BAA'
EM / /BB'
nên :
'
B
M BE 1
1
BA BA 3

Hình 19.6
Xét
CAA'
GN/ AA'
nên :
'
C
N CG 1
2
CA CA 3

T (1) và (2) suy ra :
BM CN 1
BA CA 3




. Do đó :
MN/ /BC
.
Mt khác:
ME / /BB'
nên
m
p MNGE / /mp BCC'B'
19.2. (h.19.7).
a) Các mt phng
ABB'A'
ACC'A'
là nhng hình ch nhật có cùng kích thước nên các dường chéo ca
chúng phi bng nhau:
AB' AC'
.
Xét
AB'C'
cân tại A, có AM là đường trung tuyến nên
A
M B'C' 1
Xét
A'B'C'
cân tại A’, có A’M là đường trung tuyến nên
A
'M B'C' 2
.
T (1) và (2) suy ra:
B'C' mp(AA'M)
.
b) Xét
A'B'M
vuông ti M, ta có:
22
A'M 10 6 8 cm
.
Xét
AA'M
vuông ti
A'
, ta có:
22
AA' 17 8 15 cm
.
Din tích xung quanh của hình lăng trụ :
2
xq
S 2p.h 10 10 12 .5 480 cm
.
Diện tích đáy của hình lăng trụ :
2
11
S B'C'.A'M .12.8 48 cm
22
.
Din toàn phn của hình lăng trụ :
2
tp
S 480 48.2 576 cm
.
Phát triển tư duy sáng tạo giải toán Hình học 8-Bùi Văn Tuyên (Chủ biên)
7
Hình 19.8
30
°
H
D
C
B
A
Hình 19.9
C'
B'
A'
C
B
A
19.3. Gi s cnh ca một đáy là n. Khi đótổng s cnh của hình lăng trụ đứng là 3n;tng s đỉnh là 2n và tng
s mặt là n + 2. Theo đề bài ta có:
n
2 2n 3n 26 n 4
.
Vậy hình lăng trụ đều này có đáy hình vuông.
Ta có :
32
xq
V S.h 540 cm ;S 2ph 360 cm .
xq
V 540
S 360

hay
S.h 3 S 3
2ph 2 2p 2
. Do đó :
2
a3
a 6 cm
4a 2
Chiu cao của hình lăng trụ :
d
ay
V 540
h 15 cm
S 36
.
19.4. Vì din tích toàn phn bng hai ln din tích xung quanh nên diện tích hai đáy bằng din tích xung quanh
(1).
Xét đáy là hình thoi ABCD cạnh a, góc nhn 30
0
(h.19.8).
V
AH CD
, ta có :
1a
AH AD
22

Din tích ABCD : S
đáy
=
2
aa
a. 2
22

Ta có :
xq
S
2ph 4ah 3
.
T (1), (2), (3) ta được :
2
aa
2. 4ah h
24
.
19.5. (h.19.9)
T công thc
xq
xq
S
S 2ph 2p
h
Vy chu vi của hình lăng trụ đứng là :
300
2p 30 cm
10

B
C 30 5 12 13 cm
.
Ta có :
2 2 2
BC AB AC
( Vì
2 2 2
13 5 12
).
Do đó
ABC
vuông ti A.
Din tích của hình lăng trụ là:
2
11
S AB.AC .5.12 30 cm
22
.
Phát triển tư duy sáng tạo giải toán Hình học 8-Bùi Văn Tuyên (Chủ biên)
8
Hình 19.10
O
D'
C'
B'
A'
D
C
B
A
Hình 19.11
E'
D'
C'
B'
A'
E
D
C
B
A
Th tích của hình lăng trụ là:
3
V S.h 30.10 300 cm
.
19.6. (h.19.10).
Diện tích đáy của hình lăng trụ là:
2
1
S .16.30 240 cm
2

.
Din tích xung quanh là:
2
xq
S 2860 240.2 2380 cm
Độ dài cạnh đáy là :
2 2 2 2
AB OA OB 8 15 17 cm
.
Chu vi đáy là : 17.4 = 68(cm).
Chiu cao của hình lăng trụ :
xq
S
2380
h 35 cm
2p 68
.
Th tích của hình lăng trụ là:
3
V S.h 240.35 8400 cm
.
Vy th tích ca cc là 54cm
3
.
19.7(h.19.11)
Gi h là chiu cao của hình lăng trụ.
Din tích xung quanh của hình lăng trụ đứng
ABCE.A'B'C'E'
:
1
S
AB BC CE EA .h 3a CE .h
.
Din tích xung quanh của hình lăng trụ đứng
CDE.C'D'E'
:
2
S
CD DE EC .h 2a CE .h
.
2
12
S S 4a
nên
2
3a CE 2a CE .h 4a
Hay
22
a.h 4a h 4a :a 4a
.
Vy din tích xung quanh ca hình lăng trụ đứng đã cho là:
2
xq
S 2ph 5a.4a 20a dvdt
.
Phát triển tư duy sáng tạo giải toán Hình học 8-Bùi Văn Tuyên (Chủ biên)
9
19.8. (h.19.12)
a) Xét hình thang
ABCD
vuông ti
A
.D
V
BH CD
(h.19.13)
Hình 19.12 Hình 19.13
T giác
ABHD
là hình vuông và
HBC
vuông cân ti
.H
Suy ra
; 2 ; 2.DH AB AD BH CH a CD a BC a
Xét
DAC
vuông ti
D
có:
2
2 2 2 2 2
2 5 .AC AD DC a a a
Suy ra
22
5.A C a

Trong hình lăng trụ đứng, cnh bên vuông góc với đáy nên
m
p .AA A B C D AA A C
Xét
vuông ti
A
, ta có:
2 2 2 2
9 5 2 .AA AC A C a a a
Diện tích đáy hình lăng trụ là:
2
. 2 .
3
.
2 2 2
AB CD AD a a a
a
S

Th tích hình lăng trụ là:
2
2
3
. .2 3 .
2
a
V S h a a
b) Diện tích xung quanh hình lăng trụ đứng là:
22
2 2 .2 8 2 2 .
xq
S a a a a a a a
Din tích toàn phần hình lăng trụ đứng là;
2
2 2 2 2
3
8 2 2 .2 11 2 2 .
2
tp
a
S a a a a
19.9. (h.91.14)
A
D'
A'
C'
B'
B
D
C
a
a
H
A
B
D
C
Phát triển tư duy sáng tạo giải toán Hình học 8-Bùi Văn Tuyên (Chủ biên)
10
a) b)
Hình 19.14
a) Xét trường hp th nht: Tm bạt được căng theo chiều dài (h.a).
Ta có:
2 2.
2
b
BC AB
Din tích mặt đất bên trong lu là:
1
2
. 2.
22
b ab
S BC CC a
(đvdt).
Xét trường hp th hai: Tm bạt được căng theo chiều rng (h.b).
Ta có:
2 2.
2
a
EF DE
Din tích mặt đất bên trong lu là:
2
22
..
22
a ab
S EF FF b
(đvdt).
So sánh hai kết qu ta thy
12
.SS
b) Xét trường hp th nht: Th tích không khí bên trong lu là:
2
2
1
11
.
2 2 8
b
V a ab




(đvtt).
Xét trường hp th hai: Th tích không khí bên trong lu là:
2
2
2
11
.
2 2 8
a
V b a b




(đvtt).
Ta có:
22
21
1 1 1
0
8 8 8
V V a b ab ab a b
(vì
ab
). Suy ra:
21
.VV
Vy nếu căng tấm bt theo chiu rng thì th tích không khí bên trong lu s lớn hơn.
19.10. (h.19.15)
Ta đặt
2 ; 2 .AC m BD n
Diện tích đáy
ABCD
là:
1
2 .2 2 .
2
S m n mn
Mt khác
2
1280
64( ).
20
V
S cm
h
Vy
2
2 . 64( ).m n cm
Diện tích xung quanh hình lăng trụ đứng là:
a
a
B
B'
C
C'
A
A'
b
b
F
F'
E
E'
D
D'
O
A'
D'
B'
C'
B
C
D
A
Phát triển tư duy sáng tạo giải toán Hình học 8-Bùi Văn Tuyên (Chủ biên)
11
4. .20 80 .
xq
S AB AB
Vy
xq
S
nh nht khi
AB
nh nht.
Gi
O
là giao điểm của hai đường chéo
AC
.BD
Ta có
AC BD
ti
.O
Xét
AOB
vuông ti
,O
ta có:
2 2 2 2 2
.AB OA OB m n
Hình 19.15
Mt khác
22
2.m n mn
Do đó
2
64 8( ).AB AB cm
Vy giá tr nh nht ca
AB
8cm
khi
mn
tc là khi
ABCD
là hình vuông.
Giá tr nh nht ca diên tích xung quanh là
2
4.8.20 640( ).cm
19.11. (h.19.16)
a) Chu vi đáy của đèn là:
18.6 108( ).cm
Din tích xung quanh của đèn là:
2
2 . 108.40 4320( ).
xq
S p h cm
Vy din tích giấy bóng kính để làm mt xung quanh của đèn là
2
4320( ).cm
b) Diện tích đáy đèn là:
22
2
3 18 3
.6 .6 486 3( ).
44
a
S cm
Th tích của đèn lồng là:
33
. 486 3.40 19440 3( ) 33671( ).V S h cm cm
c) Gi
a
b
lần lượt là độ dài cạnh đáy đèn lồng trước và sau
khi gim th tích. Gi
1
S
2
S
là các diện tích đáy tương ứng.
Khi đó:
11
.;V S h
22
..V S h
Ta có:
22
1 1 1
2 2 2
.
3.6 3.6
2 2 2 : 2
. 4 4
V S h S
ab
V S h S
22
: 2 : 2.a b a b
Vậy độ dài cạnh đáy phải giảm đi
2
ln.
Hình 19.16
O'
O
E
F
A
B
C
C'
E'
D'
A'
B'
F'
D
Phát triển tư duy sáng tạo giải toán Hình học 8-Bùi Văn Tuyên (Chủ biên)
12
CHUYÊN ĐỀ 20. HÌNH CHÓP ĐỀU
20.1. (h.20.8)
a) Xét
SAB
;SA SB SA SB


nên
// .
SA SB
A B AB
SA SB



Chứng minh tương tự, ta được:
// .C D CD

Mt khác
//AB CD
nên
// .A B C D
T đó suy ra bốn điểm
, , ,A B C D
cùng nm trên mt mt phng.
Ta có:
// ; //A B AB B C BC
AB

BC

ct nhau ti
;B
AB
BC
ct nhau ti B.
T đó suy ra: mp
//A B C D
mp
.A
BCD
b) Gi
O
là giao điểm ca
AC
.BD
.S ABCD
là hình chóp đều nên
;AO SO
AO DO AO
mp
.S
OD
Hình 20.8
mpA
O SAC
nên
m
p mp .SAC SBD
20.2. (h.20.9)
Ta đặt
2
2
2
a
AB a AC a OA
Xét
SAC
; 90SA SC ASC
nên
SAC
vuông cân
45 .SAO
Xét
SOA
90 ; 45SOA SAO
nên
SOA
vuông cân
.SO OA
Ta có:
22
22
2 2 2 2
22
.
2 2 2 2
a a a a
SA SO OA a
Do đó
.SA a
Hình 20.9
Xét mt bên
SAB
SA SB AB a
nên là tam giác đều. Do đó các mặt bên là những tam giác đều.
20.3. (h.20.10)
Xét
SBC
MN
là đường trung bình nên
//MN BC
(1)
2
BC
MN
Xét
ABC
PQ
là đường trung bình nên
//PQ BC
(2)
2
BC
PQ
T (1) và (2) suy ra
//MN PQ
.MN PQ
Do đó tứ giác
là hình bình hành. Ta có:
; .
2 2 2 2
BC a SA a
MN MQ
Vy
M N M Q
, suy ra hình bình hanh
là hình thoi. Hình 20.10
O
D
C
A
B
S
Q
P
N
M
C
B
A
S
B'
C'
D'
O
D
C
A
B
S
A'
Phát triển tư duy sáng tạo giải toán Hình học 8-Bùi Văn Tuyên (Chủ biên)
13
Xét
QBS
3
2
a
QB QS
nên
QBS
cân
QN SB
Xét
QNS
vuông ti
N
có:
2
2
2
2 2 2
32
2 2 2 2
a a a a
QN QS NS QN







Chứng minh tương tự, ta được
2
2
a
MP
. Do đó
QN MP
Hình thoi
MNPQ
có hai đường chéo bng nhau nên là hình vuông.
20.4 (h.20.11)
a) Ta có
; ( )SC SA SC SB SC mp SAB
Mt khác
()SC mp SAC
nên
( ) ( )mp SAC mp SAB
()SC mp SBC
nên
( ) ( )mp SBC mp SAB
Do đó mặt bên
()SAB
vuông góc vi các mt bên
()SAC
()SBC
Chứng minh tương tự ta được mi mt bên
()SBC
,
()SAC
đều vuông góc vi hai mt bên còn li.
b) Xét tam giác đều
ABC
. Gi
O
là giao điểm của các đường trung
tuyến
,CM BN
. Khi đó:
2 2 3 3
.
3 3 2 3
aa
BO BN
Xét
SAB
vuông ti
S
AB a
nên
2
2
a
SB
Xét
SOB
vuông ti
O
, ta có:
22
2
2 2 2
23
2 3 6
a a a
SO SB OB
6
6
a
SO
O
C
A
B
S
M
N
(h.20.11)
20.5 (h.20.12)
Xét hình chóp ct t giác đều
. ' ' ' 'ABCD A B C D
Gi
M
'M
lần lượt là trung điểm ca
BC
''BC
. Ta
/ / ; ' '/ / ' 'OM AB O M A B
' '/ /A B AB
nên
' '/ /O M OM
Trong hình thang
' 'MOOM
ta v
'M H OM
Ta được
' '; ' 'M H OO OH O M
Ta có
6:2 3( ); ' ' 4:2 2( ); 3 2 1( )OM cm O M cm HM cm
Tng diện tích hai đáy là:
2 2 2
12
6 4 52S S cm
Din tích xung quanh là:
2
6 4 . '
.4 20. '( )
2
xq
MM
S MM cm

Theo đề bài ta có
20. ' 52 MM' 2,6(cm)MM
O
O'
D'
C'
B'
A'
A
D
C
B
M'
M
H
(h.20.12)
Phát triển tư duy sáng tạo giải toán Hình học 8-Bùi Văn Tuyên (Chủ biên)
14
Xét
'M HM
vuông ti
H
, ta có
2
2 2 1
' ' 2,6 1 2,4( )M H M M HM cm
20.6 (h.20.13)
Gi
S
là đỉnh hình chóp sinh ra hình chóp ct
Gi din tích xung quanh ca hình chóp
.S ABCD
và hình chóp
2 2 2 2
.S A B C D
lần lượt là
S
2
S
Gọi các độ dài trung đoạn ca hình chóp
.S ABCD
và hình chóp
2 2 2 2
.S A B C D
lần lượt là
d
2
d
Ta có:
2 2 2
44
. 2 ; . 2
22
ac
S d ad S d cd
Xét
SBC
22
//BC B C
nên:
2 2 2 2
d SB BC a
d SB B C c
Do đó:
2
2
22
2
.
2
S ad a a a
S cd c c c
Chứng minh tương tự, ta được:
2
1
2
2
S
b
Sc
Theo đề bài ta có:
2 2 1
S S S S
Suy ra
21
2S S S
. Do đó :
1
2
2
SS
S
Vy :
1
22
2
S
S
SS

hay
2 2 2 2 2 2
2
2 2 2
22
2
a b a b a b
c
c c c

O2
O1
O
D1
D2
C2
C1
B1
B2
B
D
A
C
S
A2
A1
(h.20.13)
20.7 (h.20.14)
* Tìm cách gii :
Để tìm th tích của hình chóp đều khi ta đã biết cạnh đáy ta cần tính chiu cao ca hình chóp . Có th vn dng
định lí Py-ta-go để tính.
* Trình bày li gii :
ABCD
là hình vuông cnh
2a
nên
2. 2 2BD a a OB a
.S ABCD
là hình chóp đều nên
()SO mp ABCD
SOB
vuông ti
O
Ta có :
2
2 2 2 2 2
10 9 3SO SB OB a a a SO a
Th tích hình chóp là
2
3
11
. . . 2 .3 2
33
V S h a a a
O
C
A
B
D
S
(h.20.14)
20.8 (h.20.15)
Phát triển tư duy sáng tạo giải toán Hình học 8-Bùi Văn Tuyên (Chủ biên)
15
Gi
O
là trc tâm ca lục giác đều
ABCDEF
Ta có
SO AD
Din tích tam giác
ADS
là:
11
. .2 . .
22
AD SO a SO a SO
Theo đề bài ta có:
2
.a SO a SO a
Gi
SM
là một trung đoạn của hình chóp, khi đó
OM BC
Xét
OBC
đều, cnh
a
, đường cao
3
2
a
OM
Xét
SOM
vuông ti
O
, ta có :
2
2
2 2 2 2
3 7 7
2 4 2
a a a
SM SO OM a SM




Din tích xung quanh hình chóp là
2
6 7 3 7
.
2 2 2
xq
a a a
S 
O
F
A
B
C
D
E
S
M
(h.20.15)
20.9 (h.20.16)
Gi
M
là trung điểm ca
AB
Khi đó
SM
là trung đoạn ca hình chóp.
Ta đặt
AB x
thì
2
2
2 2 2 2 2
1
4.
2 4 2
xx
SM SB a SM a x



Din tích xung quanh ca hình chóp là :
2 2 2 2
3 1 3
. . 4 . 4 .
2 2 4
xq
xx
S a x a x
Vn dng bất đẳng thc
22
2a b ab
hay
22
.
2
ab
ab
ta được:
2 2 2
2 2 2
4
. 4 2 .
2
x a x
x a x a

Do đó
22
33
.2
42
xq
S a a
.
Du = xy ra khi
2 2 2 2 2 2 2
4 4 2 .x a x x a x x a
Khi đó
2 2 2
SA SB AB
( vì
2 2 2
2a a a
)
Theo định lí Py ta- go đảo thì
SAB
vuông nên
SA SB
.
Chứng minh tương tự ta có :
;SB SC SC SA
.
S
M
C
A
B
Phát triển tư duy sáng tạo giải toán Hình học 8-Bùi Văn Tuyên (Chủ biên)
16
Vy
2
3
max
2
xq
Sa
khi SA,SB,SC vuông góc với nhau đôi một
20.10 ( h. 20.17)
Ta đặt BC =2a và trung điểm đoạn SM = d ( a<d).
Khi đó
2 .4
.4
2
xq
a
S d ad
Theo đề bài ta có : 4ad = 48 suy ra ad = 12 (1)
Xét
SMC
vuông ti M, ta có
2 2 2
.MC SM SC
Do đó
22
25.ad
Suy ra
22
2 25 24a d ad
Suy ra
2
( ) 49 7a d a d
(2)
T (1) và (2) ta được
7 4; 3( )
12 3; 4( )
a d a d l
ad a d tm


Khi đó
2 2 2 2 2
16 9 7 7( ).SO SM OM d a h SO cm
Vy th tích hình chóp là :
23
11
. 6 . 7 12 7( ).
33
V s h cm
20.11 ( h.20.18)
Xét tam giác SOC vuông ti O, ta có :
2 2 2 2 2
17 15 64 8OC SC SO OC
(cm)
Suy ra CM = 12 ( cm).
Gọi độ dài cạnh đáy là a.
Ta có
3 24
3 24 ( )
2
3
a
CM a a cm
Din tích đáy của hình chóp S.ABC là :
3
1 1 1
11
. 48. 3.15 240 3( )
33
V s h cm
Theo tính cht của đường trung bình ca tam giác ta có
' ' ' '
/ / ; / /AB AB AC AC
Suy ra 2 mf
( ' ' ');( )A B C ABC
song song.
Do đó hình chóp cụt
' ' 'A B C ABC
là hình chóp ct đều.
M
O
B
A
D
C
S
N'
O'
O
M'
N
M
C
B
A
S
C'
A'
B'
Phát triển tư duy sáng tạo giải toán Hình học 8-Bùi Văn Tuyên (Chủ biên)
17
Xét tam giác SOC có:
' ' 1
7,5( ).
2
SO SC
SO cm
SO SC
Ta có :
1 12
''
2
3
A C AC
(cm). Do đó diện tích tam giác
' ' 'A B C
là:
2
2
2
12 3
. 12 3( ).
4
3
S cm




20.12 ( h. 20.19)
a) Hình chóp C.BDC’ có đáy là tam giác đều, mi cnh dài bng
2a
. Ba mt bên là nhng tam giác vuông
cân bng nhau, mi tam giác có cnh bên bng a và cạnh đáy bằng
2a
. Do đó hình chóp C.BDC’ là hình chóp
đều.
b) Din tích xung quanh ca hình chóp là :
2
2
3
.3
22
xq
a
Sa
.
Diện tích đáy hình chóp là :
2
2
2 . 3
.3
.
42
a
a
S 
T s gia din tích xung quanh và diện tích đáy hình chóp là:
2
2
33
:3
22
xq
S
a
a
S

c) Xét hình chóp C.BDC’ (h 20.20) có CB = CD = CC’= a; BD = BC’ = DC’=
2a
Gọi M là trung điểm BC ,
CO DM
Ta có:
2 . 3
6
23
a
a
DM 
Xét tam giác COD vuông ti O có:
2
2
2 2 2 2
6
33
aa
CO CD DO a




suy ra
3
.
3
a
CO
Th tích hình chóp là
2
3
1
2 . 3
1 1 3
. . .
3 3 4 4 6
a
aa
V s h
Th tích hình lập phương là
3
2
Va
.
D
A
C
D'
B'
A'
C'
B
M
D
B
C'
C
O
Phát triển tư duy sáng tạo giải toán Hình học 8-Bùi Văn Tuyên (Chủ biên)
18
Vy
3
3
1
2
1
:.
66
V
a
a
V

| 1/315